847
2009 PREP SA on CD-ROM page 1 Question: 1 You are evaluating a 6-month-old child who has a ventricular septal defect and is scheduled for cardiac surgery. The child’s weight is 6 kg (3rd percentile), length is at the 30th percentile, and head circumference is at the 50th percentile. His mother states she prepares the formula by adding 1 scoop of powder to 2 oz of water. She estimates that he drinks 24 oz of formula per day. You estimate the baby’s intake is approximately 500 kcal per day of cow milk formula, which is the recommended dietary allowance (RDA) for his age. According to his mother, he spits up three times a day and passes two soft stools daily. On physical examination, you hear a 3/6 holosystolic murmur and palpate the liver 1 cm below the right costal margin. Of the following, the BEST explanation for the child’s malnutrition is A. caloric requirements exceeding the RDA B. cow milk protein intolerance C. incorrect preparation of the formula D. pathologic gastroesophageal reflux E. undiagnosed pancreatic insufficiency

AAP PREP 2009

Embed Size (px)

DESCRIPTION

pediatrics, MCQs, AAP

Citation preview

Page 1: AAP PREP 2009

2009 PREP SA on CD-ROM

page 1

Question: 1

You are evaluating a 6-month-old child who has a ventricular septal defect and is scheduled for cardiac surgery. The child’s weight is 6 kg (3rd percentile), length is at the 30th percentile, and head circumference is at the 50th percentile. His mother states she prepares the formula by adding 1 scoop of powder to 2 oz of water. She estimates that he drinks 24 oz of formula per day. You estimate the baby’s intake is approximately 500 kcal per day of cow milk formula, which is the recommended dietary allowance (RDA) for his age. According to his mother, he spits up three times a day and passes two soft stools daily. On physical examination, you hear a 3/6 holosystolic murmur and palpate the liver 1 cm below the right costal margin.

Of the following, the BEST explanation for the child’s malnutrition is

A. caloric requirements exceeding the RDA

B. cow milk protein intolerance

C. incorrect preparation of the formula

D. pathologic gastroesophageal reflux

E. undiagnosed pancreatic insufficiency

Page 2: AAP PREP 2009

2009 PREP SA on CD-ROM

page 2

Critique: 1 Preferred Response: A

Children who have large ventricular septal defects, such as described for the child in the vignette, have increased pulmonary blood flow and may have ventricular hypertrophy and heart failure. Because their hearts have to work harder, their caloric needs often are increased above the recommended dietary allowance (RDA) for healthy children. The absence of significant vomiting, diarrhea, or rectal bleeding suggests that the patient does not have significant gastroesophageal reflux, cow milk protein intolerance, or pancreatic insufficiency. Incorrect preparation of the formula always should be considered, but the mother's reported preparation is correct.The RDA is defined as "a nutrient intake level that is ... sufficient to meet the nutrient requirements of 97% of healthy individuals" in a given group, categorized by sex and age. RDAs are developed by the Food and Nutrition Board of the Institute of Medicine in collaboration with Health Canada. A full list of RDAs may be found in many reference textbooks and at the Food and Nutrition Information Center on the United States Department of Agriculture web site (http://fnic.nal.usda.gov). RDAs estimate energy requirements of healthy children and do not adjust for chronic illness states that increase caloric needs, such as cystic fibrosis or congenital heart disease. Children who have chronic illnesses often require more calories than the RDA to grow and develop.The estimated daily caloric requirement of a patient is the aggregate of the patient's basal metabolic rate and physical activity. Various mathematic equations can be used to estimate the daily caloric intake of children and adults of different ages. Such equations usually take into account the individual's age, sex, physical activity level, and either the weight and height or, preferably, the body surface area. For children who have chronic illnesses, more accurate estimates of daily caloric requirements can be obtained by using a laboratory-based technique, such as indirect calorimetry.

References:

McDaniel NL. Ventricular and atrial septal defects. Pediatr Rev. 2001;22:265-270. Available at: http://pedsinreview.aappublications.org/cgi/content/full/22/8/265

Sonneville K. Nutritional requirements: dietary reference intakes. In: Hendricks KM, Duggan C. Manual of Pediatric Nutrition. 4th ed. Hamilton, Ontario, Canada: BC Decker; 2005:83-100

Page 3: AAP PREP 2009

2009 PREP SA on CD-ROM

page 3

Question: 2

You are admitting a 750-g female infant to the neonatal intensive care unit (NICU) for treatment of respiratory distress and presumed sepsis. The pregnancy was complicated by chorioamnionitis and preterm labor. The infant’s trachea was intubated, a single dose of exogenous surfactant administered, and both an umbilical venous catheter and umbilical arterial catheter were placed successfully in the delivery room. In the NICU, the infant is placed on a radiant warmer. The nurse caring for the infant asks if the infant will need to be transferred to an isolette incubator.

Of the following, the MOST likely reason for this infant to be relocated into an isolette incubator is

A. avoidance of light damage to the eyes

B. bronzing of the skin under the radiant warmer

C. inability to maintain core temperature on a radiant warmer

D. increased risk for infection under the radiant warmer

E. increased transcutaneous water loss under the radiant warmer

Page 4: AAP PREP 2009

2009 PREP SA on CD-ROM

page 4

Critique: 2 Preferred Response: E

The extremely low gestational-age newborn (ELGAN) who has a birthweight of less than 1,000 g is at great risk for transcutaneous evaporative water loss in an open air, nonhumidified environment such as a radiant warmer. This is most concerning in the first 24 to 72 hours of postnatal life, diminishing thereafter as the infant's skin becomes cornified. The use of a contained, convectively heated, and humidified environment can reduce transcutaneous evaporative water loss best in the first few days of the ELGAN's postnatal life. Ambient light is not a contributing factor to retinopathy of prematurity, for which the ELGAN is also at risk. Although excessive thermal warming can occur under the radiant warmer, bronzing of the skin is related to the use of phototherapy lights in newborns who have conjugated hyperbilirubinemia. The ELGAN's core temperature can be maintained using a radiant warmer, but evaporative and convective heat losses remain a concern over lengthy periods of time, and differences in extremity, head, and core temperatures may affect perfusion and acid-base status. There is no increased risk for infection on the radiant warmer compared with the isolette incubator.

References:

Dollberg S, Hoath SB. Temperature regulation in preterm infants: role of the skin-environment interface. NeoReviews. 2001;2:e282-e291. Available for subscription at: http://neoreviews.aappublications.org/cgi/content/full/2/12/e282

Korones SB. An encapsulated history of thermoregulation in the neonate. NeoReviews. 2004;5:e78-e85. Available for subscription at: http://neoreviews.aappublications.org/cgi/content/full/5/3/e78

Sedin G. The thermal environment of the newborn infant. In: Martin RJ, Fanaroff AA, Walsh MC, eds. Fanaroff and Martin's Neonatal-Perinatal Medicine. 8th ed. Philadelphia, Pa: Mosby Elsevier; 2006:585-596

Page 5: AAP PREP 2009

2009 PREP SA on CD-ROM

page 5

Question: 3

You are seeing a 1-month-old girl for follow-up after a hospitalization for acute gastroenteritis caused by rotavirus. Her diarrhea had decreased in the hospital while taking oral rehydration solution, but when her mother resumed her usual cow milk formula, the girl began to have an increased number of very watery stools. She appears well hydrated, and findings on her abdominal examination are normal.

Of the following, the MOST appropriate approach to managing this infant’s diarrhea is to

A. change to a lactose-free formula for the next few days

B. dilute the cow milk formula with oral rehydration solution for the next few days

C. give her only oral rehydration solution until the diarrhea resolves

D. readmit her to the hospital for administration of intravenous fluids

E. repeat her stool studies to confirm the diagnosis of rotavirus infection

Page 6: AAP PREP 2009

2009 PREP SA on CD-ROM

page 6

Critique: 3 Preferred Response: A

The infant described in the vignette most likely has lactase deficiency due to rotavirus infection. Lactase is an enzyme found in the most superficial villous portion of the intestinal brush border, which hydrolyzes lactose to glucose and galactose. Lactase deficiency may have several causes in children and adults. Primary lactase deficiency, the most common type, is a genetically determined condition that affects children and adults at different ages but is unusual before 5 years. Symptoms include abdominal distention, bloating, flatulence, or nausea after the ingestion of lactose, with the amount of lactose needed to cause such symptoms varying from person to person. The diagnosis is made by breath hydrogen testing, and management consists of removing some or all lactose from the diet. Congenital lactase deficiency is extremely rare. Secondary lactase deficiency may develop after an infectious gastroenteritis, such as rotavirus, giardiasis, or cryptosporidiosis. Other causes include celiac disease and enteropathy related to immunodeficiency. Secondary lactase deficiency is suggested when a child who has a recent diarrheal illness experiences worsening diarrhea or bloating after the reintroduction of lactose into the diet, as described for the girl in the vignette. Most children who have gastroenteritis do not develop lactase deficiency. For this reason, most infants can tolerate and should continue taking human milk or standard lactose-containing formula throughout a diarrheal illness. For very young infants (eg, <3 months old), such as the one described in the vignette, or those who have significant fluid losses, a lactose-free formula may be attempted until the diarrhea resolves. Infants who are breastfed should be encouraged to continue breastfeeding, even if secondary lactase deficiency is suspected. Giving full-strength formula or human milk is recommended to supply the child with sufficient calories during the recovery phase of a diarrheal illness; therefore, diluting the formula or providing only oral rehydration solution is inappropriate. If the child is not vomiting, oral hydration is optimal, and intravenous hydration is not necessary. There is no need to confirm the diagnosis of rotavirus infection; doing so would not alter management plans.

References:

Dalby-Payne J, Elliott E. Gastroenteritis in children. BMJ Clinical Evidence. 2007. Available for subscription at: http://clinicalevidence.bmj.com/ceweb/conditions/chd/0314/0314.jsp

Heyman MB; Committee on Nutrition. Lactose intolerance in infants, children, and adolescents. Pediatrics. 2006;118:1279-1286. Available at: http://pediatrics.aappublications.org/cgi/content/full/118/3/1279

King CK, Glass R, Bresee, Duggan C. Managing acute gastroenteritis among children: oral rehydration, maintenance, and nutritional therapy. MMWR Morbid Mortal Wkly Rep Recomm Rep. 2003;52(RR-16):1-16. Available at: http://www.cdc.gov/mmwr/preview/mmwrhtml/rr5216a1.htm

Page 7: AAP PREP 2009

2009 PREP SA on CD-ROM

page 7

Question: 4

During a prenatal visit with expectant parents, they report that they are strict vegans. They ask you to advise them on a healthy diet and any required supplements. The mother plans to breastfeed the newborn exclusively for the first 6 months.

Of the following, you are MOST likely to tell them that their newborn may require supplemental

A. calcium

B. folate

C. iron

D. vitamin B6

E. vitamin B12

Page 8: AAP PREP 2009

2009 PREP SA on CD-ROM

page 8

Critique: 4 Preferred Response: E

A vegan diet, by definition, excludes all foods derived from animal products. A lacto-ovo-vegetarian diet may include milk and eggs. Although a vegan diet may be healthy, there is a risk for vitamin B12 deficiency because vitamin B12 is only found in foods of animal origin. Breastfeeding vegan mothers may produce milk that is deficient in this vitamin and require supplementation that generally is achieved by continuing the consumption of prenatal vitamins containing vitamin B12. The recommended supplementation for breastfed vegan infants to prevent vitamin B12 deficiency is 0.4 mcg/day during the first 6 postnatal months and 0.5 mcg/day from 6 months to 1 year of age. Vegan infants who are not breastfed should receive iron-fortified soy infant formula until 1 year of age to avoid deficiencies in iron. Vegan infants require no other mineral or vitamin supplementation. Vegan diets in older children and adolescents may be low in calcium (similar to the typical American "teenage diet" that contains less than the recommended intake of dairy products), and the zinc consumption may be relatively low due to the absence of phytate, which renders zinc more bioavailable. Children who follow vegan diets may have relatively diminished overall energy intake because such diets commonly are low in fat and high in fiber. Review of nutrient intake and energy intake in conjunction with growth curves of children eating vegan diets in both the United States and the United Kingdom demonstrate no significant health issues. Height and weight measured in vegan populations may be slightly lower than average but not in the range of failure to thrive or short stature. Adolescents eating vegan diets are more likely than adolescents eating a typical American diet to meet nutritional goals, including recommended intake of fruits and vegetables. Vegan adolescents are less likely to be obese because they consume fewer foods high in fat. However, they remain at risk for vitamin B12 deficiency and should consume at least a daily multivitamin. They are less likely to have anemia but just as likely to have low calcium intake as their non-vegan peers. Adolescents who follow a lacto-ovo-vegetarian diet are less likely to have deficiencies in vitamin B12, calcium, and iron. Folate and vitamin B6 are not likely to be deficient in persons who consume vegan diets because those nutrients are found in many legumes, fruits, and vegetables that are the mainstays of the diet.

References:

Kleinman RE. Nutritional aspects of vegetarian diets. In: Pediatric Nutrition Handbook. 5th ed. Elk Grove Village, Ill: American Academy of Pediatrics; 2003:191-208

Mangels AR, Messina V. Considerations in planning vegan diets: infants. J Am Diet Assoc. 2001:101:670-677. Abstract available at: http://www.ncbi.nlm.nih.gov/pubmed/11424546

Messina V, Mangels AR. Considerations in planning vegan diets: children. J Am Diet Assoc. 2001:101:661-669. Abstract available at: http://www.ncbi.nlm.nih.gov/pubmed/11424545

Moilanen BC. Vegan diets in infants, children and adolescents. Pediatr Rev. 2004:25:174-176. Available at: http://pedsinreview.aappublications.org/cgi/content/full/25/5/174

Perry CL, McGuire MT, Neumark-Sztainer D, Story M. Adolescent vegetarians: how well do their dietary patterns meet the Healthy People 2010 objectives? Arch Pediatr Adolesc Med. 2002; 156:431-437. Available at: http://archpedi.ama-assn.org/cgi/content/full/156/5/431

Page 9: AAP PREP 2009

2009 PREP SA on CD-ROM

page 9

Question: 5

You are treating a child who has suffered a splenic injury and is being transfused with large volumes of packed red blood cells for severe anemia. He weighs 10 kg and has received 4 units thus far.

Of the following, the finding on electrocardiography that is MOST likely to represent a serious complication of his therapy is

A. atrial flutter

B. delta waves

C. prominent U waves

D. supraventricular tachycardia

E. tall-peaked T waves

Page 10: AAP PREP 2009

2009 PREP SA on CD-ROM

page 10

Critique: 5 Preferred Response: E

Administration of fluids and blood products can be essential for resuscitation of the trauma patient but may lead to potentially dangerous electrolyte imbalances. Recognition of these abnormalities, which may be subtle, can be important and even lifesaving. When large volumes of blood are transfused, as reported for the child in the vignette, hyperkalemia may occur, which is believed to be the result of extravasation of potassium from the red blood cells that have been irradiated and stored over time. Hyperkalemia can be associated with paresthesias, weakness, and tingling, although cardiac toxicity typically precedes such symptoms. Severe cardiac rhythym changes may begin abruptly. The classic electrocardiographic sign of hyperkalemia is tall, peaked T waves (Item C5A), particularly as the serum potassium concentration approaches or exceeds 5.0 to 6.0 mEq/L (5.0 to 6.0 mmol/L). As hyperkalemia progresses, other changes in the ECG, such as widening of the QRS complex, may be noted. The rhythm changes that occur as a result of hyperkalemia, including ventricular arrhythmias, may develop abruptly, leading to sudden changes in the patient's clinical condition. Another potential electrolyte complication of fluid and blood administration is hypocalcemia, which results from the citrate-containing anticoagulants that bind free calcium. Hypocalcemia may manifest on ECG as a prolonged QT interval, which tends to widen as the ionized calcium concentrations decrease. Atrial flutter, a primary disease of the atrial tissue, would not be expected in the patient described in the vignette. The delta wave refers to the ECG finding of pre-excitation, seen in conditions such as the Wolff-Parkinson-White syndrome that have an associated "bypass" tract allowing for excitement of the His-Purkinje system without passage through the atrioventricular node (Item C5B). The U wave may be seen in hypokalemia (Item C5C), particularly as concentrations decrease below 2.7 mEq/L (2.7 mmol/L), or hypercalcemia (calcium concentrations exceeding 12.0 mg/dL [3.0 mmol/L]), which would not be expected in the patient described in the vignette. Hypercalcemia also may lead to a diminished QT interval, and with more severe hypercalcemia, second- or third-degree heart block (Item C5D) may develop. Supraventricular tachycardia (Item C5E) would not be expected to occur as a result of a large-volume transfusion process.

References:

Galel SA, Naiman JL. Use of blood and blood products. In: Rudolph CD, Rudolph AM, eds. Rudolph's Pediatrics. 21st ed. New York, NY: McGraw-Hill Medical Publishing Division; 2003:1576-1581

Vetter V. Arrhythmias. In: Moller JH, Hoffman JIE, eds. Pediatric Cardiovascular Medicine. Philadelphia, Pa: Churchill Livingstone; 2000:833-884

Page 11: AAP PREP 2009

2009 PREP SA on CD-ROM

page 11

Critique: 5

The electrocardiogram in hyperkalemia shows peaked T waves. (Courtesy of A. Friedman)

Page 12: AAP PREP 2009

2009 PREP SA on CD-ROM

page 12

Critique: 5

In Wolff-Parkinson-White syndrome, delta waves (arrows) are present that represent pre-excitation depolarization of the QRS complex. (Courtesy of A. Friedman)

Page 13: AAP PREP 2009

2009 PREP SA on CD-ROM

page 13

Critique: 5

The U wave (arrow) may be observed in hypokalemia (shown here in which there is also ST depression and flattening of the T wave) or hypercalcemia. (Courtesy of A. Friedman)

Page 14: AAP PREP 2009

2009 PREP SA on CD-ROM

page 14

Critique: 5

In third-degree heart block that may result from severe hypercalcemia, P waves are completely dissociated from QRS complexes. (Courtesy of A. Friedman)

Page 15: AAP PREP 2009

2009 PREP SA on CD-ROM

page 15

Critique: 5

Narrow complex tachycardia that is consistent with supraventricular tachycardia or paroxysmal atrial tachycardia. (Courtesy of A. Friedman)

Page 16: AAP PREP 2009

2009 PREP SA on CD-ROM

page 16

Question: 6

A 15-year-old girl presents to the emergency department with a 4-week history of nasal drainage and face pain and a 2-week history of frontal headaches and fatigue. Her mother complains that her daughter has an "attitude" and has not been respectful or seemed to care about anything for the past 2 weeks. The daughter awoke this morning with a headache and vomited. On physical examination, the adolescent is afebrile and has normal vital signs. She responds slowly to questions and is not oriented to the date. She complains of pain to palpation of her cheeks and forehead. She has no nuchal rigidity and no focal weakness. The remainder of the physical examination findings are normal.

Of the following, the BEST initial diagnostic procedure is

A. computed tomography scan of the head with intravenous contrast

B. emergent electroencephalography to rule out nonconvulsive status epilepticus

C. lumbar puncture to rule out meningitis

D. nasal swab for bacterial culture

E. urine drug screen for barbiturates, amphetamines, and cocaine

Page 17: AAP PREP 2009

2009 PREP SA on CD-ROM

page 17

Critique: 6 Preferred Response: A

The subacute onset of mental status changes described for the adolescent in the vignette warrants an emergency evaluation. In most cases, neuroimaging is indicated, along with appropriate laboratory testing. The relatively nonspecific pain and what her mother perceives as common emotional problems (apathy in a teenager) probably represent early frontal lobe symptoms. The headache on awakening and vomiting are concerning for increased intracranial pressure (ICP). Confusion and psychomotor retardation on the mental status examination indicate involvement of the central nervous system. A focal, ischemic, ictal, infectious/inflammatory, or toxic/metabolic process must be identified urgently. A brain abscess is suggested by the prominent facial pain in this setting; the sinuses are a common source of brain abscesses (Item C6). Brain abscesses often present only with nonspecific pain and not with fever. Head computed tomography (CT) scan is preferred for this patient because the constellation of pain, confusion, and morning vomiting makes a focal intracranial mass a possibility. Increased ICP is associated with morning vomiting because ICP is highest in the morning. Contrast is recommended because of the insidious onset, which could indicate either a neoplasm or infectious process. Intravenous contrast is not needed for all neuroimaging procedures. However, it increases the diagnostic yield of imaging studies where either neoplasm or infection is suspected because both typically involve some degradation of the blood-brain barrier or hypervascularity, resulting in contrast enhancement at the site of the lesion. Magnetic resonance imaging (MRI) with contrast also is a good choice. The advantage of MRI is higher spatial and soft-tissue resolution. Disadvantages of MRI compared with CT include: 1) less availability for emergency department studies; 2) need for pharmacologic sedation in agitated patients because sedation affects mental status, thereby masking disease-related mental status; 3) longer time in the scanner, which could delay treatment decisions; and 4) cost. Thus, in most cases, a head CT scan with contrast is preferred as the initial study in the emergency department. Electroencephalography (EEG) is an important test for assessment of a patient who has encephalopathy of unclear cause to rule out nonconvulsive status epilepticus (NCSE), particularly if the patient is known to have epilepsy. If an EEG cannot be obtained rapidly, intravenous administration of 0.1 mg/kg lorazepam can treat NCSE immediately, although this would not clear confusion about other causes. For this adolescent, the facial pain makes the diagnosis of NCSE less likely than a brain abscess. A lumbar puncture may be needed to rule out meningitis or encephalitis, but the pain and morning vomiting more strongly suggest the possibility of an intracranial mass. Lumbar puncture prior to head CT is not advised in this case because it could reduce pressure below the foramen magnum and result in herniation from the supratentorial mass. Toxicology screening and nasal swabs are reasonable but not the preferred initial diagnostic tests because they will not affect emergency management of the increased intracranial pressure.

References:

Goodkin HP, Harper MB, Pomeroy SL. Intracerebral abscess in children: historical trends at Children's Hospital Boston. Pediatrics. 2004;113:1765-1770. Available at: http://pediatrics.aappublications.org/cgi/content/full/113/6/1765

Haslam RHA. Brain abscess. In: Kliegman RM, Behrman RE, Jenson HB, Stanton BF, eds. Nelson Textbook of Pediatrics. 18th ed. Philadelphia, Pa: Saunders Elsevier; 2007:2524-2525

Page 18: AAP PREP 2009

2009 PREP SA on CD-ROM

page 18

Kan L, Nagelberg J, Maytal J. Headaches in a pediatric emergency department: etiology, imaging, and treatment. Headache. 2000;40:25-29. Abstract available at: http://www.ncbi.nlm.nih.gov/pubmed/10759899

Page 19: AAP PREP 2009

2009 PREP SA on CD-ROM

page 19

Critique: 6

Brain abscess: Contrast-enhanced computed tomography scan of the head reveals a rim-enhancing lesion extending to an area of bony destruction in an opacified left frontal sinus. Vasogenic edema surrounding the abscess creates a mass effect and shift of the midline to the right. (Courtesy of D. Krowchuk)

Page 20: AAP PREP 2009

2009 PREP SA on CD-ROM

page 20

Question: 7

A 5-year-old girl who is new to your practice presents to the clinic for a prekindergarten physical examination. Her primary caretaker, the maternal grandmother, reports that the child’s mother used multiple street drugs throughout her pregnancy as well as medications prescribed for seizure and bipolar disorders. The grandmother is concerned that this child’s speech development is delayed. On physical examination, you note that the girl has wide-spaced eyes, a short nose, and midface hypoplasia.

Of the following, the substance that is MOST likely to be associated with this child’s dysmorphic features is

A. lithium

B. lysergic acid diethylamide (LSD)

C. marijuana

D. methamphetamine

E. phenobarbital

Page 21: AAP PREP 2009

2009 PREP SA on CD-ROM

page 21

Critique: 7 Preferred Response: E

The features described for the child in the vignette are most consistent with fetal anticonvulsant syndrome, which can occur following exposure to numerous medications, including phenytoin, carbamazepine, valproate, and phenobarbital. Multiple authors have observed a 10% to 20% incidence of birth defects in infants exposed to phenobarbital in utero. Anomalies include midface hypoplasia, ocular hypertelorism (Item C7A), nail hypoplasia (Item C7B), cleft lip+/-cleft palate, and heart defects as well as developmental delay and pre- and postnatal growth failure. Phenobarbital-exposed newborns may exhibit a withdrawal syndrome that is evidenced by tremulousness and increased activity. Of interest, such infants are likely to have lower serum bilirubin concentrations than nonexposed neonates. Prenatal exposure to lithium is associated with an increased risk of cardiac malformations (eg, Ebstein anomaly) in the fetus. If the mother takes lithium near term, the exposed neonate may have cyanosis, hypotonia, abnormalities of cardiac rhythm, goiter, hypothyroidism, and nephrogenic diabetes insipidus. Lithium exposure is not associated with dysmorphic features or developmental delays. Despite popular belief, lysergic acid diethylamide (LSD) generally is not associated with birth defects or withdrawal symptoms in prenatally exposed infants. Although there are isolated case reports of birth defects in exposed neonates, an increased risk for anomalies is not borne out by epidemiologic studies. It is important to note, however, that LSD users often abuse other substances, underscoring the importance of taking an in-depth drug/substance abuse history in pregnant women. Marijuana use during pregnancy is not known to be associated with an increased incidence of birth defects, dysmorphic features, or developmental delay in exposed offspring, although further study is needed in this regard. Some investigations have shown reduced fetal growth in exposed pregnancies, but this is not confirmed. Prenatally exposed newborns may have tremulousness, increased irritability, and abnormal visual response to light stimulus. Although there does not appear to be an increase in congenital anomalies associated with methamphetamine use during pregnancy, further study is necessary. There are reports of decreased birthweight in exposed neonates. A neonatal withdrawal syndrome consisting of abnormal sleep patterns, tremulousness, poor feeding, and increased tone has been observed commonly. Once again, it is important to consider polydrug abuse in these instances. As with all teratogens, the timing of exposure is critical, with the most vulnerable period of embryonic development occurring between 18 and 60 days after conception, during organogenesis. Dosage of the offending agent also is important, as are route of administration, modifying environmental factors, and genetic background of the mother and fetus.

References:

Gallagher RC, Kingham K, Hoyme HE. Fetal anticonvulsant syndrome. In: Cassidy SB, Allanson JE, eds. Management of Genetic Syndromes. 2nd ed. Hoboken, NJ: Wiley-Liss; 2005:239-250

Phenobarbital, lithium, LSD, marijuana, methamphetamine. Reprotox. Available for subscription at: http://www.reprotox.org

Phenobarbital, lithium, LSD, marijuana, methamphetamine. Teris. Available for subscription at: http://depts.washington.edu/terisweb/teris

Page 22: AAP PREP 2009

2009 PREP SA on CD-ROM

page 22

Critique: 7

Wide-spaced eyes and a short upturned nose are facial features of the fetal anticonvulsant syndrome. This child was exposed to phenytoin. (Courtesy of M. Rimsza)

Page 23: AAP PREP 2009

2009 PREP SA on CD-ROM

page 23

Critique: 7

Nail hypoplasia is observed in a child who has fetal anticonvulsant syndrome. (Courtesy of the Media Lab at Doernbecher)

Page 24: AAP PREP 2009

2009 PREP SA on CD-ROM

page 24

Question: 8

A 15-year-old girl comes to the urgent care clinic complaining of lower abdominal pain for 48 hours. She is nauseated but has had no fever, vomiting, or diarrhea. She is afebrile and denies abdominal trauma. She localizes the pain to the left lower quadrant and describes it as intermittent, stabbing pain episodes separated by intervals of more continuous dull pain. She has never been sexually active. Her last menstrual period was 1 week ago. She has had no vaginal discharge or itching. On physical examination, she has left lower quadrant guarding and rebound tenderness. Her pelvic examination shows no vaginal discharge or uterine tenderness, although there is an exquisitely tender mass in the left adnexal area.

Of the following, the MOST likely diagnosis is

A. appendicitis

B. endometritis

C. ovarian torsion

D. sacroiliitis

E. splenic rupture

Page 25: AAP PREP 2009

2009 PREP SA on CD-ROM

page 25

Critique: 8 Preferred Response: C

Torsion of any adnexal structure, including the ovary, can result in an acute pelvic mass (Item C8). The most common causes of adnexal torsion in young women are cysts and neoplasms, with about 6% of torsions in one series occurring in the setting of normal-appearing ovaries. Histopathology was benign in more than 90% of cases in this series. Torsion occurs when masses cause the ovary to swing on its vascular pedicle, and larger masses generally are associated with a greater potential for torsion until the size of the mass impedes movement. The onset of pain associated with torsion can be abrupt, sharp, and very severe. However, with intermittent or partial torsion, intense periods of intermittent pain may be separated by generalized aching during detorsion, as described for the teenager in the vignette. Pain can occur for several days or weeks prior to a complete torsion. Nausea or vomiting can accompany severe pain. If adnexal torsion is diagnosed, prompt intervention (untwisting the adnexa usually via laparoscopy) is indicated to preserve ovarian function. Appendicitis is in the differential diagnosis for lower abdominal pain, but is less likely when pain occurs in the left lower quadrant. Splenic rupture is a surgical emergency, like ovarian torsion, but the finding of an adnexal mass on examination is unlikely. Endometritis is not very likely in a young woman who is not sexually active and more commonly presents with midline uterine pain. An orthopedic cause for this patient's pain (eg, sacroiliitis) is unlikely, but it remains in the differential diagnosis of pelvic pain. Sacroiliitis usually presents with low back or hip pain.

References:

Adams Hillard PJ. Pelvic masses. In: Neinstein LS, eds. Adolescent Health Care: A Practical Guide. 5th ed. Philadelphia, Pa: Lippincott Williams & Wilkins; 2008:706-713

Growdon WB, Laufer MR. Ovarian torsion. UpToDate Online 15.3. 2008. Available for subscription at: http://www.utdol.com/utd/content/topic.do?topicKey=gyn_surg/5273

Laufer MR, Goldstein DP. Gynecologic pain: dysmenorrhea, acute and chronic pelvic pain, endometriosis, and premenstrual syndrome. In: Emans SJH, Laufer MR, Goldstein DP, eds. Pediatric and Adolescent Gynecology. 5th ed. Philadelphia, Pa: Lippincott, Williams & Wilkins; 2005:417-476

Varras M, Tsikini A, Polyzos D, Samara Ch, Hadjopoulos G, Akrivis Ch. Uterine adnexal torsion: pathologic and gray-scale ultrasonographic findings. Clin Exp Obstet Gynecol. 2004;31:34-38. Abstract available at: http://www.ncbi.nlm.nih.gov/pubmed/14998184

Page 26: AAP PREP 2009

2009 PREP SA on CD-ROM

page 26

Critique: 8

Ovarian torsion: Color flow Doppler ultrasonography of the right ovary shows abundant flow in the adjacent pelvic tissue (blue, red, and orange color seen inferiorly) but none in the ovary (the area within the dashed line).

Page 27: AAP PREP 2009

2009 PREP SA on CD-ROM

page 27

Question: 9

A 2-year-old boy who has chronic renal failure is brought to the emergency department for evaluation of nausea, fatigue, and muscle weakness. On physical examination, the boy has a heart rate of 140 beats/min, decreased perfusion, and palpable pulses. You obtain electrocardiography (Item Q9). Electrolyte measurements include a potassium concentration of 7.5 mEq/L (7.5 mmol/L) and a glucose value of 72.0 mg/dL (4.0 mmol/L).

Of the following, the MOST appropriate initial treatment is administration of

A. calcium chloride

B. insulin

C. normal saline 20 mL/kg

D. sodium polystyrene sulfonate

E. verapamil

Page 28: AAP PREP 2009

2009 PREP SA on CD-ROM

page 28

Question: 9

In hyperkalemia, electrocardiography demonstrates peaked T waves. (Courtesy of A. Friedman)

Page 29: AAP PREP 2009

2009 PREP SA on CD-ROM

page 29

Critique: 9 Preferred Response: A

Hyperkalemia can be asymptomatic or produce symptoms such as nausea, fatigue, and muscle weakness, as reported for the boy in the vignette. The most concerning effect of hyperkalemia is on cardiac membrane polarization. Electrocardiographic changes typically consist of peaked T waves and progress to widening of the QRS complexes with dampening of P waves as the hyperkalemia increases. Without treatment, the patient eventually develops ventricular fibrillation or asystole. The electrocardiographic tracing of the boy described in the vignette demonstrates peaked T waves (Item C9), and he has decreased perfusion. The immediate priority is to stabilize his cardiac membrane potential and decrease his risk of arrhythmias, which is accomplished best by administration of intravenous calcium chloride, an agent that has a rapid onset of action. Insulin causes potassium to shift to the intracellular spaces, thereby decreasing serum concentrations, but it always should be administered in conjunction with glucose to avoid hypoglycemia. Sodium bicarbonate administered intravenously also produces intracellular shifts of potassium and may be useful in patients who have metabolic acidosis. Treatments to enhance the elimination of potassium often are indicated in the treatment of hyperkalemia, but they have a slower onset of action and, therefore, should not be the first line of treatment in life-threatening hyperkalemia. Sodium polystyrene sulfonate, which can be administered orally or rectally, exchanges sodium for potassium, which subsequently is eliminated from the body. Loop diuretics also can enhance potassium elimination. Dialysis may be indicated when ongoing elevated potassium concentrations are expected, such as in renal failure, or with very high acute concentrations, as seen with tumor lysis syndrome or rhabdomyolysis. Administration of verapamil is not indicated in the treatment of hyperkalemia; it has been reported to be associated with the development of complete heart block in this setting. Normal saline has a minimal effect on hyperkalemia.

References:

Greenbaum LA. Electrolyte and acid-base disorders: potassium. In: Kliegman RM, Behrman RE, Jenson HB, Stanton BF, eds. Nelson Textbook of Pediatrics. 18th ed. Philadelphia, Pa: Saunders Elsevier; 2007:279-284

Hauser GJ, Kulick AF. Electrolyte disorders in the pediatric intensive care unit. In: Wheeler DS, Wong HR, Shanley TP, eds. Pediatric Critical Care Medicine: Basic Science and Clinical Evidence. New York, NY: Springer-Verlag; 2007:1156-1175

Page 30: AAP PREP 2009

2009 PREP SA on CD-ROM

page 30

Critique: 9

In hyperkalemia, electrocardiography demonstrates peaked T waves. (Courtesy of A. Friedman)

Page 31: AAP PREP 2009

2009 PREP SA on CD-ROM

page 31

Question: 10

A 13-year-old girl who has just moved to the United States from Brazil comes to your office because her mother is worried that she is not "developing yet." On physical examination, her height is 50 inches, and she has a triangular face, a low hairline, high-arched palate, and a shield-shaped chest (Item Q10). Breast tissue is not visible or palpable, but there is Sexual Maturity Rating 3 pubic hair. You obtain bone age radiography and a karyotype and measure serum luteinizing hormone and follicle-stimulating hormone.

Of the following, the MOST appropriate additional laboratory measurement is

A. adrenocorticotropic hormone

B. prolactin

C. 17-hydroxyprogesterone

D. testosterone

E. thyroid-stimulating hormone

Page 32: AAP PREP 2009

2009 PREP SA on CD-ROM

page 32

Question: 10

Shield-shaped chest and lack of breast development, as described for the girl in the vignette. (Courtesy of M. Rimsza)

Page 33: AAP PREP 2009

2009 PREP SA on CD-ROM

page 33

Critique: 10 Preferred Response: E

The clinical findings described for the girl in the vignette are characteristic of Turner syndrome (gonadal dysgenesis) associated with an abnormality of one X chromosome. Girls who have this disorder usually are short (mean adult height, approximately 55 inches without growth hormone treatment); have poorly developed ovaries; and often have dysmorphisms, including a triangular facies, low hairline, high-arched palate, hypoplastic nipples, and an increased carrying angle. They may have left heart disorders such as coarctation of the aorta as well as horseshoe kidney or other renal malformations. Initial screening studies to diagnose Turner syndrome include a karyotype and measurement of luteinizing hormone (LH) and follicle-stimulating hormone (FSH). Most girls who have Turner syndrome do not initiate normal puberty. Concentrations of LH and FSH rise as they reach pubertal age range because they have ovarian failure. Although concentrations of estradiol and other estrogens are low, clinical estradiol assays are not designed to provide accurate values in the low-normal range expected in early puberty. Therefore, physical findings such as breast development are a better marker of estrogen effect than measurements of estrogen. Adolescents who have Turner syndrome are at higher risk of developing chronic lymphocytic thyroiditis and hypothyroidism than the general population. Approximately 20% of affected adolescent girls have antibody-positive autoimmune chronic lymphocytic thyroiditis, and 5% to 10% develop overt hypothyroidism. Accordingly, measurement of thyroid-stimulating hormone is an appropriate laboratory test for patients such as the girl described in the vignette. An elevated value indicates primary hypothyroidism and the need for confirmatory assessment of free thyroxine and antithyroid antibodies (thyroperoxidase, antimicrosomal, or antithyroglobulin). Abnormalities of the hypothalamic-pituitary-adrenal axis are unusual in patients who have Turner syndrome. Therefore, measurement of adrenocorticotropic hormone is not useful. Measurement of prolactin would be useful if the girl had a pituitary or hypothalamic problem, but her clinical findings strongly point to Turner syndrome. A 17-hydroxyprogesterone value would be elevated in the presence of an adrenal biosynthetic defect leading to the development of the most common form of congenital adrenal hyperplasia (cyp21 or 21-hydroxylase deficiency) as well as some of the less common disorders of adrenal biosynthesis. Measuring testosterone would be reasonable if there were evidence of inappropriate masculinization, such as clitoromegaly and a growth spurt. Some girls who have Turner syndrome have functioning Y chromosomal DNA and could have androgenization, but this is unusual. The presence of Y chromosomal DNA does increase the risk of gonadal malignancy, and girls who have significant Y chromosomal DNA on testing often require prophylactic gonadectomy.

References:

Chiovato L, Larizza D, Bendinelli G, et al. Autoimmune hypothyroidism and hyperthyroidism in patients with Turner's syndrome. Eur J Endocrinol. 1996;134:568-575. Abstract available at: http://www.ncbi.nlm.nih.gov/pubmed/8664977

Doswell BH, Visootsak J, Brady AN, Graham JM Jr. Turner syndrome: an update and review for the primary pediatrician. Clin Pediatr. 2006;45:301-313. Abstract available at: http://www.ncbi.nlm.nih.gov/pubmed/16703153

Frias JL, Davenport ML, Committee on Genetics and Section on Endocrinology. Health supervision for children with Turner syndrome. Pediatrics. 2003;111:692-702. Available at: http://pediatrics.aappublications.org/cgi/content/full/111/3/692

Matura LA, Ho VB, Rosing DR, Bondy CA. Aortic dilatation and dissection in Turner syndrome.

Page 34: AAP PREP 2009

2009 PREP SA on CD-ROM

page 34

Circulation. 2007;116:1663-1670. Abstract available at: http://www.ncbi.nlm.nih.gov/pubmed/17875973

Sybert VP, McCauley E. Turner's syndrome. N Engl J Med. 2004;351:1227-1238. Extract available at: http://content.nejm.org/cgi/content/extract/351/12/1227

Turner Syndrome Society Website. Available at: http://www.turnersyndrome.org

Page 35: AAP PREP 2009

2009 PREP SA on CD-ROM

page 35

Question: 11

A 2-month-old infant has lost the vision in both of his eyes due to bilateral retinoblastoma. His distressed parents ask how the infant’s blindness will affect his behavior and development.

Of the following, the child MOST likely will

A. begin saying single words at 16 to 20 months

B. begin walking between 18 and 22 months

C. display behaviors of an autism spectrum disorder

D. have a language-based learning disorder

E. have significant cognitive impairments

Page 36: AAP PREP 2009

2009 PREP SA on CD-ROM

page 36

Critique: 11 Preferred Response: B

Legal blindness is defined as central visual acuity with corrective lenses of 20/200 or less in the strongest eye or a limited visual field that extends to an angle of 20 degrees. Congenital blindness occurs in 30 per 100,000 births. More than 50% of children who have visual impairment also have developmental disabilities, such as cognitive-adaptive disability, seizures, hearing impairments, and learning disorders. In many of these cases, the disabilities result from central nervous system pathology. Postnatal blindness, which accounts for approximately 8% to 11% of all childhood blindness, can be caused by infections, trauma, or tumors. Retinoblastoma is the most common primary malignant intraocular tumor of childhood. The initial finding in most cases is a white pupillary reflex (leukokoria) (Item C11). Advanced tumors may be treated with enucleation. Children who have congenital or acquired (eg, due to retinoblastoma) blindness without associated neurologic abnormalities should not be at increased risk for motor or cognitive impairment. They are not at increased risk for language-based learning disabilities or autism spectrum disorders. However, children who have significant visual impairment may begin to walk at an older age (18 to 22 months) than sighted children due to different exposure to motor exploration. They typically develop language skills at the same time (12 months) as sighted children. Children who have visual impairments should be provided with much physical contact that includes hugging and comforting. They should be encouraged to partake in self-help skills and exploration of their environment.

References:

Davidson PW, Burns CM. Visual impairment and blindness. In: Levine MD, Carey WB, Crocker AC, eds. Developmental- Behavioral Pediatrics. 3rd ed. Philadelphia, Pa: WB Saunders Company; 1999:571-578

Msall ME. Visual impairment. In: Parker S, Zukerman B, Augustyn M. Developmental and Behavioral Pediatrics: A Handbook for Primary Care. 2nd ed. Philadelphia, Pa: Lippincott Williams & Wilkins; 2005:366-369

Olitsky SE, Hug D, Smith LP. Disorders of vision. In: Kleigman RM, Behrman RE, Jenson HB, Stanton BF, eds. Textbook of Pediatrics. 18th ed. Philadelphia, Pa: Saunders Elsevier; 2007:2573-2576

Page 37: AAP PREP 2009

2009 PREP SA on CD-ROM

page 37

Critique: 11

Leukokoria may be observed in patients who have retinoblastoma. (Courtesy of R.G. Weaver, Jr)

Page 38: AAP PREP 2009

2009 PREP SA on CD-ROM

page 38

Question: 12

A 15-year-old boy presents to the clinic because of a persistent cough. According to his mother, his cough has been present for approximately 2 weeks, but it seems to be getting worse. He does not cough all the time, but the coughing episodes tend to come in bursts. This morning she became very worried because he passed out during a coughing spell. Physical examination reveals a healthy-appearing male in no apparent distress. He is afebrile, and his vital signs are normal. He has petechiae on his face but no other skin lesions. His lungs are clear.

Of the following, the MOST appropriate antimicrobial agent to prescribe for this patient is

A. azithromycin

B. clarithromycin

C. doxycycline

D. erythromycin

E. trimethoprim-sulfamethoxazole

Page 39: AAP PREP 2009

2009 PREP SA on CD-ROM

page 39

Critique: 12 Preferred Response: A

Any patient who has episodic coughing episodes that conclude in syncope or vomiting, such as the boy described in the vignette, should be considered to have pertussis. Other complications from pertussis described in adolescents include urinary incontinence, sleep interruption, rib fractures, and pneumonia. Despite universal immunization of children against pertussis, a marked increase in disease incidence has been demonstrated among adolescents of 11 to 18 years of age. In an attempt to address this problem, the American Academy of Pediatrics recommends that adolescents in this age group receive a single dose of tetanus toxoid, reduced diphtheria toxoid, and acellular pertussis (Tdap) for booster immunization. For those who received only a tetanus toxoid and reduced diphtheria toxoid (Td) booster, a booster with Tdap is suggested if the interval since the Td administration is at least 2 years or if the adolescent is living in a setting of increased disease, has a risk of a complicated course if he or she acquires the disease, or possibly can transmit infection to a vulnerable contact. Treatment of pertussis with antimicrobial agents once the cough has started does not affect the course of the illness but is recommended to limit the spread of disease to others. In the past, the treatment of choice was erythromycin, but recent data have demonstrated that the effectiveness of other macrolides such as azithromycin or clarithromycin is similar to that of erythromycin, and they have fewer adverse effects. The most common complaints of patients who take erythromycin preparations are gastrointestinal irritation, including epigastric distress, abdominal cramping, nausea, vomiting, and diarrhea. The occurrence of these adverse effects can result in poor adherence to treatment regimens. In addition, erythromycin cannot be used in children younger than 1 month of age due to its association with the development of hypertrophic pyloric stenosis. Azithromycin and clarithromycin attain higher tissue concentrations than erythromycin, have longer half-lives, and can be administered less often and for a shorter total duration. With the advent of these newer macrolides, erythromycin rarely is indicated as a first-line agent for any illness. Azithromycin has become the agent of choice for treatment of pertussis because of the ease of administration (once daily for 5 days) and the fact that it does not inhibit cytochrome P-450, as erythromycin and clarithromycin do. Therefore, it does not interact with other medications that are metabolized by this system (eg, digoxin, carbamazepine). Trimethoprim-sulfamethoxazole also is effective against pertussis and is considered an alternative for patients who cannot tolerate a macrolide or have a macrolide-resistant isolate if they are older than 2 months of age. Doxycycline is not recommended for the treatment of pertussis.

References:

American Academy of Pediatrics. Pertussis (whooping cough). In: Pickering LK, Baker CJ, Long SS, McMillan JA, eds. Red Book: 2006 Report of the Committee on Infectious Diseases. 27th ed. Elk Grove Village, Ill: American Academy of Pediatrics; 2006:498-520

Committee on Infectious Diseases. Prevention of pertussis among adolescents: recommendations for use of tetanus toxoid, reduced diphtheria toxoid, and acellular pertussis (Tdap) vaccine. Pediatrics. 2006;117:965-978. Available at: http://pediatrics.aappublications.org/cgi/content/full/117/3/965

Tiwari T, Murphy TV, Moran J. Recommended antimicrobial agents for the treatment and postexposure prophylaxis of pertussis: 2005 CDC guidelines. MMWR Recomm Rep. 2005;54(RR14):1-16. Available at: http://www.cdc.gov/mmwr/preview/mmwrhtml/rr5414a1.htm

Page 40: AAP PREP 2009

2009 PREP SA on CD-ROM

page 40

Question: 13

A 15-year-old male presents for evaluation of a progressively enlarging lesion on his left forearm that began 5 days ago. He explains that the lesion initially looked like a "spider bite" with a blister, but over the last several days, a black scab has developed in the center of the lesion, and there is a large area of redness around the scab. The lesion has been pruritic but not painful. Except for low-grade fevers for the last 2 days, he has had no other systemic symptoms. He returned 1 week ago from a school trip to Morocco, where he visited a leather tannery, went shopping in the large outdoor marketplace, visited some historic sites, and took a camel ride in the desert. He states that the students stayed in a hostel in Morocco, but there were no screens on the windows, and spiders, ants, and other insects were visible in the rooms. On physical examination, the boy is afebrile, and his left forearm is edematous, with a 3x3-cm black eschar surrounded by a 5-cm area of erythema and induration (Item Q13). The lesion is not tender to palpation, and there is no drainage. There are several 1.5-cm tender lymph nodes in his left axilla. Findings on the remainder of his examination are within normal limits.

Of the following, the MOST likely cause of this patient’s lesion is

A. Bacillus anthracis

B. Francisella tularensis

C. Loxosceles laeta

D. methicillin-resistant Staphylococcus aureus

E. Yersinia pestis

Page 41: AAP PREP 2009

2009 PREP SA on CD-ROM

page 41

Question: 13

Eschar, as desribed for the patient in the vignette. (Courtesy of the Centers for Disease Control and Prevention, Public Health Image Library, James H. Steele)

Page 42: AAP PREP 2009

2009 PREP SA on CD-ROM

page 42

Critique: 13 Preferred Response: A

Anthrax is a zoonotic disease caused by Bacillus anthracis, which is a gram-positive, encapsulated, spore-forming rod that occurs in many areas of the world. B anthracis spores can remain viable in the soil for decades (Item C13A), representing a major reservoir of infection for herbivorous livestock through ingestion. Human infection occurs through contact with infected animals or contaminated animal products, including carcasses, hides, hair, wool, meat, bone meal, and other contaminated foodstuffs. Depending on the route of infection, anthrax disease may manifest in three different forms: cutaneous, inhalational, and gastrointestinal. Approximately 95% of all human anthrax cases are cutaneous anthrax. The incubation period of cutaneous anthrax is 1 to 12 days. The initial skin lesion is a pruritic papule that resembles an insect or spider bite, as described for the boy in the vignette. The papule progresses to the development of a central vesicular or bullous lesion that becomes necrotic and hemorrhagic and forms a central black painless eschar, which is the classic lesion of anthrax (Item C13B). There is marked surrounding edema, swelling, induration, and erythema of the involved area but no associated tenderness. Regional lymphadenopathy, fever, malaise, and headache also may be present. In most cases, the eschar falls off in 1 to 2 weeks, and total resolution occurs in 6 weeks. Infections caused by methicillin-resistant Staphylococcus aureus (MRSA), the bite of Loxosceles laeta, Francisella tularensis, and Yersinia pestis are associated with pain, rapid progression, and in most cases, systemic symptoms. Skin and soft-tissue infections due to MRSA have been described as resembling a "spider bite" that is very tender, with progressive worsening of the erythema, warmth, and tenderness (Item C13C). It is not associated with the development of a bullous, necrotic lesion with eschar. The bite of Loxosceles laeta (the brown recluse spider) may resemble the cutaneous lesion of anthrax, but it is very painful (Item C13D). Also, symptoms develop shortly after the bite and progress rapidly within 24 hours. Francisella tularensis is the agent that causes tularemia. Signs and symptoms develop within 3 to 5 days of exposure and include the abrupt onset of fever, chills, headache, malaise, and fatigue as well as the development of a progressively enlarging, tender, localized lymphadenopathy and a red, painful papule (Item C13E) in a region draining into the involved lymph nodes. The signs and symptoms of Yersinia pestis (plague) develop abruptly and include fever; chills; weakness; headache; and extremely tender, rapidly enlarging swelling of the lymph nodes of the groin, axilla, or neck.

References:

American Academy of Pediatrics. Anthrax. In: Pickering LK, Baker CJ, Long SS, McMillan JA, eds. Red Book: 2006 Report of the Committee on Infectious Diseases. 27th ed. Elk Grove Village, Ill: American Academy of Pediatrics; 2006:208-211

Butler T, Dennis DT. Yersinia species, including plague. In: Mandell GL, Bennett JE, Dolin R, eds. Mandell, Douglas and Bennett's Principles and Practice of Infectious Diseases. 6th ed. New York, NY: Elsevier Churchill Livingstone; 2005:2691-2700

Inglesby TV, Henderson DA, Bartlett JG, et al. Anthrax as a biological weapon: medical and public health management. JAMA. 1999;281:1735-1745. Available at: http://jama.ama-assn.org/cgi/content/full/281/18/1735

King MD, Humphrey BJ, Wang YF, Kourbatova EV, Ray SM, Blumberg HM. Emergence of community-acquired methicillin-resistant Staphylococcus aureus USA 300 clone as the predominant cause of skin and soft-tissue infections. Ann Intern Med. 2006;144:309-317. Abstract available at: http://www.ncbi.nlm.nih.gov/pubmed/16520471

Page 43: AAP PREP 2009

2009 PREP SA on CD-ROM

page 43

Penn RL. Francisella tularensis (tularemia). In: Mandell GL, Bennett JE, Dolin R, eds. Mandell, Douglas and Bennett's Principles and Practice of Infectious Diseases. 6th ed. New York, NY: Elsevier Churchill Livingstone; 2005:2674-2685

Swanson DL, Vetter RS. Bites of brown recluse spiders and suspected necrotic arachnidism. N Engl J Med. 2005;352:700-707. Extract available at: http://content.nejm.org/cgi/content/extract/352/7/700

Page 44: AAP PREP 2009

2009 PREP SA on CD-ROM

page 44

Critique: 13

Scanning electron micrograph at magnification 6408X demonstrates spores of Bacillus anthracis. The spores may surive in soil for decades. (Courtesy of the Centers for Disease Control and Prevention, Public Health Image Library, Laura Rose)

Page 45: AAP PREP 2009

2009 PREP SA on CD-ROM

page 45

Critique: 13

The classic lesion of cutaneous anthrax is a black eschar with surrounding swelling, erythema, and induration. The affected area is not painful. (Courtesy of the Centers for Disease Control and Prevention, Public Health Image Library)

Page 46: AAP PREP 2009

2009 PREP SA on CD-ROM

page 46

Critique: 13

Initial lesions of staphylococcal skin infection may be small tender papules that often are attributed to spider bites (yellow arrows). Frequently, lesions enlarge and develop an overlying pustule (black arrow). (Courtesy of Bernard Cohen, MD, DermAtlas; www.dermatlas.org)

Page 47: AAP PREP 2009

2009 PREP SA on CD-ROM

page 47

Critique: 13

An eschar with surrounding painful erythema and swelling is observed following the bite of a brown recluse spider. (Courtesy of M. Smith)

Page 48: AAP PREP 2009

2009 PREP SA on CD-ROM

page 48

Critique: 13

An ulcerated papule may appear at the site of inoculation of Francisella tularensis. (Courtesy of the Centers for Disease Control and Prevention, Public Health Image Library, Dr. Thomas F. Sellers)

Page 49: AAP PREP 2009

2009 PREP SA on CD-ROM

page 49

Question: 14

A 3-month-old infant who has a history of renal dysplasia associated with obstructive uropathy has marked polyuria. He is breastfeeding and receiving supplemental cow milk-based formula. In an effort to reduce the high urine output, you consider reducing the renal solute load by changing feedings from the milk-based formula currently being used.

Of the following, the MOST appropriate change is to

A. a hydrolyzed formula containing medium-chain triglycerides

B. a more concentrated (24-kcal) milk-based formula

C. human milk exclusively

D. soy milk-based formula

E. whole cow milk

Page 50: AAP PREP 2009

2009 PREP SA on CD-ROM

page 50

Critique: 14 Preferred Response: C

The infant described in the vignette has polyuria caused by a urinary concentrating defect. The concentrating defect is the result of tubular damage due to the obstructive uropathy. The inability to concentrate the urine causes the kidneys to create an "excessive" volume of urine to excrete the solute load presented to them. One strategy to reduce polyuria is to reduce the solute burden placed on the kidneys. Potential renal solute load is affected by intake of protein, sodium, potassium, chloride, and phosphorus. The protein and phosphorus content are the most important variables when comparing infant feeding regimens. Human milk possesses a lower potential renal solute load than cow milk or cow milk-based formulas. Accordingly, the most appropriate change in feeding for the infant in the vignette is to recommend that the mother stop cow milk formula supplementation and exclusively breastfeed. If human milk is not available, a "low-solute" cow milk-based formula can be used. A low calcium-phosphorus formula has the next lowest potential renal solute load compared with human milk. Cow milk, soy milk-based formula, hydrolyzed formula with medium-chain triglycerides, and 24-kcal milk-based formula all have greater renal solute loads than human milk. Renal solute load should also be considered in nephrogenic diabetes insipidus.

References:

Fiorino KN, Cox J. Nutrition and growth. In: Robertson J, Shilkofski N, eds. Harriet Lane Handbook: A Manual for Pediatric House Officers. 17th ed. Philadelphia, Pa: Elsevier Mosby; 2005:525-608

Hall RT, Carroll RE. Infant feeding. Pediatr Rev. 2000;21:191-200. Available at: http://pedsinreview.aappublications.org/cgi/content/full/21/6/191

Linshaw MA. Congenital nephrogenic diabetes insipidus. Pediatr Rev. 2007;28:372-380. Available at: http://pedsinreview.aappublications.org/cgi/content/full/28/10/372

Ziegler EE, Fomon SJ. Potential renal solute load of infant formulas. J Nutr. 1989;119 (12 suppl):1785-1788. Available at: http://jn.nutrition.org/cgi/reprint/119/12_Suppl/1785

Page 51: AAP PREP 2009

2009 PREP SA on CD-ROM

page 51

Question: 15

A mother brings in her 13-month-old daughter for evaluation because her girl developed a perioral rash and "hives" on two occasions last week. One episode occurred while eating yogurt and another happened immediately after eating a bagel with cream cheese. She states that her daughter has eaten other foods such as eggs and bread without problems but is breastfeeding and never has been given milk-based formulas or cow milk. The infant has been given rice milk, but she became fussy and seems to prefer breastfeeding. The mother is concerned that her daughter may be allergic to milk but would like to stop breastfeeding.

Of the following, the BEST advice is to recommend

A. a cow milk food challenge in the clinic

B. avoidance of milk, egg, soy, and wheat products

C. breastfeeding until the child is 3 years old

D. switching to an amino acid-based formula

E. switching to soy milk

Page 52: AAP PREP 2009

2009 PREP SA on CD-ROM

page 52

Critique: 15 Preferred Response: E

Milk protein allergy is an immunoglobulin (Ig) E-mediated food reaction that affects 2% to 3% of infants within the first postnatal year. Typical symptoms include urticaria, angioedema, atopic dermatitis, and anaphylaxis. With IgE-mediated reactions, the quantity of milk required to result in a reaction often is minimal (eg, milk touching the face, a taste of ice cream). Taking a detailed history about the specific food(s) involved, timing of the onset of symptoms, and type of symptoms is important to distinguish IgE-mediated reactions, as described for the child in the vignette, from other adverse milk reactions, such as milk protein enterocolitis and lactose intolerance. Once an IgE-mediated food allergy is suspected, the clinician should consider allergy skin testing or serum IgE testing for the suspected food. While awaiting results from either blood testing or allergy consultation for skin testing, the first reasonable action is to switch to a soy-based formula. Approximately 10% to 15% of infants and children who have IgE-mediated milk protein allergies may not tolerate soy formula, but this risk applies to infants younger than 6 months of age. For infants older than 6 months, the risk is closer to 5%. Nonetheless, the initial soy formula trial should be performed in the clinic. Other acceptable formula options in this scenario include an extensively hydrolyzed or an amino acid-based formula, although the unpleasant taste and significantly higher cost can be limiting for many families. Also, because almost all affected infants can be fed successfully with a soy or extensively hydrolyzed formula, switching initially to an amino acid-based formula is not required. Food challenges often are used to assess adverse food reactions, but they generally are reserved for foods that are unlikely allergens or if the clinical history is inconsistent or vague (eg, a patient who eats a particular food and does not always have a reaction). Food challenges may result in anaphylaxis and generally are avoided when the history and testing results support an IgE-mediated reaction. Infants who have a specific food allergy sometimes are placed incorrectly on restricted diets that avoid multiple foods. Without a specific history of other adverse food reactions, avoidance of other foods such as egg or wheat is not recommended. However, parents should be counseled that children can develop other food allergies and should monitor their children during ingestion of other common food allergens. Breastfeeding until age 3 years old is an option that is not preferred by the mother in the vignette. Although most IgE-mediated cow milk allergies resolve by 3 years of age, milk protein allergy can persist past 5 years of age in up to 20% of affected children.

References:

American Academy of Pediatrics Committee on Nutrition. Soy protein-based formulas: recommendations for use in infant feeding. Pediatrics. 1998;101:148-153. Available at: http://pediatrics.aappublications.org/cgi/content/full/101/1/148

Bhatia J, Greer F, and the Committee on Nutrition. The use of soy protein-based formulas in infant feeding. Pediatrics. 2008;121:1062-1068. Available at: http://pediatrics.aappublications.org/cgi/content/full/121/5/1062

Klemola T, Vanto T, Juntunen-Backman K, Kalimo K, Korpela R, Varjonen E. Allergy to soy formula and to extensively hydrolyzed whey formula in infants with cow's milk allergy: a prospective, randomized study with a follow-up to the age of 2 years. J Pediatr. 2002;140:219-224. Abstract available at: http://www.ncbi.nlm.nih.gov/ pubmed/11865274

Saarinen KM, Pelkonen AS, Mäkelä MJ, Savilahti E. Clinical course and prognosis of cow's milk allergy are dependent on milk-specific IgE status. J Allergy Clin Immunol. 2005;116:869-875. Abstract

Page 53: AAP PREP 2009

2009 PREP SA on CD-ROM

page 53

available at: http://www.ncbi.nlm.nih.gov/pubmed/16210063

Sampson HA, Leung DYM. Adverse reactions to foods. In: Kleigman RM, Behrman RE, Jenson HB, Stanton BF, eds. Nelson Textbook of Pediatrics. 18th ed. Philadelphia, Pa: Saunders Elsevier; 2007:986-989

Page 54: AAP PREP 2009

2009 PREP SA on CD-ROM

page 54

Question: 16

A 16-year-old girl is brought to the emergency department after being found unresponsive in her bedroom. Her parents report finding a note in which she wrote of "wanting to end the pain." In addition, they found several empty, unlabeled pill vials on her dresser. On physical examination, the girl is responsive only to painful stimuli. Her heart rate is 60 beats/min, respiratory rate is 16 breaths/min, blood pressure is 90/60 mm Hg, and oxygen saturation is 92%. Her pupils are 3 mm, equal in size, and sluggishly reactive. The remainder of findings on her physical examination are normal.

Of the following, the MOST important diagnostic test to obtain when evaluating this patient is a

A. carboxyhemoglobin concentration

B. complete blood count

C. serum acetaminophen concentration

D. serum ammonia concentration

E. serum osmolality

Page 55: AAP PREP 2009

2009 PREP SA on CD-ROM

page 55

Critique: 16 Preferred Response: C

Treatment of a patient who has ingested an unknown substance or substances, such as the one described in the vignette, should focus initially on stabilization of vital functions; treatment of the patient's symptoms; and identification of agents that are potentially fatal, have delayed clinical toxicity, or for which antidotal therapy is indicated. The evaluation should include a screening history to identify circumstances surrounding the event, potential environmental exposures, available medications or toxins, and previous medical or psychiatric history. The physical examination should focus on vital sign abnormalities, pupillary size and reactivity, skin findings, and mental status. These components are likely to yield the most useful clues to toxins that have identifiable symptom complexes (Item C16A). Laboratory testing should assess the patient's acid/base status, oxygenation and ventilation, glucose concentration, and anion gap. Qualitative urine drug testing may be obtained but has significant limitations, including the small number of drugs tested (typically drugs of abuse) and screening thresholds that may produce false-negative results. Therefore, the results of urine toxicologic screening rarely affect treatment plans. Quantitative drug concentrations should be measured based on the information gleaned from the initial history, physical examination, and screening laboratory testing, although acetaminophen and salicylate concentrations should be measured for every patient who has an unknown, mixed, or intentional ingestion. Acetaminophen is of particular concern because it causes few initial symptoms, is a common agent in adolescent and adult intentional ingestions, and may require antidotal therapy to prevent potential fatal liver damage. Abdominal radiographs may be considered to examine the patient for radio-opaque substances (Item C16B). A complete blood count is not likely to provide information leading to the identification of a toxin. Measurement of serum ammonia may be indicated if acetaminophen is identified as the ingested agent because toxicity can lead to hepatic damage. Carboxyhemoglobin should be measured in the patient whose history is suggestive of carbon monoxide exposure. Increased serum osmolality may provide indirect evidence of alcohol ingestion, although a quantitative serum test for alcohols is more useful in this setting.

References:

Erikson TB, Thompson TM, Lu JJ. The approach to the patient with an unknown overdose. Emerg Med Clin North Am. 2007;25:249-281. Abstract available at: http://www.ncbi.nlm.nih.gov/pubmed/17482020

Lavallee M, Olsson J Jr, Cheng TL. In brief: unknown poison. Pediatr Rev. 2004;25:370-371. Available at: http://pedsinreview.aappublications.org/cgi/content/full/25/10/370

McKay CA Jr. Can the laboratory help me? Toxicology laboratory testing in the possibly poisoned pediatric patient. Clin Pediatr Emerg Med. 2005;6:116-122

Valez LI, Shepherd JG, Goto CS. Approach to the child with occult toxic exposure. UpToDate Online 15.3. 2008. Available for subscription at: http://www.utdol.com/utd/content/topic.do?topicKey=ped_tox/3023&selectedTitle=4~150&source=search_result

Page 56: AAP PREP 2009

2009 PREP SA on CD-ROM

page 56

Critique: 16

Page 57: AAP PREP 2009

2009 PREP SA on CD-ROM

page 57

Critique: 16

Page 58: AAP PREP 2009

2009 PREP SA on CD-ROM

page 58

Question: 17

You are following a 3-month-old infant who was born at 30 weeks’ gestation, underwent a distal ileal resection for necrotizing enterocolitis at 2 weeks of age, and subsequently was placed on parenteral nutrition for 2 months. The baby has residual cholestasis from the parenteral nutrition (total bilirubin, 5.0 mg/dL [85.5 mcmol/L]; direct bilirubin, 3.0 mg/dL [51.3 mcmol/L]). Currently, she is receiving a cow milk protein hydrolysate formula concentrated to 24 kcal/oz (0.8 kcal/mL). You are considering adding a dietary supplement to increase the caloric density of the formula.

Of the following, the supplement that is the MOST likely to be tolerated and cause less diarrhea in this infant is

A. flaxseed oil

B. medium-chain triglyceride oil

C. olive oil

D. omega-3 polyunsaturated fatty acid (fish oil)

E. soybean oil

Page 59: AAP PREP 2009

2009 PREP SA on CD-ROM

page 59

Critique: 17 Preferred Response: B

Infants who have chronic illnesses may have specialized nutritional requirements and often do not tolerate the standard 20-kcal/oz formula given to healthy term infants. For example, children who have some forms of congenital heart disease or renal disease may require a more concentrated formula because the standard formula may lead to volume overload. Children who have intestinal disease or malabsorption, such as the child described in the vignette, also may require a more concentrated formula to decrease the likelihood of feeding intolerance or diarrhea. Formula may be concentrated by increasing the concentration of protein, carbohydrate, or fat. Perhaps the easiest method of increasing the caloric density of a formula is to mix more powder with the same amount of water. For example, four scoops of most commercially available formulas mixed with 8 oz of water yields standard 0.67-kcal/mL (20-kcal/oz) formula, but mixing five scoops in 8 oz results in 0.83-kcal/mL (25-kcal/oz) formula. However, exceeding 25-kcal/oz formula by increasing the amount of powder may yield too high a concentration of protein, which could result in an excessive renal solute load. For this reason, carbohydrate and fat supplements are available to concentrate infant and toddler feedings further. The most common adverse effect of carbohydrate supplements is diarrhea, and lipid supplements may cause either diarrhea or delayed gastric emptying. For these reasons, caution is recommended when increasing the caloric density of a formula, especially when the caloric density is increased to greater than 1 kcal/mL (30 kcal/oz). Because the patient in the vignette has had an ileal resection and cholestasis, the best fat supplement for him is oil composed of medium-chain triglycerides (MCT oil). MCT oil can be absorbed directly across the enterocyte and does not require intraluminal digestion by bile acids. In contrast, soy, olive, flaxseed, and fish oils are long-chain fatty acids that require bile acids for digestion and might cause diarrhea in a child who has cholestasis and ileal resection.

References:

Courtney E, Grunko A, McCarthy T. Enteral nutrition. In: Hendricks KM, Duggan C. Manual of Pediatric Nutrition. 4th ed. Hamilton, Ontario, Canada: BC Decker; 2005:252-316

Suchy FJ. Neonatal cholestasis. Pediatr Rev. 2004;25:388-396. Available at: http://pedsinreview.aappublications.org/cgi/content/full/25/11/388

Page 60: AAP PREP 2009

2009 PREP SA on CD-ROM

page 60

Question: 18

You have admitted a 750-g male infant to the neonatal intensive care unit (NICU) for treatment of respiratory distress and presumed sepsis. The Apgar scores were 1, 5, and 7 at 1, 5, and 10 minutes, respectively. The infant received one dose of exogenous surfactant in the delivery room. In the NICU, the infant is being cared for on a radiant warmer. At 4 hours after birth, physical examination reveals a temperature of 97.0°F (36.1°C), heart rate of 180 beats/min, respiratory rate of 40 breaths/min (assisted breaths on the ventilator), blood pressure of 45/27 mm Hg, mean arterial blood pressure of 30 mm Hg, and pulse oximetry of 92%. The infant is receiving synchronized intermittent mechanical ventilation with a peak inflation pressure of 18 cm H2O over a positive end-expiratory pressure of 4 cm H2O at a rate of 40 breaths/min and an FiO2 of 0.40. Umbilical catheters are present in the umbilical artery and vein. On physical examination, you note a soft, flat anterior fontanelle. You auscultate equal mechanical breath sounds bilaterally over the chest and note minimal subcostal retractions. The skin is thin and somewhat moist, and many veins are visible through it. The ears are flattened against the cranium and lack any cartilage or recoil. There is a small phallus and an empty scrotum. The infant is laying on the warmer with legs and arms extended. The neuromotor tone is decreased, and the infant does not appear to be very active, but he does respond to tactile stimuli with movement of the arms and legs in seemingly random and purposeless activity.

Of the following, the MOST important next step is to

A. administer another dose of exogenous surfactant

B. consult the urologist for cryptorchidism

C. increase the inspired oxygen concentration

D. move the infant into an isolette incubator

E. obtain head ultrasonography to evaluate for any hemorrhage

Page 61: AAP PREP 2009

2009 PREP SA on CD-ROM

page 61

Critique: 18 Preferred Response: D

The extremely low gestational-age newborn (ELGAN) whose birthweight is less than 1,000 g may present with physical examination findings indicative of marked physical and neurodevelopmental immaturity. These findings have been studied and correlated with gestational age in scoring systems such as those published by Dubowitz in 1980 and Ballard in 1979 and (revised) 1991. Even the best efforts to estimate the gestational age of very low-birthweight and immature infants at dates less than 28 completed weeks of gestation by maternal last menstrual period and obstetric ultrasonography may be imprecise and tend to overestimate maturity (gestational age) by 2 weeks. Nevertheless, the thin, moist, transparent skin; flattened thin ears without cartilage or recoil; and small phallus and empty scrotum described for the infant in the vignette indicate a degree of physical immaturity consistent with 28 weeks' gestation or less. Similarly, the low neuromotor tone, absence of flexed posture, relative inactivity, and random purposeless flailing of extremities with tactile stimulation indicate significant neurologic immaturity. Moving the infant into an isolette incubator is an important next step in the care of this ELGAN. The absolute benefit of using a radiant warmer bed (access to the infant to conduct early examinations, admission nursing care, and procedures such as placement of umbilical catheters) is eclipsed by its associated increased ambient convection, evaporative heat loss, and transcutaneous water loss compared with the contained, convectively heated, and humidified environment of the isolette. The isolette environment still allows access, visibility, and treatment with phototherapy, but with less insensible water loss or variability in patient temperature. This ELGAN is being treated for respiratory distress syndrome and has had a good response to exogenous surfactant administration, oxygen, and assisted ventilation. At 4 hours of postnatal age, it is too early to administer a second dose of surfactant. Developmentally cryptorchid testes may yet descend into the scrotal sac in coming weeks. An oxygen saturation of 92% is good for this infant, and increasing the inspired oxygen concentration may be harmful, contributing to oxidative injury of pulmonary tissues or retinopathy of prematurity. Cranial ultrasonography is indicated in this sick and preterm infant, but without clinical indications of hemorrhage, shock, and seizure or encephalopathy, such a study is performed best at 3 to 7 days of postnatal life.

References:

Donovan EF, Tyson JE, Ehrenkranz RA, et al. Inaccuracy of Ballard scores before 28 weeks' gestation. National Institute of Child Health and Human Development Neonatal Research Network. J Pediatr. 1999;135:147-152. Abstract available at: http://www.ncbi.nlm.nih.gov/pubmed/10431107

Marín GMA, Martín Moreiras J, Llitera Fleixas G, et al. Assessment of the new Ballard score to estimate gestational age [in Spanish]. An Pediatr (Barc). 2006;64:140-145. English abstract available at: http://www.ncbi.nlm.nih.gov/pubmed/16527066

Sedin G. The thermal environment of the newborn infant. In: Martin RJ, Fanaroff AA, Walsh MC, eds. Fanaroff and Martin's Neonatal-Perinatal Medicine. 8th ed. Philadelphia, Pa: Mosby Elsevier; 2006:585-596

Thilo EH, Rosenberg AA. The newborn infant. In: Hay WW Jr, Levin MJ, Sondheimer JM, Deterding RR, eds. Current Pediatric Diagnosis & Treatment. 18th ed. New York, NY: The McGraw-Hill Companies, Inc; 2007:chap 1

Page 62: AAP PREP 2009

2009 PREP SA on CD-ROM

page 62

Question: 19

You are called to the newborn nursery to evaluate a 1-day-old girl whose hands and feet are blue. She was born at term via a cesarean section, and there were no complications. Apgar scores were 9 at both 1 and 5 minutes. Her respiratory rate is 40 breaths/min, heart rate is 140 beats/min, and blood pressure is normal. Pulse oximetry is 98% on room air. Her lungs are clear, and there is no murmur. Her lips are pink, but her hands and feet are cyanotic (Item Q19), and capillary refill is less than 2 seconds.

Of the following, the MOST likely cause of her peripheral cyanosis is

A. cold environment

B. polycythemia

C. retained fetal lung fluid

D. sepsis

E. transposition of the great arteries

Page 63: AAP PREP 2009

2009 PREP SA on CD-ROM

page 63

Question: 19

Cyanosis of the feet, as exhibited by the infant in the vignette. (Courtesy of the Media Lab at Doernbecher)

Page 64: AAP PREP 2009

2009 PREP SA on CD-ROM

page 64

Critique: 19 Preferred Response: A

Cyanosis is a common finding in newborns. The first step in the evaluation of an infant who exhibits cyanosis is to determine whether it is central or peripheral (acrocyanosis). The infant described in the vignette has clinical features suggestive of acrocyanosis, a bluish discoloration of the hands and feet (Item C19A) in response to vasomotor instability or cold environment. In this condition, the lips and mucous membranes are spared, although the perioral area may be affected. The extremities may be cool to touch. Acrocyanosis is believed to be due to vasoconstriction of small arterioles and resolves in the first few postnatal months. Infants who have acrocyanosis require no further evaluation; parental reassurance is all that is needed. Central cyanosis is the bluish discoloration of the tissues best observed in the lips, tongue, mucous membranes, and occasionally nail beds (Item C19B). It occurs when the concentration of deoxygenated (reduced) hemoglobin exceeds 4.0 to 6.0 g/dL (40.0 to 60.0 g/L) within the capillary bed. In an individual who has normal hemoglobin, cyanosis usually becomes apparent when the oxygen saturation drops below 75%-80%. Causes of central cyanosis in the neonatal period typically are related to cyanotic heart disease, such as transposition of the great arteries, or respiratory disease, such as pneumonia and retained fetal lung fluid (transient tachypnea of the newborn). The absence of central cyanosis, a cardiac murmur, and respiratory distress for the infant in the vignette makes these diagnoses less likely than acrocyanosis of infancy. Polycythemia, defined as a venous hematocrit of at least 65% (0.65), causes a deep reddish-purple discoloration of the skin and mucous membranes. Infants may be otherwise asymptomatic or may exhibit lethargy, anorexia, or respiratory distress. Polycythemia does not cause either peripheral or central cyanosis per se, but it does affect the percent saturation at which cyanosis becomes clinically apparent. Cyanosis in infants who have polycythemia occurs at higher oxygen saturations than in those who have anemia. For example, if an infant has a hemoglobin of 24.0 g/dL (240.0 g/L), he is likely to appear cyanotic when his oxygen saturation reaches 87%. In an infant who has a hemoglobin of 12.0 g/dL (120.0 g/L), cyanosis may not be seen until the oxygen saturation falls to 75%. Newborns who have sepsis may develop cyanosis as a component of multisystem involvement, but other signs, such as respiratory distress, poor peripheral perfusion, lethargy, or poor feeding, also are evident.

References:

Bernstein D. Evaluation of the cardiovascular system: history and physical examination. In: Kliegman RM, Behrman RE, Jenson HB, Stanton BF, eds. Nelson Textbook of Pediatrics. 18th ed. Philadelphia, Pa: Saunders Elsevier; 2007:1857-1863

Sasidharan P. An approach to diagnosis and management of cyanosis and tachypnea in term infants. Pediatr Clin North Am. 2004;51:999-1021. Abstract available at: http://www.ncbi.nlm.nih.gov/pubmed/15275985

Page 65: AAP PREP 2009

2009 PREP SA on CD-ROM

page 65

Critique: 19

Acrocyanosis is a blue discoloration of the hands and feet. (Courtesy of the Media Lab at Doernbecher)

Page 66: AAP PREP 2009

2009 PREP SA on CD-ROM

page 66

Critique: 19

Central cyanosis, as exhibited by this infant who has tetralogy of Fallot, is observed best in highly vascularized areas, such as the lips, tongue, mucous membranes, and nail beds. (Courtesy of M. Rimsza)

Page 67: AAP PREP 2009

2009 PREP SA on CD-ROM

page 67

Question: 20

A frustrated mother requests referral of her 15-month-old child to an allergy and asthma specialist because the boy never seems to have stopped coughing and wheezing over the 6 months of the past respiratory virus season. During the history taking, the mother states that the baby only occasionally is exposed to wood smoke at the family’s barbecue restaurant and to cosmetic chemicals used at the grandmother’s hair salon. Both the father and grandfather smoke cigarettes in the home.

Of the following, the environmental exposure that is MOST likely to be causing the child’s respiratory symptoms is exposure to

A. cigarette smoke

B. cleaning fluids

C. dust mites

D. hairspray

E. wood smoke

Page 68: AAP PREP 2009

2009 PREP SA on CD-ROM

page 68

Critique: 20 Preferred Response: A

The child described in the vignette may have multiple triggers for his wheezing exacerbations, although tobacco smoke probably is the one to which he is exposed most often that should be diminished. Well-known tobacco toxins include nicotine, carbon monoxide, formaldehyde, hydrogen cyanide, sulfur dioxide, nitrogen oxide, ammonia, polycyclic aromatic hydrocarbons, and the nitrosamines. These substances produce both irritant and immunologic effects on the respiratory tract. It is important to note that smoking cessation is difficult, and most pediatricians have little training in the process. However, there is evidence that a brief discussion of smoking cessation in the context of medical visits is effective. Wood smoke is another significant irritant to the respiratory tract; in addition to chemical irritants, wood smoke contains particulate matter. Assuming that the family's restaurant kitchen meets current standards for air filtration/exhaust, the child's exposure should be minimal. However, outdoor barbecue pits that discharge smoke directly into the environment should be avoided. Exposure to cooking spray/oils at the restaurant and cosmetic chemicals such as hairspray at the salon also should be avoided, although the use of volatile organic compounds, including chlorofluorocarbons in hairspray, permanent wave solution, cleaning solutions, and cooking sprays, has decreased sharply since passage of the Clean Air Act and other Environmental Protection Agency measures in the 1990s. Material Safety Data Sheets for most hairsprays, as well as other cosmetics and cleaning fluids, reveal little inhalation toxicity, except in the case of prolonged exposure under improper ventilation. Although dust mite exposure may contribute to asthma exacerbation, there is no evidence that environmental control can reduce the concentration of mites or exacerbations related to them. Finally, serial respiratory virus infections themselves may contribute significantly to this child's history of wheezing and cough. The child in the vignette may benefit from therapeutic advances in asthma management such as those contained in the National Heart, Lung and Blood Institute Guidelines for asthma, including attention to number of symptomatic days, use of inhaled corticosteroids, and use of a metered dose inhaler with spacer for proper delivery of medication, which may be more effective than nebulization in older infants and children.

References:

Brunnhuber K, Cummings KM, Feit S, Sherman S, Woodcock J.Putting evidence into practice: smoking cessation. BMJ Clinical Evidence. 2007. Available for subscription at: http://clinicalevidence.bmj.com/ceweb/resources/index.jsp

Kum-Nji P, Meloy L, Herrod HG. Environmental tobacco smoke exposure: prevalence and mechanisms of causation of infections in children. Pediatrics. 2006;117:5:1745-1754. Available at: http://pediatrics.aappublications.org/cgi/content/full/117/5/1745

Roseby R, Waters E, Polnay A, Campbell R, Webster P, Spencer N. Family and carer smoking control programmes for reducing children's exposure to environmental tobacco smoke. Cochrane Database Syst Rev. 2003;3:CD001746. Available at: http://www.cochrane.org/reviews/en/ab001746.html

Stein RT, Holberg CJ, Sherrill D, et al. Influence of parental smoking on respiratory symptoms during the first decade of life: The Tucson Children's Respiratory Study. Am J Epidemiol. 1999;149:1030-1037. Available at: http://aje.oxfordjournals.org/cgi/reprint/149/11/1030

Page 69: AAP PREP 2009

2009 PREP SA on CD-ROM

page 69

U.S. Environmental Protection Agency. Health effects of wood smoke. Available at: http://www.epa.gov/woodstoves/healtheffects.html

U.S. Environmental Protection Agency. National volatile organic compound emission standards for consumer products. Available at: http://www.epa.gov/fedrgstr/EPA-AIR/1998/September/Day-11/a22660.htm

Page 70: AAP PREP 2009

2009 PREP SA on CD-ROM

page 70

Question: 21

You are evaluating a newborn 6 hours after his birth. Labor and delivery were uncomplicated, but amniocentesis performed during the pregnancy revealed trisomy 21. Fetal echocardiography at 20 weeks’ gestation showed normal findings. The infant currently is sleeping and is well-perfused, with a heart rate of 140 beats/min and no audible murmurs. His physical features are consistent with Down syndrome.

Of the following, the MOST appropriate diagnostic study to perform is

A. barium swallow

B. cervical spine radiography

C. echocardiography

D. head ultrasonography

E. radiography of the abdomen

Page 71: AAP PREP 2009

2009 PREP SA on CD-ROM

page 71

Critique: 21 Preferred Response: C

Congenital heart defects (CHDs) are the most common of the congenital anomalies, occurring with an incidence of approximately 5 to 8 per 1,000 live births (0.5% to 0.8%). The incidence of CHD is greater in stillbirths and there is an increased incidence of CHD in those who have aneuploidy. It is believed by some that the abnormal chromosomal composition rather than the cardiac abnormality is responsible for the fetal demise of those who have CHDs. The strong association between chromosomal abnormality and CHD has been shown in a number of studies that focus on identification of fetal cardiac abnormality. A number of well-defined chromosomal anomalies are associated with CHD (Item C21). Early and accurate diagnosis of CHD is important in counseling parents of children in whom a chromosomal abnormality or syndrome is suspected. Such diagnoses may have significant effects on the health and well-being of the newborns because some CHDs may require ductal patency for perfusion of either the systemic or pulmonary circulation. Therefore, echocardiography now is considered an important component of the routine health supervision of infants who have Down syndrome and other syndromes predisposing to structural heart defects. Although results of fetal echocardiography for the infant in the vignette were interpreted as normal, not all CHDs can be diagnosed routinely with fetal echocardiography because of the shunting pathways of the fetal circulation. For example, persistent patency of the ductus arteriosus and secundum atrial septal defects are diagnosed postnatally. Small ventricular septal defects frequently are not seen during fetal echocardiography because the pressure in the right and left ventricles are equal due to the ductus arteriosus, resulting in minimal flow across the defect prenatally. Given the strong association between Down syndrome and CHD, echocardiography should be performed in the newborn described in the vignette. Although children born with Down syndrome have an increased incidence of duodenal atresia and other types of gastrointestinal obstruction, a barium swallow is not an appropriate initial test for an asymptomatic newborn. Cervical radiography, beginning at age 3 years, is important because of the risk of atlantoaxial (C1-C2) subluxation. There is no indication for routine head ultrasonography or abdominal radiography in an otherwise asymptomatic newborn who has Down syndrome.

References:

Committee on Genetics. Health supervision for children with Down syndrome. Pediatrics. 2001;107:442-449. Available at: http://pediatrics.aappublications.org/cgi/content/full/107/2/442

Silberbach M, Hannon D. Presentation of congenital heart disease in the neonate and young infant. Pediatr Rev. 2007;28:123-131. Available at: http://pedsinreview.aappublications.org/cgi/content/full/28/4/123

Tennstedt C, Chaoui R, Körner H, Dietel M. Spectrum of congenital heart defects and extracardiac malformations associated with chromosomal abnormalities: results of a seven year necropsy study. Heart. 1999;82:34-39. Abstract available at: http://www.ncbi.nlm.nih.gov/pubmed/10377306

Page 72: AAP PREP 2009

2009 PREP SA on CD-ROM

page 72

Critique: 21

Page 73: AAP PREP 2009

2009 PREP SA on CD-ROM

page 73

Question: 22

A 4-year-old boy presents with headache and difficulty walking. On physical examination, he is afebrile, all growth parameters are within normal limits, and his mentation appears normal. The optic discs are clearly visible and appear normal. He has normal eye position in primary gaze but cannot abduct his right eye fully. He has normal tone, strength, and reflexes in his upper limbs, but has bilateral hyperreflexia at the knees and ankle clonus. On gait examination, he toe-walks.

Of the following, the MOST important next step is to obtain

A. computed tomography scan of the head

B. electromyography/nerve conduction studies of the legs

C. lumbar puncture

D. magnetic resonance imaging of the thoracolumbar spine

E. visual evoked potentials

Page 74: AAP PREP 2009

2009 PREP SA on CD-ROM

page 74

Critique: 22 Preferred Response: A

The boy described in the vignette presents with a very concerning constellation of symptoms and signs requiring urgent evaluation of the central nervous system. The first step in the diagnostic process is to localize the problem to the proper level of the nervous system: brain, brainstem/cerebellum, spinal cord, nerve, junction, or muscle. Such a determination not only allows for urgent appropriate diagnosis and treatment but also reduces unnecessary discomfort, risks, and costs of inappropriate diagnostic testing. Consultation with a neurologist can be useful before ordering testing. Headache, gait disturbance (toe-walking), and acquired ocular misalignment (right eye cannot move to the right) localizes the problem to the central nervous system and raises concerns for both hydrocephalus and a brainstem lesion. Computed tomography (CT) scan of the head is the preferred neuroimaging technique for this patient. Although brain magnetic resonance imaging (MRI) can document the brainstem/posterior fossa better, it may not be readily available. Moreover, a head CT scan in the emergency department is adequate to rule out hydrocephalus that requires emergent neurosurgical consultation. Unfortunately, a common cause for hydrocephalus in a child of this age is a brainstem or cerebellar neoplasm such as an astrocytoma, glioma, medulloblastoma, or ependymoma. When these tumors enlarge in the posterior fossa, they can obstruct the flow of cerebrospinal fluid and cause acute hydrocephalus, which is a neurosurgical emergency. Primary headache disorders such as migraine do not often present before the age of 6 years and are not accompanied by the neurologic findings described in the vignette. Toe-walking is a nonspecific finding that can occur in the context of relatively benign delayed development, although if this represents a change for a child, an upper motor neuron (brain/spinal cord) lesion must be ruled out. The ankle clonus described for the boy in the vignette mandates this approach. Similarly, inability to abduct one eye can be congenital, but if this is a new finding, a brain or brainstem lesion must be ruled out. In this case, headache and the oculomotor findings make a brain or brainstem lesion likely. Gait impairment and hyperreflexia only in the legs can result from hydrocephalus or can localize to the mid- to lower spinal cord. A spinal cord lesion would not explain the headache, and bilateral motor findings without sensory, bowel, or bladder involvement are uncommon for a spinal cord lesion. Therefore, MRI of the spine is not needed. Similarly, the hyperreflexia is not evidence of a nerve or muscle problem, obviating the need for electromyography and nerve conduction studies. Lumbar puncture is therapeutic for headache due to pseudotumor cerebri, which could present with this constellation of symptoms, but this degree of gait abnormality would be uncommon in pseudotumor, and the presence of true hydrocephalus or an intracranial mass must be excluded with a head CT or brain MRI prior to lumbar puncture. Visual evoked potentials occasionally are used to determine whether the axons from eye to occipital cortex function normally, but such a test generally is performed when demyelinating disorders are suspected.

References:

Avellino AM. Hydrocephalus. In: Singer HS, Kossoff EH, Hartman AL, Crawford TO, eds. Treatment of Pediatric Neurologic Disorders. Boca Raton, Fla: Taylor & Francis; 2005:25-36

Garton HJ, Piatt JH Jr. Hydrocephalus. Pediatr Clin North Am. 2004;51:305-325. Abstract available at: http://www.ncbi.nlm.nih.gov/pubmed/15062673

Kestle JR. Pediatric hydrocephalus: current management. Neurol Clin. 2003;21:883-895. Abstract available at: http://www.ncbi.nlm.nih.gov/pubmed/14743654

Page 75: AAP PREP 2009

2009 PREP SA on CD-ROM

page 75

Kuttesch J Jr, Ater JL. Brain tumors in childhood. In: Behrman RE, Kliegman RM, Jenson HB, Stanton BF, eds. Nelson Textbook of Pediatrics. 18th ed. Philadelphia, Pa: Saunders Elsevier; 2007:2128-2136

Piatt JH Jr. Recognizing neurosurgical conditions in the pediatrician's office. Pediatr Clin North Am. 2004;51:237-270. Abstract available at: http://www.ncbi.nlm.nih.gov/pubmed/15062671

Page 76: AAP PREP 2009

2009 PREP SA on CD-ROM

page 76

Question: 23

Parents who are new to your area bring in their 3-year-old daughter for evaluation because they are concerned about her delayed speech. They say that she uses about 50 single words. The girl has had tetralogy of Fallot repaired surgically and recurrent upper respiratory tract infections with otitis media, for which tympanostomy tubes have been placed. Findings on physical examination include microcephaly, underfolded pinnae, a broad nasal bridge, cleft uvula, and a small chin. In addition, the child’s speech has a hypernasal quality. The family history is negative for birth defects and developmental delays.

Of the following, the contiguous gene deletion syndrome that BEST fits this child’s features is

A. Angelman

B. Beckwith-Wiedemann

C. 4p-

D. Prader-Willi

E. 22q11

Page 77: AAP PREP 2009

2009 PREP SA on CD-ROM

page 77

Critique: 23 Preferred Response: E

The contiguous gene deletion syndromes comprise a group of disorders defined by characteristic dysmorphisms or behaviors in association with a submicroscopic chromosome deletion that is detected using fluorescence in situ hybridization (FISH) or microarray technologies. In each case, the deleted segment contains multiple genes that play varying roles in producing a particular phenotype. Angelman, Beckwith-Wiedemann, Wolf-Hirschhorn (4p-), Prader-Willi, and 22q11 deletion syndromes are among the best delineated contiguous gene deletion syndromes. The child described in the vignette has features most consistent with 22q11 (the long arm of chromosome 22 at band 11) deletion syndrome, which occurs in approximately 1 in 4,000 births, making it the most common of the contiguous gene deletion syndromes. Confusion has arisen regarding the relationship of 22q11 deletion syndrome to the DiGeorge and velo-cardio-facial syndromes. It now is known that the condition described by DiGeorge in 1965, consisting of hypoplastic-to-absent thymus and parathyroid glands together with conotruncal heart defects, is due to 22q11 deletion in most cases. Similarly, the condition described by Shprintzen and associates in 1978 that has come to be called velo-cardio-facial syndrome, which includes cleft palate, speech disorder, broad nasal root, immune dysfunction, and learning difficulties, is also caused by 22q11 deletion. The two disorders represent different manifestations of the same submicroscopic deletion. 22q11 deletion syndrome should be suspected in any child who has a conotruncal heart defect (eg, tetralogy of Fallot, double-outlet right ventricle, truncus arteriosus), especially if accompanied by developmental delay, dysmorphic features, or recurrent upper respiratory tract infections. Testing for the condition involves sending blood for FISH (it must be specified that the region of interest is 22q11) or for microarray analysis, assuming the array covers the 22q11 region. Angelman and Prader-Willi syndromes (AS and PWS, respectively) are caused by aberrations (usually contiguous gene deletions) on the long arm of chromosome 15 at bands 11 through 13. This region of the human genome is "imprinted," such that when the paternal region is deleted, the result is PWS, and when the maternal region is deleted, the result is AS. Children who have PWS typically present in infancy with poor feeding, hypotonia, mild facial dysmorphisms, and undescended testes with or without micropenis in boys. At the age of 2 to 3 years, affected children start eating excessively and gain weight rapidly, becoming obese (Item C23A). They are mildly to moderately mentally retarded and often exhibit excessive skin picking and a facility for jigsaw puzzles. Children who have AS typically appear normal at birth, but by 1 to 2 years of age, they have acquired microcephaly and severe speech delay (Item C23B). During this same period, most (~85%) develop generalized seizures. Children who have AS have an ataxic gait and jerky arm movements, resulting in a "puppet-like" gait. They are typically nonverbal and severely mentally retarded and have paroxysms of inappropriate laughter. 4p- (4p minus), previously called Wolf-Hirschhorn syndrome, is a multiple congenital anomaly syndrome characterized by prenatal-onset growth deficiency, prominent glabella, microcephaly, and cleft lip+/-cleft palate. Profound mental retardation is the rule. Affected individuals may have a cytogenetically detectable deletion on the short arm of chromosome 4 or they may have a submicroscopic deletion involving 4p16.3. Beckwith-Wiedemann syndrome (BWS) is characterized by pre- and postnatal overgrowth, hypoglycemia in the newborn period, large tongue (Item C23C), unusual earlobe creases, omphalocele in some (Item C23D), and organomegaly. Affected children are at increased risk for certain neoplasias, including hepatoblastoma and Wilms tumor, making it important to follow a tumor surveillance protocol up to 7 to 10 years of age. Intelligence typically is normal. The genetics of BWS are complex; the condition is caused by submicroscopic deletions/alterations in genes located at 11p15, which is a highly imprinted region. The management of each of the contiguous gene deletion syndromes involves referrals to multiple specialists, depending on the systems involved, together with anticipatory guidance based on natural

Page 78: AAP PREP 2009

2009 PREP SA on CD-ROM

page 78

history information.

References:

Battaglia A, Carey JC, Wright TJ. Wolf-Hirschhorn syndrome. GeneReviews. 2006. Available at: http://www.geneclinics.org/servlet/access?db=geneclinics&site=gt&id=8888891&key=OvKiicpzcfvnc&gry=&fcn=y&fw=vH7o&filename=/profiles/whs/index.html

Bishara N, Clericuzio CL. Common dysmorphic syndromes in the NICU. NeoReviews. 2008;9:e29-e38. Available for subscription at: http://neoreviews.aappublications.org/cgi/content/full/9/1/e29

Cassidy SB, Schwartz S. Prader-Willi syndrome. GeneReviews. 2006. Available at: http://www.geneclinics.org/servlet/access?db=geneclinics&site=gt&id=8888891&key=OvKiicpzcfvnc&gry=&fcn=y&fw=vWzr&filename=/profiles/pws/index.html

Lin RJ, Cherry AM, Bangs CD, Hoyme HE. FISHing for answers: the use of molecular cytogenetic techniques in neonatology. NeoReviews. 2003;4:e94-e98. Available for subscription at: http://neoreviews.aappublications.org/cgi/content/full/4/4/e94

McDonald-McGinn DM, Emanuel BS, Zackai EH. 22q11.2 deletion syndrome. GeneReviews. 2005. Available at: http://www.ncbi.nlm.nih.gov/bookshelf/br.fcgi?book=gene&part=gr_22q11deletion

Shprintzen RJ. Velo-cardio-facial syndrome. In: Cassidy SB, Allanson JE, eds. Management of Genetic Syndromes. 2nd ed. Hoboken, NJ: Wiley-Liss; 2005:615-632

Shuman C, Amith AC, Weksberg R. Beckwith-Wiedemann syndrome. GeneReviews. 2005. Available at: http://www.geneclinics.org/servlet/access?db=geneclinics&site=gt&id=8888891&key=OvKiicpzcfvnc&gry=&fcn=y&fw=0F2J&filename=/profiles/bws/index.html

Weksberg R, Shuman C. Beckwith-Wiedemann syndrome and hemihyperplasia. In: Cassidy SB, Allanson JE, eds. Management of Genetic Syndromes. 2nd ed. Hoboken, NJ: Wiley-Liss; 2005:101-116

Williams CA. Angelman syndrome. In: Cassidy SB, Allanson JE, eds. Management of Genetic Syndromes. 2nd ed. Hoboken, NJ: Wiley-Liss; 2005:53-62

Williams CA, Driscoll DJ. Angelman syndrome. GeneReviews. 2007. Available at: http://www.geneclinics.org/servlet/access?db=geneclinics&site=gt&id=8888891&key=OvKiicpzcfvnc&gry=&fcn=y&fw=tkPG&filename=/profiles/angelman/index.html

Page 79: AAP PREP 2009

2009 PREP SA on CD-ROM

page 79

Critique: 23

Prader-Willi syndrome is characterized by short stature, obesity, hypogonadism, and small hands and feet. (Courtesy of Y. Lacassie)

Page 80: AAP PREP 2009

2009 PREP SA on CD-ROM

page 80

Critique: 23

Angelman syndrome: Note the prominent mandible and happy expression. (Reprinted with permission from Jonas DM, Demmer LA. Genetic syndromes determined by alterations in genomic imprinting pathways. NeoReviews. 2007;8:e120-e126.)

Page 81: AAP PREP 2009

2009 PREP SA on CD-ROM

page 81

Critique: 23

Note the macroglossia, salmon patch on the forehead, prominent eyes, and infraorbital creases in a 3-month-old infant who has Beckwith-Wiedemann syndrome. (Reprinted with permission from Jonas DM, Demmer LA. Genetic syndromes determined by alterations in genomic imprinting pathways. NeoReviews. 2007;8:e120-e126.)

Page 82: AAP PREP 2009

2009 PREP SA on CD-ROM

page 82

Critique: 23

An omphalocele is observed in some patients who have Beckwith-Wiedemann syndrome. (Courtesy of the Media Lab at Doernbecher)

Page 83: AAP PREP 2009

2009 PREP SA on CD-ROM

page 83

Question: 24

A 13-year-old girl presents with severe lower abdominal pain of 24 hours’ duration. She states that the pain is sharp and constant and that she has had similar pain for several days approximately monthly over the past 4 months. She has no vomiting or diarrhea with the pain, but she is constipated frequently, having a bowel movement about every 3 to 4 days. She feels that her jeans are getting tighter around the waist, although she remains active, playing soccer daily. She has never had a menstrual period and denies ever being sexually active. On physical examination, she is afebrile, her heart rate is 85 beats/min, and her blood pressure is 110/70 mm Hg. Her weight is at the 60th percentile and her height at the 50th percentile for age. Her breasts and genitalia are at Sexual Maturity Rating 5. Abdominal examination reveals a firm and tender midline mass that is inferior to the umbilicus.

Of the following, the MOST likely diagnosis is

A. bladder obstruction

B. endometriosis

C. hematocolpos

D. megacolon

E. ovarian cyst

Page 84: AAP PREP 2009

2009 PREP SA on CD-ROM

page 84

Critique: 24 Preferred Response: C

The adolescent described in the vignette has a clinical history and physical examination findings compatible with an imperforate hymen, which probably is the most common obstructive anomaly of the female reproductive tract. An adolescent patient who has an imperforate hymen may be asymptomatic or may have a history of cyclic abdominal pain that may occur for several years before the diagnosis is made. A bluish, bulging hymen may be seen on genital inspection (Item C24), and a distended vagina may be palpated on rectoabdominal or abdominal examination. If the vagina becomes substantially enlarged with accumulated blood, the patient may experience back pain, pain with defecation that can result in constipation, nausea and vomiting, or difficulty in urinating. Bladder outlet obstruction occurs rarely, and although it produces a suprapubic mass, it does not cause cyclic abdominal pain. Megacolon also is unlikely and does not cause cyclic pain, although colonic irritation may develop from the pressure produced by the mass. An ovarian cyst typically causes a right- or left-sided (not midline) mass, and endometriosis is an unlikely cause of a palpable mass, although it can cause cyclic and acyclic pain in adolescents.

References:

Adams Hillard PJ, Deitch HF. Gynecologic disorders. In: Osborn LM, DeWitt TG, First LR, Zenel JA, eds. Pediatrics. Philadelphia, Pa: Elsevier Mosby;2005:1461-1471

Laufer MR, Goldstein DP, Hendren WH. Structural abnormalities of the female reproductive tract. In: Emans SJH, Laufer MR, Goldstein DP, eds. Pediatric and Adolescent Gynecology. 5th ed. Philadelphia, Pa: Lippincott, Williams & Wilkins; 2005:334-416

Page 85: AAP PREP 2009

2009 PREP SA on CD-ROM

page 85

Critique: 24

Bulging of an imperforate hymen. (Reprinted with permission from Torok K, Bhende MS. Index of suspicion (case 2). Pediatr Rev. 2008;29:25-30.)

Page 86: AAP PREP 2009

2009 PREP SA on CD-ROM

page 86

Question: 25

During teaching rounds, the pediatric ward resident reports on a 4-month-old circumcised male infant who was admitted to the pediatric ward for fever that morning. The infant is now afebrile and has had respiratory rates of 40 breaths/min while sleeping and greater than 60 breaths/min when awake. The infant has a soft, flat fontanelle on physical examination and is not irritable. The only diagnostic studies obtained on admission were a urinalysis and complete blood count, the results of which were normal, except for a white blood cell count of 16.0x103/mcL (16.0x109/L).

Of the following, the MOST appropriate next step is

A. administration of 100 mL normal saline

B. chest radiography

C. lumbar puncture

D. reassurance of the resident that this represents normal respiratory variation

E. urine culture

Page 87: AAP PREP 2009

2009 PREP SA on CD-ROM

page 87

Critique: 25 Preferred Response: B

Respiratory rates vary across a relatively wide range in pediatric patients, depending on factors such as age and activity status. Therefore, strict definitions of tachypnea and bradypnea are difficult to determine and always must be considered in association with other factors such as current clinical status and individual history. Because tachypnea is a sensitive indicator of lower airway disease, patients who have elevated respiratory rates deserve a clinical evaluation in the context of other associated symptoms. The tachypnea, history of fever, and elevated white blood cell count described for the boy in the vignette warrant chest radiography. Although lumbar puncture and a urine culture often are indicated to evaluate infants who have fever, the elevated respiratory rate combined with reassuring neurologic examination results and normal urinalysis make pneumonia a more likely diagnosis. Administration of a normal saline bolus would not be expected to improve the abnormal respiratory rate.

References:

Bloomfield D. In brief: tachypnea. Pediatr Rev. 2002;23:294-295. Available at: http://pedsinreview.aappublications.org/cgi/content/full/23/8/294

Sectish TC, Prober CG. Pneumonia. In: Kliegman RM, Behrman RE, Jenson HB, Stanton BF, eds. Nelson Textbook of Pediatrics. 18th ed. Philadelphia, Pa: Saunders Elsevier;2007:1795-1799

Page 88: AAP PREP 2009

2009 PREP SA on CD-ROM

page 88

Question: 26

A 6-year-old boy who has severe vomiting and dehydration is admitted to the hospital. Initial laboratory studies demonstrate a serum sodium concentration of 126.0 mEq/L (126.0 mmol/L), potassium of 5.3 mEq/L (5.3 mmol/L), and pH of 7.26. After 24 hours of rehydration with 0.9% saline, his serum sodium concentration is 129.0 mEq/L (129.0 mmol/L) and potassium is 4.9 mEq/L (4.9 mmol/L). On physical re-examination, you note that his knees, elbows, dorsal fingers, and tongue are somewhat pigmented (Item Q26), and his skin is darker than that of other family members.

Of the following, the MOST useful diagnostic laboratory study at this time is measurement of serum

A. antidiuretic hormone (ADH) and alpha-melanocortin-stimulating hormone

B. cortisol and adrenocorticotropic hormone (ACTH)

C. cortisol and dehydroepiandrosterone

D. dehydroepiandrosterone and ACTH

E. insulin-like growth factor 1 and ADH

Page 89: AAP PREP 2009

2009 PREP SA on CD-ROM

page 89

Question: 26

Hyperpigmentation (arrows), as exhibitied by the boy in the vignette. (Courtesy of the Media Lab at Doernbecher)

Page 90: AAP PREP 2009

2009 PREP SA on CD-ROM

page 90

Critique: 26 Preferred Response: B

Acute or chronic vomiting and dehydration associated with hyponatremia and elevated potassium concentrations, as described for the boy in the vignette, suggest adrenocortical insufficiency. Primary adrenal insufficiency is associated with skin pigmentation (Item C26) because of ACTH overproduction. ACTH acts directly on the melanocortin receptors of skin to activate melanin production. Normally, ACTH controls the amount of cortisol produced by the adrenal cortex by stimulating adrenal steroidogenesis. Cortisol then feeds back both to the pituitary and hypothalamus to inhibit pituitary ACTH release. Measurement of low serum cortisol and markedly elevated serum ACTH concentrations at any time of the day usually confirms the diagnosis of primary adrenal insufficiency. The presence of skin pigmentation in the boy described in the vignette suggests that identifying an elevated ACTH value will be easy, but because there is diurnal variation in ACTH and cortisol, with highest concentrations during the early morning hours and lowest in the late afternoon and evening, children who have less severe adrenal insufficiency should have these hormones assessed in the early morning. In addition, an ACTH stimulation test might be necessary for diagnostic confirmation. In this test, synthetic ACTH1-24 is administered intravenously, and the adrenal cortisol response is measured before injection and at 1 hour postinjection. An adequate cortisol response at 1 hour rules out adrenal insufficiency. Although ADH concentrations might be elevated in primary adrenal insufficiency because of loss of intravascular fluid volume, measurement of this hormone does not help in the diagnosis of a child who has low serum sodium and somewhat elevated potassium values. Low serum sodium is associated with inappropriate ADH release, but the potassium would not be elevated. ACTH and melanocyte-stimulating hormone (a melanocortin) both are produced from enzymatic cleavage of a larger molecule, proopiomelanocortin. Although melanocyte-stimulating hormone might be overproduced to some extent in the presence of excess ACTH, elevated concentrations of this hormone are not necessary for skin pigmentation with ACTH excess. Dehydroepiandrosterone (DHEA) is a weak androgen precursor produced by the fetal adrenal initially; production increases again in mid-childhood with the onset of adrenarche. DHEA values rise slowly from age 4 to 6 years. Elevated values may be found with some adrenal tumors and some relatively rare types of congenital adrenal hyperplasia, but these disorders are associated with some degree of early puberty. DHEA-S, the sulfated product of DHEA, usually has stable serum values without diurnal variation. Therefore, normal DHEA or DHEA-S values in an adolescent or adult confirm the presence of a functioning adrenal gland. This test is less useful in a young child because onset of adrenarche is variable. Cortisol is low in primary or secondary adrenal insufficiency but tends toward low ranges in most people after the early morning hours. Insulin-like growth factor 1 is a marker for growth hormone sufficiency, and normal concentrations vary with age and sex. Because this child does not seem to have a growth problem, the result of this assay should be normal for age.

References:

Auchus RJ, Rainey WE. Adrenarche-physiology, biochemistry and human disease. Clin Endocrinol. 2004;60:288-296. Available at: http://www.blackwell-synergy.com/doi/full/10.1046/j.1365-2265.2003.01858.x

Coco G, Dal Pra XC, Presotto F, et al. Estimated risk for developing autoimmune Addison's disease in patients with adrenal cortex antibodies. J Clin Endocrinol Metab. 2006;91:1637-1645. Available at: http://jcem.endojournals.org/cgi/content/full/91/5/1637

Donohoue PA. Diagnosis of adrenal insufficiency in children. UpToDate Online 15.3. 2008. Available for

Page 91: AAP PREP 2009

2009 PREP SA on CD-ROM

page 91

subscription at: http://www.uptodateonline.com/utd/content/topic.do?topicKey=pediendo/20697

Perry R, Kecha O, Paquette J, Huot C, van Vliet G, Deal C. Primary adrenal insufficiency in children: twenty years experience at the Sainte-Justine Hospital, Montreal. J Clin Endocrinol Metab. 2005;90:3243-3250. Available at: http://jcem.endojournals.org/cgi/content/full/90/6/3243

Wilson TA, Speiser P. Adrenal insufficiency. eMedicine Specialties, Pediatrics: General Medicine, Endocrinology. 2007. Available at: http://www.emedicine.com/ped/TOPIC47.HTM

Page 92: AAP PREP 2009

2009 PREP SA on CD-ROM

page 92

Critique: 26

In Addison disease, excess adrenocorticotropic hormone acts on melanocortin, causing increased melanin production and hyperpigmentation of the mucous membranes and skin (arrows). (Courtesy of the Media Lab at Doernbecher)

Page 93: AAP PREP 2009

2009 PREP SA on CD-ROM

page 93

Question: 27

A mother of a 6-year-old boy in your practice is concerned that her son may have dyslexia. She has brought a sample of his printing to the visit in which the boy wrote "ded" instead of "bed" and "dad" instead of "bad." She wants your advice on what she should do to help her son learn how to write properly.

Of the following, the MOST appropriate response is to

A. reassure the mother that letter reversal can be normal through 7 years of age

B. recommend a comprehensive psychoeducational evaluation for a learning disability

C. recommend neurologic evaluation

D. refer the child for an occupational therapy evaluation and services to improve his writing skills

E. refer the child for vision therapy

Page 94: AAP PREP 2009

2009 PREP SA on CD-ROM

page 94

Critique: 27 Preferred Response: A

Letter reversal in writing can be normal in children through 7 years of age. Dyslexia, a word recognition defect, is a specific learning disability that is neurobiologically based. It is characterized by problems with the ability to recognize words accurately and poor spelling and decoding skills. Its prevalence is as high as 17.4% of the school-age population. Affected children have problems attaching the correct labels or names to letters and words. They may call a "b" a "d" or read "saw" as "was." Because the problem is linguistic, not visual, affected children do not have problems copying letters. Backward writing and letter reversal occur commonly in early development for all children whether or not they have learning disabilities. All children should receive routine vision screening, but a visual acuity problem would not be the cause of the letter reversal for the boy described in the vignette. There is no scientific evidence that vision therapy (eye exercise) is effective in the remediation of language-based learning disorders. Because letter reversal still can be considered in the normal range of development at 6 years of age, psychoeducational evaluation, neurologic evaluation, and occupational therapy are not indicated for this child.

References:

Committee on Children With Disabilities, American Academy of Pediatrics (AAP) and American Academy of Ophthalmology (AAO), and American Association for Pediatric Ophthalmology and Strabismus (APOS). Learning disabilities, dyslexia, and vision: a subject review. Pediatrics. 1998;102:1217-1219. Available at: http://pediatrics.aappublications.org/cgi/content/full/102/5/1217

Fletcher JM, Lyon GR, Fuchs LS, Barnes MA. Reading disabilities: word recognition. In: Learning Disabilities: From Identification to Intervention. New York, NY: The Guilford Press: 2007:85-163

Shaywitz SE, Shawitz BA. Dyslexia (specific reading disability). Pediatr Rev. 2003;24:147-153. Available at: http://pedsinreview.aappublications.org/cgi/content/full/24/5/147

Page 95: AAP PREP 2009

2009 PREP SA on CD-ROM

page 95

Question: 28

A 14-year-old girl presents to the emergency department with a 2-day history of fever and a rash. The rash has been progressive, and now her mouth and eyes hurt. Upon further questioning, she reports that she was started on an antibiotic 7 days ago for some complaints of dysuria, but she does not remember its name. Physical examination reveals a moderately toxic-appearing female whose temperature is 102.6°F (39.2°C), respiratory rate is 25 breaths/min, heart rate is 105 beats/min, and blood pressure is 105/70 mm Hg. Her bulbar conjunctivae are erythematous (Item Q28A), and she has some early bullous lesions developing in her mouth. She has right upper quadrant tenderness and multiple target lesions (Item Q28B) on her chest, abdomen, arm, back, upper thighs, buttocks, and face.

Of the following, the antimicrobial agent that is MOST likely to be associated with these clinical findings is

A. amoxicillin

B. azithromycin

C. cefdinir

D. clindamycin

E. trimethoprim-sulfamethoxazole

Page 96: AAP PREP 2009

2009 PREP SA on CD-ROM

page 96

Question: 28

Conjunctival erythema, as exhibited by the girl in the vignette. (Reprinted with permission from Bullen LK, Zenel J. Visual diagnosis: a 15-year-old who has cough, rash, and painful swallow. Pediatr Rev. 2005;26:176-181

Page 97: AAP PREP 2009

2009 PREP SA on CD-ROM

page 97

Question: 28

(Courtesy of D. Krowchuk)

Page 98: AAP PREP 2009

2009 PREP SA on CD-ROM

page 98

Critique: 28 Preferred Response: E

Stevens-Johnson syndrome, the condition described in the vignette, may be caused by infectious agents (eg, Mycoplasma pneumoniae, herpes simplex virus) or medications such as nonsteroidal anti-inflammatory agents (eg, ibuprofen, salicylates), anticonvulsants (eg, phenytoin, carbamazepine), and other antimicrobial agents (eg, trimethoprim-sulfamethoxazole [TMP-SXT], cephalosporins). Although generally well tolerated, TMP-SXT has several adverse effects. From 3% to 8% of patients may experience mild gastrointestinal symptoms, including nausea, vomiting, anorexia, diarrhea, glossitis, and stomatitis. Approximately 3% to 4% of patients who receive TMP-SXT develop skin lesions. These drug eruptions include maculopapular rashes, urticaria (Item C28A), pruritus, photodermatitis (Item C28B), exfoliative dermatitis, erythema multiforme, toxic epidermal necrolysis (Item C28C), and Stevens-Johnson syndrome. Although any of the antimicrobial agents listed in the vignette (eg, amoxicillin, azithromycin, cefdinir, clindamycin) can cause these skin reactions, sulfa-containing agents such as TMP-SXT most commonly are responsible. Patients infected with the human immunodeficiency virus and taking TMP-SXT for Pneumocystis prophylaxis also are at increased risk for the development of such reactions. Bone marrow suppression can occur with prolonged administration of TMP-SXT, resulting in pancytopenia, agranulocytosis, anemia, or thrombocytopenia. TMP-SXT also competes with bilirubin for plasma protein binding sites and is not recommended for infants younger than 2 months of age. TMP-SXT can potentiate the effects of warfarin, phenytoin, methotrexate, and oral hypoglycemic agents, leading to bleeding, phenytoin toxicity, severe pancytopenia, and hypoglycemia, respectively. The half-life of digoxin also is increased, resulting in elevated concentrations. Oral contraception metabolism can be accelerated when these medications are administered in conjunction with TMP-SXT, leading to ineffective contraception. TMP-SXT is recommended as first-line therapy for acute, uncomplicated urinary tract infections; urinary tract prophylaxis; selected bacterial gastrointestinal infections (eg, Shigella); and for treatment and prophylaxis of Pneumocystis infections. The use of amoxicillin for the empiric treatment of urinary tract infections is limited due to drug resistance among Escherichia coli. Although cefdinir has very good activity against most community-acquired isolates of E coli and other gram-negative organisms, its use is often cost prohibitive. Clindamycin should be used only in certain situations (eg, when the isolated organism is Staphylococcus aureus), and azithromycin is not indicated for the treatment of urinary tract infections.

References:

Libecco JA, Powell KR. Trimethoprim/sulfamethoxazole: clinical update. Pediatr Rev. 2004;25:375-380. Available at: http://pedsinreview.aappublications.org/cgi/content/full/25/11/375

Mittmann N, Chan B, Knowles S, Cosentino L, Shear N. Intravenous immunoglobulin use in patients with toxic epidermal necrolysis and Stevens-Johnson syndrome. Am J Clin Dermatol. 2006;7:359-368. Abstract available at: http://www.ncbi.nlm.nih.gov/pubmed/17173470

Saidinejad M, Ewald MB, Shannon MW. Transient psychosis in an immune-competent patient after oral trimethoprim-sulfamethoxazole administration. Pediatrics. 2005;115:e739-e741. Available at: http://pediatrics.aappublications.org/cgi/content/full/115/6/e739

Page 99: AAP PREP 2009

2009 PREP SA on CD-ROM

page 99

Critique: 28

Wheals with unusul shapes are characteristic of urticaria, a hypersensitivity reaction that may be triggered by antibiotics such as trimethoprim-sulfamethoxazole. (Courtesy of D. Krowchuk)

Page 100: AAP PREP 2009

2009 PREP SA on CD-ROM

page 100

Critique: 28

A phototoxic eruption has the appearance of a severe sunburn, and bullae may be present. The eruption may be caused by a number of drugs, including tetracyclines, sulfonamides, and fluoroquinolones. In this patient, the eruption was caused by doxycycline. (Courtesy of D. Krowchuk)

Page 101: AAP PREP 2009

2009 PREP SA on CD-ROM

page 101

Critique: 28

In toxic epidermal necrolysis, erythematous patches appear (top), bullae form (bottom arrow), and later rupture, leaving widespread erosions. (Courtesy of D. Krowchuk)

Page 102: AAP PREP 2009

2009 PREP SA on CD-ROM

page 102

Question: 29

A 14-year-old girl presents for evaluation after 4 days of a temperature to 103.0°F (39.5°C), nausea, abdominal cramping, and profuse bloody diarrhea. She reports that she has not traveled anywhere, has no pets, and has had no ill contacts or unusual food exposures. One week ago, she was diagnosed with a methicillin-sensitive Staphylococcus aureus chronic osteomyelitis of her distal radius and has been receiving intravenous cefazolin therapy via a peripherally inserted central catheter line. Physical examination reveals an uncomfortable teenager who complains of severe abdominal pain and has a temperature of 102.8°F (39.4°C) and moist mucous membranes. Her abdomen is diffusely tender, with voluntary guarding but no rebound tenderness on palpation. Rectal examination demonstrates normal sphincter tone with no fissures or other lesions. Laboratory findings include a peripheral white blood cell count of 15.0x103/mcL (15.0x109/L); hemoglobin of 13.0 g/dL (130.0 g/L); platelet count of 300.0x103/mcL (300.0x109/L); and a differential count of 65% neutrophils, 25% lymphocytes, and 10% monocytes. Her stool appears watery and grossly bloody.

Of the following, the MOST appropriate treatment for this patient’s condition is

A. ceftriaxone

B. clindamycin

C. metronidazole

D. trimethoprim-sulfamethoxazole

E. vancomycin

Page 103: AAP PREP 2009

2009 PREP SA on CD-ROM

page 103

Critique: 29 Preferred Response: C

The patient described in the vignette has findings of antibiotic-associated Clostridium difficile disease due to her cefazolin therapy. C difficile accounts for approximately 20% to 30% of cases of antibiotic-associated diarrhea, 50% to 70% of antibiotic-associated colitis cases, and more than 90% of antibiotic-associated pseudomembranous colitis cases. The incidence of antibiotic-associated diarrhea and colitis varies greatly, depending on the antibiotic and its spectrum of activity and pharmacokinetic properties. Almost all antibiotic classes have been associated with disease, but clindamycin, the penicillins, and the cephalosporins are implicated most commonly. The pathogenesis of C difficile-mediated diarrhea and colitis includes: 1) disruption and eradication of normal colonic organisms by antibacterial agents; 2) colonization with toxigenic C difficile; and 3) production of toxin A or toxin B by C difficile, both of which mediate cytoskeletal damage of target cells, resulting in mucosal injury and inflammation. Infection with toxigenic C difficile causes a spectrum of disease, ranging from asymptomatic carriage to a fulminant, relapsing, and occasionally fatal colitis. Signs and symptoms may develop as early as the first day of therapy to as late as 10 weeks after therapy has ended, with a typical range of 5 to 10 days after the initiation of antibiotic therapy. The gastrointestinal manifestations of C difficile are variable, ranging from watery diarrhea to colitis with blood. Other findings include fever (seen in 30% to 50% of patients), leukocytosis (mean peripheral leukocyte count of more than 15.0 x 103/mcL [15.0 x 109/L), nausea, malaise, abdominal pain, and cramping. Oral metronidazole and oral vancomycin are equally effective for the treatment of C difficile-associated diarrhea and colitis, but metronidazole is considered the drug of choice because of cost and concerns regarding the emergence of vancomycin-resistant enterococci. Ceftriaxone, clindamycin, and trimethoprim-sulfamethoxazole are not effective in the treatment of this infection.

References:

American Academy of Pediatrics. Clostridium difficile. In: Pickering LK, Baker CJ, Long SS, McMillan JA, eds. Red Book: 2006 Report of the Committee on Infectious Diseases. 27th ed. Elk Grove Village, Ill: American Academy of Pediatrics; 2006:261-263

Benson L, Song X, Campos J, Singh N. Changing epidemiology of Clostridium difficile-associated disease in children. Infect Control Hosp Epidemiol. 2007;28:1233-1235. Abstract available at: http://www.ncbi.nlm.nih.gov/pubmed/17926272

Klein EJ, Boster DR, Stapp JR, et al. Diarrhea etiology in a children's hospital emergency department: a prospective cohort study. Clin Infect Dis. 2006;43:807-813. Abstract available at: http://www.ncbi.nlm.nih.gov/pubmed/16941358

Thielman NM, Wilson KH. Antibiotic-associated colitis. In: Mandell GL, Bennett JE, Dolin R, eds. Mandell, Douglas and Bennett's Principles and Practice of Infectious Diseases. 6th ed. New York, NY: Elsevier Churchill Livingstone; 2005:1249-1263

Page 104: AAP PREP 2009

2009 PREP SA on CD-ROM

page 104

Question: 30

A 14-year-old girl who has a history of insulin-dependent diabetes mellitus (IDDM) presents with vomiting, increased urination, and decreased energy. Physical examination reveals Kussmaul breathing and delayed capillary refill. Laboratory findings include:· Sodium, 136.0 mEq/L (136.0 mmol/L)· Potassium, 5.2 mEq/L (5.2 mmol/L)· Chloride, 100.0 mEq/L (100.0 mmol/L)· Bicarbonate, 10.0 mEq/L (10.0 mmol/L) · Blood urea nitrogen, 24.0 mg/dL (8.6 mmol/L) · Creatinine, 0.9 mg/dL (79.6 mcmol/L)· Glucose, 550.0 mg/dL (30.5 mmol/L) The patient receives initial hydration with 20 mL/kg of normal saline.

Of the following, a TRUE statement regarding this patient is that the

A. initial maintenance fluid regimen should avoid supplemental potassium

B. serum potassium concentration reflects adequate intracellular potassium

C. serum potassium concentration should increase with correction of the acidosis

D. serum potassium concentration should increase with correction of the hyperglycemia

E. total body potassium content is depleted

Page 105: AAP PREP 2009

2009 PREP SA on CD-ROM

page 105

Critique: 30 Preferred Response: E

Sodium and potassium are cations that often are accompanied by chloride as an anion when consumed in the diet. Within the body, these cations play very different roles. The cell membrane is permeable to both sodium and potassium, but due to Na+-K+-ATPase, they are contained primarily on opposite sides of the cell membrane, with sodium distributed almost exclusively extracellularly and potassium contained intracellularly. Sodium is the principal extracellular cation and acts as the primary osmole to maintain extracellular volume-including plasma volume, which is important for tissue perfusion. Sodium balance is regulated in the kidney. Excessive sodium intake results in reduced sodium conservation within the nephron and increased excretion to keep balance with the increased intake. One tissue in which sodium is contained is the bone, and growing children who have normal kidney function require 2 to 3 mEq/kg per day of sodium for growth. Potassium is important for maintaining the resting membrane potential of cells. The typical intracellular concentration of potassium (140.0 mEq/L [140.0 mmol/L]) is in marked contrast to the 3.5 to 5.5 mEq/L (3.5 to 5.5 mmol/L) of potassium in the extracellular compartment. The ratio of extracellular-to-intracellular potassium determines the resting membrane potential and subsequent action potentials in tissues such as skeletal/cardiac muscle and neuronal tissues. Accordingly, hypokalemia and hyperkalemia can result in muscle paralysis or cardiac dysrhythmias such as ventricular tachycardia, fibrillation, and ultimately cardiac arrest. The growing child requires 1 to 2 mEq/kg per day of potassium for cellular growth and to avoid potassium deficiency. Certain conditions alter the measured extracellular potassium concentrations, often providing misleading information about intracellular stores. In the setting of acidosis, potassium shifts from intracellular to extracellular locations, increasing measured extracellular potassium values. Conversely, alkalosis or correction of metabolic acidosis results in a lowering of measured potassium values. Therapies that result in intracellular shifting of potassium include: insulin, beta-2 agonists such as albuterol or epinephrine, and sodium bicarbonate (due to quenching of acidosis). The patient described in the vignette has severe metabolic acidosis with increased urine output, and the serum potassium value of 5.2 mEq/L (5.2 mmol/L) is misleading. In fact, potassium should be added to the intravenous fluids because this girl's total body potassium is depleted, which will become very apparent upon correction of the acidosis and of the hyperglycemia (with insulin).

References:

Plotnick L. Insulin-dependent diabetes mellitus. Pediatr Rev. 1994;15:137-148. Available at: http://pedsinreview.aappublications.org/cgi/reprint/15/4/137

Rose BD, Post TW. Potassium homeostasis. In: Clinical Physiology of Acid-base and Electrolyte Disorders. 5th ed. New York, NY: McGraw-Hill Medical Publishing Division; 2001:372-375

Rose BD, Post TW. The total body water and the plasma sodium concentration. In: Clinical Physiology of Acid-base and Electrolyte Disorders. 5th ed. New York, NY: McGraw-Hill Medical Publishing Division; 2001:241-243

Page 106: AAP PREP 2009

2009 PREP SA on CD-ROM

page 106

Question: 31

You have just assisted in the delivery of a 38-week gestational age male infant who was born via cesarean section to a 25-year-old woman. As you are completing the infant’s initial physical examination, the father mentions that he and his wife have allergic rhinitis and asthma. He asks whether his son is at increased risk for allergies and how they can reduce the boy’s chance for developing such allergic disorders.

Of the following, the MOST appropriate next step is to

A. explain that because both parents have asthma, breastfeeding will not reduce the risk of eczema

B. explain that breastfeeding or formula choices do not matter now because the mother did not restrict her diet during pregnancy

C. measure the cord blood immunoglobulin E concentration to help establish the newborn’s risk for atopic disorders

D. recommend exclusive breastfeeding for 4 months with the addition of a hypoallergenic formula if needed

E. start the newborn on a cow milk formula for the first month, then switch to strict breastfeeding if he develops eczema

Page 107: AAP PREP 2009

2009 PREP SA on CD-ROM

page 107

Critique: 31 Preferred Response: D

The incidence of atopy (allergic rhinitis, asthma, eczema) has increased significantly over the past few decades. The ability to intervene and either delay or prevent atopic disease in infants born to atopic parents has been the subject of numerous studies. Application of these studies to the population as a whole is difficult because the specific interventions and endpoints for each study often differ. However, one aspect that is agreed on is that atopy risk for infants increases significantly when both parents have a history of atopy (30% to 60%) compared with a history for just one parent (20% to 40%) or neither parent (10% to 15%). Prior to delivery, two prevention strategies have been studied: maternal diet restriction and supplementation with probiotics. Currently, no evidence supports maternal dietary restriction to common allergenic foods. Some studies have supported administration of probiotics (eg, Lactobacillus rhamnosus) to the mother 2 to 4 weeks before delivery and to the infant for 6 months after birth. One study demonstrated a reduction in eczema at 2 years but no reduction in asthma, immunoglobulin (Ig) E concentrations, or allergen sensitization. Further, the dose and type of probiotic has differed in various investigations, making generalized recommendations difficult. Even if both parents have atopy, as described in the vignette, breastfeeding or formula choices may affect atopy outcomes for the infant. In "high-risk" newborns (ie, both parents have atopy or one parent and one sibling have atopy), the American Academy of Pediatrics Committee on Nutrition recommends exclusive breastfeeding for at least 4 months, with supplementation of a hypoallergenic formula if needed. Although it is difficult to compare studies because the duration of breastfeeding and atopic outcome (ie, eczema, allergic rhinitis, asthma) differ, breastfeeding for at least 3 months reduces the risk for eczema. The protective benefit becomes more complex when controlling for the specific maternal atopic condition. For "high-risk" infants born to women who choose not to breastfeed, most studies and experts support starting an extensively hydrolyzed formula. Starting a cow or soy milk formula, compared with an extensively hydrolyzed formula, increases the risk for early eczema. Or note, interventions resulting in decreased atopy early in life may not predict later atopic outcomes. Cord IgE concentrations can be used to assess a newborn's risk for atopy, but its measurement currently is not recommended as a routine screening tool. Furthermore, because both parents in the vignette have a history of atopy, the child already is considered "high risk." The ability to predict atopy based on cord IgE concentrations also depends on the cutoff value used. In one study, 80% of newborns whose cord IgE concentrations were greater than 0.9 kU/L subsequently developed atopy by 5 years of age, but the specific IgE value did not correlate with atopy severity.

References:

American Academy of Pediatrics Committee on Nutrition. Hypoallergenic infant formulas. Pediatrics. 2000;106: 346-349. Available at: http://pediatrics.aappublications.org/cgi/content/full/106/2/346

Greer FR, Sicherer SH, Burks AW, Committee on Nutrition and Section on Allergy and Immunology. Effects of early nutritional interventions on the development of atopic disease in infants and children: the role of maternal dietary restriction, breastfeeding, timing of introduction of complementary foods and hydrolyzed formulas. Pediatrics. 2008;121:183-191. Available at: http://pediatrics.aappublications.org/cgi/content/full/121/1/183

Mihrshahi S, Ampon R, Webb K, et al for the CAPS Team. The association between infant feeding practices and subsequent atopy among children with a family history of asthma. Clin Exp Allergy. 2007;37:671-679. Abstract available at: http://www.ncbi.nlm.nih.gov/pubmed/17456214

Page 108: AAP PREP 2009

2009 PREP SA on CD-ROM

page 108

Prescott SL, Björkstön B. Probiotics for the prevention or treatment of allergic disease. J Allergy Clin Immunol. 2007;120:255-262. Abstract available at: http://www.ncbi.nlm.nih.gov/pubmed/17544096

Snijders BEP, Thijs C, Dagnelie PC, et al. Breast-feeding duration and infant atopic manifestations, by maternal allergic status, in the first two years of life (KOALA study). J Pediatr. 2007;151:347-351. Abstract available at: http://www.ncbi.nlm.nih.gov/pubmed/17889066

Page 109: AAP PREP 2009

2009 PREP SA on CD-ROM

page 109

Question: 32

A 2-year-old girl who has a 4-day history of varicella presents to the office with agitation. Her mother reports that she treated the fever, rash, and pruritus with acetaminophen and diphenhydramine regularly, which provided some relief. This morning her daughter seemed more irritable, had a higher fever than yesterday, and "seemed delirious." On physical examination, the agitated and inconsolable child has a temperature of 104.2°F (40.1°C), heart rate of 160 beats/min, respiratory rate of 36 beats/min, and blood pressure of 135/87 mm Hg. Her pupils are dilated and sluggishly reactive. Examination of the skin reveals numerous small, crusted erosions without surrounding erythema. Neurologic examination demonstrates no focal findings, and the patient is not ataxic.

Of the following, the MOST likely explanation for these symptoms is

A. diphenhydramine overdose

B. hypoglycemia

C. intracranial hemorrhage

D. Reye syndrome

E. varicella cerebellitis

Page 110: AAP PREP 2009

2009 PREP SA on CD-ROM

page 110

Critique: 32 Preferred Response: A

The patient described in the vignette is exhibiting symptoms of anticholinergic toxicity. The classic mnemonic "hot as a hare, dry as a bone, blind as a bat, red as a beet, and mad as a hatter" aptly describes a number of the commonly seen signs and symptoms, including hyperpyrexia, dry skin, dilated pupils, flushing, and delirium. In addition, affected patients typically exhibit tachycardia and hypertension. The clinical syndrome caused by anticholinergic toxicity is due to competitive inhibition of acetylcholine binding to the postganglionic parasympathetic muscarinic receptors by the offending agent. Many commonly used medications have anticholinergic properties, including antihistamines, tricyclic antidepressants, antispasmodics, and mydriatics. For this patient, diphenhydramine is the most likely culprit. Treatment of anticholinergic poisoning is primarily supportive, with stabilization of vital functions (the ABCs) of greatest importance, followed by appropriate decontamination (eg, activated charcoal for ingestions, skin cleansing and removal of transdermal delivery devices for dermal absorption). Agitation and seizures may be treated with benzodiazepines. Physostigmine, a cholinergic agent that competitively inhibits acetylcholinesterase, may be more effective for the treatment of severe agitation than benzodiazepines, but significant toxicity risks make its use controversial. Hypoglycemia can cause altered mental status but typically produces bradycardia, hypotension, and coma, rather than agitation. Intracranial hemorrhage can be seen in patients who have varicella-related thrombocytopenia, although affected patients frequently present with hemorrhagic skin lesions (Item C32). In addition, patients who have intracranial hemorrhage typically present with focal neurologic findings and bradycardia with hypertension, if intracranial pressure is increased. The classic presentation of Reye syndrome includes sudden onset of protracted vomiting without fever in association with lethargy that progresses to delirium, seizures, stupor, and coma. Reye syndrome typically occurs 5 to 7 days after the onset of a viral illness, following apparent improvement in the child's initial symptoms. It results from mitochondrial dysfunction leading to liver failure and cerebral edema. Ninety percent of cases have been reported to follow an upper respiratory tract infection; varicella has been associated with 5% of cases. Central nervous system complications of varicella are uncommon and may include cerebellitis, transverse myelitis, peripheral neuritis, and optic neuritis. Cerebellitis is the most common presentation of encephalitis and typically presents on the third to eighth day of illness with ataxia. Meningoencephalitis presents with fever, meningeal signs, seizures, and altered mental status.

References:

Burns JJ Jr. Toxicity, anticholinergic. eMedicine Specialties, Emergency Medicine, Toxicology. 2006. Available at: http://www.emedicine.com/EMERG/topic36.htm

Burns MJ, Linden CH, Graudins A, Brown RM, Fletcher KE. A comparison of physostigmine and benzodiazepines for the treatment of anticholinergic poisoning. Ann Emerg Med. 2000;35:374-381. Abstract available at: http://www.ncbi.nlm.nih.gov/pubmed/10736125

Carey RG, Balisteri WF. Mitochondrial hepatopathies. In: Kleigman RM, Behrman RE, Jenson HB, Stanton BF, eds. Nelson Textbook of Pediatrics. 18th ed. Philadelphia, Pa: Saunders Elsevier; 2007:1696-1697

Gershon AA, LaRussa P. Varicella-zoster virus infections. In: Gershon AA, Hotez PJ, Katz SL, eds. Krugman's Infectious Diseases of Children. 11th ed. Philadelphia, Pa: Mosby; 2004:785-816

Page 111: AAP PREP 2009

2009 PREP SA on CD-ROM

page 111

Su M, Goldman M. Anticholinergic poisoning. UpToDate Online 15.3. 2008. Available for subscription at: http://www.utdol.com/utd/content/topic.do?topicKey=ad_tox/13958&selectedTitle=1~376&source=search_result

Page 112: AAP PREP 2009

2009 PREP SA on CD-ROM

page 112

Critique: 32

Purpura fulminans may follow varicella and is characterized by hemorrhagic necrosis of the skin and disseminated intravascular coagulation. (Courtesy of Bernard Cohen, MD, DermAtlas, www.dermatlas.org)

Page 113: AAP PREP 2009

2009 PREP SA on CD-ROM

page 113

Question: 33

A 7-month-old child presents for a follow-up office visit after undergoing a Kasai procedure for biliary atresia at 6 weeks of age. The mother states that the boy is irritable when his right arm is moved. On physical examination, the infant is jaundiced. You detect tenderness in the anterior radial head. Radiography of the affected region demonstrates metaphyseal fraying (Item Q33) and a fracture.

Of the following, the MOST appropriate laboratory studies to obtain next are

A. calcium and phosphorus measurement and bone densitometry (DEXA scan)

B. calcium and phosphorus measurement and urinary calcium-to-creatinine ratio

C. calcium, phosphorus, and 25-hydroxyvitamin D measurement

D. calcium, phosphorus, and magnesium measurement

E. magnesium, phosphorus, and parathyroid hormone measurement

Page 114: AAP PREP 2009

2009 PREP SA on CD-ROM

page 114

Question: 33

Metaphyseal fraying, cupping, and widening, as described for the infant in the vignette. (Couretsy of R. Schwartz)

Page 115: AAP PREP 2009

2009 PREP SA on CD-ROM

page 115

Critique: 33 Preferred Response: C

Chronic cholestasis due to biliary atresia results in decreased bile flow into the intestine. The absence of intraluminal bile acids, in turn, causes decreased digestion of lipids, leading to fat malabsorption. In addition, absorption of fat-soluble vitamins (A, D, E, and K) is impaired, which may lead to clinical sequelae of fat-soluble vitamin deficiency. Finally, steatorrhea may impair calcium absorption because intraluminal free fatty acids may bind calcium. The clinical presentation of the patient in the vignette strongly suggests the presence of rickets from vitamin D deficiency. Therefore, the most helpful initial laboratory testing is determination of calcium, phosphorus, and 25-hydroxyvitamin D concentrations. The 25-hydroxyvitamin D assay is the best measure of hepatic stores of vitamin D and is a better marker of vitamin D status than either serum vitamin D or 1,25-dihydroxyvitamin D. Although bone density testing, measurement of serum magnesium and parathyroid hormone, and determination of the urinary calcium-to-creatinine ratio may provide useful additional information, they will not help establish the diagnosis of vitamin D-deficient rickets. Rickets is a potentially preventable complication of biliary atresia, but requires monitoring of calcium, phosphorus, and 25-hydroxyvitamin D concentrations two to four times a year. Infants who have biliary atresia routinely receive supplementation with approximately 8,000 IU of ergocalciferol (vitamin D2) daily. This dose of vitamin D is approximately 20 times the recommended dietary allowance for a healthy toddler. If rickets develops or the vitamin D concentration cannot be maintained within the normal range, the patient should receive either calcitriol (1,25-dihydroxyvitamin D3) or intramuscular vitamin D.

References:

Campbell KM, Bezerra JA. Biliary atresia. In: Walker WA, Goulet O, Kleinman RE, Sherman PM, Shneider BL, Sanderson IR, eds. Pediatric Gastrointestinal Disease. 4th ed. Hamilton, Ontario, Canada: BC Decker; 2004:1122-1138

Suchy FJ. Neonatal cholestasis. Pediatr Rev. 2004;25:388-396. Available at: http://pedsinreview.aappublications.org/cgi/content/full/25/11/388

Page 116: AAP PREP 2009

2009 PREP SA on CD-ROM

page 116

Question: 34

You are examining a 3.5-kg term infant 48 hours after his birth. Results of the physical examination are normal, and you are considering discharging him from the hospital. He is being fed formula from a bottle, and the nurses report intakes of 30 mL every 3 hours. He has wet at least six diapers daily for the past 2 days, but he has not passed any meconium or expressed any stool since birth.

Of the following, the MOST likely diagnosis is

A. ileal atresia

B. imperforate anus

C. meconium ileus

D. meconium plug syndrome

E. neonatal small left colon syndrome

Page 117: AAP PREP 2009

2009 PREP SA on CD-ROM

page 117

Critique: 34 Preferred Response: D

Ninety-five percent of term infants express meconium or pass a stool in the first 24 hours of postnatal life. The infant described in the vignette is term and appropriately grown and has been feeding and voiding well, but he has failed to pass meconium or any stool in the first 2 days of postnatal life. Although he does not have any abdominal distention, emesis, or systemic illness, the pediatric clinician should be concerned about potential bowel, particularly colonic, obstruction. The most likely explanation for the symptoms described for the infant in the vignette is meconium plug syndrome, which typically is an isolated phenomenon that is not associated with anatomic obstruction (eg, atresia). It occurs commonly in term and preterm infants and may be associated with maternal magnesium sulfate treatment for pre-eclampsia/eclampsia. Meconium plug obstruction generally is related to hypomotility. Clinically, there may be no abdominal findings or a gradual increase in girth but no other signs of illness. Plain radiographs of the abdomen generally provide nonspecific findings, but may show a paucity of gas in the rectosigmoid. A contrast enema characteristically illuminates the plugs of meconium and facilitates their evacuation. On occasion, a firm, paraffin-like formed plug may be expressed spontaneously by affected infants during the second postnatal day. Although some infants who have retained meconium may exhibit a small left colon on contrast enema, colonic motility usually is normal upon evacuation of the meconium plug(s). Although meconium plug syndrome is the most common cause of delayed passage of stool, the clinician also should consider Hirschsprung disease, a congenital absence of ganglion cells. A failure to pass meconium in the first 24 hours of postnatal life characterizes 95% of affected infants. The area of affected bowel typically is in the rectosigmoid, where a transition zone may be observed on contrast enema, although this finding is less common in neonates. If Hirschsprung disease is considered, diagnostic rectal biopsy should be performed. The neonatal small left colon syndrome is seen in infants of diabetic mothers and is diagnosed using a contrast enema. The enema may be both diagnostic and therapeutic, as seen in meconium plug syndrome. Gradual feeding and monitoring of the stooling pattern generally results in resolution of the condition over the early weeks of postnatal life. Imperforate anus occurs in about 1 in 4,000 to 5,000 births, and typically is apparent on physical examination. In some cases, a fistulous tract may exist, and the expression of meconium may occur anywhere along the perineal-scrotal-urethral line. Imperforate anus may be an isolated finding or seen in conjunction with other anomalies such as vertebral malformations, cardiac malformation, tracheoesophageal fistula/esophageal atresia, renal anomalies, and limb malformation (VACTERL association). Ileal atresia (proximal or distal) occurs as part of a spectrum of jejunal-ileal bowel atresia that likely reflects a mesenteric vascular defect or interruption in development. Its absolute frequency is not well reported, although it commonly is diagnosed prenatally (dilated bowel, polyhydramnios) on obstetric ultrasonography. With a distal obstruction, the newborn may take early feedings well, but becomes ill, with bile-stained emesis and abdominal distention, in the first 24 to 48 postnatal hours. Plain radiographic findings may include multiple stacked loops of air-filled bowel and air-fluid levels. Meconium ileus is a condition of intestinal obstruction related to thickened, inspissated mucus mixed with meconium that is characteristic of cystic fibrosis and is related to altered chloride and water balance in mucus. The meconium may be beadlike, in small, dense pellets, and even visible on prenatal obstetric ultrasonography. A microcolon may exist distal to the small bowel obstruction. Affected infants may have visible and palpable loops of bowel on examination, in addition to abdominal distention, bilious emesis, and failure to pass meconium in the first 24 to 48 hours of postnatal life. Plain radiographs may reveal a soap-bubble appearance characteristic of meconium stool. A contrast enema may reveal a microcolon and failure to see contrast reflux past the ileocecal valve.

Page 118: AAP PREP 2009

2009 PREP SA on CD-ROM

page 118

References:

Albanese CT, Sylvester KG. Pediatric surgery. In: Doherty GM, Way LW, eds. Current Surgical Diagnosis and Treatment. 12th ed. New York, NY: The McGraw-Hill Companies, Inc; 2006:chap 45

Burge D, Drewett M. Meconium plug obstruction. Pediatr Surg Int. 2004;20:108-110. Abstract available at: http://www.ncbi.nlm.nih.gov/pubmed/14760494

Casaccia G, Trucchi A, Spirydakis I, et al. Congenital intestinal anomalies, neonatal short bowel syndrome, and prenatal/neonatal counseling. J Pediatr Surg. 2006;41:804-807. Abstract available at: http://www.ncbi.nlm.nih.gov/pubmed/16567197

Hajivassiliou CA. Intestinal obstruction in neonatal/pediatric surgery. Semin Pediatr Surg. 2003;12:241-253. Abstract available at: http://www.ncbi.nlm.nih.gov/pubmed/14655163

Magnuson DK, Parry RL, Chwals WJ. Selected abdominal gastrointestinal anomalies. In: Martin RJ, Fanaroff AA, Walsh MC, eds: Fanaroff and Martin's Neonatal-Perinatal Medicine. 8th ed. Philadelphia, Pa: Mosby Elsevier; 2006:1381-1402

Nurko S. Motility of the colon and anorectum. NeoReviews. 2006;7:e34-e48. Available for subscription at: http://neoreviews.aappublications.org/cgi/content/full/7/1/e34

Sutton TL. Index of suspicion in the nursery. NeoReviews. 2006;7:e269-e271. Available for subscription at: http://neoreviews.aappublications.org/cgi/content/full/7/5/e269

Thilo EH, Rosenberg AA. The newborn infant. In: Hay WW Jr, Levin M, Sondheimer JM, Deterding RR, eds. Current Pediatric Diagnosis & Treatment. 18th ed. New York, NY: The McGraw-Hill Companies, Inc; 2007:chap 1

Page 119: AAP PREP 2009

2009 PREP SA on CD-ROM

page 119

Question: 35

A 4-year-old boy who recently emigrated from Central America is brought to your clinic because of 2 weeks of colicky abdominal pain that recently has worsened. His vital signs are normal, and he is afebrile. Physical examination reveals mild diffuse tenderness, but there is no rebound or guarding. After your examination, he has an episode of vomiting. Examination of the vomitus reveals long, slim objects that resemble worms (Item Q35).

Of the following, the BEST treatment choice is

A. albendazole

B. iodoquinol

C. metronidazole

D. praziquantel

E. voriconazole

Page 120: AAP PREP 2009

2009 PREP SA on CD-ROM

page 120

Question: 35

(Courtesy of M. Rimsza)

Page 121: AAP PREP 2009

2009 PREP SA on CD-ROM

page 121

Critique: 35 Preferred Response: A

Ascariasis is caused by infestation with the roundworm Ascaris lumbricoides. It occurs most commonly in tropical regions and areas that have poor sanitation, although many cases also occur in the United States each year. Adult worms (Item C35A) live in the small intestine and produce eggs (Item C35B) that are excreted in the stool into the soil. Infestation occurs when the eggs in the contaminated soil are ingested. Larvae pass from the small intestine into the bloodstream, traveling to the liver and lungs. They migrate from the lung to the pharynx, where they are swallowed. The worms mature in the small intestine, where they produce their eggs, completing the cycle. Many patients who have ascariasis are asymptomatic, but symptoms such as nonspecific gastrointestinal complaints and abdominal pain can occur, as described for the boy in the vignette. Intestinal obstruction and symptoms related to the migration of the larvae, such as obstructive jaundice and peritonitis, also may be seen. Adult worms may pass from the rectum, nose, or mouth if there is heavy worm burden. The diagnosis is made by seeing either ova on microscopic stool examination or the adult worm itself, as described for the boy in the vignette. Treatment of ascariasis that is not associated with intestinal obstruction consists of a single dose of either albendazole or pyrantel pamoate or a 3-day course of mebendazole. Intestinal obstruction may require treatment with piperazine citrate solution or, rarely, surgical intervention. Iodoquinol is a luminal amebicide that is effective for the treatment of asymptomatic amebic cyst excreters, and metronidazole is the drug of choice for patients who have symptomatic amebiasis as well as giardiasis. Neither is appropriate for the treatment of ascariasis. Praziquantel is useful for the treatment of liver fluke infestations, schistosomiasis, and tapeworm infestations. Voriconazole is an intravenous or oral medication used in the treatment of fungal infections.

References:

American Academy of Pediatrics. Amebiasis. In: Pickering LK, Baker CJ, Long SS, McMillan JA, eds. Red Book: 2006 Report of the Committee on Infectious Diseases. 27th ed. Elk Grove Village, Ill: American Academy of Pediatrics; 2006:204-208

American Academy of Pediatrics. Ascaris lumbricoides infections. In: Pickering LK, Baker CJ, Long SS, McMillan JA, eds. Red Book: 2006 Report of the Committee on Infectious Diseases. 27th ed. Elk Grove Village, Ill: American Academy of Pediatrics; 2006:218-219

Dent AE, Kazura JW. Ascariasis (Ascaris lumbricoides). In: Kliegman RM, Behrman RE, Jenson HB, Stanton BF, eds. Nelson Textbook of Pediatrics. 18th ed. Philadelphia, Pa: Saunders Elsevier; 2007:1495

Page 122: AAP PREP 2009

2009 PREP SA on CD-ROM

page 122

Critique: 35

Ascaris sp causes the most common human roundworm infection. Adult worms may reach 30 cm in length. (Courtesy of M. Rimsza)

Page 123: AAP PREP 2009

2009 PREP SA on CD-ROM

page 123

Critique: 35

Fertilized Ascaris lumbricoides eggs are rounded and have a thick shell. (Courtesy of the Centers for Disease Control and Prevention, Public Health Image Library, M Melvin)

Page 124: AAP PREP 2009

2009 PREP SA on CD-ROM

page 124

Question: 36

An 8-month-old boy who has Down syndrome and a large ventriculoseptal defect has had recurrent otitis media and sinusitis during the respiratory virus season that required four separate courses of antibiotics in 4 months. At today’s visit, his mother states that his rhinitis and otitis media symptoms have resolved, but she is concerned about a recurrent diaper rash that is unresponsive to both barrier creams and repeated use of the nystatin cream prescribed last month. Examination reveals white plaques (Item Q36A) on the buccal mucosa just inside the lips and a diaper rash (Item Q36B).

Of the following, the BEST therapeutic option for this child is

A. oral fluconazole

B. oral griseofulvin

C. oral itraconazole

D. topical clotrimazole alone

E. topical clotrimazole combined with triamcinolone

Page 125: AAP PREP 2009

2009 PREP SA on CD-ROM

page 125

Question: 36

White plaques, as described for the infant in the vignette. (Courtesy of D. Krowchuk)

Page 126: AAP PREP 2009

2009 PREP SA on CD-ROM

page 126

Question: 36

(Reprinted with permission from Krowchuk DP, Mancini AJ, eds. Pediatric Dermatology. A Quick Reference Guide. Elk Grove Village, Ill: American Academy of Pediatrics; 2007)

Page 127: AAP PREP 2009

2009 PREP SA on CD-ROM

page 127

Critique: 36 Preferred Response: A

The child described in the vignette has recurrent Candida diaper dermatitis and thrush that is probably due to his four courses of antibiotics. Candida infections are a common problem in infants, toddlers, and children who have underlying medical conditions that require frequent administration of antibiotics. The administration of antibiotics disrupts normal skin and mucous membrane microbiologic flora. The boy described in the vignette has been prescribed topical antifungal preparations without sustained improvement. Prescribing alternative topical antifungal preparations such as clotrimazole for the diaper rash might treat Candida sp that are resistant to nystatin, but it will not treat thrush. Once-daily oral fluconazole has been proven superior to oral nystatin for resistant thrush and effective for Candida diaper dermatitis. Recurrence rates are similar for both fluconazole and nystatin. Oral itraconazole is another effective antifungal, but it may be associated with more gastrointestinal adverse effects, including hepatotoxicity, and is dosed twice daily. Griseofulvin is not the drug of choice for Candida infection, although it is useful in fungal hair and nail infections. Combining antifungal agents with steroids may diminish inflammation temporarily, but steroids also delay the resolution of fungal infection by decreasing T-cell response in the skin, resulting in worsening of the dermatitis after a few days. Further, high systemic absorption of steroids may occur in areas of thin and occluded skin, such as the diaper area.

References:

American Academy of Pediatrics. Candidiasis (moniliasis, thrush). In: Pickering LK, Baker CJ, Long SS, McMillan JA, eds. Red Book: 2006 Report of the Committee on Infectious Diseases. 27th ed. Elk Grove Village, Ill: American Academy of Pediatrics; 2006:242-246

American Academy of Pediatrics. Drugs for invasive and other serious fungal infections in children. In: Pickering LK, Baker CJ, Long SS, McMillan JA, eds. Red Book: 2006 Report of the Committee on Infectious Diseases. 27th ed. Elk Grove Village, Ill: American Academy of Pediatrics; 2006:780

American Academy of Pediatrics. Recommended doses of parenteral and oral antifungal drugs. In: Pickering LK, Baker CJ, Long SS, McMillan JA, eds. Red Book: 2006 Report of the Committee on Infectious Diseases. 27th ed. Elk Grove Village, Ill: American Academy of Pediatrics; 2006:777-779

American Academy of Pediatrics. Topical drugs for superficial fungal infections. In: Pickering LK, Baker CJ, Long SS, McMillan JA, eds. Red Book: 2006 Report of the Committee on Infectious Diseases. 27th ed. Elk Grove Village, Ill: American Academy of Pediatrics; 2006:781-784

Goins RA, Ascher D, Waecker N, Arnold J, Moorefield E. Comparison of fluconazole and nystatin oral suspensions for treatment of oral candidiasis in infants. Pediatr Infect Dis J. 2002;21:1165-1167. Abstract available at: http://www.ncbi.nlm.nih.gov/pubmed/12506950

Pankhurst CL. Antifungal treatment in immunocompetent or immunocompromised infants and children. BMJ Clinical Evidence. 2007. Available for subscription at: http://clinicalevidence.bmj.com/ceweb/conditions/orh/1304/1304_I4.jsp#REF22

Weisse ME, Aronoff SC. Candida. In: Kliegman RM, Behrman RE, Jenson HB, Stanton BF, eds. Nelson's Textbook of Pediatrics. 18th ed. Philadelphia, Pa: Saunders Elsevier; 2007:1207-1310

Page 128: AAP PREP 2009

2009 PREP SA on CD-ROM

page 128

Question: 37

You are called to the newborn nursery to evaluate a 2-hour-old male who was born at term. The pregnancy was uncomplicated, but meconium staining was noted at delivery. The baby weighs 3.8 kg, is afebrile, and has a heart rate of 165 beats/min and a respiratory rate of 70 breaths/min. You note tachypnea and hyperpnea with clear breath sounds, no murmurs, and strong distal pulses. His oxygen saturation in room air is 68%. You place a nonrebreather mask to deliver an Fio2 of 1.0. After 5 minutes, the oxygen saturation is 72%.

Of the following, the BEST explanation for the findings of the hyperoxia test is

A. meconium aspiration syndrome

B. persistent pulmonary hypertension of the newborn

C. pneumonia

D. retained fetal lung liquid syndrome

E. transposition of the great arteries

Page 129: AAP PREP 2009

2009 PREP SA on CD-ROM

page 129

Critique: 37 Preferred Response: E

Most commonly, the practitioner is alerted to hypoxemia in the newborn by the finding of a low oxygen saturation value. Among the various causes of abnormal oxygenation in the newborn are pulmonary pathologies, congenital cardiovascular malformations, persistent pulmonary hypertension of the newborn, and disturbances of the hematologic and metabolic systems. Right-to-left shunting can be thought of as a diversion of desaturated blood away from the lungs and to the systemic circulation. This can occur because blood does not perfuse the ventilated portions of the lung (intrapulmonary right-to-left shunting). Intrapulmonary shunting resulting from infection such as pneumonia, pneumothorax, retained fetal lung liquid, and pulmonary prematurity is the most frequently encountered reason for desaturation in a newborn. Conversely, abnormal oxygenation can result from the situations in which the desaturated blood does not perfuse the pulmonary artery from the heart (intracardiac right-to-left shunting) or is diverted from the pulmonary circuit through the ductus arteriosus (extracardiac right-to-left shunting). Examples of these pathologies include pulmonary atresia, transposition of the great arteries, tricuspid atresia, and pulmonary hypertension. Whether the cause of the desaturation is intrapulmonary, intracardiac, or extracardiac right-to-left shunting, cyanosis (blue, maroon, or purple discoloration of the skin) is likely to be present. Typically, clinicians discern cyanosis in patients who have oxygen saturations of less than 85%, although it may be apparent to some when the saturation is 90% or less. If cyanosis is suspected in the newborn, cyanotic heart disease must be considered. It is reasonable to place the patient in a high-oxygen atmosphere (near FiO2 of 1.0) to determine if the high-dose oxygen can overcome the shunting. If the degree of cyanosis improves and oxygen saturations become normal, the problem likely is intrapulmonary shunting rather than cyanotic heart disease. If hyperoxia does not lead to increased oxygen saturation and higher PaO2 (=150 torr), cyanotic congenital heart disease should be considered and the infant should undergo further cardiac evaluation. The newborn described in the vignette has cyanosis but no murmurs. His tachypnea and hyperpnea (deep breathing) represent the physiologic response to hypoxemia. His saturation improves slightly with the delivery of high-dose oxygen. Transposition of the great arteries is the best explanation for the infant's hypoxemia (Item C37). No amount of oxygen delivered to the patient's alveoli can improve oxygenation of the pulmonary blood flow because the pulmonary blood flow in transposition already is well saturated. The neonate remains desaturated until oxygenated blood from the left atrium adequately crosses the atrial septum to be delivered to the system through the aorta. In contrast, the oxygenation defect associated with meconium aspiration syndrome, persistent pulmonary hypertension of the newborn, pneumonia, and retained fetal lung liquid syndrome is improved with high-dose oxygen delivery to the alveoli.

References:

Driscoll D, Allen HD, Atkins DL, et al. Guidelines for evaluation and management of common congenital cardiac problems in infants, children, and adolescents. A statement for healthcare professionals from the Committee on Congenital Cardiac Defects of the Council on Cardiovascular Disease in the Young, American Heart Association. Circulation. 1994;90:2180-2188. Available at: http://circ.ahajournals.org/cgi/reprint/90/4/2180

Ranjit MS. Common congenital cyanotic heart defects--diagnosis and management. J Indian Med Assoc. 2003;101:71-72, 74. Abstract available at: http://www.ncbi.nlm.nih.gov/pubmed/12841486

Silberbach M, Hannon D. Presentation of congenital heart disease in the neonate and young infant. Pediatr Rev. 2007;28:123-131. Available at:

Page 130: AAP PREP 2009

2009 PREP SA on CD-ROM

page 130

http://pedsinreview.aappublications.org/cgi/content/full/28/4/123

Page 131: AAP PREP 2009

2009 PREP SA on CD-ROM

page 131

Critique: 37

Transposition of the great arteries: Computed tomography angiography of the heart with three-dimensional reconstruction shows the aorta (Ao) arising from the right ventricle (RV) and the pulmonary artery (PA) arising from the left ventricle (LV). (Courtesy of D. Mulvihill)

Page 132: AAP PREP 2009

2009 PREP SA on CD-ROM

page 132

Question: 38

The mother of a 10-month-old child who has mild hypotonia brings him to the office after he has an unprovoked seizure. On physical examination, you note several hypopigmented macules on the trunk (Item Q38A). Magnetic resonance imaging of the brain reveals several thickened areas of cerebral cortex (Item Q38B), with abnormal signal and abnormalities along the walls of the lateral ventricles (Item Q38C).

Of the following, the MOST likely diagnosis is

A. incontinentia pigmenti

B. neurofibromatosis type 1

C. Sturge-Weber syndrome

D. tuberous sclerosis

E. von Hippel-Lindau syndrome

Page 133: AAP PREP 2009

2009 PREP SA on CD-ROM

page 133

Question: 38

(Courtesy of D. Krowchuk)

Page 134: AAP PREP 2009

2009 PREP SA on CD-ROM

page 134

Question: 38

T2-weighted axial magnetic resonance imaging reveals dysplastic cortical gyri (arrows), as described for the infant in the vignette. (Courtesy of D. Krowchuk)

Page 135: AAP PREP 2009

2009 PREP SA on CD-ROM

page 135

Question: 38

Axial magnetic resonance imaging demonstrates lesions along the walls of the lateral ventricles (arrows), as described for the infant in the vignette. (Courtesy of D. Krowchuk)

Page 136: AAP PREP 2009

2009 PREP SA on CD-ROM

page 136

Critique: 38 Preferred Response: D

Low tone and seizures are relatively common neurologic problems. Low tone is a nonspecific finding that may be due to disease in the central or peripheral nervous system, but the occurrence of a seizure suggests a central cerebral cause. Neuroimaging with brain magnetic resonance imaging (MRI) generally is recommended for any infant who has a seizure because congenital brain malformations are more common at this age. The combination of such cerebral symptoms and pigmentary abnormalities of the skin are an indication to obtain brain MRI to assess for a possible neurocutaneous disorder. The seizures, hypotonia, hypopigmented macules, and MRI findings described for the child in the vignette are most suggestive of tuberous sclerosis complex (TSC) (Item C38A). Incontinentia pigmenti is characterized by spasticity rather than hypotonicity, and skin findings include swirled hyperpigmentation following the line Blaschko (Item C38B). Skin findings associated with neurofibromatosis type 1 include café au lait macules (Item C38C), axillary freckling, and iris Lisch nodules (Item C38D). Although Sturge-Weber syndrome can be associated with seizures in the first postnatal year, the primary skin finding is a port wine stain (Item C38E). Neither seizures nor skin abnormalities are seen with von Hippel-Lindau syndrome. Initial management of this patient centers on the chief complaint, which is the seizure. In most cases, no treatment is recommended in a child after a first unprovoked seizure. However, the recurrence risk is much greater in TSC. Subsequently, diagnostic assessment should be directed toward confirming whether this child has TSC. In many cases, this is a clinical diagnosis based on the characteristic findings of skin examination and the cerebral complications. However, many other organs may be involved at presentation or during the child's lifetime, including the eyes, kidneys, lungs, and heart. In the future, this child is at risk for developmental learning difficulties, behavior problems that can include features of autistic spectrum disorders, and malignancies. Given the autosomal dominant inheritance, proper management involves assessment of the parents and genetic counseling. Commercial genetic testing is available and is helpful in cases where clinical features, particularly early in the disease, do not confirm the diagnosis. Genetic testing may have false-negative results due to mosaicism, ie, some organs may be affected due to TSC1 or TSC2 mutations that are not present in blood. Given the complexity of this diagnosis, many experts recommend that children who have TSC be cared for in multidisciplinary specialty clinics.

References:

Ferner RE. Neurofibromatosis 1 and neurofibromatosis 2: a twenty first century perspective. Lancet Neurol. 2007;6:340-351. Abstract available at: http://www.ncbi.nlm.nih.gov/pubmed/17362838

Haslam RHA. Neurocutaneous syndromes. In: Kliegman RM, Behrman RE, Jenson HB, Stanton BF, eds. Nelson Textbook of Pediatrics. 18th ed. Philadelphia, Pa: Saunders Elsevier; 2007:2483-2488

Kandt RS. Tuberous sclerosis complex. In: Singer HS, Kossoff EH, Hartman AL, Crawford TO, eds. Treatment of Pediatric Neurologic Disorders. Boca Raton, Fla: Taylor & Francis; 2005:553-560

Northrup H, Au K-S. Tuberous sclerosis complex. GeneReviews. 2005. Available at: http://www.ncbi.nlm.nih.gov/bookshelf/br.fcgi?book=gene&part=tuberous-sclerosis

Page 137: AAP PREP 2009

2009 PREP SA on CD-ROM

page 137

Critique: 38

Page 138: AAP PREP 2009

2009 PREP SA on CD-ROM

page 138

Critique: 38

Swirled hyperpigmentation following the lines of Blaschko is characteristic of the third stage of incontinentia pigmenti. (Courtesy of D. Krowchuk)

Page 139: AAP PREP 2009

2009 PREP SA on CD-ROM

page 139

Critique: 38

Café au lait macules are a feature of neurofibromatosis type 1. (Courtesy of D. Krowchuk)

Page 140: AAP PREP 2009

2009 PREP SA on CD-ROM

page 140

Critique: 38

Lisch nodules (iris hamartomas [arrow]) are one of the diagnostic criteria for neurofibromatosis type 1. (Courtesy of Wake Forest University Eye Center)

Page 141: AAP PREP 2009

2009 PREP SA on CD-ROM

page 141

Critique: 38

A port wine stain involving the distribution of the first and second branches of the trigeminal nerve is observed in Sturge-Weber syndrome. (Courtesy of M. Rimsza)

Page 142: AAP PREP 2009

2009 PREP SA on CD-ROM

page 142

Question: 39

You are called to the emergency department to evaluate a 5-month-old boy who has new-onset seizures. On physical examination, you note that he is thin and has marked hepatomegaly. The mother tells you that he has been irritable the past several mornings when he awakened from a full night’s sleep. This morning, she found him seizing in his crib and called 911. Laboratory tests performed on specimens taken prior to starting intravenous fluids reveal hypoglycemia, lactic acidosis, hyperuricemia, and hyperlipidemia. You suspect a diagnosis of glycogen storage disease.

Of the following, the MOST appropriate long-term management of this disorder includes

A. coenzyme Q10 administration

B. oral administration of cornstarch

C. oral carnitine supplementation

D. protein restriction

E. restriction of long-chain fats

Page 143: AAP PREP 2009

2009 PREP SA on CD-ROM

page 143

Critique: 39 Preferred Response: B

The hepatomegaly, severe fasting hypoglycemia, lactic acidosis, hyperuricemia, hyperlipidemia, and ketonuria described for the thin child in the vignette are most consistent with glycogen storage disease type I (GSD I) (von Gierke disease). GSD I is an autosomal recessive disorder resulting from deficiency of the enzyme glucose-6-phosphatase, and it is the most serious of all the hepatic glycogenoses. The laboratory findings result from complete blockage of the release of glycogen. Affected children typically have massive hepatomegaly without splenomegaly on physical examination, and they may have a wasted appearance. Kidneys are enlarged and may be palpable on examination. Parents may give a history of irritability and pallor, especially prior to feedings (after fasting). Some of the children develop seizures. The mainstays of treatment for GSD I are the avoidance of fasting and frequent administration of free glucose. The approaches that have been most successful include continuous nocturnal nasogastric or gastrostomy feedings or administration of uncooked cornstarch every 4 hours during sleep or other times of fasting. Maintenance of euglycemia reverses clinical and biochemical abnormalities in most patients. Coenzyme Q often is administered to individuals who have mitochondrial disorders and is of unclear benefit, but it plays no role in the management of GSD I. Similarly, carnitine supplementation and protein and long-chain fat restrictions are of no benefit in GSD I. The management of disorders of carbohydrate metabolism, regardless of their cause, is aimed at ensuring the availability of energy for cellular metabolism without compromising necessary fat and protein stores. This requires frequent delivery of carbohydrates, and gastrostomy tube placement or venous access may be necessary to ensure success.

References:

Hoffmann GF, Nyhan WL, Zschocke J, Kahler SG, Mayatepek E. Approach to the patient with hepatic disease. In: Inherited Metabolic Diseases. Philadelphia, Pa: Lippincott Williams & Wilkins; 2002:191-214

Nyhan WL, Barshop BA, Ozand PT. Disorders of carbohydrate metabolism. In: Atlas of Metabolic Diseases. 2nd ed. London, England: Hodder Arnold; 2005:371-402

Page 144: AAP PREP 2009

2009 PREP SA on CD-ROM

page 144

Question: 40

A 13-year-old girl comes to your office because her menstrual periods are irregular. She attained menarche at 12 years of age and states that she has had only four menstrual periods over the past year. The periods last for 5 to 7 days and require the use of four pads per day. She has never been sexually active. She plays no sports, but she swims in the summer for fun. On physical examination, her weight and height are at the 50th percentile for age. She has minimal facial acne and no hirsutism or other skin lesions. Her breast and genital development is at Sexual Maturity Rating 5.

Of the following, the MOST appropriate management strategy for this patient is to

A. measure follicle-stimulating hormone, luteinizing hormone, and prolactin

B. measure free thyroxine and thyroid-stimulating hormone

C. monitor menses and reassure the girl

D. obtain a bone age by radiography

E. obtain pelvic ultrasonography

Page 145: AAP PREP 2009

2009 PREP SA on CD-ROM

page 145

Critique: 40 Preferred Response: C

Young women of low gynecologic age (ie, a few years after the onset of menstruation) often have anovulatory cycles due to immaturity of the hormonal feedback system of the hypothalamic-pituitary-ovarian axis. During the adolescent years, when ovulation does not occur with every cycle, both frequent and infrequent menstruation can result. By 2 years after menarche, 55% to 82% of cycles are ovulatory, and by 5 years after menarche, 80% to 90% are ovulatory, resulting in more regular menses. The girl described in the vignette has no signs of androgen excess or evidence of weight loss that may be associated with an eating disorder but does have scant irregular periods without cramps or heavy prolonged menses. Because these findings suggest physiologic anovulatory cycles, no further laboratory studies, such as measurement of follicle-stimulating hormone, luteinizing hormone, and prolactin, are necessary. Her continued normal linear growth obviates the need for a bone age measurement. The girl's normal growth coupled with no evidence of fatigue or skin dryness make thyroid disease unlikely. Pelvic ultrasonography is not indicated in the absence of signs of androgen excess or other symptoms suggestive of ovarian pathology. The possibility of pregnancy always should be assessed in the presence of amenorrhea.

References:

Emans SJ. Amenorrhea in the adolescent. In: Emans SJH, Laufer MR, Goldstein DP, eds. Pediatric and Adolescent Gynecology. 5th ed. Philadelphia, Pa: Lippincott, Williams & Wilkins; 2005:214-269

Ohlemeyer CL. Menstrual disorders. In Osborn LM, DeWitt TG, First LR, Zenel JA eds. Pediatrics. Philadelphia, Pa: Elsevier Mosby; 2005:1455-1460

Page 146: AAP PREP 2009

2009 PREP SA on CD-ROM

page 146

Question: 41

You are evaluating a 20-month-old boy who has a rectal temperature of 106°F (41.1°C) and a history of coughing. His mother reports that the child has had a decrease in activity and eating over the past 2 days. On physical examination, the boy appears moderately ill but is alert and easily interacts with you. He occasionally grunts, has a heart rate of 140 beats/min, and has a respiratory rate of 55 breaths/min. His neck is supple, he is circumcised, and he has no evidence of otitis media.

Of the following, the BEST initial test in the evaluation of this child is

A. chest radiography

B. C-reactive protein measurement

C. erythrocyte sedimentation rate

D. lumbar puncture

E. urinalysis

Page 147: AAP PREP 2009

2009 PREP SA on CD-ROM

page 147

Critique: 41 Preferred Response: A

Physical examination, a detailed history, and direct observation can help the clinician recognize up to 90% of children who have a serious illness. Carefully selected laboratory and diagnostic tests can enhance the detection of invasive bacterial infections in children who have fever. Infants younger than 3 months of age who present with a fever are at high risk of invasive bacterial infection due to their still-developing immune system. Specific high-risk factors, including a history of prematurity, evidence of an abnormally elevated or depressed white blood cell count, a focal source of infection such as otitis media or soft-tissue infection, or abnormal chest findings on radiography are associated with serious bacterial infection in almost 25% of cases. Infants younger than 3 months of age who do not have such high-risk factors still have a nearly 3% incidence of a serious bacterial infection. Infants between the ages of 3 months and 3 years remain at risk for serious bacterial infections, with reports of a 3% incidence of bacteremia associated with a temperature of 102.2°F (39.0°C) and 7% with a temperature of 104.0°F (40.0°C). A temperature higher than 105.8°F (41.0°C) frequently is associated with invasive bacterial infection. The child described in the vignette has both an elevated temperature and signs and symptoms suggestive of a primary respiratory infection that make chest radiography the best initial diagnostic test. A lumbar puncture can be deferred because of the patient's age and appropriate interaction with the examiner. The patient's age, sex, and circumcised status make a urinary tract infection unlikely; as a result, urinalysis also could be deferred pending results of chest radiography. Positive urinalysis can be suggestive of a urinary tract infection, but a urine culture remains the definitive test when a urinary tract infection is suspected. Elevated erythrocyte sedimentation rates and C-reactive protein measurements are indicative of the presence of acute inflammatory processes, but they are nonspecific findings. Their elevation in young children who have fever may be useful in guiding the clinician toward additional studies in selected children.

References:

Brook I. Unexplained fever in young children: how to manage severe bacterial infection. BMJ. 2003;327:1094-1097. Available at: http://www.bmj.com/cgi/content/full/327/7423/1094

McCarthy PL. Evaluation of the sick child in the office and clinic. In: Kliegman RM, Behrman RE, Jenson HB, Stanton BF, eds. Nelson Textbook of Pediatrics. 18th ed. Philadelphia, Pa: Saunders Elsevier; 2007:363-365

Page 148: AAP PREP 2009

2009 PREP SA on CD-ROM

page 148

Question: 42

You are called to the emergency department to see an 8-year-old girl in whom congenital adrenal hyperplasia was diagnosed at birth. She is being treated with oral mineralocorticoid daily (9-alpha-fludrocortisone 0.1 mg) and hydrocortisone 5 mg orally every 8 hours. She is febrile (temperature of 102.0°F [38.9°C]) and has vomited twice. According to her mother, other family members recently recovered from a gastrointestinal illness that started with fever and vomiting.

Of the following, the MOST appropriate treatment for this child is to

A. administer usual oral medication

B. administer parenteral hydrocortisone hemisuccinate

C. administer parenteral mineralocorticoid

D. double the usual oral dose of mineralocorticoid

E. triple the usual oral dose of hydrocortisone

Page 149: AAP PREP 2009

2009 PREP SA on CD-ROM

page 149

Critique: 42 Preferred Response: B

The girl described in the vignette has adrenal insufficiency and is vomiting and febrile. She needs stress doses of glucocorticoids but cannot keep down orally administered agents. No parenteral preparation of mineralocorticoid is available. In contrast to synthetic glucocorticoids, hydrocortisone has about 1% of the mineralocorticoid effect of aldosterone (the most important natural mineralocorticoid), and a large dose of hydrocortisone can act as a mineralocorticoid. Accordingly, this child needs a rapid-acting parenteral hydrocortisone preparation. Hydrocortisone hemisuccinate administered subcutaneously, unless she has poor perfusion, which would necessitate intramuscular or intravenous administration, can stabilize her course. The girl's usual oral medication is not sufficient for the increased need of stress. Increasing the oral medication may not be sufficient because she probably will not be able to keep it down or absorb it if she is vomiting. Mineralocorticoid need generally does not increase during stress. If she is vomiting and receiving parenteral hydrocortisone, she might require intravenous 0.9% saline with glucose to maintain her sodium stores and her blood glucose in the normal range. For children who have usual febrile illnesses or require surgery, administration of glucocorticoid in doses that supply three to six times the usual cortisol secretion rate of 3 to 7 mg/M2 per day is reasonable treatment. However, children who have adrenal insufficiency and develop varicella because of failure or lack of immunization must be managed very carefully. There are no controlled studies, but some anecdotal data suggest that infection with varicella virus can become devastating in glucocorticoid-treated individuals. For such children, careful titration of hydrocortisone dose to only twice the usual need may provide the small amount of needed additional glucocorticoid without compromising the children's immune responses.

References:

Donohoue PA. Treatment of adrenal insufficiency in children. UptoDate Online 15.3. 2008. Available for subscription at: http://www.uptodateonline.com/utd/content/topic.do?topicKey=pediendo/19876

Shulman DI, Palmert MR, Kemp SF, for the Lawson Wilkins Drug and Therapeutics Committee. Adrenal insufficiency: still a cause of morbidity and death in childhood. Pediatrics. 2007;119:e484-e494. Available at: http://pediatrics.aappublications.org/cgi/content/full/119/2/e484

Wilson TA, Speiser P. Adrenal insufficiency. eMedicine Specialties, Pediatrics: General Medicine, Endocrinology. 2007. Available at: http://www.emedicine.com/ped/TOPIC47.HTM

Page 150: AAP PREP 2009

2009 PREP SA on CD-ROM

page 150

Question: 43

A 9-year-old girl has been evaluated by a learning consultant and found to have a slow reading rate, weakness in short-term memory, and problems with reading comprehension. Her parents ask you what subjects other than reading will be most challenging for her due to these learning difficulties.

Of the following, the subject that this child should find MOST challenging is

A. art

B. creative writing

C. mathematics

D. music

E. social studies

Page 151: AAP PREP 2009

2009 PREP SA on CD-ROM

page 151

Critique: 43 Preferred Response: E

A student who has a slow reading rate, reduced reading comprehension, and impaired short-term memory, such as the girl described in the vignette, will encounter problems in "content" classes, which include subjects such as science, history, and social studies. When children read texts in these subjects, they need to read factual information and use the material in the text to obtain knowledge about the subject. This requires comprehension of the text, which involves identifying and understanding the words. To determine the meaning of a word, a reader first must decode and identify the word on the page. A slow reader takes much longer to complete assignments and test questions. Further, individuals who have weakness in understanding and remembering the text will have much more difficulty with homework assignments and on examinations. The girl in the vignette will not have the same difficulty with art, music, creative writing, or mathematics because these subjects do not demand the same emphasis on reading a text and recalling facts.

References:

Fletcher JM, Lyon GR, Fuchs LS, Barnes MA. Reading disabilities: comprehension. In: Learning Disabilities: From Identification to Intervention. New York, NY: The Guilford Press; 2007:184-206

Shaywitz SE, Shaywitz BA. Dyslexia (specific reading disability). Pediatr Rev. 2003;24:147-153. Available at: http://pedsinreview.aappublications.org/cgi/content/full/24/5/147

Page 152: AAP PREP 2009

2009 PREP SA on CD-ROM

page 152

Question: 44

As you are leaving the supermarket, the cashier tells you that she is worried because her child recently had a positive tuberculin skin test. She had to take him to the health department for skin testing because he had been in contact with her father, who recently was diagnosed with active pulmonary tuberculosis. They told her that the boy’s skin test was positive at "25," but his chest radiograph was normal. She is concerned because the doctor told her that the case is a little unusual because of the type of tuberculosis her father has. She asked the physician at the health department to write it down, and she hands you a piece of paper that says "INH resistant." The mother asks you what type of medication her boy should receive.

Of the following, the MOST appropriate antituberculous agent to prescribe for this boy is

A. ciprofloxacin

B. ethambutol

C. isoniazid

D. pyrazinamide

E. rifampin

Page 153: AAP PREP 2009

2009 PREP SA on CD-ROM

page 153

Critique: 44 Preferred Response: E

With a positive tuberculin skin test, a negative chest radiograph, and no indications of active disease, the patient described in the vignette meets the classification of a latent tuberculosis infection (LTBI). LTBI usually is treated with isoniazid (INH) once daily for 9 months, but when the source case is known to have INH resistance, this agent should not be used. Instead, a 6-month course of rifampin is recommended. An exception to this approach would be if the source case was known to be resistant to both INH and rifampin, in which case a tuberculosis expert should be consulted to determine the best course of treatment. Ciprofloxacin, ethambutol, and pyrazinamide are used in combination with other antituberculous agents for the treatment of active tuberculous disease; they are not indicated for monotherapy in treating LTBI. Although usually very well tolerated, patients who receive rifampin should be aware of possible adverse effects. Urine, tears, and saliva change to a reddish-orange color, which may stain clothes or contact lenses. Rifampin therapy also may be associated with mild "flulike" symptoms (eg, myalgias) that resolve with continued therapy. Rifampin also induces cytochrome P-450 activity and, therefore, decreases the half-life of medications such as warfarin, digoxin, thyroxine, oral contraceptives, and some antimicrobial agents (eg, chloramphenicol), making them less effective. When rifampin is used in combination with INH, patients are twice as likely to develop hepatitis as are patients treated with rifampin in combination with other antituberculous medications. Thrombocytopenia and leukopenia both have been associated with rifampin therapy exceeding 1 month's duration.

References:

Alsayyed B, Adam HM. In brief: rifampin. Pediatr Rev. 2004;25:216-217. Available at: http://pedsinreview.aappublications.org/cgi/content/full/25/6/216

American Academy of Pediatrics. Tuberculosis. In: Pickering LK, Baker CJ, Long SS, McMillan JA, eds. Red Book: 2006 Report of the Committee on Infectious Diseases. 27th ed. Elk Grove Village, Ill: American Academy of Pediatrics; 2006:678-698

Bliziotis IA, Ntziora F, Lawrence KR, Falagas ME. Rifampin as adjuvant treatment of Gram-positive bacterial infections: a systemic review of comparative clinical trials. Eur J Clin Microbiol Infect Dis. 2007;26:849-856. Abstract available at: http://www.ncbi.nlm.nih.gov/pubmed/17712583

Page 154: AAP PREP 2009

2009 PREP SA on CD-ROM

page 154

Question: 45

You are speaking to a group of medical students about different antibiotic classes that can be used in the treatment of meningitis. One student asks you about chloramphenicol, a drug with which he is not familiar.

Of the following, the MOST common adverse effect associated with chloramphenicol therapy is

A. dose-related bone marrow suppression

B. drug eruption

C. gray baby syndrome

D. idiosyncratic aplastic anemia

E. optic neuritis

Page 155: AAP PREP 2009

2009 PREP SA on CD-ROM

page 155

Critique: 45 Preferred Response: A

Chloramphenicol is an antibiotic that inhibits protein synthesis by reversibly binding to the 50S subunit of the 70S ribosome. Against most sensitive organisms, chloramphenicol produces a static effect by blocking protein synthesis. However, it is bactericidal against some meningeal pathogens, such as Haemophilus influenzae, Streptococcus pneumoniae, and Neisseria meningitidis. Chloramphenicol is not bactericidal against group B streptococci or enteric gram-negative bacilli because drug concentrations in the cerebrospinal fluid sufficient for effective eradication of these organisms cannot be achieved at therapeutic doses. With the possible exception of typhoid fever in some developing areas of the world where cost and availability make it the primary therapy, chloramphenicol no longer is the drug of choice for any specific infection. Chloramphenicol is extremely active against a variety of organisms, including bacteria, spirochetes, rickettsiae, chlamydiae, and mycoplasmas. Most gram-positive and gram-negative aerobic bacteria are inhibited by concentrations achievable in the serum, and chloramphenicol is one of the most effective antibiotics against anaerobic bacteria, including Bacteroides strains. Salmonellae, including S typhi, generally are susceptible, although chloramphenicol-resistant strains have emerged rapidly since the late 1980s in India, Korea, Vietnam, Peru, Mexico, and Thailand. Plasma concentrations of chloramphenicol vary, depending on the time and route of administration. Oral chloramphenicol is well absorbed from the gastrointestinal tract and produces peak serum concentrations of 12 mcg/mL of active antibiotic after a 1-g dose. The intravenous preparation of the drug produces active chloramphenicol concentrations in the serum that are 70% of those obtained after an oral dose. Intramuscular administration is well tolerated and produces peak serum values that are similar to those seen with intravenous administration. However, for patients who have enteric fever, peak serum concentrations of the drug after intramuscular administration are only one half to two thirds of those obtained by intravenous administration due to delayed drug absorption from the injection site. Metabolism and excretion of chloramphenicol vary widely in the pediatric population and are age- and patient-dependent. Therefore, monitoring of serum concentrations is critical. Significant drug-drug interactions occur between chloramphenicol and other agents. Chloramphenicol prolongs the half-life of tolbutamide, chlorpropamide, phenytoin, cyclophosphamide, and warfarin by inhibiting hepatic microsomal enzymes. Drugs such as phenytoin, rifampin, and phenobarbital decrease the serum concentration and increase the total body clearance of chloramphenicol by inducing hepatic microsomal enzymes. Chloramphenicol may delay the response of anemias to iron, folic acid, and vitamin B12, and in vitro it antagonizes the bactericidal activity of the penicillins, cephalosporins, and aminoglycoside antibiotics. The most common adverse effect associated with chloramphenicol is reversible bone marrow depression that is characterized by a combination of reticulocytopenia, anemia, leucopenia, and thrombocytopenia. Other adverse effects include idiosyncratic aplastic anemia that is rare but may be fatal; hemolytic anemia in patients who have the Mediterranean form of glucose-6-phosphate dehydrogenase deficiency; gray baby syndrome in neonates, which is characterized by abdominal distention, vomiting, flaccidity, cyanosis, circulatory collapse, and death; and optic neuritis that may occur in patients receiving prolonged therapy. Hypersensitivity reactions (eg, rashes, drug fevers, anaphylaxis) are rare.

References:

Kauffman RE, Miceti JN, Strebel L, Buckley JA, Done AK, Dajani AS. Pharmacokinetics of chloramphenicol and chloramphenicol succinate in infants and children. J Pediatr. 1981;98:315-320. Abstract available at: http://www.ncbi.nlm.nih.gov/pubmed/7463235

Page 156: AAP PREP 2009

2009 PREP SA on CD-ROM

page 156

Myers B, Salvatore M. Tetracyclines and chloramphenicol. In: Mandell GL, Bennett JE, Dolin R, eds. Mandell, Douglas and Bennett's Principles and Practice of Infectious Diseases. 6th ed. New York, NY: Elsevier Churchill Livingstone, 2005:356-373

Pickering LK, Hoecker JL, Kramer WG, Kohl S, Cleary TG. Clinical pharmacology of two chloramphenicol preparations in children: sodium succinate (IV) and palmitate (oral) esters. J Pediatr. 1980;96:757-761. Abstract available at: http://www.ncbi.nlm.nih.gov/pubmed/6987361

Rahal JJ Jr, Simberkoff MS. Bactericidal and bacteriostatic action of chloramphenicol against meningeal pathogens. Antimicrob Agents Chemother. 1979;16:13-18. Available at: http://aac.asm.org/cgi/reprint/16/1/13?view=long&pmid=38742

Yunis AA. Chloramphenicol-induced bone marrow suppression. Semin Hematol. 1973;10:225-234

Page 157: AAP PREP 2009

2009 PREP SA on CD-ROM

page 157

Question: 46

A 2-year-old boy presents with fever and abdominal pain. Urinalysis reveals a specific gravity of 1.010, pH of 5.5, 2+ protein, no blood, and negative leukocyte esterase and nitrite tests. Microscopy findings are negative.

Of the following, the MOST appropriate diagnostic test to assess the severity of proteinuria in this child is

A. a random urine sample measurement for quantitative protein and creatinine concentrations

B. measurement of serum albumin concentration and correlation with urine protein concentration measured by dipstick

C. measurement of urine microalbumin excretion

D. sulfosalicylic acid treatment of a freshly voided urine sample

E. 24-hour urine collection for protein and creatinine concentrations

Page 158: AAP PREP 2009

2009 PREP SA on CD-ROM

page 158

Critique: 46 Preferred Response: A

The 2+ protein measurement on a dilute urine sample (urine specific gravity <1.015) described for the boy in the vignette is abnormal and requires quantitation of the proteinuria. Although traditional teaching favored 24-hour urine testing for quantitative assessment of proteinuria in the past, this technique no longer is necessary. Spot/random urine samples for proteinuria correlate closely with 24-hour urine collections and are much easier to collect via a urine bag, especially in the child who is not yet toilet trained. A normal spot urine protein-to-creatinine ratio is less than 0.2. The urine dipstick is a good screening test for proteinuria, but because it is a qualitative test, it is a suboptimal indicator of the severity of proteinuria. Similarly, sulfosalicylic acid treatment of freshly voided urine is a qualitative, not quantitative test. The sulfosalicylic acid induces precipitation of proteins. Urine microalbumin is an ultrasensitive technique (using a radioimmunoassay, enzyme-linked immunoassay, or nephelometry) to detect albumin excretion in the urine that falls below that measured on a urine dipstick. Assessment of microalbuminuria is used commonly in patients who have diabetes as a screening test for renal disease before overt proteinuria occurs. Microalbuminuria screens also have been used in those who have hypertension. Although this technique is quantitative, it has no role when overt proteinuria is present, as measured on a urine dipstick. Measurement of serum albumin should be deferred until the severity of the proteinuria can be quantified. Although 24-hour urine testing for proteinuria no longer is necessary, the test remains useful for evaluating children for risk factors for nephrolithiasis/urolithiasis or to screen for catecholamine metabolites in children who have suspected neural crest tumors (neuroblastoma or pheochromocytoma).

References:

Abitbol C, Zilleruelo G, Freundlich M, Strauss J. Quantitation of proteinuria with urinary protein/creatinine ratios and random testing with dipsticks in nephrotic children. J Pediatr. 1990;116:243-247. Abstract available at: http://www.ncbi.nlm.nih.gov/pubmed/2299494

Chahar OP, Bundella B, Chahar CK, Purohit M. Quantitation of proteinuria by use of single random spot urine collection. J Indian Med Assoc. 1993;91:86-87. Abstract available at: http://www.ncbi.nlm.nih.gov/pubmed/8409488

Gregianin LJ, McGill AC, Pinheiro CM, Brunetto AL. Vanilmandelic acid and homovanillic acid levels in patients with neural crest tumor: 24-hour urine collection versus random sample. Pediatr Hematol Oncol. 1997;14:259-265. Abstract available at: http://www.ncbi.nlm.nih.gov/pubmed/9185210

Pontremoli R, Leoncini G, Ravera M, et al. Microalbuminuria, cardiovascular, and renal risk in primary hypertension. J Am Soc Nephrol. 2002;13:S169-S172. Available at: http://jasn.asnjournals.org/cgi/content/full/13/suppl_3/S169

Page 159: AAP PREP 2009

2009 PREP SA on CD-ROM

page 159

Question: 47

A 12-month-old girl presents with a 3-month history of a pruritic rash that involves her cheeks, neck, anterior trunk, and antecubital and popliteal areas. The rash improves after use of an over-the-counter topical steroid cream but still is present most days, and the infant often wakes up at night scratching. On physical examination, you observe a raised erythematous rash that has areas of lichenification (Item Q47).

Of the following, the MOST helpful intervention is to

A. eliminate fruit and acidic juices from the diet

B. eliminate milk, eggs, soy, and wheat from the diet

C. perform aeroallergen allergy testing

D. perform food allergy testing

E. recommend a skin biopsy

Page 160: AAP PREP 2009

2009 PREP SA on CD-ROM

page 160

Question: 47

(Courtesy of D. Krowchuk)

Page 161: AAP PREP 2009

2009 PREP SA on CD-ROM

page 161

Critique: 47 Preferred Response: D

Some 30% to 40% of infants who have moderate-to-severe atopic dermatitis (AD), such as described for the infant in the vignette, may have an underlying immunoglobulin (Ig) E-mediated food allergy exacerbating the AD. For some infants, food ingestion may result in immediate worsening of AD severity, although most infants do not demonstrate this immediate reaction. Many foods have been implicated in AD, but five (milk, eggs, soy, wheat, and peanut) account for 90% of the causative allergens. Both allergy skin testing and measurement of serum IgE concentrations to these foods can help to identify and eliminate likely triggers. Either a negative IgE blood test (<0.35 kU/L) or a negative skin test for a specific food provides a high negative predictive value. On the other hand, the positive predictive value for a skin or blood test may be only 50%. Although the most commonly implicated foods often are eliminated from the diet, such an approach does not improve symptoms in most (60% to 70%) children because they do not have IgE-mediated AD. The unnecessary elimination of multiple foods can have an adverse effect on nutrition, and food avoidance should be guided by the dietary history, eczema severity, and skin or blood testing. Frequently, children experience perioral rashes after drinking fruit juice. Such rashes typically are nonpruritic, limited to the area of contact, and resolve within a few hours. The mechanism of such rashes is unknown, but children generally outgrow such reactions by age 4 years. In cases involving more widespread cutaneous symptoms, such as described in the vignette, elimination of fruit or acidic juices is unnecessary. Parents often request testing for environmental allergies. House dust mites have been implicated in some cases of AD, although they are less likely a cause for moderate-to-severe atopic dermatitis than food allergies. Climate changes such as cold, dry air or hot, humid weather can worsen AD, but specific seasonal allergens such as oak tree or ragweed are not associated with eczema in infants. A skin biopsy can provide insight into the pathophysiology of chronic rashes or lesions. Generally, skin biopsies neither are advised nor provide insight into the causes of typical AD manifestations in infants, but atypical presentations or lack of expected improvement with appropriate therapy should prompt consideration of a dermatology referral.

References:

Burks W. Skin manifestations of food allergy. Pediatrics. 2003;111:1617-1624. Available at: http://pediatrics.aappublications.org/cgi/content/full/111/6/S2/1617

Greer FR, Sicherer SH, Burks AW, Committee on Nutrition and Section on Allergy and Immunology. Effects of early nutritional interventions on the development of atopic disease in infants and children: the role of maternal dietary restriction, breastfeeding, timing of introduction of complementary foods and hydrolyzed formulas. Pediatrics. 2008;121:183-191. Available at: http://pediatrics.aappublications.org/cgi/content/full/121/1/183

Sampson HA, Leung DYM. Adverse reactions to foods. In: Kleigman RM, Behrman RE, Jenson HB, Stanton BF, eds. Nelson Textbook of Pediatrics. 18th ed. Philadelphia, Pa: Saunders Elsevier; 2007:986-989

Page 162: AAP PREP 2009

2009 PREP SA on CD-ROM

page 162

Question: 48

The mother of a 2-year-old boy calls you because she found her son holding an open bottle of liquid dishwasher detergent. He is crying, drooling profusely, and has vomited three times. In answer to your questions, she reports that he is not sleepy and did not seem to get it in his eyes or on his skin. On examination in your office, you note an ulcer on his lower lip and several ulcers on his tongue.

Of the following, the most appropriate next step in the evaluation and management of this boy is to

A. administer activated charcoal

B. administer ipecac syrup

C. reassure the mother that the product is nontoxic

D. refer the boy to a gastroenterologist for urgent endoscopy

E. refer the boy to the emergency department for gastric lavage

Page 163: AAP PREP 2009

2009 PREP SA on CD-ROM

page 163

Critique: 48 Preferred Response: D

Dishwasher detergent is a highly caustic alkali that can cause significant oropharyngeal, upper airway, and esophageal injury if ingested. Alkaline agents cause deep liquefaction necrosis of the affected tissues, with ulceration and perforation likely complications. Common household products such as oven and drain cleaners, dishwasher detergents, and cleaning products that contain ammonia pose a risk of caustic injury. Substances that have a pH greater than 11 are likely to cause injury, even in small amounts. Children who have ingested caustic or corrosive agents typically present with drooling, dysphagia, or emesis, as reported for the boy in the vignette, although the presence and severity of symptoms is not predictive of esophageal injury. The presence or absence of oral lesions also is not predictive of esophageal injury. The initial evaluation and treatment of patients who have ingested caustic substances includes stabilization of vital functions, pain management, and other supportive measures. Decontamination with activated charcoal is not indicated because it will not adsorb acids or alkalis and is likely to impair endoscopic evaluation. Use of ipecac syrup or gastric lavage is contraindicated because of the potential to cause further injury to the esophagus. In the symptomatic patient, endoscopy should be undertaken within 6 to 24 hours of ingestion to determine the presence of esophageal burns and the degree of severity. Evaluation of the asymptomatic patient is controversial. Because the available data suggest that esophageal burns may be found in as many as 45% of those who do not have oral lesions, endoscopy is recommended in clear cases of ingestion. If the history is questionable and the patient remains asymptomatic, with normal swallowing, after several hours of observation, some authors have suggested that it is safe to forego endoscopy.

References:

Cordero B, Savage RR, Cheng TL. In brief: corrosive ingestions. Pediatr Rev. 2006;27:154-155. Available at: http://pedsinreview.aappublications.org/cgi/content/full/27/4/154

Ferry GD. Caustic esophageal injury in children. UpToDate. 2008. Available for subscription at: http://www.utdol.com/utd/content/topic.do?topicKey=pedigast/11441&view=print

Gaudreault P, Parent M, McGuigan MA, Chicoine L, Lovejoy FH Jr. Predictability of esophageal injury from signs and symptoms: a study of caustic ingestion in 378 children. Pediatrics. 1983;71:767-770. Available at: http://pediatrics.aappublications.org/cgi/content/full/71/5/767

Page 164: AAP PREP 2009

2009 PREP SA on CD-ROM

page 164

Question: 49

A 12-year-old boy has had cholestasis since infancy from Alagille syndrome. He has been lost to medical follow-up for the last several years. He now presents to your office with pain in his right upper thigh after a fall. His thigh is intensely tender, and ultrasonography demonstrates a large hematoma in his quadriceps. The parents state that he has tended to bruise easily in the past few months.

Of the following, the condition MOST likely to account for this patient’s symptoms is

A. factor VIII deficiency

B. idiopathic thrombocytopenic purpura

C. vitamin C deficiency

D. vitamin K deficiency

E. von Willebrand disease

Page 165: AAP PREP 2009

2009 PREP SA on CD-ROM

page 165

Critique: 49 Preferred Response: D

Alagille syndrome is characterized by cardiac disease (especially peripheral pulmonary stenosis), vertebral anomalies, ocular anomalies (posterior embryotoxon), facial dysmorphism (triangular facies, macrocephaly, large ears) (Item C49A), and paucity of the intrahepatic bile ducts. The hepatic manifestations of this syndrome account for much of the medical morbidity. Specifically, impaired bile flow results in chronic cholestasis, which leads to severe pruritus, jaundice, malabsorption of nutrients, and malabsorption of fat-soluble vitamins (Item C49B). Although most affected children have their jaundice improve as they grow older, a subset progresses to cirrhosis and requires liver transplantation. Patients who have hepatic disease must have their nutritional status monitored carefully. Chronic anorexia, recurrent illnesses, and fat malabsorption may result in caloric deficiency and growth failure. Caloric supplementation by nasogastric tube or gastrostomy may be necessary to ensure adequate caloric intake. In addition, patients who have cholestasis are at risk for fat-soluble vitamin deficiency. Vitamin D deficiency typically causes osteopenia and rickets, vitamin E deficiency causes peripheral neuropathy and ataxia, and vitamin A deficiency may cause night blindness or corneal lesions. The bruising described for the patient in the vignette most likely is due to vitamin K deficiency. Vitamin K is a cofactor essential in posttranscriptional carboxylation of the clotting factors II, VII, IX, and X. Thus, vitamin K deficiency leads to prolonged prothrombin and partial thromboplastin time, which predisposes to bruising. Although factor VIII deficiency, vitamin C deficiency, von Willebrand disease, and idiopathic thrombocytopenia purpura also may cause bruising, the patient who has Alagille syndrome is not at increased risk for developing these conditions. In addition to supplementing patients who have chronic liver disease with fat-soluble vitamins, the clinician caring for these patients also must supply adequate calories. Patients who have advanced chronic liver disease may have both anorexia and increased caloric requirements. In addition, patients who have portal hypertension and ascites may need to have total fluid intake restricted, which, in turn, means that they may require a more concentrated and less palatable formula. For these reasons, nasogastric or gastrostomy feedings sometimes are necessary to achieve optimal growth, especially when preparing a patient for liver transplantation.

References:

Cranenburg ECM, Shurgers LJ, Vermeer C. Vitamin K: the coagulation vitamin that became omnipotent. Thromb Haemost. 2007;98:120-125. Available at: http://www.schattauer.de/index.php?id=1268&pii=th07070120&no_cache=1

Kamath BM, Piccoli DA. Heritable disorders of the bile ducts. Gastroenterol Clin North Am. 2003;32:857-875. Abstract available at: http://www.ncbi.nlm.nih.gov/pubmed/14562578

Sokol RJ. Fat-soluble vitamins and their importance in patients with cholestatic liver diseases. Gastroenterol Clin North Am. 1994;23:673-705. Abstract available at: http://www.ncbi.nlm.nih.gov/pubmed/7698827

Page 166: AAP PREP 2009

2009 PREP SA on CD-ROM

page 166

Critique: 49

The broad forehead and pointed chin characteristic of Alagille syndrome. (Courtesy of M. Rimsza)

Page 167: AAP PREP 2009

2009 PREP SA on CD-ROM

page 167

Critique: 49

Page 168: AAP PREP 2009

2009 PREP SA on CD-ROM

page 168

Question: 50

A term infant is delivered by emergency cesarean section following the acute onset of maternal vaginal bleeding and profound fetal bradycardia on electronic fetal heart rate monitoring. The Apgar scores are 1, 2, and 3 at 1, 5, and 10 minutes, respectively. Resuscitation includes intubation and assisted ventilation, chest compressions, and intravenous epinephrine. The infant is admitted to the neonatal intensive care unit and has seizures at 6 hours of age.

Of the following, a TRUE statement about infants who have seizures following perinatal asphyxia is that most

A. develop epilepsy

B. develop microcephaly

C. do not have severe long-term neurodevelopmental delay

D. experience hearing loss

E. require multiple anticonvulsant medications

Page 169: AAP PREP 2009

2009 PREP SA on CD-ROM

page 169

Critique: 50 Preferred Response: C

The infant described in the vignette has concerning perinatal events, low Apgar scores through 10 minutes of postnatal age, the need for vigorous resuscitation, and subsequent seizure activity 6 hours after birth. Seizures occurring in the first 24 hours after birth in newborns who have perinatal asphyxia are considered evidence of hypoxic-ischemic encephalopathy. Accompanied by profound acidemia, these seizures also may reflect significant cerebral cellular injury or cell death. The causes of neonatal seizure broadly include hypoxia-ischemia, which may be global (as in perinatal asphyxia) or focal (as in arterial or venous thrombosis); hemorrhagic injury; brain malformation; transient metabolic disorders (such as hypoglycemia or hypocalcemia); infectious (viral, bacterial, or fungal); inborn errors of metabolism; or unknown (Item C50A). As demonstrated in Item C50B, most term infants who have neonatal seizures do not manifest long-term neurodevelopmental sequelae. Further, even though motor abnormalities may be found on examination in a slight majority (53%) of newborns having a seizure for any cause, few have cerebral palsy. However, when seizures occur in newborns due to asphyxia (generalized hypoxic-ischemic injury), the prevalence of abnormal neurodevelopmental outcomes is more concerning. Although severe impairment in neurodevelopmental outcomes may occur in fewer than 50% of these infants, the presence of mild-to-moderate neurodevelopmental impairment in cognitive and motor function as well as some (~33%) children having long-term epilepsy brings the level of abnormal outcomes to approximately 50% (Item C50B). Early predictions of outcome for such children may be determined by evaluating the worst early electroencephalography (EEG) finding, the follow-up 1-week EEG, and the findings evident on cranial magnetic resonance imaging. Following perinatal asphyxia and resultant seizures, most newborns can be treated with a single anticonvulsant drug. Longstanding epilepsy and microcephaly occur in fewer than 50% of patients. Hearing loss, though more common in asphyxiated infants who have neurologic injury than in uncomplicated term infants, still occurs in fewer than 10% of infants.

References:

Hill A. Neonatal seizures. Pediatr Rev. 2000;21:117-121. Available at: http://pedsinreview.aappublications.org/cgi/content/full/21/4/117

Ronen GM, Buckley D, Penney S, Streiner DL. Long-term prognosis in children with neonatal seizures: a population-based study. Neurology. 2007;69:1816-1822. Abstract available at: http://www.ncbi.nlm.nih.gov/pubmed/17984448

Tekgul H, Gauvreau K, Soul J, et al. The current etiologic profile and neurodevelopmental outcome of seizures in term newborn infants. Pediatrics. 2006;117:1270-1280. Available at: http://pediatrics.aappublications.org/cgi/content/full/117/4/1270

Page 170: AAP PREP 2009

2009 PREP SA on CD-ROM

page 170

Critique: 50

Page 171: AAP PREP 2009

2009 PREP SA on CD-ROM

page 171

Critique: 50

Page 172: AAP PREP 2009

2009 PREP SA on CD-ROM

page 172

Question: 51

A 16-year-old girl who is new to your practice complains of a nearly constant headache for the past year. She describes the pain as a band around her head that often is throbbing and is worse during the middle of the day. She denies nausea or vomiting but reports occasional fatigue. There is no family history of headaches. She has missed more than 20 days of school this year because of the headache, and she is struggling to maintain a C average. She admits to hating school and does not participate in extracurricular activities because she "doesn’t like anything." Findings on her physical examination, including complete neurologic and funduscopic evaluation, are normal.

Of the following, the BEST next step in the management of this girl’s headaches is to

A. advise her to keep a headache diary and return in 2 months

B. obtain a lumbar puncture

C. obtain computed tomography scan of the brain

D. prescribe oral sumatriptan

E. refer her for psychosocial evaluation and counseling

Page 173: AAP PREP 2009

2009 PREP SA on CD-ROM

page 173

Critique: 51 Preferred Response: E

Chronic headache is a common complaint in children and adolescents. A careful history and complete neurologic examination are indicated in the evaluation of headache. The most important initial consideration is to identify characteristics of the headache that suggest serious diagnoses, such as brain tumors or other diseases causing increased intracranial pressure. Such characteristics include worsening pain at night or immediately upon awakening, association with vomiting, and worsening pain with coughing or straining. Papilledema or focal neurologic findings may be found in patients who have increased intracranial pressure or the examination results may be normal. Migraine headaches are periodic, may be accompanied by an aura, and typically are relieved by sleep. A family history of migraines usually can be elicited. The neurologic examination typically yields normal results, although complicated migraines can be accompanied by focal neurologic deficits such as hemiparesis, cranial nerve palsies, and visual disturbances. Pain from stress-related or tension headache generally is diffuse and may be described as "bandlike" or throbbing, as reported for the girl in the vignette. Pain usually occurs on most days, and school absence is frequent. The neurologic examination yields normal results. It is also very important for the clinician to obtain further information regarding any emotional, social, or academic difficulties the patient may be experiencing. Such data can help to determine both an underlying cause for the headache and the effect the headache is having on the child's quality of life. Family stressors and depression are known causes of headache, and questions regarding sleep patterns, anhedonia, school performance, and relationships with family and friends can help to screen for these conditions. Studies have shown that children and adolescents who have frequent or severe headaches have greater impairment in academic and social functioning than those who do not have headaches. A thorough psychosocial evaluation can aid in evaluating patients for depression and assessing the effect of headaches on their daily functioning. Nonpharmacologic therapies, such as rest, relaxation techniques, and removal of stress from the environment, can be effective once the stress is identified. Advising the girl in the vignette to keep a diary for 2 months likely would prolong her difficulties and not address the underlying cause of her headaches. Lumbar puncture and computed tomography scan of the brain may play some role in the evaluation of an acute headache, especially if meningitis or a mass lesion of the brain is suspected, but these diagnoses are very unlikely in a patient who has had a headache for a year and normal results on neurologic examination. Oral sumatriptan is used in the treatment of migraine, but this girl's history points to depression or another psychological cause for her headache, and prescribing medications without a psychosocial evaluation probably would not be effective.

References:

Forsyth R, Farrell K. Headache in childhood. Pediatr Rev. 1999;20:39-45. Available at: http://pedsinreview.aappublications.org/cgi/content/full/20/2/39

Silver N. Headache (chronic, tension type). BMJ Clinical Evidence. 2007. Available for subscription at: http://clinicalevidence.bmj.com/ceweb/conditions/nud/1205/1205_guidelines.jsp

Strine TW, Okoro CA, McGuire LC, Balluz LS. The associations among childhood headaches, emotional and behavioral difficulties, and health care use. Pediatrics. 2006;117: 1728-1735. Available at: http://pediatrics.aappublications.org/cgi/content/full/117/5/1728

Page 174: AAP PREP 2009

2009 PREP SA on CD-ROM

page 174

Question: 52

When a 14-year-old girl had frequent complaints of shoulder pain made worse by pitching softball a few months ago, you diagnosed overuse injury. Nonsteroidal anti-inflammatory drugs and rest have provided some relief. She presents today with complaints of recurrent upper arm pain that is unrelated to exercise and sometimes awakens her from sleep. Physical examination reveals a slightly larger circumference of the left proximal humerus compared with the right. There is minimal tenderness on palpation over the area, although the girl reports a constant ache. She has full range of motion of the arm at the shoulder and elbow. You obtain a shoulder radiograph (Item Q52).

Of the following, the MOST likely diagnosis is

A. acromioclavicular separation

B. acute osteomyelitis

C. chronic osteomyelitis

D. osteosarcoma

E. supracondylar fracture of the humerus

Page 175: AAP PREP 2009

2009 PREP SA on CD-ROM

page 175

Question: 52

(Courtesy of the Media Lab at Doernbecher)

Page 176: AAP PREP 2009

2009 PREP SA on CD-ROM

page 176

Critique: 52 Preferred Response: D

The girl described in the vignette has radiographic findings and clinical history that are most suggestive of osteogenic sarcoma of the humerus. The most common presenting symptom of osteosarcoma is pain, particularly with activity, as described for the girl. The affected patient may have a history of swelling, depending on the size of the lesion and its location. Patients or their parents may complain of a "sprain," "arthritis," or so-called "growing pains." Symptoms may be present for weeks, months, or occasionally longer before osteosarcoma is diagnosed. The child often has a history of trauma, as in this patient, because trauma occurs frequently in teenagers. Pathologic fractures are not particularly common with this tumor compared with leukemia and other malignancies. Systemic symptoms, such as fever and night sweats, are rare. Osteosarcoma is the third most common cancer in adolescence, after lymphomas and brain tumors. Peak age at diagnosis is 10 to 25 years. The incidence is 400 cases per year in the United States (4.8 cases per 1 million persons younger than 20 years) and is slightly higher in African Americans than in whites. An increased incidence during adolescence corresponds with the growth spurt. Osteosarcoma occurs in the long bones near metaphyseal growth plates (Item C52A). Most are high-grade intramedullary osteosarcomas, with only about 5% being low-grade lesions. The most common sites are the femur (42%, with 75% of femoral tumors occurring distally), tibia (19%, with 80% in the proximal tibia), and humerus (10%, with 90% in the proximal humerus). The overall 5-year survival rate for patients whose condition was diagnosed between 1974 and 1994 was 63% (59% for males, 70% for females). The current 5-year survival rate is estimated to be 65%. The mainstay of therapy is excision of the lesion. Chemotherapy is required to treat micrometastatic disease, which is present but not detectable in most patients at diagnosis. Metastatic spread to the lungs only rarely results in respiratory symptoms; such symptoms usually indicate extensive lung involvement. Metastasis to other sites is extremely rare. The exact cause of osteosarcoma is unknown, but it is believed to be a tumor of osteoprogenitor cells, which are multipotential, hormone-responsive stromal cells in the periosteum and marrow that are capable of differentiating into many lineages, depending on their environment. Among the known risk factors are rapid bone growth, exposure to radiation, and potentially a genetic predisposition. Children who have a prior personal or family history of retinoblastoma and those who have received radiation therapy for a previous malignancy are at higher risk of developing osteosarcoma. Initial evaluation of an adolescent or older child presenting with bone pain and swelling, especially with a palpable mass, should include: o Plain radiographs (two views) of the suspected lesions, although no single feature on radiographs is diagnostic. Osteosarcomatous lesions can be purely osteolytic (about 30% of patients), purely osteoblastic (about 45% of patients), or a mixture of both. Elevation of the periosteum may appear as the characteristic Codman triangle. Extension of tumor through the periosteum may result in a so-called "sunburst appearance" (about 60% of patients). o Both magnetic resonance imaging of the primary lesion and computed tomography scan of the chest are necessary to confirm the diagnosis and for staging purposes. Such scans frequently are performed at the tertiary center using their protocols. The differential diagnosis of bone pain and swelling includes stress fracture, hematoma, bone cysts, and other bony tumors such as Ewing sarcoma. Although acute osteomyelitis is common in children, it is relatively less common in teenagers; approximately 50% of cases occur in preschool-age children. Infection frequently is characterized by overlying erythema, warmth, and more acute systemic signs such as fever and malaise. The white blood cell count is elevated in 50% of patients, and the erythrocyte sedimentation rate or C-reactive protein value is increased. Plain radiographs may be read as normal early in the disease course. Chronic osteomyelitis also may cause pain. However, results of blood tests often are normal. The

Page 177: AAP PREP 2009

2009 PREP SA on CD-ROM

page 177

radiograph for the patient in the vignette, however, shows a classic sunburst pattern, which is not consistent with chronic osteomyelitis. Acromioclavicular (AC) separation typically results in sudden pain and limited range of motion. The diagnosis can be made by an anteroposterior radiograph, which can demonstrate excessive separation of the AC joint (Item C52B). Supracondylar fracture of the humerus usually is caused by falling onto the extremity (often outstretched) and may be associated with acute pain, swelling, and deformity near the elbow (Item C52C).

References:

Arndt CAS. Neoplasms of bone. In: Kliegman RM, Behrman RE, Jenson HB, Stanton BF, eds. Nelson's Textbook of Pediatrics. 18th ed. Philadelphia, Pa: Saunders Elsevier; 2007:2146-2150

Cripe TP. Osteosarcoma. eMedicine Specialties, Pediatrics, Oncology. 2006. Available at: http://www.emedicine.com/ped/topic1684.htm

Gorlick R, Anderson P, Andrulis I, et al. Biology of childhood osteogenic sarcoma and potential targets for therapeutic development. Clin Cancer Res. 2003;9:5442-5453. Available at: http://clincancerres.aacrjournals.org/cgi/content/full/9/15/5442

Gurney JG, Swensen AR, Bulterys M. Malignant bone tumors. In: Ries LAG, Smith MA, Gurney JG, et al, eds. Cancer Incidence and Survival Among Children and Adolescents: United States SEER Program 1975-1995. Bethesda, Md: National Cancer Institute, SEER Program; 1999:88-110. Available at: http://seer.cancer.gov/publications/childhood/bone.pdf

Kalyoussef S, Tolan RW Jr. Osteomyelitis. eMedicine Specialties, Pediatrics, Infectious Diseases. 2006. Available at: http://www.emedicine.com/PED/topic1677.htm

Seade LE, Bryan, WJ, Bartz RL, Josey R. Acromioclavicular joint injury. eMedicine Specialties, Sports Medicine, Shoulder. 2006. Available at: http://www.emedicine.com/sports/TOPIC3.HTM

Page 178: AAP PREP 2009

2009 PREP SA on CD-ROM

page 178

Critique: 52

Intense periosteal new bone formation extending into soft tissue is characteristic of osteogenic sarcoma. (Courtesy of the Media Lab at Doernbecher)

Page 179: AAP PREP 2009

2009 PREP SA on CD-ROM

page 179

Critique: 52

Anteroposterior radiograph of a normal shoulder (top): A line drawn on the underside of the clavicle intersects the achromion. In acromioclavicular separation (bottom), this line does not intersect the achromion, indicating that the distal end of the clavicle is no longer in contact with the achromion. There is an incidental finding of separation of the VP shunt tubing in the neck (arrow). (Courtesy of D. Mulvihill)

Page 180: AAP PREP 2009

2009 PREP SA on CD-ROM

page 180

Critique: 52

Lateral radiograph of the elbow in supracondylar fracture of the humerus: the posterior fat pad (darker area noted by arrow) is displaced from the supracondylar fossa by hemorrhage into the joint. There is posterior displacement of the capitellum (the anterior humeral line passes along the anterior edge of the capitellum rather than through its center). (Courtesy of D. Mulvihill)

Page 181: AAP PREP 2009

2009 PREP SA on CD-ROM

page 181

Question: 53

A 7-month-old female has undergone the second stage of surgical palliation (Glenn operation) for hypoplastic left heart syndrome. She was discharged from the hospital 1 week ago, and her mother brings her to the office because of irritability that began this morning. On physical examination, the infant is awake and irritable, with a heart rate of 150 beats/min and a respiratory rate of 50 breaths/min. She has cyanosis of the face and mucosal surfaces and swelling of the arms and head.

Of the following, the BEST explanation for this patient’s clinical presentation is

A. polycythemia

B. postpericardiotomy syndrome

C. protein-losing enteropathy

D. superior vena cava syndrome

E. thoracic duct injury

Page 182: AAP PREP 2009

2009 PREP SA on CD-ROM

page 182

Critique: 53 Preferred Response: D

The child described in the vignette has had surgery involving her superior vena cava, which has been sewn by an end-to-side anastomosis to her right pulmonary artery (Glenn operation). She is at risk for stenosis at the surgical site, thrombosis within the superior vena cava, and altered hemodynamics if the pulmonary vascular resistance (and, thus, the pulmonary artery pressure) increases, which raises the pressure against which the venous drainage must occur. Her symptoms and physical findings are consistent with superior vena cava syndrome (Item C53), and she should undergo an aggressive evaluation and rapid institution of treatment. Obstruction of the systemic venous return may result from one of three primary causes: extrinsic compression of either the superior or inferior vena cava, intrinsic obstruction of systemic return, or abnormal hemodynamics with significantly elevated right atrial pressures. The systemic veins are thin-walled vascular structures that typically drain at low pressure into the superior and inferior vena cavae. Normally, these large veins drain into the right atrium at low pressure. Flow in any system moves from high to low pressure, and the cardiovascular system is no exception. The right atrium in the healthy individual typically has a pressure of less than 10 mm Hg, often less than 5 mm Hg. Mechanisms that facilitate venous drainage to the right atrium include gravity for the vessels of the upper body and muscular contraction of the lower extremities, which serves to "push blood up" the valved veins of the caudal portion of the body. As long as the pressure in the right atrium ("downstream") remains lower than the pressures in the veins, forward flow ensues. Any process that increases the pressure in the right atrium raises the pressure needed to ensure forward drainage of the systemic veins. Similarly, any obstruction of the superior or inferior vena cava raises the pressure "upstream" and may limit normal venous drainage. Extrinsic compression can result from a mediastinal mass or tumor that physically compresses the vena cava, thereby raising the pressure needed to pass blood through the narrowing and into the right atrium. Intrinsic obstructions can result from surgical anastomotic sites, baffle stenosis, thrombosis (eg, from an indwelling catheter), or cardiac tumors that physically obstruct blood return through the vena cava. When blood return from the superior vena cava into the right atrium is obstructed significantly, patients may demonstrate signs of superficial venous distention, venous congestion, and facial and upper body edema, as described for the infant in the vignette. As the venous pressure increases proximal to the obstruction, the venous drainage of the brain may become engorged, leading to discomfort, irritability, and seizure and brain injury. Polycythemia (elevated hemoglobin concentrations) might lead to sluggish blood flow through the small capillaries, but would not cause the findings described in this child. Postpericardiotomy syndrome, which can occur in children who have had cardiac surgery, generally presents with fever and systemic symptoms. Protein-losing enteropathy is a serious complication that can occur in patients who have increased pressure in the venous drainage of the gut, including those who undergo single ventricle palliation. Such patients typically present with diarrhea and edema of the entire body, not localized to the upper compartments, as in this patient. Thoracic duct injury can occur in any patient undergoing cardiac surgery and often leads to a chylothorax.

References:

Moore P. Obstructive lesions. In: Rudolph CD, Rudolph AM, eds. Rudolph's Pediatrics. 21st ed. New York, NY: McGraw-Hill Medical Publishing Division; 2003:1800-1813

Silberbach M, Hannon D. Presentation of congenital heart disease in the neonate and young infant. Pediatr Rev. 2007;28:123-131. Available at:

Page 183: AAP PREP 2009

2009 PREP SA on CD-ROM

page 183

http://pedsinreview.aappublications.org/cgi/content/full/28/4/123

Page 184: AAP PREP 2009

2009 PREP SA on CD-ROM

page 184

Critique: 53

Superior vena cava syndrome is characterized by swelling of the head and neck and engorgement of veins on the chest wall. (Courtesy of M. Rimsza)

Page 185: AAP PREP 2009

2009 PREP SA on CD-ROM

page 185

Question: 54

A 6-year-old boy presents with a sudden-onset loss of awareness characterized by staring, drooling, and chewing movements for more than 15 minutes, followed by confusion, then deep sleep. On physical examination in the emergency department, the child is afebrile and appears to be returning to normal. Vital signs and general examination findings are normal, and there are no focal findings. Head computed tomography scan shows a large, contrast-enhancing cerebral mass (Item Q54) without edema or midline shift.

Of the following, the MOST likely diagnosis is

A. arteriovenous malformation

B. brain damage from a seizure

C. ependymoma

D. glioblastoma multiforme

E. herpes encephalitis

Page 186: AAP PREP 2009

2009 PREP SA on CD-ROM

page 186

Question: 54

(Courtesy of D. Gilbert)

Page 187: AAP PREP 2009

2009 PREP SA on CD-ROM

page 187

Critique: 54 Preferred Response: A

The most common cause of hemorrhagic stroke in children is vascular malformation, of which there are two types: arteriovenous malformations (AVMs) and cavernous malformations. These can present in childhood with hemorrhage that leads to headache and seizures, as described for the boy in the vignette. Severe, "worst-ever" headache; seizure; and mental status change are indications for emergency evaluation and imaging of the central nervous system. The computed tomography scan obtained for this boy shows a contrast-enhancing tortuous vascular mass without surrounding edema or midline shift (Item C54A). Seizures are not believed to cause brain damage in children, except in rare cases. A large hemorrhage can cause seizures, but seizures cannot cause a large hemorrhage. Ependymomas are one of the more common brain tumors in children. They emerge from the ependymal lining of ventricles, do not hemorrhage, and typically occur in the posterior fossa of young children (Item C54B). Therefore, they present more indolently, with headache and cranial nerve findings. Glioblastoma multiforme, the highest grade astrocytoma, is uncommon in young children. Although glioblastoma multiforme can hemorrhage, it does not have a tortuous vascular appearance on imaging and has surrounding edema. Herpes encephalitis causes hemorrhagic necrosis (Item C54C) and can present with seizures, but usually the patient is febrile and the characteristic imaging appearance does not involve a tortuous vascular-appearing mass.

References:

Huang J, Gailloud PH, Tamargo RJ. Vascular malformations. In: Singer HS, Kossoff EH, Hartman AL, Crawford TO, eds. Treatment of Pediatric Neurologic Disorders. Boca Raton, Fla: Taylor & Francis; 2005:409-414

Klimo P Jr, Rao G, Brockmeyer D. Pediatric arteriovenous malformations: a 15-year experience with an emphasis on residual and recurrent lesions. Childs Nerv Syst. 2007;23:31-37. Abstract available at: http://www.ncbi.nlm.nih.gov/pubmed/17053936

Thai Q, Moriarty JL, Tamargo RJ. Central nervous system vascular malformations in pediatric patients. In: Maria BL, ed. Current Management in Child Neurology. 3rd ed. Hamilton, Ontario, Canada: BC Decker Inc; 2005:595-605

Page 188: AAP PREP 2009

2009 PREP SA on CD-ROM

page 188

Critique: 54

Computed tomography scan with contrast demonstrates a tortuous vascular mass without midline shift or edema. (Courtesy of D. Gilbert)

Page 189: AAP PREP 2009

2009 PREP SA on CD-ROM

page 189

Critique: 54

Ependymoma: Sagittal T1-weighted magnetic resonance imaging with gadolinium contrast demonstrates a contrast-enhancing cerebellar mass (arrow) causing brainstem compression and obstructive hydrocephalus. (Courtesy of D. Gilbert)

Page 190: AAP PREP 2009

2009 PREP SA on CD-ROM

page 190

Critique: 54

Axial computed tomography scan 2 weeks after the onset of symptoms of herpes simplex virus encephalitis shows necrosis in the right temporal lobe. (Courtesy of D. Gilbert)

Page 191: AAP PREP 2009

2009 PREP SA on CD-ROM

page 191

Question: 55

While examining a newborn, you note a persistent curve in the spine regardless of the baby’s position. You order spine radiographs, which reveal multiple vertebral malformations and segmentation defects (Item Q55).

Of the following, the MOST appropriate studies to guide further management are

A. chromosome analysis and renal ultrasonography

B. echocardiography and chromosome analysis

C. echocardiography and renal ultrasonography

D. head ultrasonography and ophthalmology consultation

E. renal and head ultrasonography

Page 192: AAP PREP 2009

2009 PREP SA on CD-ROM

page 192

Question: 55

Vertebral malformations (arrows), as described for the newborn in the vignette. (Courtesy of T. Jewett)

Page 193: AAP PREP 2009

2009 PREP SA on CD-ROM

page 193

Critique: 55 Preferred Response: C

Congenital scoliosis, as described for the infant in the vignette, is caused by errors in vertebral development during embryogenesis, resulting in abnormal segmentation or formation (Item C55). It is associated with other malformations in more than 50% of affected individuals, most commonly involving the heart, kidneys, and spinal cord. Most cases of congenital scoliosis are not hereditary, although numerous syndromes include spinal malformations. The child who has congenital scoliosis should receive serial posteroanterior and lateral spine radiographs beginning, ideally, at birth. After 1 year of age, spine radiographs should be taken with the child standing and the pelvis maintained in a level position. Computed tomography scan with thin slices and three-dimensional reconstruction may be helpful, especially if malformations are multiple or complex. Magnetic resonance imaging (MRI) with cartilage sequences is recommended to determine growth potential. Significant progression of scoliosis usually warrants surgery. Bracing typically is not helpful as a primary treatment modality but may be recommended after surgery. Every child who has congenital scoliosis and vertebral anomalies should be evaluated clinically for associated malformations. Up to 20% of affected individuals have abnormalities of the urinary tract, and up to 25% have cardiac anomalies. Therefore, children should receive screening renal ultrasonography and echocardiography. Up to 40% of affected children may have abnormalities of the spinal cord such as syrinx and tethered cord, warranting spinal MRI. Chromosome analysis, ophthalmology evaluation, and head ultrasonography typically are not helpful in directing management but must be considered on a case-by-case basis.

References:

Arlet V, Odent T, Aebi M. Congenital scoliosis. Eur Spine J. 2003;12:456-463. Abstract available at: http://www.ncbi.nlm.nih.gov/pubmed/14618384

Ferguson RL. Medical and congenital comorbidities associated with spinal deformities in the immature spine. J Bone Joint Surg Am. 2007;89:34-41

Hedequist D, Emans J. Congenital scoliosis. J Am Acad Orthop Surg. 2004;12:266-275. Abstract available at: http://www.ncbi.nlm.nih.gov/pubmed/15473678

Page 194: AAP PREP 2009

2009 PREP SA on CD-ROM

page 194

Critique: 55

Congenital scoliosis: Left thoracic scoliosis is associated with multiple vertebral anomalies (red arrows) and rib abnormalities (eg, rib fusion, yellow arrow). (Courtesy of T. Jewett)

Page 195: AAP PREP 2009

2009 PREP SA on CD-ROM

page 195

Question: 56

A community group asks you to speak at a forum on teenage pregnancy. The number of pregnancies among young adolescents at the local middle school has increased this year, and several community members want more information about adolescent pregnancy and its long-term effects.

Of the following, the MOST appropriate statement to include in your talk about pregnant and parenting adolescents in the United States is that

A. adolescent fathers do not have increased school drop-out rates

B. adolescents who become pregnant have the same vocational opportunities as their nonpregnant female peers

C. most adolescent pregnancies occur in 14- to 16-year-old young women

D. poverty is correlated significantly with adolescent pregnancy

E. the adolescent pregnancy rate is increasing in the United States

Page 196: AAP PREP 2009

2009 PREP SA on CD-ROM

page 196

Critique: 56 Preferred Response: D

Although the proportion of teenagers reporting sexual activity has dropped and adolescent pregnancy rates have declined in recent years, the United States still has the highest adolescent birth rate among comparable developed countries. Most adolescent pregnancies occur among older adolescents (ie, 18 and 19 year olds), and young women in this age group have a lower risk of medical complications involving the mother or child than those younger than 17 years of age. The adverse consequences of adolescent pregnancy are myriad, including school interruption, persistent poverty, limited vocational opportunities, separation from the father of the baby, divorce, and repeat pregnancy. Poverty is an important factor correlated with adolescent pregnancy, with as many as 83% of adolescents who gave birth and 61% of adolescents who had abortions coming from poor or low-income homes compared with 38% of nonpregnant adolescent females. Adolescent fathers have been shown to have similar outcomes to adolescent mothers, with a higher likelihood of having poor academic performance, higher school drop-out rates, and decreased income potential compared with their peers who are not fathers. In an attempt to decrease adolescent child-bearing rates as well as unsafe sexual practices, every pediatrician should integrate sexuality education into clinical practice with children from early childhood through adolescence. Pediatricians also should consider participating in development and implementation of sexuality education curricula for schools or public efforts.

References:

American Academy of Pediatrics Committee on Psychosocial Aspects of Child and Family Health and Committee on Adolescence. Sexuality education for children and adolescents. Pediatrics. 2001;108:498-502. Available at: http://pediatrics.aappublications.org/cgi/content/full/108/2/498

Klein JD, and the AAP Committee on Adolescence. Adolescent pregnancy: current trends and issues. Pediatrics. 2005;116:281-286. Available at: http://pediatrics.aappublications.org/cgi/content/full/116/1/281

Page 197: AAP PREP 2009

2009 PREP SA on CD-ROM

page 197

Question: 57

You are treating a 14-year-old boy in the pediatric intensive care unit who suffered a traumatic brain injury in a motor vehicle crash earlier today and underwent surgery to drain a right-sided epidural hematoma. He is currently receiving mechanical ventilation and is sedated. The nurse calls you to the bedside because the intraventricular catheter is clotted and no intracranial pressure waveform is seen on the monitor. On physical examination, you note that his right pupil is dilated and unresponsive to light, which differs from findings on your examination immediately after surgery.

Of the following, the MOST appropriate immediate next step is

A. administration of fentanyl

B. administration of mannitol

C. cerebral angiography

D. replacement of the intraventricular catheter

E. ophthalmology consultation

Page 198: AAP PREP 2009

2009 PREP SA on CD-ROM

page 198

Critique: 57 Preferred Response: B

Increased intracranial pressure can cause herniation of the cerebral contents, which is a neurologic emergency. Transtentorial herniation results from downward or upward displacement of the brain through the tentorium at the level of the incisura. Temporal lobe (uncal) herniations are a subcategory of transtentorial herniations that are characterized by dilation of a unilateral pupil due to compression of the oculomotor nerve, as described for the boy in the vignette (Item C57). Uncal herniations usually occur with rapid expansion of the contents of the temporal lobe fossa, such as seen with epidural hematomas, focal injury, or infection. Findings for the boy in the vignette suggest rebleeding of the epidural hematoma, which requires prompt action to reduce the associated swelling and reverse the herniation. Osmotic agents such as mannitol or hypertonic saline should be administered while calling the neurosurgeon and planning for emergent radiologic evaluation. Pain or anxiety may produce bilateral, not unilateral, pupillary dilation. Therefore, treatment with fentanyl would not address this patient's neurologic emergency. Ischemic stroke can produce pupil dilation but is less likely than uncal herniation in this scenario. Cerebral angiography to evaluate blood vessel characteristics usually is performed after stroke is diagnosed by computed tomography scan or magnetic radiographic imaging. Replacement of the clotted intraventricular catheter is indicated for overall management, but acute measures to reverse the uncal herniation are of much higher priority. Ophthalmology consultation for anisocoria is not indicated at this time because the most likely cause of the patient's signs and symptoms is acute herniation.

References:

Avner JR. Altered states of consciousness. Pediatr Rev. 2006:27:331-338. Available at: http://pedsinreview.aappublications.org/cgi/content/full/27/9/331

Frankel LR. Neurological emergencies and stabilization. In: Kliegman RM, Behrman RE, Jenson HB, Stanton BF, eds. Nelson Textbook of Pediatrics. 18th ed. Philadelphia, Pa: Saunders Elsevier; 2007:405-410

Page 199: AAP PREP 2009

2009 PREP SA on CD-ROM

page 199

Critique: 57

Uncal herniation may result in ipsilateral pupillary dilation, diminished level of consciousness, and contralateral hemiparesis. (Courtesy of the Media Lab at Doernbecher)

Page 200: AAP PREP 2009

2009 PREP SA on CD-ROM

page 200

Question: 58

The mother of a 10-year-old boy, whom you have been following since he was 3 years old, complains that he is always hungry and is gaining weight. The mother, who is overweight, reports that the boy refuses to exercise, and she cannot control his diet. She just read an article in a magazine about weight gain from Cushing syndrome and wonders if he could have this condition.

Of the following, the growth chart shown in Item 58 that suggests Cushing syndrome is

A. Growth chart A

B. Growth chart B

C. Growth chart C

D. Growth chart D

E. Growth chart E

Page 201: AAP PREP 2009

2009 PREP SA on CD-ROM

page 201

Question: 58

(Courtesy of L. Levitsky)

Page 202: AAP PREP 2009

2009 PREP SA on CD-ROM

page 202

Critique: 58 Preferred Response: C

Weight gain from exogenous obesity can be confused with Cushing syndrome, but glucocorticoid excess, as seen in Cushing syndrome, almost always is associated with attenuation of normal growth, as documented with Growth Chart C (Item C58A). The other growth charts are more typical for exogenous obesity, with height either enhanced or unchanged in the presence of weight gain. Other signs and symptoms of Cushing syndrome include hypertension, violaceous skin striae (Item C58B), "buffalo hump" and muscle weakness because of loss of muscle mass, centripetal obesity, cushingoid facies, easy bruisability, hirsutism, failure of pubertal progression or amenorrhea in women, loss of libido in men, headache, depression, and dysphoria. Comparison of school photographs from past years can be a useful exercise, although the clinical diagnosis of pituitary Cushing syndrome (Cushing disease) can be difficult. Documentation of several elevated 24-hour urine free cortisol measurements as well as elevated overnight dexamethasone-suppressed serum cortisol, evening salivary cortisol, or midnight serum cortisol values aid in diagnosis. However, children who are very obese, stressed, or depressed may have inappropriate elevations in serum or urine cortisol values, and children who have mild Cushing disease may have normal values on one or more occasions.

References:

Batista DL, Riar J, Keil M, Stratakis CA. Diagnostic tests for children who are referred for the investigation of Cushing syndrome. Pediatrics. 2007;120:e575-e586. Available at: http://pediatrics.aappublications.org/cgi/content/full/120/3/e575

Greening JE, Storr HL, McKensie SA, et al. Linear growth and body mass index in pediatric patients with Cushing's disease or simple obesity. J Endocrinol Invest. 2006;29:885-887. Abstract available at: http://www.ncbi.nlm.nih.gov/pubmed/17185896

Klish WJ. Clinical evaluation of the obese child and adolescent. UpToDate Online 15.3. 2008. Available for subscription at: http://www.uptodateonline.com/utd/content/topic.do?topicKey=pedigast/11089

Magiakou MA, Mastorakas G, Oldfield EH, et al. Cushing's syndrome in children and adolescents. Presentation, diagnosis, and therapy. N Engl J Med. 1994;331:629-636. Abstract available at: http://www.ncbi.nlm.nih.gov/pubmed/8052272

Nieman L, Ilias I. Evaluation and treatment of Cushing's syndrome. Am J Med. 2005;118:1340-1346. Abstract available at: http://www.ncbi.nlm.nih.gov/pubmed/16378774

Page 203: AAP PREP 2009

2009 PREP SA on CD-ROM

page 203

Critique: 58

Attenuation of linear growth is observed in most patients who have Cushing syndrome. (Courtesy of L. Levitsky)

Page 204: AAP PREP 2009

2009 PREP SA on CD-ROM

page 204

Critique: 58

Violaceous striae may be seen in patients who have Cushing syndrome. (Courtesy of M. Rimsza)

Page 205: AAP PREP 2009

2009 PREP SA on CD-ROM

page 205

Question: 59

The parents of an 8-year-old boy are concerned because he recently has begun to struggle in school. In the past, he always had been an attentive and motivated student. His current teacher reports that at times when he is speaking in class, he stops speaking abruptly, stares with glassy eyes, then resumes speaking. At home, his parents note that he "spaces out" when eating dinner. His parents ask your input and the best approach to treat his issues.

Of the following, the MOST appropriate initial evaluation is

A. computed tomography scan of the brain

B. educational evaluation

C. electroencephalography

D. formal audiologic evaluation

E. magnetic resonance imaging of the brain

Page 206: AAP PREP 2009

2009 PREP SA on CD-ROM

page 206

Critique: 59 Preferred Response: C

The child described in the vignette appears to be experiencing absence seizures. These present as multiple daily episodes of abrupt arrest of consciousness and voluntary movement that last a few seconds (Item C59), followed by immediate return to normal consciousness. Decline in school function is common at the onset of this disorder. Electroencephalography (EEG) confirms the diagnosis, showing a pattern of spike and wave electrical complexes that have a frequency of 3 Hz (3/second). Computed tomography scan and magnetic resonance imaging are not indicated in the evaluation of children who have uncomplicated absence seizures. An audiologic evaluation would be indicated if the child was suspected of not hearing properly. Decisions regarding an educational evaluation should be deferred until the diagnosis of epilepsy is clarified. Occasionally, inattention associated with attention-deficit/hyperactivity disorder (ADHD) may mimic absence epilepsy. However, children who have ADHD do not have a sudden cessation of motor activity, blank facial expression, or flickering of the eyelids characteristic of absence seizures. Computed tomography scan, magnetic resonance imaging, and EEG generally are not indicated for the diagnosis of either ADHD or a learning disability. Children who have learning disabilities are identified by results from standardized psychoeducational testing. ADHD is diagnosed if the child meets American Psychiatric Association Diagnostic and Statistical Manual of Mental Disorders, fourth edition, text revision (DSM-IV-TR) criteria and has symptoms of inattention and hyperactivity/impulsivity that have lasted for more than 6 months and were observed prior to age 7 years. The behaviors cause "clinically significant impairment in social, academic or occupational functioning" and present in more that one setting. Symptoms do not include the type of "staring spells" described for the boy in the vignette.

References:

American Academy of Pediatrics. Committee on Quality Improvement, Subcommittee on Attention-Deficit/Hyperactivity Disorder. Clinical practice guideline: diagnosis and evaluation of the child with attention-deficit/hyperactivity disorder. Pediatrics. 2000;105:1158-1170. Available at: http://pediatrics.aappublications.org/cgi/content/full/105/5/1158

American Psychiatric Association. Diagnostic criteria for ADHD. In: Diagnostic and Statistical Manual of Mental Disorders. 4th ed. Text revision. Arlington, Va: American Psychiatric Association; 2000:85-94

American Psychiatric Association. Diagnostic criteria for learning disability. In: Diagnostic and Statistical Manual of Mental Disorders. 4th ed. Text revision. Arlington, Va: American Psychiatric Association; 2000:49-56

Posner E. Absence seizures in children. BMJ Clinical Evidence. 2007. Available for subscription at: http://clinicalevidence.bmj.com/ceweb/conditions/chd/0317/0317_background.jsp

Pritchard D. Attention deficit hyperactivity disorder in children. BMJ Clinical Evidence. 2006. Available for subscription at: http://clinicalevidence.bmj.com/ceweb/conditions/chd/0312/0312_background.jsp

Page 207: AAP PREP 2009

2009 PREP SA on CD-ROM

page 207

Question: 60

You are seeing a young girl for a health supervision visit. Her older brother recently underwent a bone marrow transplant, and you inquire about his health. The mother is tearful as she tells you it has been difficult, explaining that he has had fever for about 10 days, his "counts are still down," and they are planning to start amphotericin B just in case he has a fungal infection. She is concerned because she was told about potential adverse effects of the medication and how they need to watch the "electrolytes in his blood" very closely. She doesn’t know what "electrolytes" are, but asks what parameter in his blood might be affected.

Of the following, the MOST clinically important parameter to monitor during the initiation of amphotericin B therapy is

A. bicarbonate

B. creatinine

C. glucose

D. potassium

E. sodium

Page 208: AAP PREP 2009

2009 PREP SA on CD-ROM

page 208

Critique: 60 Preferred Response: D

The major metabolic abnormality that occurs early in therapy with amphotericin B administration is hypokalemia, which can be difficult to correct, especially if there is concurrent hypomagnesemia. Amphotericin B augments potassium and magnesium excretion by the kidney. Therefore, potassium concentrations cannot be corrected in most patients until the hypomagnesemia also is corrected. The hypokalemia can occur whether amphotericin B deoxycholate or the lipid formulations of amphotericin B are used. Problems with sodium, glucose, and bicarbonate usually do not occur with amphotericin B therapy. Elevation of the blood urea nitrogen or creatinine values associated with amphotericin B therapy occurs slowly and is due to the cumulative renal effects of the medication. Once the medication is stopped, renal function returns to normal in most patients. Other common adverse reactions associated with amphotericin B therapy include fever, chills, rigors, malaise, nausea, vomiting, and headaches. Younger children and infants generally do not have these reactions. Older children and adolescents may require premedication with ibuprofen or acetaminophen to lessen such reactions. In addition, lengthening of infusion times to between 4 and 6 hours can blunt such symptoms. In severe cases, patients may require antihistamines, antiemetics, meperidine, or the addition of hydrocortisone to the infusion before administration of amphotericin B.

References:

American Academy of Pediatrics. Antifungal drugs for systemic fungal infections. In: Pickering LK, Baker CJ, Long SS, McMillan JA, eds. Red Book: 2006 Report of the Committee on Infectious Diseases. 27th ed. Elk Grove Village, Ill: American Academy of Pediatrics; 2006:774-776

Zaoutis TE, Benjamin DK, Steinbach WJ. Antifungal treatment in pediatric patients. Drug Resist Update. 2005;8:235-245. Abstract available at: http://www.ncbi.nlm.nih.gov/pubmed/16054422

Page 209: AAP PREP 2009

2009 PREP SA on CD-ROM

page 209

Question: 61

You are evaluating an 8-year-old boy who has acute lymphoblastic leukemia and is in septic shock caused by Klebsiella pneumoniae. The antibiotic susceptibilities for the organism reveal that it is resistant to ampicillin, cefazolin, ceftriaxone, and gentamicin.

Of the following, the MOST appropriate antibiotic to use in this patient is

A. cefuroxime

B. clindamycin

C. meropenem

D. penicillin G

E. piperacillin

Page 210: AAP PREP 2009

2009 PREP SA on CD-ROM

page 210

Critique: 61 Preferred Response: C

Imipenem and meropenem are carbapenems, a type of beta-lactam antibiotic that has a very broad antibacterial spectrum of activity. These agents are beta-lactamase-stable and bind with high affinity to the penicillin-binding proteins of gram-positive and gram-negative bacteria. Imipenem is slightly more active against gram-positive bacteria than meropenem; meropenem is slightly more active against gram-negative aerobic organisms. Both have excellent activity against aerobic hemolytic streptococci, Streptococcus pneumoniae (including resistant strains), methicillin-susceptible S aureus and S epidermidis, Listeria sp, and Bacillus sp. Penicillin-susceptible strains of Enterococcus faecalis are susceptible to imipenem (bacteriostatic), but resistant to meropenem. As a class, carbapenems also are highly active against Neisseria gonorrhoeae and N meningitidis, Haemophilus influenzae and other Haemophilus sp, Morganella sp, Proteus sp, most Enterobacteriaceae, Citrobacter sp, Enterobacter sp, Providencia sp, Pseudomonas sp, and most anaerobic species, including anaerobic gram-positive cocci. They also are active against Nocardia sp and Actinomyces sp. Carbapenems are useful for the treatment of a wide variety of infections. Imipenem and meropenem are most appropriate for treatment of infections caused by cephalosporin-resistant Enterobacteriaceae, especially Serratia marcescens, Providencia sp, C freundii, and Enterobacter sp; as empiric therapy for serious infections in patients previously treated with multiple antibiotics; as single agents in the treatment of febrile neutropenia; and as treatment for polymicrobial and nosocomial infections. Thus, meropenem is appropriate therapy for the boy described in the vignette, who is infected with a multiple drug-resistant strain of Klebsiella pneumoniae (a gram-negative organism) that is resistant to the cephalosporins (including the third-generation cephalosporins). An infrequent adverse effect of imipenem (<1%) is seizures. This effect has been reported in patients receiving very high doses who have underlying central nervous system pathology and in patients who have decreased renal function in whom the dose has not been adjusted. Seizures are not a major problem with meropenem, even in patients being treated for meningitis. Cefuroxime is a second-generation cephalosporin to which the organism infecting the patient in the vignette is resistant. Clindamycin only has activity against gram-positive organisms. Because the K pneumoniae strain is resistant to ampicillin, it also is resistant to penicillin G. The antibiotic resistance pattern of the identified pathogen indicates that one of its major resistance mechanisms is the production of extended-spectrum beta-lactamases, which are enzymes that hydrolyze many different penicillins and cephalosporins. Because of the production of these enzymes, piperacillin is not an effective treatment option.

References:

Balfour JA, Bryson HM, Brogden RN. Imipenem/cilastatin: an update of its antibacterial activity, pharmacokinetics and therapeutic efficacy in the treatment of serious infections. Drugs. 1996;51:99-136. Abstract available at: http://www.ncbi.nlm.nih.gov/pubmed/8741235

Nicolau DP. Carbapenems: a potent class of antibiotics. Expert Opin Pharmacother. 2008;9:23-37. Abstract available at: http://www.ncbi.nlm.nih.gov/pubmed/18076336

Norrby SR. Carbapenems. Med Clin North Am. 1995;79:745-759. Abstract available at: http://www.ncbi.nlm.nih.gov/pubmed/7791421

Wiseman LR, Wagstaff AJ, Brogden RN, Bryson HM. Meropenem: a review of its antibacterial activity, pharmacokinetic properties and clinical efficacy. Drugs. 1995;50:73-101. Abstract available at: http://www.ncbi.nlm.nih.gov/pubmed/7588092

Page 211: AAP PREP 2009

2009 PREP SA on CD-ROM

page 211

Zhanel GG, Wiebe R, Diley L, et al. Comparative review of the carbapenems. Drugs. 2007;67:1027-1052. Abstract available at: http://www.ncbi.nlm.nih.gov/pubmed/17488146

Page 212: AAP PREP 2009

2009 PREP SA on CD-ROM

page 212

Question: 62

A 10-year-old boy has marked fluid intake, frequent urination, and decreased visual acuity. On physical examination, the boy is short (<5th percentile), neurologic evaluation findings are normal, and no edema is present. His electrolyte values are normal. Other laboratory results include: · Blood urea nitrogen, 36.0 mg/dL (12.9 mmol/L) · Creatinine, 2.0 mg/dL (176.8 mcmol/L) · Hemoglobin, 6.5 g/dL (65.0 g/L)· Urine specific gravity, 1.005· Urine pH, 6· Urine protein, 1+

Ophthalmologic evaluation reveals features of retinitis pigmentosa.

Of the following, the MOST likely diagnosis is

A. Alport syndrome

B. diabetic nephropathy

C. juvenile nephronophthisis

D. Lowe syndrome

E. nephropathic cystinosis

Page 213: AAP PREP 2009

2009 PREP SA on CD-ROM

page 213

Critique: 62 Preferred Response: C

The symptoms of polyuria and polydipsia; clinical finding of short stature; and laboratory findings of azotemia, anemia, and proteinuria described for the patient in the vignette point to the diagnosis of juvenile nephronophthisis (NPH). This autosomal recessive inherited disorder is characterized by chronic tubulointerstitial disease. NPH begins with tubular dysfunction, which progresses to azotemia and ultimately, end-stage renal disease (ESRD). The clinical presentation includes symptoms consistent with a urinary concentrating defect (polyuria and polydipsia) at 2 to 3 years of age that progresses to more overt symptoms in the next few years. Tubular wasting of sodium can lead to hyponatremia and salt craving. Signs of acidosis, anemia, and azotemia appear next. The median age of onset of ESRD is 13 years. NPH is estimated to account for 5% to 10% of all cases of ESRD in children. The diagnosis is based primarily on clinical symptoms; renal ultrasonography usually is not helpful because the finding of mild increased echogenicity of the kidneys is nonspecific. Medullary cysts, which characterize the disease, may be missed on ultrasonography but usually are seen on computed tomography scan. Renal biopsy, which is not always performed, demonstrates tubular damage, with interstitial fibrosis and normal glomeruli. Cysts may be seen in advanced disease. NPH may be associated with other disorders, such as the Senior-Loken syndrome, which is seen in 15% of cases. This syndrome involves the eye, with tapetoretinal degeneration, retinitis pigmentosa (Item C62A), and the development of blindness. Other findings include nystagmus, coloboma (Item C62B), and cataracts (Item C62C). NPH also is associated with Joubert syndrome (NPH with aplasia of the cerebellar vermis causing ataxia and retinal coloboma/retinitis pigmentosa), hepatic fibrosis, and skeletal defects (cone-shaped epiphyses). Four genes have been identified in association with NPH. Most cases of juvenile NPH are related to the NPHP1 gene, which is located on chromosome 2q12.3 and encodes for nephrocystin 1. NPHP2 and NPHP3 encode for proteins for the infantile and adolescent forms of NPH, respectively. Alport syndrome is an X-linked disorder associated with a defect of the alpha 5 chain of type IV collagen (Col4A5). It manifests with hematuria, high-frequency sensorineural hearing loss, and anterior lenticonus (and occasionally cataracts). Diabetic nephropathy can result in retinal and renal disease, but the patient in the vignette lacks a history of diabetes and has no glycosuria. Lowe syndrome is an X-linked disorder characterized by Fanconi syndrome (glycosuria, metabolic acidosis, aminoaciduria, and hypophosphatemia), mental retardation, and congenital cataracts, but it does not result in azotemia, anemia, or retinal abnormalities. Nephropathic cystinosis is a lysosomal storage disorder that has an autosomal recessive inheritance pattern and involves the Fanconi syndrome, failure to thrive, and cystine crystal deposition within the cornea (Item C62D), resulting in severe photophobia.

References:

Hildebrandt F. Nephronophthisis-medullary cystic kidney disease. In: Avner ED, Harmon WE, Niaudet P, eds. Pediatric Nephrology. 5th ed. Philadelphia, Pa: Lippincott Williams & Wilkins; 2004:665-673

Niaudet P. Inherited nephropathies. In: Kher KK, Schnaper HW, Makker SP, eds. Clinical Pediatric Nephrology. 2nd ed. London, England: Informa Healthcare; 2007:195-212.

Saunier S, Calado J, Benessy F, et al. Characterization of the NPHP1 locus: mutational mechanism involved in deletions in familial juvenile nephronophthisis. Am J Hum Genet. 2000;66:778-789. Available at: http://www.pubmedcentral.nih.gov/articlerender.fcgi?tool=pubmed&pubmedid=10712196

Page 214: AAP PREP 2009

2009 PREP SA on CD-ROM

page 214

Critique: 62

Retinitis pigmentosa is observed in a number of inherited disorders. Progressive atrophy of retinal pigment epithelium results in poor night vision and tunnel vision. Abnormalities observed on direct ophthalmoscopy include peripheral "bone spicule" hyperpigmentation (white arrows), optic atrophy, and attenuation of retinal vessels (black arrow).

Page 215: AAP PREP 2009

2009 PREP SA on CD-ROM

page 215

Critique: 62

Keyhole-shaped pupil representing an iris coloboma. (Courtesy of M. Rimsza)

Page 216: AAP PREP 2009

2009 PREP SA on CD-ROM

page 216

Critique: 62

A cataract may result in leukocoria. (Courtesy of M. Rimsza)

Page 217: AAP PREP 2009

2009 PREP SA on CD-ROM

page 217

Critique: 62

Slitlamp examination of the cornea in a boy who has nephropathic cystinosis reveals fine crystals packing the full thickness of the cornea. This produces the speckled appearance of the cornea. (Courtesy of National Eye Institute, National Institute of Health)

Page 218: AAP PREP 2009

2009 PREP SA on CD-ROM

page 218

Question: 63

A mother brings in her 11-month-old son after he broke out in "hives" today during breakfast. The infant had stayed home from child care with a low-grade fever, and the mother had let him eat eggs for the first time. Immediately after breakfast, the mother noted a diffuse erythematous, pruritic rash covering the boy’s trunk and extremities. She is concerned that her son may have an egg allergy.

Of the following, the BEST statement regarding immunoglobulin E-mediated egg food allergy is that

A. cooking the egg eliminates its allergic potential

B. egg is the most common food allergy in the first postnatal year

C. egg white is more allergenic than egg yolk

D. most children do not outgrow their egg allergy

E. the measles-mumps-rubella vaccine is contraindicated in children who have egg allergy

Page 219: AAP PREP 2009

2009 PREP SA on CD-ROM

page 219

Critique: 63 Preferred Response: C

Immunoglobulin (Ig) E-mediated egg allergy is one of the more common childhood food allergies, affecting approximately 1% to 2% of children. As described in the vignette, cutaneous features are common, including atopic dermatitis, urticaria, and pruritus. Once the diagnosis of egg allergy is determined, patients generally are advised to avoid all egg food products with the hope that most children will outgrow their egg allergy within 3 to 5 years (Item C63). The primary allergenic egg protein is ovomucoid, a protein predominantly in the egg white. Approximately 50% of children may be able to tolerate small amounts of egg protein that has been heated extensively (eg, baked goods). Prolonged heating at high temperatures can denature proteins from a conformational form to a linear form. Some children who are allergic to eggs do not recognize the linear protein form as an allergen and, therefore, do not experience a reaction. Of note, the brief cooking used to make scrambled eggs will not denature heat-stable proteins. The relationship between egg allergy and vaccination is a common question. The measles-mumps-rubella vaccine is safe for children who have egg allergy and should be administered without special precautions. The trivalent influenza and live attenuated influenza vaccines contain small amounts of egg protein and are contraindicated for patients who have egg allergy. However, studies have supported a two-dose protocol for the administration of the influenza vaccine in egg-allergic patients. The two-dose protocol involves administering one tenth of the vaccine, observing the recipient for a period of time, and administering the rest of the vaccine, followed by a similar observation period. In westernized countries, milk generally is regarded as the most common food allergen in infants, with an incidence of 2.5%, compared with an incidence of 1.5% for egg allergy.

References:

Boyano-Martínez, T, García-Ara C, Díaz-Pena JM, and Martín-Esteban M. Prediction of tolerance on the basis of quantification of egg white-specific IgE antibodies in children with egg allergy. J Allergy Clin Immunol. 2002;110:304-309. Abstract available at: http://www.ncbi.nlm.nih.gov/pubmed/12170273

Piquer-Gibert M, Plaza-Martín A, Martorell-Aragonés A, et al. Recommendations for administering the triple viral vaccine and antiinfluenza vaccine in patients with egg allergy. Allergol Immunopathol (Madr). 2007;35:209-212. Available at: http://db.doyma.es/cgi-bin/wdbcgi.exe/doyma/mrevista.pubmed_full?inctrl=05ZI0102&rev=105&vol=35&num=5&pag=209

Sampson HA, Leung DYM. Adverse reactions to foods. In: Kleigman RM, Behrman RE, Jenson HB, Stanton BF, eds. Nelson Textbook of Pediatrics. 18th ed. Philadelphia, Pa: Saunders Elsevier; 2007:986-989

Page 220: AAP PREP 2009

2009 PREP SA on CD-ROM

page 220

Critique: 63

Page 221: AAP PREP 2009

2009 PREP SA on CD-ROM

page 221

Question: 64

A 12-year-old boy is brought to the emergency department by emergency medical services after sustaining a lower leg injury sliding into home plate during a baseball game. He tells you that he thinks his leg twisted when he slid. He reports that he had immediate pain in his right ankle and has been unable to walk since the injury occurred. Prior to transport, the paramedics splinted his right lower leg. On physical examination, he has significant swelling and ecchymosis around his distal tibia and fibula. Following the administration of analgesia, radiographs are obtained (Item Q64).

Of the following, the MOST likely complication of this injury is

A. avascular necrosis of the distal tibia

B. osteochondritis desiccans

C. osteomyelitis

D. tibial growth arrest

E. unicameral bone cyst

Page 222: AAP PREP 2009

2009 PREP SA on CD-ROM

page 222

Question: 64

Radiograph findings as exhibited by the boy in the vignette. (Courtesy of D. Mulvihill)

Page 223: AAP PREP 2009

2009 PREP SA on CD-ROM

page 223

Critique: 64 Preferred Response: D

Pediatric fracture patterns differ from those in adults for a variety of biomechanical, physiologic, and anatomic reasons. Most importantly, children are at risk of orthopedic injuries involving the growth plates (physes) that may lead to adverse sequelae if not diagnosed acutely and managed appropriately. Physeal injuries occur most commonly in school-age children and account for 15% to 30% of all pediatric fractures. The physis, or growth plate, of the pediatric bone is located between the metaphysis and the epiphysis. This region of the pediatric skeleton is composed predominantly of cartilage cells that multiply rapidly, calcify along the metaphyses, and lead to longitudinal bone growth. Although the growth activity at the physes is responsible for rapid fracture healing, damage to this area can impair healing and future growth. This region of the bone is particularly vulnerable to injury because the weaker physeal cartilage is less resistant to traumatic forces than the mineralized bone and ligaments. Epiphyseal fractures typically are categorized using the five-tier Salter-Harris classification, a scale based on the degree of physeal, bone, and joint involvement (Item C64A and Item C64B). This categorization largely determines fracture management and prognosis. The radiograph of the boy described in the vignette indicates that he has suffered a type IV tibial fracture (Item C64C). His fracture is unique in that it occurs through three anatomic planes: coronally through the distal tibial metaphysis, transversely through the physis, and sagittally through the epiphysis (ie, a triplane fracture). These fractures occur because the physis has not fused completely. An associated fibular fracture is a common finding. Osteochondritis desiccans most commonly affects the medial femoral condyle of the knee. Caused by repetitive stress, it is characterized by bone fragment development and separation at the posterolateral aspect of the distal femur (Item C64D). Unicameral, or simple, bone cysts usually are found in the metaphyses of long bones of children between 4 and 12 years (Item C64E). They may lead to pathologic fractures but most often are asymptomatic and stop expanding after skeletal maturity is reached. Unless the fracture is open, osteomyelitis is not a likely complication. Distal tibial avascular necrosis has been reported as an extremely rare complication of severe fractures in adults whose physes already are closed.

References:

Dinolfo EA, Adam HM. In brief: fractures. Pediatr Rev. 2004;25:218-219. Available at: http://pedsinreview.aappublications.org/cgi/content/full/25/6/218

Gholve PA, Hosalkar HS, Wells L. Common fractures. In: Kleigman RM, Behrman RE, Jenson HB, Stanton BF, eds. Nelson Textbook of Pediatrics. 18th ed. Philadelphia, Pa: Saunders Elsevier; 2007:2834-2841

Perron AD, Miller MD, Brady WJ. Orthopedic pitfalls in the ED: pediatric growth plate injuries. Am J Emerg Med. 2002;20:50-54. Abstract available at: http://www.ncbi.nlm.nih.gov/pubmed/11781914

Page 224: AAP PREP 2009

2009 PREP SA on CD-ROM

page 224

Critique: 64

Salter-Harris classification system for fractures of the growth plate. See Item C64B for description of fractures, treatment, and prognosis. M= metaphysis, E=epiphysis (Reprinted with permission from Metzl JD, Sports Medicine in the Pediatric Office. Elk Grove Village, Ill: American Academy of Pediatrics; 2008.

Page 225: AAP PREP 2009

2009 PREP SA on CD-ROM

page 225

Critique: 64

Page 226: AAP PREP 2009

2009 PREP SA on CD-ROM

page 226

Critique: 64

Oblique view of the ankle reveals a Salter-Harris type IV fracture of the tibia that passes through the metaphysis, growth plate, and epiphysis (yellow arrows); there is also a frature of the distal fibula (red arrow). This unique fracture occurs in adolescence before there is complete closure of the growth plate. (Courtesy of D. Mulvihill)

Page 227: AAP PREP 2009

2009 PREP SA on CD-ROM

page 227

Critique: 64

Anteroposterior radiograph of the knee in osteochondritis dessicans (dissecans) shows a well-demarcated fragment of subchondral bone surrounded by a lucent line (arrows) that separates it from the lateral condyle of the femur. (Courtesy of D. Rosenbaum)

Page 228: AAP PREP 2009

2009 PREP SA on CD-ROM

page 228

Critique: 64

Unicameral bone cyst: Anteroposterior radiograph of the knee shows a metaphyseal bony defect in the tibia. There is minimal expansion of the bony contours and few septations. (Courtesy of D. Mulvihill)

Page 229: AAP PREP 2009

2009 PREP SA on CD-ROM

page 229

Question: 65

A 4-month-old male infant presents for his initial examination. The family recently immigrated to the United States from southeast Asia. They describe progressive abdominal distention (Item Q65A) in the infant over the past 2 months. Physical examination demonstrates a firm liver edge 2 cm below the right costal margin and a spleen tip palpable 3 cm below the costal margin. Abdominal ultrasonography demonstrates a fluid-filled abdomen (Item Q65B).

Of the following complications from his underlying disorder, this child is MOST at risk for

A. acute intussusception

B. emphysematous chronic lung disease

C. gastrointestinal bleeding

D. pneumococcal meningitis

E. renal failure

Page 230: AAP PREP 2009

2009 PREP SA on CD-ROM

page 230

Question: 65

Abdominal distention, as described for the infant in the vignette. (Courtesy of M. Rimsza)

Page 231: AAP PREP 2009

2009 PREP SA on CD-ROM

page 231

Question: 65

Abdominal ultrasonography of the abdomen demonstrates anechoic fluid surrounding the bladder. (Courtesy of A. Bousvaros)

Page 232: AAP PREP 2009

2009 PREP SA on CD-ROM

page 232

Critique: 65 Preferred Response: C

The firm liver, enlarged spleen, and fluid in the abdomen seen on ultrasonography described for the infant in the vignette is consistent with a number of clinical scenarios, including infection and malignancy. However, the findings strongly suggest chronic liver disease, with secondary portal hypertension resulting in ascites and splenomegaly. Accordingly, he is at risk for esophageal variceal bleeding. Acute intussusception, emphysematous chronic lung disease, pneumococcal meningitis, and renal failure are not associated directly with portal hypertension. Portal hypertension may occur from either hepatic venous thrombosis (Budd-Chiari syndrome), portal venous thrombosis ("cavernous transformation of the portal vein"), or hepatic cirrhosis. Hepatic cirrhosis is the most common cause in both adults and children. In adults, alcoholic cirrhosis is the most common cause. In children, cirrhosis may occur from a number of congenital, hereditary, and infectious conditions. These include biliary atresia, Alagille syndrome, alpha-1-antitrypsin deficiency, cystic fibrosis, hepatitis C, congenital hepatic fibrosis, and congestive heart failure. The portal vein drains the intestinal blood and lymph that subsequently enter the liver, where blood and nutrients are filtered through the hepatic sinusoids. The cirrhotic liver has increased resistance to the flow of portal blood, resulting in increased portal venous pressure. This increase in pressure is transmitted to the splenic vein (resulting in splenomegaly) and the esophageal and hemorrhoidal venous systems (resulting in esophageal varices and hemorrhoids). The changes in oncotic pressure also may result in hypoalbuminemia and accumulation of ascitic fluid in the abdominal cavity, as seen in this infant. The earliest clinical sign that suggests portal hypertension usually is an enlarged spleen. As liver disease progresses, ascites, prominent abdominal veins, and esophageal or gastric varices may develop. Many patients who have portal hypertension exhibit laboratory signs of chronic liver disease, including transaminase elevation and jaundice. However, patients who have some conditions associated with portal hypertension, including congenital hepatic fibrosis and alpha-1-antitrypsin deficiency, may not have jaundice. In such patients, portal hypertension should be suspected if the liver or spleen is chronically enlarged.

References:

Molleston JP. Variceal bleeding in children. J Pediatr Gastroenterol Nutr. 2003;37:538-545

Shepherd RW, Ramm GA. Liver function and dysfunction: fibrogenesis and cirrhosis. In: Walker WA, Goulet O, Kleinman RE, Sherman PM, Shneider BL, Sanderson IR. Pediatric Gastrointestinal Disease. 4th ed. Hamilton, Ontario, Canada: BC Decker; 2004:80-88

Page 233: AAP PREP 2009

2009 PREP SA on CD-ROM

page 233

Question: 66

A 2.1-kg, 34-week gestation infant is delivered to a mother who has chorioamnionitis and had a positive group B streptococcal urinary tract infection at 30 weeks of gestation. Four hours after birth, the infant requires admission to the intensive care nursery because of respiratory distress. Physical examination reveals a temperature of 96.8°F (36.0°C), heart rate of 160 beats/min, respiratory rate of 80 breaths/min, blood pressure of 60/30 mm Hg, mean arterial pressure of 40 mm Hg, and pulse oximetry of 82% on room air. The infant audibly grunts, has flaring of the ala nasi and intercostal and subcostal chest wall retractions, and is poorly perfused, with a capillary refill time of 4 seconds and mild acrocyanosis. There is no heart murmur.

Of the following, the MOST likely radiographic findings expected for this infant are

A. air bronchograms, diffusely hazy lung fields, and low lung volume

B. cardiomegaly, hazy lung fields, and pulmonary vascular engorgement

C. fluid density in the horizontal fissure, hazy lung fields with central vascular prominence, and normal lung volume

D. gas-filled loops of bowel in the left hemithorax and opacification of the right lung field

E. patchy areas of diffuse atelectasis, focal areas of air-trapping, and increased lung volumes

Page 234: AAP PREP 2009

2009 PREP SA on CD-ROM

page 234

Critique: 66 Preferred Response: A

Infants delivered at any gestational age are at risk for respiratory distress from many causes. Respiratory distress syndrome (RDS) is due to surfactant deficiency and characteristically affects newborns delivered at less than 32 weeks' gestation. In the term or late preterm infant, such as the infant described in the vignette, additional or alternative causes of respiratory distress should be considered, including infection, aspiration, cardiac disease, or congenital anomalies (eg, tracheoesophageal fistula, diaphragmatic hernia). Infants born to mothers who have a history of prolonged rupture of membranes, chorioamnionitis, or known maternal group B streptococcal carriage have a high risk for infection, including congenital pneumonia. Respiratory distress may be seen anytime after birth but characteristically is present in the first 24 hours. Tachypnea, tachycardia, and increased work of breathing with a supplemental oxygen requirement are the most common clinical signs. Temperature instability, irritability, lethargy, and poor feeding also may be present. The chest radiograph for a newborn who has congenital pneumonia may resemble that of RDS with hyaline membrane disease (air bronchograms, diffusely hazy lung fields, and low lung volumes) (Item C66A). Rarely, a lobar infiltrate may be seen. The most likely infectious agents in congenital pneumonia include those that ascend from the genital tract before or during labor: group B streptococci, Escherichia coli, and Klebsiella sp. No specific clinical sign or laboratory finding in the sick newborn confirms the presence or absence of pneumonia. Therefore, all newborns who exhibit respiratory distress should have blood cultures obtained and be started on broad-spectrum antibiotic therapy (eg, ampicillin and gentamicin) until a diagnosis of bacterial infection can be excluded. Obtaining a Gram stain of tracheal aspirates from newborns in whom pneumonia is suspected and who have required tracheal intubation and assisted ventilation may be helpful. The chest radiograph depicting cardiomegaly, hazy lung fields, and pulmonary vascular engorgement is characteristic of congenital heart disease with pulmonary overcirculation (eg, critical aortic stenosis, total anomalous pulmonary venous return with obstruction, truncus arteriosus, transposition of the great arteries) (Item C66B). Fluid densities in the horizontal fissure, hazy lung fields with central vascular prominence, and normal lung volumes are characteristic of retained fetal lung fluid, referred to clinically as transient tachypnea of the newborn (Item C66C). Gas-filled loops of bowel in either hemithorax are suspicious for a diaphragmatic hernia, more commonly seen on the left, with a mediastinal shift to the right and compression atelectasis and opacification of the right lung field (Item C66D). Patchy atelectasis with focal areas of air-trapping and increased lung volumes are characteristic of aspiration syndromes, most notably meconium (Item C66E).

References:

Aly H. Respiratory disorders in the newborn: identification and diagnosis. Pediatr Rev. 2004;25:201-208. Available at: http://pedsinreview.aappublications.org/cgi/content/full/25/6/201

Herting E, Gefeller O, Land M, van Sonderen L, Harms K, Robertson B, and Members of the Collaborative European Multicenter Study Group. Surfactant treatment of neonates with respiratory failure and group B streptococcal infection. Pediatrics. 2000;106:957-964. Available at: http://pediatrics.aappublications.org/cgi/content/full/106/5/957

Sivit CJ. Diagnostic imaging. In: Martin RJ, Fanaroff AA, Walsh MC, eds. Fanaroff and Martin's Neonatal-Perinatal Medicine. 8th ed. Philadelphia, Pa: Mosby Elsevier; 2006:713-732

Thilo EH, Rosenberg AA. The newborn infant. In: Hay WW Jr, Levin M, Sondheimer JM, Deterding RR,

Page 235: AAP PREP 2009

2009 PREP SA on CD-ROM

page 235

eds. Current Pediatric Diagnosis & Treatment. 18th ed. New York, NY: The McGraw-Hill Companies, Inc; 2007:chap 1

Page 236: AAP PREP 2009

2009 PREP SA on CD-ROM

page 236

Critique: 66

The chest radiograph in neonatal pneumonia may show features identical to those of respiratory distress syndrome, including underinflation, a "ground glass" appearance, and air bronchograms (arrows). (Courtesy of B. Carter)

Page 237: AAP PREP 2009

2009 PREP SA on CD-ROM

page 237

Critique: 66

Chest radiograph of an infant who has transposition of the great vessels reveals prominent pulmonary vessels (arrows), suggesting pulmonary overcirculation. (Reprinted with permission from Aly H. Respiratory disorders in the newborn: identification and diagnosis. Pediatr Rev. 2004;25:201-208.)

Page 238: AAP PREP 2009

2009 PREP SA on CD-ROM

page 238

Critique: 66

Chest radiograph of transient tachypnea of the newborn shows increased pulmonary interstitial markings and fluid in the interlobar fissures (arrows). (Reprinted with permission from Aly H. Respiratory disorders in the newborn: identification and diagnosis. Pediatr Rev. 2004;25:201-208.)

Page 239: AAP PREP 2009

2009 PREP SA on CD-ROM

page 239

Critique: 66

Plain radiograph of the chest and abdomen in a patient who has congenital diaphragmatic hernia shows bowel in the left chest, with displacement of the heart to the right. (Courtesy of B. Carter)

Page 240: AAP PREP 2009

2009 PREP SA on CD-ROM

page 240

Critique: 66

Meconium aspiration is characterized by areas of atelectasis (arrows) and hyperinflation (seen best at the bases). (Reprinted with permission from Aly H. Respiratory disorders in the newborn: identification and diagnosis. Pediatr Rev. 2004;25:201-208.)

Page 241: AAP PREP 2009

2009 PREP SA on CD-ROM

page 241

Question: 67

A 16-month-old boy is brought to your clinic because his mother says he is "walking funny" today. She states that he has been walking for 4 months and is very active, but she is unaware of any trauma or falls. She denies fever or other symptoms. He appears well and has normal vital signs. Physical examination reveals mild tenderness to palpation over the medial aspect of the lower leg just above the ankle. There is no overlying bruising, erythema, or edema, and you can elicit full range of motion in the hips, knees, and ankles.

Of the following, the MOST likely diagnosis is

A. aneurysmal bone cyst

B. ankle sprain

C. fracture

D. osteomyelitis

E. transient synovitis

Page 242: AAP PREP 2009

2009 PREP SA on CD-ROM

page 242

Critique: 67 Preferred Response: C

The differential diagnosis of the child who has an acute limp includes infectious causes such as osteomyelitis and septic arthritis, postinfectious entities such as transient synovitis and rheumatic fever, and injuries related to accidental or nonaccidental trauma. The presence of fever or other signs and symptoms, including erythema and limitation of joint motion, should lead the clinician to investigate infectious or postinfectious causes. However, in the absence of constitutional symptoms and typical external signs of infection, such as described for the boy in the vignette, an occult fracture should be considered, even if no history of trauma is elicited. Careful questioning regarding any falls or injuries, observation of the parent-child interaction, and a complete physical examination that includes the skin are important because many children who have fractures from inflicted trauma come to medical attention because of vague limb pain or limp. However, the presence of a fracture in a toddler does not necessarily indicate inflicted injury (eg, child abuse) because children who are just learning to walk and run can fall and sustain injury without the parent's awareness of the event. Anteroposterior and lateral radiographs of the lower extremities are indicated if a fracture is suspected, and the type of fracture seen can provide some clues as to the mechanism of injury. One of the most common fractures in children is a nondisplaced spiral fracture of the tibia, otherwise as known as a "toddler's fracture" (Item C67A). The mechanism of injury is a twisting movement of the lower leg that occurs as the toddler learns to move around his or her environment. Examination often reveals only a limp or tenderness over the affected area. Initial radiographs can appear normal, with a periosteal reaction appearing in follow-up films. Torus (buckle) and greenstick (Item C67B) fractures also can be seen with relatively minor trauma because children's bones are more porous and more likely to "buckle" or bend with trauma than adult bones. Fractures of the growth plate are common with trauma and are particularly concerning because of the risk of growth retardation at the site of the fracture. The Salter-Harris system is used to classify types of growth plate fractures and identify patients who are at high risk of growth failure. Finally, some types of fractures should alert the clinician immediately to the possibility of child abuse, including multiple fractures in different stages of healing, metaphyseal "chip" fractures (Item C67C), femoral fracture in children younger than 1 year of age (Item C67D), and scapular fracture. The minimal external signs and no constitutional symptoms such as fever reported for the child in the vignette make the diagnosis of osteomyelitis unlikely. Transient synovitis occurs after a viral illness and generally affects the knees and hips. Some limitation of motion at the joint is expected, but external signs such as erythema typically are absent. Aneurysmal bone cysts are metaphyseal lesions (Item C67E) that usually occur in adolescence and cause pain with or without swelling. The ligaments and tendons in children generally are stronger than the growing bones underneath them, in contrast to adults, whose bones are stronger. The amount of injuring force required to cause a sprain in an adult is more likely to cause a fracture in a child.

References:

Dinolfo EA. Fractures. Pediatr Rev. 2004;25:218-219. Available at: http://pedsinreview.aappublications.org/cgi/content/full/25/6/218

Eiff MP, Hatch RL. Boning up on common pediatric fractures. Contemp Pediatr. 2003;20:30-59

Kellogg ND and the Committee on Child Abuse and Neglect. Evaluation of suspected child physical abuse. Pediatrics. 2007;119:1232-1241. Available at: http://pediatrics.aappublications.org/cgi/content/full/119/6/1232

Page 243: AAP PREP 2009

2009 PREP SA on CD-ROM

page 243

Critique: 67

Oblique (left) and anteroposterior (right) views of the distal tibia reveal a nondisplaced spiral (toddler's) fracture. (Courtesy of D. Mulvihill)

Page 244: AAP PREP 2009

2009 PREP SA on CD-ROM

page 244

Critique: 67

A greenstick fracture is characterized by a fracture in one cortex (yellow arrow) and buckling of the opposite side. This phenomenon occurs because of the elasticity of children's bones. (Courtesy of D. Mulvihill)

Page 245: AAP PREP 2009

2009 PREP SA on CD-ROM

page 245

Critique: 67

Metaphyseal corner (chip) fractures may be observed in children who are the victims of nonaccidental trauma. (Courtesy of D. Krowchuk)

Page 246: AAP PREP 2009

2009 PREP SA on CD-ROM

page 246

Critique: 67

Spiral fracture of the femur in a 6-week-old infant who had been physically abused. (Courtesy of D. Krowchuk)

Page 247: AAP PREP 2009

2009 PREP SA on CD-ROM

page 247

Critique: 67

Aneurysmal bone cyst: Lateral radiograph of the humerus shows a lytic, expansile lesion with thin internal strands and a thin continuous rim of bone. (Courtesy of D. Mulvihill)

Page 248: AAP PREP 2009

2009 PREP SA on CD-ROM

page 248

Question: 68

An 11-year-old girl presents 2 weeks after an office visit for a presumed viral illness characterized by fever, malaise, and flushing of the cheeks. Today, her mother notes that she no longer has a fever, but she complains of pain in her knees and elbows. On physical examination, the left knee is slightly swollen and warm but not erythematous. The girl reports pain on movement of both elbows, but there are no physical findings on examination of the elbows or other joints. The remainder of the physical examination findings are normal, except for an oral temperature of 100.6°F (38.1°C). Results of laboratory studies include a white blood cell count of 8.9x103/mcL (8.9x109/L) with 40% polymorphonuclear leukocytes, 45% lymphocytes, and 15% monocytes; hemoglobin of 11.0 g/dL (110.0 g/L); platelet count of 472.0x103/mcL (472.0x109/L); and erythrocyte sedimentation rate of 20 mm/hr.

Of the following, the MOST likely pathogen to cause this child’s joint complaints is

A. Borrelia burgdorferi

B. Coxsackievirus

C. group A beta-hemolytic streptococci

D. influenza A virus

E. parvovirus B19

Page 249: AAP PREP 2009

2009 PREP SA on CD-ROM

page 249

Critique: 68 Preferred Response: E

The girl described in the vignette has developed swelling of the knee and arthralgias following a recent febrile illness, which is typical of a postinfectious arthritis. The flushing of her cheeks suggests that her febrile illness was due to human parvovirus B19 infection, commonly known as erythema infectiosum (EI) (Item C68). Parvovirus B19 infection is a common cause of postinfectious arthritis. Multiple viruses can cause postinfectious arthritis, including influenza, hepatitis B, rubella, and Epstein-Barr. Arthralgias may occur in 10% of children who have clinical or laboratory evidence of EI. Older children, particularly girls and young women, frequently experience involvement of the knees, although involvement of both large and small joints has been reported. Parvovirus B19 has been detected in synovial fluid and serum samples of such patients. Some clinical features of parvovirus B19 infection are similar to those of autoimmune connective tissue diseases, and some children who have EI may develop positive antinuclear antibody serum test results or rheumatoid factor-positive serology. Lyme disease results from infection with the spirochete Borrelia burgdorferi, which is transmitted by deer tick bites. In North America, Lyme disease is most prevalent in the northeastern, midwestern, and southern and western coastal areas of the United States as well as in Ontario, Canada. School-age children are affected most commonly, with boys and girls affected equally. Arthritis is the second most frequent presentation of Lyme disease, following the cutaneous signs of erythema migrans. Arthralgias usually develop in the early phase, but the onset of arthritis may occur months to years after the original infection. Initially, the arthritis is episodic, but it may evolve to a recurrent and prolonged condition. Two thirds of affected children present with monoarthritis of the knee, but oligoarticular involvement of the large joints and, rarely, a polyarthritis of the small joints also can occur. The diagnosis of Lyme arthritis is based on history and physical examination findings as well as laboratory tests to document infection with B burgdorferi. The immunoglobulin G titers to B burgdorferi can remain positive for years and, therefore, cannot be used to monitor treatment response or failure. In contrast to adults, the prognosis for Lyme arthritis in children generally is good, and symptoms resolve over time without permanent damage to joints. Group A beta-hemolytic streptococcal infection can be associated with arthritis that may occur during the acute illness or after the acute illness has resolved (poststreptococcal reactive arthritis). Other bacterial causes of postinfectious arthritis include Neisseria gonorrhoeae, Staphylococcus aureus, and other streptococcal species. Coxsackieviruses are not often associated with arthritis. Enterovirus, hepatitis B, rubella, and mumps infections may cause transient arthritis. Many other infections result in malaise and myalgias associated with a prodrome, but they do not cause true arthritis.

References:

American Academy of Pediatrics. Enterovirus (nonpoliovirus) infections (group A and B Coxsackieviruses, echoviruses, numbered enteroviruses. In: Pickering LK, Baker CJ, Long SS, McMillan JA, Eds. Red Book: 2006 Report of the Committee on Infectious Diseases. 27th ed. Elk Grove Village, Ill: American Academy of Pediatrics; 2006:284-285

American Academy of Pediatrics. Group A streptococcal infections. In: Pickering LK, Baker CJ, Long SS, McMillan JA, Eds. Red Book: 2006 Report of the Committee on Infectious Diseases. 27th ed. Elk Grove Village, Ill: American Academy of Pediatrics; 2006:610-620

American Academy of Pediatrics. Influenza. In: Pickering LK, Baker CJ, Long SS, McMillan JA, Eds. Red Book: 2006 Report of the Committee on Infectious Diseases. 27th ed. Elk Grove Village, Ill: American Academy of Pediatrics; 2006:401-411

Page 250: AAP PREP 2009

2009 PREP SA on CD-ROM

page 250

American Academy of Pediatrics. Parvovirus B19/erythema infectiosum, fifth disease). In: Pickering LK, Baker CJ, Long SS, McMillan JA, Eds. Red Book: 2006 Report of the Committee on Infectious Diseases. 27th ed. Elk Grove Village, Ill: American Academy of Pediatrics; 2006:484-487

Khouqeer R, Cohen M. Viral arthritis. eMedicine Specialties, Rheumatology, Infectious Arthritis. 2006. Available at: http://www.emedicine.com/med/topic3414.htm

Koch WC. Parvovirus B19. In: Kliegman RM, Behrman RE, Jenson HB, Stanton BF, eds. Nelson's Textbook of Pediatrics. 18th ed. Philadelphia, Pa: Saunders Elsevier; 2007:1357-1359

Lehman HW, Knöll, A, Küster RM, Modrow S. Frequent infection with a viral pathogen, parvovirus B19, in rheumatic diseases of childhood. Arthritis Rheum. 2003;48:1631-1638. Available at: http://www3.interscience.wiley.com/cgi-bin/fulltext/104536478/HTMLSTART

Siegel DM. In brief: antinuclear antibody (ANA) testing. Pediatr Rev. 2003;24:320-321. Available at: http://pedsinreview.aappublications.org/cgi/content/full/24/9/320

Tse SML, Laxer RM. Approach to acute limb pain in childhood. Pediatr Rev. 2006;27:170-180. Available at: http://pedsinreview.aappublications.org/cgi/content/full/27/5/170

Page 251: AAP PREP 2009

2009 PREP SA on CD-ROM

page 251

Critique: 68

A feature of erythema infectiosum is flushed cheeks. (Courtesy of D. Krowchuk)

Page 252: AAP PREP 2009

2009 PREP SA on CD-ROM

page 252

Question: 69

You are prescribing atenolol for a 15-year-old boy in whom you diagnosed hypertrophic cardiomyopathy. There is a family history of asthma. He is concerned about the potential adverse effects of medicines.

Of the following, a TRUE statement about treatment with this drug is that

A. he cannot use atenolol because of his family history of asthma

B. he has an increased risk for developing diabetes mellitus

C. he may develop gynecomastia

D. his school performance may be adversely affected

E. tachyphylaxis may develop over time

Page 253: AAP PREP 2009

2009 PREP SA on CD-ROM

page 253

Critique: 69 Preferred Response: D

Beta-blocking drugs have an antagonistic effect on the beta-adrenergic receptors. This diverse and widely used group of medications can have multiple and various effects on the heart and other organ systems. In the heart, they typically exhibit some degree of negative chronotropic (slowing of the heart rate), negative dromotropic (slowing of the conduction through the atrioventricular node), and negative inotropic (decrease in the ventricular force of contraction) effects. As a result, this class of medications is used in the treatment of many pediatric conditions, including hypertension, arrhythmias, hypertrophic cardiomyopathy, dilated cardiomyopathy, Marfan syndrome, and migraine prophylaxis. The adverse effects of beta-blocker drugs have been well-documented and may be divided into three broad categories based on end-organ effects (Item C69). The patient described in the vignette has been prescribed a beta blocker that has relative cardiac selectivity. Some children and adolescents experience a decrease in school performance while being treated with beta blockers, most likely related to the central nervous penetration, and it may be difficult to differentiate this effect from the known fatigue or depression that also can occur. Although bronchospasm or asthma is a contraindication to the use of some beta blockers (eg, propranolol), a family history alone is not a contraindication for their use. Some beta blockers can cause a decreased reaction to hypoglycemia, leading to a relative contraindication of their use in those who have diabetes, but their use does not increase the risk of developing diabetes mellitus. Similarly, beta blocker use typically does not lead to gynecomastia. Although reflex tachycardia may be noted when chronic beta blocker use is discontinued, these drugs are not associated with tachyphylaxis.

References:

Feld LG, Corey H. Hypertension in childhood. Pediatr Rev. 2007;28:283-298. Available at: http://pedsinreview.aappublications.org/cgi/content/full/28/8/283

Opie LH, Sonnenblick EH, Frishman WH, Thadani U. Beta-blocking agents. In: Opie LH, ed. Drugs for the Heart. 4th ed. Philadelphia, Pa: W.B. Saunders Co; 1995:1-30

Page 254: AAP PREP 2009

2009 PREP SA on CD-ROM

page 254

Critique: 69

Page 255: AAP PREP 2009

2009 PREP SA on CD-ROM

page 255

Question: 70

A 14-year-old girl is brought to the emergency department because she has back pain and a sudden inability to walk. Neurologic examination shows normal upper limb strength. However, her legs are flaccid, relatively symmetrically weak, areflexic, and numb to pinprick. Vibratory and position sense in the legs persists. A sensory deficit exists below the sixth thoracic dermatome. Rectal examination shows low rectal tone. The remainder of her physical examination findings, including vital signs, are normal.

Of the following, the MOST appropriate next diagnostic study is emergent

A. brain magnetic resonance imaging

B. lumbar puncture

C. nerve conduction/electromyography of the legs

D. somatosensory evoked potentials

E. spine magnetic resonance imaging

Page 256: AAP PREP 2009

2009 PREP SA on CD-ROM

page 256

Critique: 70 Preferred Response: E

The child described in the vignette has findings consistent with an acute spinal cord lesion, which is a neurologic emergency. In some cases, emergency neurosurgery is needed for lesions causing acute spinal cord compression, and the initial diagnostic emphasis should be directed toward identifying such lesions. The first step is to localize the problem to the proper level of the nervous system: brain, brainstem/cerebellum, spinal cord, nerve, junction, or muscle. Such localization not only allows for urgent appropriate diagnosis and treatment but can reduce discomfort, risks, and costs of inappropriate diagnostic testing. Urgent phone or in-person consultation with a neurologist or neurosurgeon can be useful before ordering testing. The back pain, acute bilateral flaccid weakness, and sensory loss below the level of the lesion described for the girl in the vignette localize this problem to the spinal cord. In this case, the rapid onset of symptoms and the preservation of vibratory and proprioceptive sensation point to the anterior cord, bilaterally, which is consistent with an anterior spinal artery stroke. Spinal cord strokes in children are uncommon but can occur after aortic surgery or as a consequence of thrombotic disorders, infection, inflammatory diseases, or trauma. Based on the history and examination findings, magnetic resonance imaging (MRI) of the spine is the test of choice. This should aid in determining the specific location of the lesion and whether emergency neurosurgery to decompress the spinal cord is needed. Brain MRI is not the initial modality of choice because of the urgency of the presentation and the localization of the lesion suggested by clinical findings. If a spinal cord lesion is radiologically consistent with the diagnosis of transverse myelitis, a subsequent brain MRI with gadolinium should be obtained to look for evidence of demyelinating lesions in the brain, which is supportive of a diagnosis of multiple sclerosis. A subsequent lumbar puncture also may help identify the cause of the patient's symptoms, but it may not be necessary and should not delay obtaining the spine MRI. Lumbar puncture is the appropriate test for suspected Guillain-Barré syndrome, an acute inflammatory demyelinating polyneuropathy, which can present with weakness beginning in the proximal legs but not with the sensory findings seen in this child. Lumbar puncture is also helpful for identifying inflammatory causes of spinal cord disease. Somatosensory evoked potentials are neurophysiologic tests that document the integrity of sensory pathways from peripheral nerves through the spinal cord to the brain. They are not part of standard emergency management in this clinical setting. Nerve conduction studies and electromyography are not needed to assess the nerves because the problem localizes to spinal cord.

References:

Hakimi KN, Massagli TL. Anterior spinal artery syndrome in two children with genetic thrombotic disorders. J Spinal Cord Med. 2005;28:69-73. Abstract available at: http://www.ncbi.nlm.nih.gov/pubmed/15832907

Haslam RHA. Spinal cord disorders. In: Kliegman RM, Behrman RE, Jenson HB, Stanton BF, eds. Nelson Textbook of Pediatrics. 18th ed. Philadelphia, Pa: Saunders Elsevier; 2007:2526-2530

Menkes JH, Ellenbogen RC. Traumatic brain and spinal cord injuries in children. In: Maria BL, ed. Current Management in Child Neurology. 3rd ed. Hamilton, Ontario, Canada: BC Decker Inc; 2005:515-527

Nance JR, Golomb MR. Ischemic spinal cord infarction in children without vertebral fracture. Pediatr Neurol. 2007;36:209-216. Abstract available at: http://www.ncbi.nlm.nih.gov/pubmed/17437902

Page 257: AAP PREP 2009

2009 PREP SA on CD-ROM

page 257

Question: 71

An infant in the newborn nursery is normally grown and normally formed, except for a preauricular pit (preauricular sinus) bilaterally (Item Q71). He has passed his newborn hearing screening. When you meet the baby’s mother, you learn that she has progressive, bilateral sensorineural hearing loss for which she uses hearing aids.

Of the following, the MOST helpful test to aid in diagnosis and management of this baby’s condition is

A. chromosome analysis

B. head ultrasonography

C. ophthalmology consultation

D. radiographic skeletal survey

E. renal ultrasonography

Page 258: AAP PREP 2009

2009 PREP SA on CD-ROM

page 258

Question: 71

(Courtesy of P. Sagerman)

Page 259: AAP PREP 2009

2009 PREP SA on CD-ROM

page 259

Critique: 71 Preferred Response: E

For many years, the association between ear anomalies and other malformations, especially involving the kidneys, have been noted in the medical literature. Based on numerous studies, the documented frequency of defects in other organ systems is 5% to 40% in children who have ear anomalies. Largely due to the association between crumpled, deformed ears and renal agenesis (as seen in Potter sequence), it has become customary in many centers to evaluate the kidneys in the newborn, even if only a preauricular pit/sinus (PPS) is noted (Item C71). However, recent studies have shown that PPS alone is not associated with an increased risk for renal anomalies, and renal ultrasonography is not warranted in these cases. It is critical to evaluate newborns and children who have auricular anomalies carefully for dysmorphic features and evidence of organ malformations. It is important to know if the mother has diabetes because diabetic embryopathy is associated with ear anomalies. In addition, a family history should be obtained, with special attention to any individuals who have ear anomalies, deafness, and renal malformations. Should such a history be elicited, as for the mother in the vignette, renal ultrasonography is warranted, and further evaluations may be in order. Chromosome analysis, head ultrasonography, ophthalmology consultation, and radiographic skeletal survey are not necessary initial tests for a child who has auricular anomalies.

References:

Adam M, Hudgins L. The importance of minor anomalies in the evaluation of the newborn. NeoReviews. 2003;4:e99-e104. Available for subscription at: http://neoreviews.aappublications.org/cgi/content/full/4/4/e99

Arora RS, Pryce R. Is ultrasonography required to rule out renal malformations in babies with isolated preauricular tags? Arch Dis Child. 2004;89:492-493

Huang XY, Tay GS, Wansaicheong GK-L, Low WK. Preauricular sinus: clinical course and associations. Arch Otolaryngol Head Neck Surg. 2007;133:65-68. Available at: http://archotol.ama-assn.org/cgi/content/full/133/1/65

Wang RY, Earl DL, Ruder RO, Graham JM Jr. Syndromic ear anomalies and renal ultrasounds. Pediatrics. 2001;108:e32. Available at: http://pediatrics.aappublications.org/cgi/content/full/108/2/e32

Page 260: AAP PREP 2009

2009 PREP SA on CD-ROM

page 260

Critique: 71

A preauricular pit. (Courtesy of P. Sagerman)

Page 261: AAP PREP 2009

2009 PREP SA on CD-ROM

page 261

Question: 72

A 16-year-old girl comes to your office with complaints of a thick white vaginal discharge. She is sexually active with one partner with whom she always uses condoms. She has no complaints of fever or abdominal pain, but she reports external "burning" of the vaginal area when she urinates. On physical examination, she is afebrile. Pelvic examination reveals fiery red labia majora and minora and an adherent white discharge on the vaginal walls, with a moderate amount of white discharge in the vaginal vault. The speculum examination is uncomfortable for her, but there is no cervical motion, uterine, or adnexal tenderness, and the cervix shows no friability or discharge.

Of the following, the MOST likely pathogen responsible for this patient’s symptoms is

A. Candida albicans

B. Chlamydia trachomatis

C. group A Streptococcus

D. Neisseria gonorrhoeae

E. Trichomonas vaginalis

Page 262: AAP PREP 2009

2009 PREP SA on CD-ROM

page 262

Critique: 72 Preferred Response: A

Candidal vulvovaginitis is a common problem for young women and usually is caused by Candida albicans, although other candidal species also may be involved. Typical symptoms of vulvovaginal candidiasis include a thick, white, creamy vaginal discharge; pruritus; vaginal discomfort; dyspareunia; and external dysuria. The diagnosis is suggested clinically by the previously noted symptoms and the presence of vulvar swelling, erythema, and fissures or erosions, as described for the girl in the vignette. The diagnosis may be confirmed by a wet preparation or Gram stain showing pseudohyphae or yeasts (Item C72A) or by culture. Chlamydia trachomatis and Neisseria gonorrhoeae, both of which may present with abnormal vaginal discharge, produce cervical, not vaginal infections. C trachomatis and N gonorrhoeae infections can be asymptomatic, but they often present with a yellowish purulent or mucopurulent endocervical discharge, friability of the endocervix, and cervical motion tenderness; vulvar inflammation does not occur. Group A Streptococcus (S pyogenes) is a respiratory pathogen that can cause vaginitis in prepubertal girls, but rarely causes vaginal discharge in the adolescent. Infection with Trichomonas vaginalis usually causes a diffuse, malodorous, yellow-green vaginal discharge (Item C72B) with vulvar irritation, although some affected women can have minimal or no symptoms.

References:

American Academy of Pediatrics. Chlamydia trachomatis. In: Pickering LK, Baker CJ, Long SS, McMillan JA, eds. Red Book: 2006 Report on the Committee on Infectious Diseases. 27th ed. Elk Grove Village, Ill: American Academy of Pediatrics; 2006:252-257

American Academy of Pediatrics. Gonococcal infections. In: Pickering LK, Baker CJ, Long SS, McMillan JA, eds. Red Book: 2006 Report on the Committee on Infectious Diseases. 27th ed. Elk Grove Village, Ill: American Academy of Pediatrics; 2006:301-309

American Academy of Pediatrics. Group A streptococcal infections. In: Pickering LK, Baker CJ, Long SS, McMillan JA, eds. Red Book: 2006 Report on the Committee on Infectious Diseases. 27th ed. Elk Grove Village, Ill: American Academy of Pediatrics; 2006:610-620

Emans SJ. Office evaluation of the child and adolescent. In: Emans SJH, Laufer MR, Goldstein DP, eds. Pediatric and Adolescent Gynecology. 5th ed. Philadelphia, Pa: Lippincott, Williams & Wilkins; 2005:1-50

Workowski KA, Berman SM, Centers for Disease Control and Prevention. Sexually transmitted diseases treatment guidelines, 2006. MMWR Recomm Rep. 2006;55(RR11):1-94. Available at: http://www.cdc.gov/mmwr/preview/mmwrhtml/rr5511a1.htm

Page 263: AAP PREP 2009

2009 PREP SA on CD-ROM

page 263

Critique: 72

Numerous pseudohyphae (arrows) observed on a normal saline preparation of vaginal secretions from a patient who has vaginal candidiasis. (Courtesy of D. Krowchuk)

Page 264: AAP PREP 2009

2009 PREP SA on CD-ROM

page 264

Critique: 72

Vaginal discharge caused by trichomoniasis often is yellow-green and frothy.

Page 265: AAP PREP 2009

2009 PREP SA on CD-ROM

page 265

Question: 73

An 18-month-old boy fell into a swimming pool 12 hours ago. He had no heart rate when he was pulled from the pool, and cardiopulmonary resuscitation (CPR) was initiated at the scene. The CPR was continued for 30 minutes until spontaneous circulation was restored in the emergency department. He is now in the pediatric intensive care unit, receiving mechanical ventilation with maximal intensive care support. Over the past several hours, his blood pressure has increased, he has developed persistent bradycardia, and he exhibits no movement in response to stimulation. He has not received any neuromuscular blockers or sedation. In addition, his pupils are dilated bilaterally and do not respond to light. Bedside electroencephalography demonstrates generalized burst suppression with loss of reactivity to external stimuli.

In discussion with his parents, you inform them that these recent changes are MOST likely a result of

A. agitation

B. increasing intracranial pressure

C. myocardial failure

D. ongoing seizure activity

E. physiologic response to the ventilator

Page 266: AAP PREP 2009

2009 PREP SA on CD-ROM

page 266

Critique: 73 Preferred Response: B

Hypoxic-ischemic injury, which occurs after periods of impaired perfusion, such as with cardiopulmonary arrest, produces a broad spectrum of organ failure. Unfortunately, the brain is one of the organs most vulnerable to decreased oxygen and nutrient delivery, and hypoxic-ischemic central nervous system (CNS) injury is common following asphyxia. Irreversible CNS injury may occur after as little as 3 to 5 minutes of interrupted blood flow or oxygen delivery. Both ischemia and hypoxia trigger numerous pathophysiologic processes that result in cellular injury, cell death, and subsequent development of cerebral edema that compromises blood flow to adjacent areas of the brain. These areas are initially either potentially recoverable or uninjured but at risk due to impaired perfusion. The boy described in the vignette is exhibiting evidence of increased intracranial pressure due to significant global hypoxic-ischemic injury. His hypertension and bradycardia represent the Cushing reflex (the raising of systemic arterial pressure to increase cerebral perfusion with associated stimulation of the carotid bodies and subsequent bradycardia). Cushing triad (systemic arterial hypertension, bradycardia, and depressed or irregular respirations) is a late sign of increased intracranial pressure that often develops just prior to cerebral herniation. The lack of spontaneous movements and dilated, unresponsive pupils described for the boy are consistent with severe CNS injury and a poor outcome. Agitation or pain due to underlying injuries, ventilator asynchrony, or anxiety can produce hypertension, but this typically is associated with tachycardia and increased motor movements. Myocardial hypoxic-ischemic injury normally is manifested by decreased ventricular function and hypotension. Although patients who have hypoxic-ischemic injuries are at risk for the development of seizures, the electroencephalographic findings in this patient are consistent with a severe hypoxic-ischemic encephalopathy and portend a poor prognosis. Abnormal motor movements or vital signs in combination with neurologic examination results that are not consistent with a known cause should prompt the consideration of seizure and subsequent evaluation and treatment.

References:

Doherty DR, Hutchison JS. Hypoxic ischemic encephalopathy after cardiorespiratory arrest. In: Wheeler DS, Wong HR, Shanley T, eds. Pediatric Critical Care Medicine: Basic Science and Clinical Evidence. New York, NY: Springer-Verlag; 2007:935-946

Kallas HJ. Drowning and submersion injury. In: Kliegman RM, Behrman RE, Jenson HB, Stanton BF, eds. Nelson Textbook of Pediatrics. 18th ed. Philadelphia, Pa: Saunders Elsevier; 2007:438-449

Meyer RJ, Theodorou AA, Berg RA. Childhood drowning. Pediatr Rev. 2006;27:163-169. Available at: http://pedsinreview.aappublications.org/cgi/content/full/27/5/163

Page 267: AAP PREP 2009

2009 PREP SA on CD-ROM

page 267

Question: 74

On the initial health supervision visit of a 7-year-old boy who is new to your practice, you note that his height is 43 inches, which is at the 50th percentile for a 5-year-old, and that his weight is appropriate for his age. His parents say that he has been wearing the same size clothes for at least the past year. The boy also has dry skin. You suspect he has hypothyroidism and decide to measure thyroid-stimulating hormone concentrations.

Of the following, the MOST appropriate additional study needed to evaluate this child for hypothyroidism is

A. bone age radiography

B. measurement of free thyroxine

C. measurement of total thyroxine

D. measurement of tri-iodothyronine

E. thyroid ultrasonography

Page 268: AAP PREP 2009

2009 PREP SA on CD-ROM

page 268

Critique: 74 Preferred Response: B

The initial evaluation of the boy described in the vignette should be directed at determining whether he has hypothyroidism and defining its severity. The assay of free thyroxine (FT4) generally provides the best estimate of circulating active concentrations of thyroxine. Total thyroxine (TT4) concentrations are dependent on circulating thyroxine and on circulating binding proteins, including thyroxine-binding globulin (TBG) and thyroxine-binding prealbumin (TBPA). In general, TT4 is an accurate measure of thyroid hormone sufficiency, but about 1 in 8,000 individuals has familial sex-linked deficiency or excess of TBG and, therefore, has a TT4 value that cannot be interpreted properly. Although hypothyroidism in children usually is caused by primary thyroid gland failure, a substantial proportion of affected individuals could have a hypothalamic or pituitary disease, in which case the FT4 value is low and the thyroid-stimulating hormone value is normal or low. Tri-iodothyronine (T3) is the active form of intracellular thyroid hormone. Only about 15% of T3 is produced by the thyroid gland; the rest is produced by the enzymatic deiodination of T4 in most cells of the body, including the hepatocyte. Because the process of enzymatic deiodination is nutritionally controlled, individuals who are particularly well-fed or obese or small children who eat frequently have elevated T3 values compared with adult normal values. In contrast, sick people or people who are not eating well have very low values. Therefore, T3 cannot be used to diagnose hypothyroidism. Children who have hypothyroidism often have delayed bone age, which may be used as an indicator of how long the hypothyroidism has been present. Although a bone age radiograph might be part of the extended evaluation of hypothyroidism, the diagnosis first must be established by measuring FT4. Most children who have acquired primary hypothyroidism have chronic lymphocytic thyroiditis, with measurable serum antibodies directed against the thyroperoxidase enzyme, the thyroid microsomes that contain these antibodies, or thyroglobulin. In addition, they often have palpable thyroid glands. If they do not have positive antithyroid antibodies, thyroid ultrasonography can aid in assessing the shape and position of the thyroid gland. Some affected children could have congenitally abnormal thyroid glands with maldescent but enough thyroid hormone production not to become hypothyroid until early childhood. Assessment using technetium or radioactive iodine imaging also might be appropriate.

References:

Hunter I, Greene SA, MacDonald TM, Morris AD. Prevalence and aetiology of hypothyroidism in the young. Arch Dis Child. 2000;83:207-210. Abstract available at: http://www.ncbi.nlm.nih.gov/pubmed/10952634

LaFranchi S. Acquired hypothyroidism in childhood and adolescence. UpToDate Online 15.3. 2008. Available for subscription at: http://www.uptodateonline.com/utd/content/topic.do?topicKey=pediendo/4633

Ferry RJ Jr, Bauer AJ. Hypothyroidism. eMedicine Specialties, Pediatrics, Endocrinology. 2006. Available at: http://www.emedicine.com/ped/TOPIC1141.HTM

Page 269: AAP PREP 2009

2009 PREP SA on CD-ROM

page 269

Question: 75

An 8-year-old boy has difficulty in academics and a short attention span. His father states that he had the same problems when he was a child. Physical examination reveals macrocephaly, multiple café au lait macules (Item Q75A) and axillary freckles (Item Q75B). Upon questioning, the father explains that he has similar skin findings.

Of the following, the MOST likely diagnosis is

A. fragile X syndrome

B. hypomelanosis of Ito

C. neurofibromatosis type 1

D. tuberous sclerosis

E. velocardiofacial syndrome

Page 270: AAP PREP 2009

2009 PREP SA on CD-ROM

page 270

Question: 75

Café au lait macules, as described for the child in the vignette. (Courtesy of P. Fisher)

Page 271: AAP PREP 2009

2009 PREP SA on CD-ROM

page 271

Question: 75

Axillary freckling and a café au lait macule, as described for the child in the vignette. (Courtesy of D. Krowchuk)

Page 272: AAP PREP 2009

2009 PREP SA on CD-ROM

page 272

Critique: 75 Preferred Response: C

The frequency of learning difficulties is increased in individuals who have a positive family history for learning disorders or genetic syndromes such as neurofibromatosis type 1 (NF-1), fragile X syndrome, or velocardiofacial syndrome (VCFS). It is important to obtain a family history when evaluating children who have suspected learning disorders to ascertain if there is a possible genetic cause. A thorough family history also may identify speech or language problems in up to 30% of first-degree relatives of children who have language delays, a risk factor for learning difficulties. The multiple café au lait macules and axillary freckling described for the boy in the vignette meet diagnostic criteria for NF-1. The offspring of an affected individual has a 50% risk of inheriting the altered NF1 gene, which can be detected by molecular genetic testing, although this rarely is necessary for diagnosis. Learning disabilities occur with increased frequency in those who have a family history of NF-1, ranging from 40% to 60%. Fragile X syndrome, the most common genetic cause of intellectual disabilities, occurs in 1 in 1,200 males and 1 in 2,500 females. A mutation in the FMR1 gene (Xq27.3) leads to expansion of the CGG trinucleotide repeat. Women who have alleles in the range of 29 to 200 CGG repeats are at risk for giving birth to affected children. Males who have fragile X syndrome have large heads, long faces, large ears, and macro-orchidism but no skin findings. Males who have the full mutation have mean intelligence quotient (IQ) scores of 40; affected females have mean IQ scores in the low-average to borderline range. Velocardiofacial syndrome affects 1 in 400 individuals, and 10% of cases are inherited in an autosomal dominant pattern. The condition is caused by a microdeletion on chromosome 22q11.2 and may be associated with cardiac and facial anomalies, but not cutaneous abnormalities. Learning difficulties are reported in 82% to 100% of affected children. Hypomelanosis of Ito is a congenital skin disorder that is characterized by linear or whorled hypopigmentation that follows the lines of Blaschko, the paths of embryonic neural crest cell migration (Item C75A). There is no evidence for genetic transmission. Associated intellectual disability has been reported in as many as 70% of cases, seizures in 40%, and microcephaly in 25%. However, these high percentages may be the result of selection bias, and the actual frequency of associated abnormalities may be lower. Tuberous sclerosis complex (TSC) is inherited in an autosomal dominant pattern, but as many as two thirds of cases represent new mutations. TSC may present during infancy with infantile spasms and a hypsarrhythmic electroencephalographic pattern. Autism is seen 1% of affected individuals. The most common skin finding is hypopigmented macules that have been likened to an ash leaf; they are seen in more than 90% of affected individuals (Item C75B).

References:

Haslam RHA. Neurocutaneous syndromes. In: Kliegman RM, Behrman RE, Jenson HB, Stanton BF, eds. Nelson Textbook of Pediatrics. 18th ed. Philadelphia, Pa: Saunders Elsevier; 2007:2483-2488

Kates WR, Antshel KM, Femont W, Roizen, NJ, Shprintzen RJ. Velocardiofacial syndrome. In: Accardo PJ, ed. Capute & Accardo's Neurodevelopmental Disabilities in Infancy and Childhood. Volume II: The Spectrum of Neurodevelomental Disabilities. 3rd ed. Baltimore, Md: Paul H. Brookes Publishing Co; 2008:363-373

Lyon GR, Shaywitz SE, Shaywitz BA. Dyslexia. In: Kliegman RM, Behrman RE, Jenson HB, Stanton BF, eds. Nelson Textbook of Pediatrics. 18th ed. Philadelphia, Pa: Saunders Elsevier; 2007:150-151

Page 273: AAP PREP 2009

2009 PREP SA on CD-ROM

page 273

Morelli JG. Hyperpigmented lesions. In: Kliegman RM, Behrman RE, Jenson HB, Stanton BF, eds. Nelson Textbook of Pediatrics. 18th ed. Philadelphia, Pa: Saunders Elsevier; 2007:2682-2685

National Fragile X Foundation Web site. Available at: http://www.fragilex.org/

Nowicki ST, Hansen RL, Hagerman RJ. X-linked intellectual disabilities In: Accardo PJ, ed. Capute & Accardo's Neurodevelopmental Disabilities in Infancy and Childhood. Volume II: The Spectrum of Neurodevelomental Disabilities. 3rd ed. Baltimore, Md: Paul H Brookes Publishing Co; 2008:331-351

Page 274: AAP PREP 2009

2009 PREP SA on CD-ROM

page 274

Critique: 75

Hypomelanosis of Ito is characterized by swirled, hypopigmented patches that follow the lines of Blaschko, the paths of embryonic neural crest cell migration. (Courtesy of D. Krowchuk)

Page 275: AAP PREP 2009

2009 PREP SA on CD-ROM

page 275

Critique: 75

Ash leaf macules are a feature of tuberous sclerosis. (Courtesy of D. Krowchuk)

Page 276: AAP PREP 2009

2009 PREP SA on CD-ROM

page 276

Question: 76

A 6-year-old girl presents with a history of swelling on her jaw of 1 month’s duration. The mother has been to a "couple of emergency rooms," but nobody can tell her what is wrong with the girl. The child’s father died about 3 years ago from pneumonia, and the mother reports that she has "no energy," but she has not sought medical care. The mother states that her daughter has been fairly healthy except for frequent ear infections. On physical examination, the girl is afebrile; her weight is 16 kg (3rd percentile); her height is 105 cm (3rd percentile); and she has scarred tympanic membranes, bilateral parotid swelling, mild clubbing, and some fine crackles on lung examination.

Of the following, the MOST likely diagnosis is

A. bacterial parotitis

B. common variable immunodeficiency

C. human immunodeficiency virus infection

D. lymphoma

E. mumps

Page 277: AAP PREP 2009

2009 PREP SA on CD-ROM

page 277

Critique: 76 Preferred Response: C

The chronic parotid swelling, clubbing of the digits, history of chronic otitis media, and paternal and maternal illness described for the girl in the vignette are most characteristic of human immunodeficiency virus (HIV) infection. Among HIV-infected children, digital clubbing and wheezing are due to a chronic lung disease known as lymphocytic interstitial pneumonitis. Patients who have bacterial parotitis usually are febrile and have purulent material coming from Stensen's duct (Item C76A). Although patients who have HIV infection may have lymphoma, the presentation usually is not bilateral and is much less common than chronic parotitis. Sinopulmonary infections can occur with common variable immunodeficiency (CVID), but HIV is more common and also is more likely to affect both parents, unlike CVID. Mumps can present with parotid swelling (Item C76B) but is an uncommon infection in developed countries, and the duration of the swelling is not long. The parotid swelling associated with mumps usually peaks in 3 days and resolves over the next 7 days. If mumps is being considered, the diagnosis can be confirmed by isolation of the virus in cell culture from throat washings, saliva, urine, or cerebrospinal fluid. In addition, a positive mumps immunoglobulin M titer, mumps polymerase chain reaction test, or a significant increase between acute and convalescent serum mumps titers can help establish the diagnosis.

References:

American Academy of Pediatrics. Mumps. In: Pickering LK, Baker CJ, Long SS, McMillan JA, eds. Red Book: 2006 Report of the Committee on Infectious Diseases. 27th ed. Elk Grove Village, Ill: American Academy of Pediatrics; 2006:464-468

Burchett SK, Pizzo PA. HIV infection in infants, children, and adolescents. Pediatr Rev. 2003;24:186-194. Available at: http://pedsinreview.aappublications.org/cgi/content/full/24/6/186

Page 278: AAP PREP 2009

2009 PREP SA on CD-ROM

page 278

Critique: 76

Purulent material emanating from Stensen's duct in an infant who has staphylococcal parotitis. (Courtesy of the Red Book® Online.)

Page 279: AAP PREP 2009

2009 PREP SA on CD-ROM

page 279

Critique: 76

Mumps may present with unilateral (as shown here) or bilateral parotid swelling. (Courtesy of M. Rimsza)

Page 280: AAP PREP 2009

2009 PREP SA on CD-ROM

page 280

Question: 77

You are evaluating a 2-year-girl who recently was adopted from Russia for a 4-day history of temperature to 102.5°F (39.2°C), rash, coryza, malaise, conjunctivitis, and cough that have worsened over the last 24 hours. She had nasal congestion and rhinorrhea for 5 days prior to developing the fever, rash, and cough. The girl has been in the United States for 7 days. She was adopted from a rural orphanage, where she was exposed to farm animals, but information regarding her past medical history and immunizations is unavailable. Physical examination shows a tired-appearing, irritable toddler who is clinging to her adopted mother. She has a temperature of 103.0°F (39.5°C), bilateral conjunctival injection, profuse clear rhinorrhea, an erythematous buccal mucosa with scattered whitish specks (Item Q77A) on the left side, and an erythematous posterior pharynx with no tonsillar exudates. There is a confluent erythematous maculopapular rash on her face, trunk, and abdomen (Item Q77B), with scattered patches on her legs.

Of the following, the test MOST likely to confirm the diagnosis for this child is

A. blood culture

B. C-reactive protein measurement

C. serology

D. throat culture

E. urine culture

Page 281: AAP PREP 2009

2009 PREP SA on CD-ROM

page 281

Question: 77

White papules on the buccal mucosa (arrow), as described for the girl in the vignette. (Courtesy of W.W. Tunnessen, Jr)

Page 282: AAP PREP 2009

2009 PREP SA on CD-ROM

page 282

Question: 77

(Courtesy of the Centers for Disease Control and Prevention, Public Health Image Library, Dr. Heinz F. Eichenwald)

Page 283: AAP PREP 2009

2009 PREP SA on CD-ROM

page 283

Critique: 77 Preferred Response: C

The patient described in the vignette has history and physical examination findings that are classic for measles. Measles is caused by the measles (rubeola) virus, which is an RNA virus in the Paramyxoviridae family. The simplest and preferred method for establishing the diagnosis is by demonstrating a positive serologic test for measles immunoglobulin M (IgM) antibody on a single serum specimen obtained during the first encounter with a person suspected of having the disease. Measles IgM antibody is detectable for at least 1 month after the onset of the rash. The measles vaccine alone does not give an IgM response. Other methods of diagnosis include detection of measles virus antigen in respiratory epithelial cells or tissue by immunofluorescent methods and detection of viral genome by polymerase chain reaction. Isolation of the measles virus from blood, throat, and urine cultures is possible but is very difficult and is not readily available. C-reactive protein is a nonspecific measurement of inflammation that is not specific for the diagnosis of measles. References:

American Academy of Pediatrics. Measles. In: Pickering LK, Baker CJ, Long SS, McMillan JA, eds. Red Book: 2006 Report of the Committee on Infectious Diseases. 27th ed. Elk Grove Village, Ill: American Academy of Pediatrics; 2006:441-452

Centers for Disease Control and Prevention. Measles, mumps, and rubella - vaccine use and strategies for elimination of measles, rubella, and congenital rubella syndrome and control of mumps: recommendations of the Advisory Committee on Immunization Practices (ACIP). MMWR Recomm Rep. 1998;47(RR-8):1-57. Available at: http://www.cdc.gov/mmwr/preview/mmwrhtml/00053391.htm

Maldonado YA. Rubeola virus (measles and subacute sclerosing panencephalitis). In: Long SS, Pickering LK, Prober CG, eds. Principles and Practice of Pediatric Infectious Diseases. 2nd ed. Philadelphia, Pa: Churchill Livingstone; 2003:1148-1155

Page 284: AAP PREP 2009

2009 PREP SA on CD-ROM

page 284

Question: 78

You employ voiding cystourethrography (VCUG) to evaluate a 4-year-old girl who had a febrile urinary tract infection 1 month ago. The study reveals a smooth-walled bladder, absence of vesicoureteral reflux, and a mildly narrowed urethra.

Of the following, the MOST appropriate management in this situation is

A. administration of prophylactic antibiotics until 6 years of age

B. no treatment

C. placement of the child on a clean intermittent catheterization program

D. repeat VCUG in 6 months

E. urologic consultation for cystoscopic evaluation

Page 285: AAP PREP 2009

2009 PREP SA on CD-ROM

page 285

Critique: 78 Preferred Response: B

The patient described in the vignette has had a febrile urinary tract infection (UTI), but voiding cystourethrography (VCUG) failed to demonstrate vesicoureteral reflux. It did, however, demonstrate a narrow urethra with an otherwise normal bladder. The question that arises is whether the "narrow urethra" is associated with potential voiding dysfunction and development of the UTI. The recognition of a distal urethral ring in some females who had UTIs in the 1960s led to the hypothesis that a narrowed urethra or "urethral stenosis" was responsible for UTIs. However, it is now recognized that the previously termed "narrow urethra" in females is a normal variant. Indeed, dilatations led to more invasive surgeries of the internal urethra to prevent "restenosis." A recent survey of Fellows in the Section of Urology of the American Academy of Pediatrics found that 61% of respondents never use dilatation under any circumstance, and only 2% perform this procedure regularly. Therefore, no treatment is needed for the child in the vignette. Children who have recurrent UTIs may have vesicoureteral reflux or dysfunctional voiding with incomplete bladder emptying. More recently, greater emphasis has been placed on the effects of pelvic floor muscle dysfunction in the child who has dysfunctional voiding and symptoms of enuresis, UTI, and constipation. It is theorized that the pelvic floor muscles responsible for continence in such children are hyperactive, resulting in abnormal bladder emptying and high postvoid residual volumes. Biofeedback is being used to teach children how to improve relaxation of pelvic floor muscles and reduce urinary retention and UTI risk. There is no indication for prophylactic antibiotics for a child who has a single UTI and no vesicoureteral reflux. Only children who have significant urinary retention, such as seen with neurogenic bladder (myelomeningocele) or detrusor sphincter dyssynergia (Hinman syndrome), warrant intermittent catheterization. Follow-up VCUG is not needed for the patient in the vignette. Cystoscopy is not indicated in a child who has a smooth wall bladder and no urinary symptoms.

References:

Brock WA, Kaplan GW. Abnormalities of the lower urinary tract. In: Edelmann CM Jr, Bernstein J, Meadow SR, Spitzer A, Travis LB, eds. Pediatric Kidney Disease. 2nd ed. Boston, Ma: Little, Brown, and Company; 1992:2037-2076

McKenna PH, Herndon CD, Connery S, Ferrer FA. Pelvic floor muscle retraining for pediatric voiding dysfunction using interactive computer games. J Urol. 1999;162:1056-1063. Abstract available at: http://www.ncbi.nlm.nih.gov/pubmed/10458431

Metwalli AR, Cheng EY, Kropp BP, Pope JC 4th. The practice of urethral dilation for voiding dysfunction among fellows of the Section on Urology of the American Academy of Pediatrics. J Urol. 2002;168:1764-1767. Abstract available at: http://www.ncbi.nlm.nih.gov/pubmed/12352355

Page 286: AAP PREP 2009

2009 PREP SA on CD-ROM

page 286

Question: 79

An 18-year-old girl is admitted to the hospital for intravenous therapy for a complicated urinary tract infection that failed to respond to outpatient therapy with a sulfa-based antibiotic. Her urine culture shows more than 100,000 colony-forming units/mL of Pseudomonas aeruginosa that is sensitive to aztreonam and imipenem. As you take her medical history, she mentions she is "highly allergic" to penicillin.

Of the following, a TRUE statement regarding penicillin drug reactions is that

A. a nonpruritic maculopapular rash that occurs in patients who receive amoxicillin during mononucleosis is a contraindication for future penicillin therapy

B. aztreonam can be administered safely to patients who have a history of immunoglobulin E (IgE)-mediated penicillin allergy

C. desensitization can be used to administer penicillin safely to patients who have experienced Stevens-Johnson reactions to penicillin

D. skin testing to major and minor determinants of penicillin can exclude IgE-mediated and non-IgE-mediated reactions

E. a patient who can only recall a childhood history of penicillin allergy but does not remember the details is very likely to react to future penicillin courses

Page 287: AAP PREP 2009

2009 PREP SA on CD-ROM

page 287

Critique: 79 Preferred Response: B

Penicillin is the most common cause of drug-induced anaphylaxis. Because the likelihood of an immunoglobulin (Ig) E-mediated penicillin allergy is less than 20% based on history alone, patients often are referred to an allergist-immunologist for evaluation and skin testing. The negative predictive value of penicillin skin testing for IgE-mediated reactions is 97% to 99%. However, standardized skin testing materials such as benzylpenicilloyl polylysine or minor determinants either no longer are available or are not approved by the United States Food and Drug Administration, respectively. In addition, non-IgE-mediated penicillin reactions, such as Stevens-Johnson syndrome (Item C79A), result in negative IgE skin tests, even though drug administration causes adverse effects. Because appropriate penicillin skin testing material is not available in most allergy clinics, administration of a non-beta-lactam antibiotic is advised for the patient who has a history of suspected IgE-mediated penicillin allergy. If a penicillin, cephalosporin, or carbapenem drug is required, consultation with an allergist-immunologist should be considered. A detailed discussion regarding avoidance, graded challenge, or drug desensitization usually ensues from the consultation. Carbapenems have a high cross-reactivity with penicillin and should be avoided, but monobactams such as aztreonam do not cross-react and may be administered safely. Desensitization is a process that allows the gradual administration of a drug to a patient who has an IgE-mediated drug allergy. Thus, desensitization cannot be successful for a patient who experienced a non-IgE-mediated reaction such as Stevens-Johnson syndrome. Also, once the desensitization is complete and the patient finishes the appropriate antibiotic therapy, he or she still is considered "allergic" to that drug and must undergo repeat desensitizations to receive subsequent courses of that antibiotic. One common non-IgE-mediated reaction to penicillin occurs when an antibiotic such as amoxicillin is administered to a patient who has an Epstein-Barr virus infection (eg, infectious mononucleosis). Typical symptoms include a nonpruritic, maculopapular rash that occurs a few days into therapy (Item C79B). Clinically, it may be difficult to distinguish this reaction from a true drug allergy, but this rash is not due to an allergic reaction, and future drug use need not be restricted.

References:

Boguniewicz M, Leung DYM. Adverse reactions to drugs. In: Kleigman RM, Behrman RE, Jenson HB, Stanton BF, eds. Nelson Textbook of Pediatrics. 18th ed. Philadelphia, Pa: Saunders Elsevier; 2007:990-994

Wolf R, Orion E, Marcos B, Matz H. Life-threatening acute adverse cutaneous drug reactions. Clin Dermatol. 2005;23:171-181. Abstract available at: http://www.ncbi.nlm.nih.gov/pubmed/15802211

Page 288: AAP PREP 2009

2009 PREP SA on CD-ROM

page 288

Critique: 79

Stevens-Johnson syndrome is characterized by erythematous macules and patches that develop bullae or erosions. Target lesions are absent or few in number. Involvement of multiple mucosal sites (eg, mouth and conjunctiva) is typical. (Courtesy of M. Rimsza)

Page 289: AAP PREP 2009

2009 PREP SA on CD-ROM

page 289

Critique: 79

Morbilliform eruption that occurred in an adolescent who had infectious mononucleosis and received amoxicillin. (Courtesy of D. Krowchuk)

Page 290: AAP PREP 2009

2009 PREP SA on CD-ROM

page 290

Question: 80

A 5-year-old boy is brought to the emergency department because of a nose bleed that has lasted 1} hours. His mother reports that he has had nose bleeds in the past that usually stopped when she pinched his nose, but this time he continued to bleed. She says that he does pick his nose and that he has had cold symptoms for the past 3 days. There is no family history of bleeding disorders, and he had no excessive bleeding after circumcision. On physical examination, the awake, alert, and anxious patient is holding a bloody washcloth to his nose. His heart rate is 140 beats/min, respiratory rate is 24 breaths/min, blood pressure is 100/60 mm Hg, and oxygen saturation is 98%. There is active bleeding from his right naris, but an active anterior bleeding site is not visible. Bleeding is controlled with phenylephrine instillation and packing.

Of the following, the MOST appropriate further evaluation is

A. chest radiograph

B. computed tomography scan of the sinuses

C. magnetic resonance arteriography of the sinuses

D. nasopharyngoscopy

E. no further evaluation

Page 291: AAP PREP 2009

2009 PREP SA on CD-ROM

page 291

Critique: 80 Preferred Response: D

Nosebleeds are common in children, most often associated with trauma (eg, nose-picking), mucosal friability due to upper respiratory tract infections, and mucosal drying related to environmental conditions. Most epistaxis episodes are self-limited and require only bleeding control instructions and comfort care. Recurrent nosebleeds are seen in fewer than 10% of patients and only rarely are related to an underlying anatomic or hematologic abnormality. Evidence suggests that antibacterial creams instilled in the affected naris may decrease the incidence of recurrent idiopathic epistaxis as the child "outgrows" the predisposition. Patients who have nosebleeds that recur frequently or are difficult to control or localize, have a family history suggestive of a bleeding disorder, or have signs and symptoms suggestive of a blood dyscrasia (eg, petechiae) may require further evaluation for an underlying cause. Evaluation should be directed at likely causes, including vascular anomalies, foreign bodies, benign and malignant masses, and primary or secondary bleeding diatheses such as thrombocytopenia, von Willebrand disease, platelet dysfunction, hemophilias, leukemia, liver disease, or those related to medications. The history and physical examination should guide further evaluation. The inability to localize the bleeding to the anterior vestibule for the boy described in the vignette suggests bleeding from the posterior nasopharynx. Ninety percent of epistaxis events result from injury to the Kiesselbach vascular plexus in the anterior nasal septum, and the bleeding site should be visible on nasal inspection. Bleeding from the anterior or posterior ethmoidal or sphenopalatine arteries in the posterior nasopharynx is more difficult to see and control. Therefore, nasopharyngoscopy is indicated to identify the site and to aid in directing treatment. If a mass or vascular anomaly is identified, computed tomography scan or magnetic resonance angiography/magnetic resonance venography should be undertaken. A chest radiograph is not likely to provide information that would guide evaluation or treatment of epistaxis. When a bleeding disorder is suspected, the initial hematologic evaluation should include complete blood and platelet counts, with evaluation of the blood smear, prothrombin time, and partial thromboplastin time and typing and cross-matching if transfusion is anticipated. If abnormalities are identified, consultation with a hematologist can help to guide further evaluation, which may include a closure test to evaluate for platelet dysfunction and von Willebrand factor or ristocetin cofactor assays to assess for von Willebrand disease. Of note, in one study of patients who had recurrent epistaxis and were referred to a hematologist, one third ultimately were diagnosed with bleeding disorders, and 50% of these patients had von Willebrand disease. Family history was the only predictor that identified patients who had primary bleeding diatheses.

References:

Haddad J Jr. Acquired disorders of the nose. In: Kleigman RM, Behrman RE, Jenson HB, Stanton BF, eds. Nelson Textbook of Pediatrics. 18th ed. Philadelphia, Pa: Saunders Elsevier; 2007:1744-1745

McGarry G. Nosebleeds in children. BMJ Clinical Evidence. 2006. Available for subscription at: http://clinicalevidence.bmj.com/ceweb/conditions/chd/0311/0311.jsp

Messner AH. Epidemiology and etiology of epistaxis in children. UpToDate Online 15.3. 2008. Available for subscription at: http://www.utdol.com/utd/content/topic.do?topicKey=ped_lryn/5986

Messner AH. Evaluation of epistaxis in children. UpToDate Online 15.3. 2008. Available for subscription at:http://www.utdol.com/utd/content/topic.do?topicKey=ped_lryn/6248&selectedTitle=4~150&source=searc

Page 292: AAP PREP 2009

2009 PREP SA on CD-ROM

page 292

h_result

Sandoval C, Dong S, Visintainer P, Ozkaynak MF, Jayabose S. Clinical and laboratory features of 178 children with recurrent epistaxis. J Pediatr Hematol Oncol. 2002;24:47-49. Abstract available at: http://www.ncbi.nlm.nih.gov/pubmed/11902740

Page 293: AAP PREP 2009

2009 PREP SA on CD-ROM

page 293

Question: 81

A 4-month-old infant who has gastroschisis underwent surgical repair on the first day after birth, but continues to require support with parenteral nutrition and lipids. He now has developed poor feeding, irritability, and progressive diarrhea. Radiography demonstrates metaphyseal fraying, but calcium, phosphorus, and 25-hydroxyvitamin D concentrations are normal. When you review his prior laboratory studies, you note he has had neutropenia for the past 4 weeks.

Of the following, this child’s symptoms are MOST consistent with

A. copper deficiency

B. magnesium deficiency

C. vitamin A deficiency

D. vitamin B6 deficiency

E. zinc deficiency

Page 294: AAP PREP 2009

2009 PREP SA on CD-ROM

page 294

Critique: 81 Preferred Response: A

The poor feeding, irritability, metaphyseal irregularity, normal vitamin D value, and neutropenia described for the infant in the vignette are consistent with copper deficiency, a rare disorder that develops when children receive parenteral nutrition without trace element supplementation. Copper is a trace element that has many important biochemical functions. It is an important component of respiratory chain enzymes (eg, cytochrome C) and lysyl oxidase (an enzyme critical to collagen production and bone formation). Menkes disease, a rare syndrome involving a defect in copper transport and profoundly low serum copper concentrations, is characterized by hypotonia, developmental delay, seizures, and "steely hair." Copper deficiency also may be seen in preterm infants who do not receive appropriate trace element supplementation. The clinical presentation of copper deficiency in the preterm infant is more subtle, but can include pallor, poor feeding, hypochromic anemia, neutropenia, and skeletal changes (including metaphyseal fraying and osteoporosis). The patient's symptoms are more consistent with copper deficiency than with magnesium, vitamin B6, zinc, or vitamin A deficiency. Specifically, magnesium deficiency can cause hypotonia and apnea, vitamin B6 deficiency can cause hypotonia and seizures, zinc deficiency can cause diarrhea and skin rashes, and vitamin A deficiency can lead to corneal lesions and impaired vision. To prevent copper deficiency, most parenteral nutrition is supplemented with 200 mcg/L of copper. Infants who have cholestasis may have impaired copper excretion into the bile, necessitating a decrease in the concentration of copper in the parenteral nutrition to prevent copper overload.

References:

Collier S, Gura KM, Richardson D, Duggan C. Parenteral nutrition. In: Hendricks KM, Duggan C. Manual of Pediatric Nutrition. 4th ed. Hamilton Ontario, Canada: BC Decker; 2005:317-375

Giles E, Doyle LW. Copper in extremely low-birthweight or very preterm infants. NeoReviews. 2007;8:e159-e164. Available for subscription at: http://neoreviews.aappublications.org/cgi/content/full/8/4/e159

Page 295: AAP PREP 2009

2009 PREP SA on CD-ROM

page 295

Question: 82

You are counseling a 23-year-old woman who has diabetes mellitus and has been your patient for the past 18 years. She recently found out that she is pregnant and asks you about potential complications for her unborn child.

Of the following, the MOST likely complications to expect for this woman’s child are

A. hyperacusis, hypercalcemia, hydronephrosis

B. hyperbilirubinemia, hypercalcemia, polydactyly

C. hyperglycemia, hypocalcemia, polysplenia

D. hypoglycemia, hypocalcemia, polycythemia

E. hypogonadism, hypocalcemia, polyuria

Page 296: AAP PREP 2009

2009 PREP SA on CD-ROM

page 296

Critique: 82 Preferred Response: D

Disorders of glucose regulation such as diabetes mellitus may complicate as many as 5% of pregnancies. This includes women who develop diabetes (insulin resistance) during pregnancy, known as gestational diabetes; those who have pre-existing insulin resistance (type 2 diabetes); and those who have pre-existing insulin-dependent diabetes mellitus (IDDM) (type 1 diabetes). The pregnant woman described in the vignette has IDDM. Risks posed to her developing fetus and newborn are numerous (Item C82). Postnatal hypoglycemia frequently is encountered in the first 4 to 12 hours in infants of diabetic mothers (IDMs) and is related to macrosomia, an increased metabolic rate, and fetal hyperinsulinemia that takes a few days after birth to diminish. Postnatal hypocalcemia results from the effects of poor late-trimester transfer of calcium across the placenta in pregnancies affected by diabetes, a delay in normal postnatal parathyroid hormone elevation, and poor fetal and neonatal bone mineralization (ie, poor calcium stores). If hypomagnesemia is found in the IDM, it must be corrected to allow normal parathyroid function to resume. Polycythemia (hematocrit >65% [0.65]) represents the fetal response to its increased metabolic rate and a relative fetal hypoxemia in utero when pregnancy is complicated by diabetes and fetal macrosomia. Resultant hyperbilirubinemia needs to be anticipated. Hyperacusis is not seen in IDMs, whose risk for hearing impairment is similar to that of other newborns requiring intensive care. Hypercalcemia is seen in Williams syndrome, but not in IDMs. Hydronephrosis, seen on prenatal ultrasonography, may be present in a number of high-risk pregnancies, but not IDMs. Polydactyly may be seen in trisomies and some other congenital syndromes but does not occur with greater frequency in IDMs. Polysplenia is seen in defects of left-right asymmetry but does not have an increased incidence in IDMs. Hypogonadism is characteristic of congenital adrenal hyperplasia, Prader-Willi syndrome, and Turner syndrome; it is not more common in IDMs.

References:

Cowett RM. The infant of the diabetic mother. NeoReviews. 2002;3:e173-e189. Available for subscription at: http://neoreviews.aappublications.org/cgi/content/full/3/9/e173

Sivit CJ. Diagnostic imaging. In: Martin RJ, Fanaroff AA, Walsh MC, eds. Fanaroff and Martin's Neonatal-Perinatal Medicine. 8th ed. Philadelphia, Pa: Mosby Elsevier; 2006:713-732

Page 297: AAP PREP 2009

2009 PREP SA on CD-ROM

page 297

Critique: 82

Page 298: AAP PREP 2009

2009 PREP SA on CD-ROM

page 298

Question: 83

A 17-year-old girl complains of an itchy rash all over her back and trunk for 2 weeks. Topical hydrocortisone has not relieved the rash or itching. She denies fever or other symptoms, and her vital signs are normal. Examination of the skin reveals multiple 5- to 8-mm salmon-colored thin scaling plaques over her trunk (Item Q83). There is one similar lesion on her abdomen that measures 2x3 cm. There are no other lesions, and the remaining findings of her physical examination are normal.

Of the following, the BEST approach to managing this girl’s rash is to

A. administer intramuscular penicillin

B. administer topical coal tar lotion

C. administer topical selenium sulfide shampoo

D. prescribe oral corticosteroids

E. reassure her that the rash will resolve in several weeks

Page 299: AAP PREP 2009

2009 PREP SA on CD-ROM

page 299

Question: 83

(Courtesy of D. Krowchuk)

Page 300: AAP PREP 2009

2009 PREP SA on CD-ROM

page 300

Critique: 83 Preferred Response: E

The rash described for the girl in the vignette is most consistent with the diagnosis of pityriasis rosea, a self-limited condition believed to have a viral etiology, although this remains unproven. A nonspecific prodrome of fever and malaise is seen occasionally. Typical features include a salmon-colored patch surrounded by a darker rim and a fine scale, known as a "herald patch" (Item C83A). Several days after the appearance of the herald patch, multiple similar but smaller lesions develop over the trunk and back along lines of skin cleavage, giving the impression of a "Christmas tree" pattern (Item C83B). The palms and soles typically are spared. Some pruritus occurs in approximately 25% of patients. The rash is generally present for 2 to 12 weeks and resolves spontaneously. Exposure to sunlight may hasten resolution of the lesions. Numerous therapies have been prescribed for the treatment of pityriasis rosea, including oral antibiotics, oral antihistamines, and oral steroids, but most have not proven to be effective. Therefore, reassurance is all that is necessary. Topical antipruritic agents may relieve itching in some cases. Coal tar preparations are useful in the treatment of psoriasis and other chronic dermatoses. Topical selenium sulfide shampoo may be helpful for tinea versicolor and seborrheic dermatitis, but these conditions are not suggested by the appearance of the girl's rash. One of the most important diagnoses to consider in the differential diagnosis of pityriasis rosea is secondary syphilis. The skin lesions can look similar to those of pityriasis rosea and may follow the same lines of distribution, but the palms and soles generally are affected (Item C83C). Patients also may complain of vague constitutional symptoms, such as headaches, fatigue, and lymph node swelling. It is critical to obtain an accurate sexual history in any adolescent who presents with this type of rash. Intramuscular penicillin is the drug of choice for secondary syphilis.

References:

Chuh AAT, Dofitas BL, Comisel GG, et al. Interventions for pityriasis rosea. Cochrane Database Syst Rev. 2007;2:CD005068. Available at: http://www.mrw.interscience.wiley.com/cochrane/clsysrev/articles/CD005068/frame.html

Morelli JG. Diseases of the epidermis. In: Kliegman RM, Behrman RE, Jenson HB, Stanton BF, eds. Nelson Textbook of Pediatrics. 18th ed. Philadelphia, Pa: Saunders Elsevier; 2007:2702-2707

Wolfrey JD, Billica WH, Gulbranson SH, et al. Pediatric exanthems. Clin Fam Pract. 2003;5:557-588

Page 301: AAP PREP 2009

2009 PREP SA on CD-ROM

page 301

Critique: 83

The herald patch of pityriasis rosea is a round or oval erythematous scaling patch that may be mistaken for tinea corporis. (Courtesy of D. Krowchuk)

Page 302: AAP PREP 2009

2009 PREP SA on CD-ROM

page 302

Critique: 83

In pityriasis rosea, the long axes of lesions are aligned parallel to lines of skin stress. As a result, the distribution of lesions may have the appearance of the branches of a fir tree. (Courtesy of D. Krowchuk)

Page 303: AAP PREP 2009

2009 PREP SA on CD-ROM

page 303

Critique: 83

The eruption of secondary syphilis often involves the palms and soles. (Courtesy of C. Haverstock)

Page 304: AAP PREP 2009

2009 PREP SA on CD-ROM

page 304

Question: 84

A 4-year-old boy who has had mild eczema in the past that was treated successfully with emollients presents with the worst exacerbation he ever has had. He has multiple lichenified lesions, especially in the antecubital fossae (Item Q84) and popliteal fossa, which is usual for him, but he also has nummular lesions on the trunk. His mother reports no changes in detergents or personal hygiene products. The boy has been going to a summer day camp at the local community center for the first time.

Of the following, the factor MOST likely involved in his eczema exacerbation is

A. activities in the air-conditioned gymnasium

B. craft activities using water-soluble paint and glue

C. daily snacks of popcorn and fruit juice

D. outdoor soccer practice every afternoon

E. wearing a cotton t-shirt

Page 305: AAP PREP 2009

2009 PREP SA on CD-ROM

page 305

Question: 84

Lichenified and crusted plaques, as described for the boy in the vignette. (Courtesy of D. Krowchuk)

Page 306: AAP PREP 2009

2009 PREP SA on CD-ROM

page 306

Critique: 84 Preferred Response: D

A number of factors may worsen atopic dermatitis by causing itching and scratching. Exposure to heat (as may occur during warm weather months or because of overdressing during cold weather months), sunlight, chemicals (including swimming pool chemicals such as chlorine), and sweat retention cause exacerbations. Cold weather and low humidity may cause skin to become dry, leading to increased pruritus. Wool or synthetic materials, fragrances, harsh soaps or detergents, and some fabric softeners also may cause itching. For the child described in the vignette, overheating during outdoor play is the most likely activity that is exacerbating his eczema. Activities in an air-conditioned gymnasium would be beneficial. Although the use of glues and craft materials may cause irritation, they would be anticipated to produce a rash on the hands at sites of exposure. The snacks the child is receiving are unlikely to worsen atopic dermatitis. Food allergy as an exacerbating factor is observed most often in infants, not older children. The allergens most commonly implicated are milk, eggs, soy, wheat, and peanuts.

References:

Ashcroft DM, Chen L-C, Garside R, Stein K, Williams HC. Topical pimecrolimus for eczema. Cochrane Database Syst Rev. 2007;4:CD005500. Available at: http://www.mrw.interscience.wiley.com/cochrane/clsysrev/articles/CD005500/frame.html

Bath-Hextall F, Williams H. Eczema (atopic). BMJ Clinical Evidence. 2006. Available for subscription at: http://clinicalevidence.bmj.com/ceweb/conditions/skd/1716/1716_I15.jsp

Beattie PE, Lewis-Jones MS. A comparative study of impairment of quality of life in children with skin disease and children with other chronic childhood diseases. Br J Dermatol. 2006;155:145-151. Abstract available at: http://www.ncbi.nlm.nih.gov/pubmed/16792766

Byremo G, Rød G, Carlsen KH. Effect of climatic change in children with atopic eczema. Allergy. 2006;61:1403-1410. Abstract available at: http://www.ncbi.nlm.nih.gov/pubmed/17073869

Ersser SJ, Latter S, Sibley A, Satherley PA, Welbourne S. Psychological and educational interventions for atopic eczema in children. Cochrane Database Syst Rev. 2007;3:CD004054. Available at: http://www.mrw.interscience.wiley.com/cochrane/clsysrev/articles/CD004054/frame.html

Kramer MS, Kakuma R. Maternal dietary antigen avoidance during pregnancy or lactation, or both, for preventing or treating atopic disease in the child. Cochrane Database Syst Rev. 2006;3:CD000133. Available at: http://www.mrw.interscience.wiley.com/cochrane/clsysrev/articles/CD000133/frame.html

Osborn DA, Sinn J. Formulas containing hydrolysed protein for prevention of allergy and food intolerance in infants. Cochrane Database Syst Rev. 2003;4:CD003664. Available at: http://www.mrw.interscience.wiley.com/cochrane/clsysrev/articles/CD003664/frame.html

Osborn DA, Sinn JK. Probiotics in infants for prevention of allergic disease and food hypersensitivity. Cochrane Database Syst Rev. 2007;4:CD006475. Available at: http://www.mrw.interscience.wiley.com/cochrane/clsysrev/articles/CD006475/frame.html

Osborn DA, Sinn J. Soy formula for prevention of allergy and food intolerance in infants. Cochrane Database Syst Rev. 2006;4:CD003741. Available at:

Page 307: AAP PREP 2009

2009 PREP SA on CD-ROM

page 307

http://www.mrw.interscience.wiley.com/cochrane/clsysrev/articles/CD003741/frame.html

Page 308: AAP PREP 2009

2009 PREP SA on CD-ROM

page 308

Question: 85

You are called by the mother of 3-year-old girl because the child appears confused and is pale and sweating. The mother thinks the child may have taken some of her grandmother’s imipramine. You advise her to contact emergency medical services for immediate transport to the emergency department, where you plan to meet them.

Of the following, the MOST appropriate action to take in the emergency department is

A. chest radiography to evaluate for pulmonary edema

B. continuous cardiac monitoring for dysrhythmias

C. echocardiography to assess cardiac function

D. electroencephalography to identify a seizure focus

E. measurement of the serum concentration of imipramine

Page 309: AAP PREP 2009

2009 PREP SA on CD-ROM

page 309

Critique: 85 Preferred Response: B

The child described in the vignette has symptoms suggestive of tricyclic antidepressant (TCA) ingestion. This class of antidepressants is used less frequently today for treatment of depression because of the availability of selective serotonin reuptake inhibitors (SSRIs). TCAs are not the drugs of choice for treatment of depression in children and adolescents but are used occasionally in the treatment of other disorders (eg, enuresis, narcolepsy). SSRIs are much safer to use than TCAs because they have fewer adverse effects and are unlikely to result in death when overdose occurs. When TCAs are ingested in toxic amounts, they primarily affect the central nervous and cardiovascular systems. Central nervous system signs and symptoms of TCA toxicity include irritability, euphoria, seizures, and unresponsiveness. Autonomic nervous system symptoms such as mydriasis, dry skin, dry mouth, urinary retention, and tachycardia also may be evident. Among the direct effects on the cardiac system are a delay in signal conduction through the bundle of His, depression of myocardial contractile function, and prolongation of the QRS and the QT intervals. These latter cardiac effects may potentiate arrhythmia formation. Continuous cardiac monitoring for the possible occurrence of arrhythmia is essential in patients who are suspected of toxic TCA ingestion. Electrocardiography is the best tool for assessing the function of the conduction system. The voltage intervals should be measured, with particular attention to the QRS duration and the QT interval. A QRS duration of greater than 100 msec is associated with the development of seizures; a QRS duration of more than 160 msec is associated with ventricular dysrhythmias that may be particularly difficult to treat. Echocardiography is an excellent tool to evaluate cardiac structure and function but is not necessarily indicated as part of the emergency department evaluation of a child in whom toxic TCA ingestion is suspected and does not obviate the need for continuous cardiac monitoring. Similarly, chest radiography does not play a role in the acute management of such a patient. Electroencephalography has no place in the initial evaluation and management of children in whom a TCA toxic ingestion is suspected. Seizures, when they occur, usually do so early in the course and often resolve by the time anticonvulsants are administered. Measuring serum drug concentrations of TCAs is not helpful for the prognosis or management of TCA ingestion. Serum electrolyte concentrations may be measured, but they have no predictive value in the management of TCA toxicity.

References:

Boehnert MT, Lovejoy FH Jr. Value of the QRS duration versus the serum drug level in predicting seizures and ventricular arrhythmias after an acute overdose of tricyclic antidepressants. N Engl J Med. 1985;313:474-479. Abstract available at: http://www.ncbi.nlm.nih.gov/pubmed/4022081

Hatcher-Kay C, King CA. Depression and suicide. Pediatr Rev. 2003;24:363-371. Available at: http://pedsinreview.aappublications.org/cgi/content/full/24/11/363

Liebelt EL, Francis PD, Woolf AD. ECG lead aVR versus QRS interval in predicting seizures and arrhythmias in acute tricyclic antidepressant toxicity. Ann Emerg Med. 1995;26:195-201. Abstract available at: http://www.ncbi.nlm.nih.gov/pubmed/7618783

McGuigan ME. Poisoning potpourri. Pediatr Rev. 2001;22:295-302. Available at: http://pedsinreview.aappublications.org/cgi/content/full/22/9/295

Pérez-Fontán J, Lister G. The acutely ill infant and child. In: Rudolph CD, Rudolph AM, eds. Rudolph's Pediatrics. 21st ed. New York, NY: McGraw-Hill Medical Publishing Division; 2003:364-365

Page 310: AAP PREP 2009

2009 PREP SA on CD-ROM

page 310

Question: 86

A 4-year-old boy who has neuroblastoma presents with back pain and an inability to urinate. He is alert, with normal general examination findings and normal mental status. Strength and tone in the arms are normal, but tone is low in the legs, and patellar reflexes are diminished.

Of the following, the MOST appropriate next step for diagnosis is

A. lumbar puncture

B. magnetic resonance imaging with contrast of the spine

C. postvoid bladder residual measurement

D. radiograph of the spine

E. voiding cystourethrography

Page 311: AAP PREP 2009

2009 PREP SA on CD-ROM

page 311

Critique: 86 Preferred Response: B

Back pain in a young child is unusual and requires prompt evaluation. When there are additional signs that can localize to the spinal cord, such as inability to urinate, low tone, and diminished reflexes, as described for the boy in the vignette, immediate evaluation in the emergency department is needed. Lesions within or near the spinal cord that are causing symptoms can progress to irreversible damage that may be avoided with emergency neurosurgery. The child described in the vignette has a history of neuroblastoma, an aggressive tumor that is prone to metastasis, and emerging flaccid paralysis. Therefore, magnetic resonance imaging (MRI) of the spine with gadolinium contrast is indicated to assess the nature of the problem and determine the location of the lesion more exactly (Item C86). Hypervascularity or vascular wall breakdown in a metastasis may result in contrast enhancement, improving visualization of the lesion. Lumbar puncture may be needed subsequently to evaluate for other causes or for tumor cytology, but this procedure should be deferred until after imaging the spine and, if necessary, emergent neurosurgical consultation. Similarly, spine imaging takes precedence over urologic dynamic studies such as postvoid bladder residual measurement or voiding cystourethrography. A plain radiograph may provide some information, but if MRI is available, it is preferred because it can assist in emergency surgical planning. Management of spinal cord compression due to a tumor includes immediate neurosurgical consultation. Intravenous dexamethasone typically is used, along with a proton pump inhibitor or histamine2 blocker. Aggressive pain management may be needed.

References:

Haslam RHA. Spinal cord disorders. In: Kliegman RM, Behrman RE, Jenson HB, Stanton BF, eds. Nelson Textbook of Pediatrics. 18th ed. Philadelphia, Pa: Saunders Elsevier; 2007:2526-2530

Kim S, Chung DH. Pediatric solid malignancies: neuroblastoma and Wilms' tumor. Surg Clin North Am. 2006;86:469-487

Page 312: AAP PREP 2009

2009 PREP SA on CD-ROM

page 312

Critique: 86

T1-weighted sagittal magnetic resonance imaging of the spine demonstrating spinal cord compression by an extramedullary mass (arrow). (Courtesy of P. Fisher)

Page 313: AAP PREP 2009

2009 PREP SA on CD-ROM

page 313

Question: 87

The mother of a boy in your practice is contemplating another pregnancy and asks for your advice. The woman is tall and thin and works as a model part-time. She had previously reported to you a history of bulimia. She is extremely concerned about any "extra" weight she may gain during the pregnancy, and she confides that she sometimes smokes cigarettes to avoid eating. Additionally, she occasionally has taken her son’s methylphenidate to suppress her appetite. When asked about alcohol use, she describes herself as a "social drinker."

Of the following, the MOST accurate statement to make in counseling this woman is that

A. bulimia during pregnancy is not associated with birth defects

B. cigarette smoking increases the risk of sudden infant death syndrome in the exposed infant

C. one or two alcoholic beverages per day will do no harm to the embryo/fetus

D. prenatal methylphenidate exposure is associated with craniofacial malformations

E. vitamin supplements reduce the risk of defects associated with prenatal alcohol exposure

Page 314: AAP PREP 2009

2009 PREP SA on CD-ROM

page 314

Critique: 87 Preferred Response: B

The extent of the impact that environmental exposures have on pregnancy outcome is still largely unknown. However, some large, well-designed epidemiologic studies offer insight into complications that can arise from fetal exposure to maternal eating disorders, tobacco smoking, and substances of abuse. Undoubtedly, genetic and other factors modify outcomes. The impact of cigarette smoking on fetal development and pregnancy has been, and continues to be, an active area of study. Intrauterine growth restriction is the most consistent negative effect of maternal smoking on the fetus, with an average 200-g reduction in the birthweights of term infants born to mothers who smoke during pregnancy. The more a woman smokes, the greater the reduction in fetal weight. Cigarette smoking during pregnancy also has been associated with increased health risks to exposed children as they grow. Among the reported complications are abnormal pulmonary function, increased cancer risk, and lower sperm counts. Numerous studies have found an increased risk for sudden infant death in prenatally exposed infants. Studies of the effects of eating disorders on pregnancy outcome are limited. However, both prospective, controlled studies and literature reviews indicate that pregnant women who have past or active eating disorders, including bulimia, are at increased risk for delivering babies of significantly lower-than-average birthweight and head circumference (including microcephaly). Prenatal exposure to alcohol may result in varied outcomes collectively termed fetal alcohol spectrum disorder. To date, no safe quantity of alcohol has been established for consumption during pregnancy. Therefore, the strong recommendation is that women abstain from drinking alcoholic beverages from at least the time of conception and throughout pregnancy. The effects of methylphenidate on pregnancy outcome have been studied in experimental animals and a limited number of humans. To date, there does not appear to be an increase in congenital anomalies associated with prenatal exposure to methylphenidate. The question of the potential benefits of vitamin and micronutrient supplementation for women who smoke cigarettes and drink alcohol during pregnancy and their babies is under investigation. At present, it is not clear whether supplementation can mitigate poor outcomes, and no specific recommendations exist. However, it is clear that good nutrition and vitamin supplementation during pregnancy reduce the risk for pregnancy complications.

References:

Cigarette smoking, methamphetamine. Reprotox. Available for subscription at: http://www.reprotox.org

Cigarette smoking, methamphetamine. Teris. Available for subscription at http://depts.washington.edu/terisweb/teris/

Cogswell ME, Weisberg P, Spong C. Cigarette smoking, alcohol use and adverse pregnancy outcomes: implications for micronutrient supplementation. J Nutr. 2003;133:1722S-1731S. Available at: http://jn.nutrition.org/cgi/content/full/133/5/1722S

Kouba S, Hällström T, Londholm C, Lindén Hirschbe A. Pregnancy and neonatal outcomes in women with eating disorders. Obstet Gynecol. 2005;105:255-260. Available at: http://www.greenjournal.org/cgi/content/full/105/2/255

Kunz LH, King JC. Impact of maternal nutrition and metabolism on health of the offspring. Semin Fetal Neonatal Med. 2007;12;71-77. Abstract available at: http://www.ncbi.nlm.nih.gov/pubmed/17200031

Page 315: AAP PREP 2009

2009 PREP SA on CD-ROM

page 315

Micali N, Simonoff E, Treasure J. Risk of major adverse outcomes in women with eating disorders. Br J Psychiatry. 2007;190:255-259. Abstract available at: http://www.ncbi.nlm.nih.gov/pubmed/17329747

Page 316: AAP PREP 2009

2009 PREP SA on CD-ROM

page 316

Question: 88

An 18-year-old young man comes to your office with complaints of burning pain with urination over the past 24 hours. He has seen a small amount of yellowish discharge from his penis during this time. He also complains of some lower back pain over the past 48 hours. He denies fever or rashes, but his eyes are a little irritated. He is sexually active and uses condoms "most of the time." On physical examination, he is afebrile, his palpebral and bulbar conjunctivae are mildly injected (Item Q88), and his back is tender at the lower lumbar area, but there is no costovertebral angle tenderness. Genital examination reveals no scrotal tenderness and scant yellow discharge at the urethral orifice.

Of the following, the MOST likely cause of this patient’s symptoms is

A. Chlamydia trachomatis

B. Gardnerella vaginalis

C. Neisseria gonorrhoeae

D. Treponema pallidum

E. Trichomonas vaginalis

Page 317: AAP PREP 2009

2009 PREP SA on CD-ROM

page 317

Question: 88

(Courtesy of P. Sagerman)

Page 318: AAP PREP 2009

2009 PREP SA on CD-ROM

page 318

Critique: 88 Preferred Response: A

Urethritis can have infectious and noninfectious causes. Symptoms include mucopurulent or purulent discharge, dysuria, and urethral pruritus. Several organisms, including Neisseria gonorrhoeae and Chlamydia trachomatis, cause urethritis. Ureaplasma urealyticum, Mycoplasma genitalium, Gardnerella vaginalis, herpes simplex virus, adenovirus, and Trichomonas vaginalis are implicated in nonchlamydial, nongonococcal urethritis (NGU), but they are more difficult to detect than N gonorrhoeae and C trachomatis. The constellation of conjunctivitis, urethritis, and arthritis reported for the young man in the vignette represents the classic symptoms of a form of reactive arthritis once called Reiter syndrome. The term reactive arthritis refers to rheumatic disorders that appear after an infection, but in which the responsible pathogen is not detected in the affected joint. C trachomatis is the only genital pathogen commonly accepted to be a cause of reactive arthritis. N gonorrhoeae can cause septic arthritis or disseminated gonococcal infection (ie, a rash and tenosynovitis) but does not produce reactive arthritis. Although T vaginalis and G vaginalis may cause urethritis, they do not produce the other symptoms exhibited by the boy described in the vignette. Syphilis, caused by infection with Treponema pallidum, may affect bones congenitally (osteochondritis) or in late stages of the disease (with gummas, granulomatous lesions that involve bones as well as soft tissue or viscera) but does not produce urethral discharge or conjunctivitis. Reactive arthritis caused by C trachomatis is treated with a single 1-g oral dose of azithromycin or with 100 mg doxycycline orally twice a day for 7 days, after testing for both N gonorrhoeae and C trachomatis is completed. This is also the recommended regimen for all NGUs. First-line treatment of uncomplicated gonococcal urethritis is accomplished with ceftriaxone 125 mg intramuscularly or cefixime 400 mg orally, both in a single dose.

References:

Centers for Disease Control and Prevention. Update to CDC's sexually transmitted diseases treatment guidelines, 2006: fluoroquinolones no longer recommended for treatment of gonococcal infections. MMWR Morbid Mortal Wkly Rep. 2007;56:332-336. Available at: http://www.cdc.gov/mmwr/preview/mmwrhtml/mm5614a3.htm

Fortenberry JD, Neinstein LS. Syphilis. In: Neinstein LS, ed. Adolescent Health Care: A Practical Guide. 5th ed. Philadelphia, Pa: Lippincott Williams & Wilkins; 2008:825-833

Workowski KA, Berman SM, Centers for Disease Control and Prevention. Sexually transmitted diseases treatment guidelines, 2006. MMWR Recomm Rep. 2006;55(RR11):1-94. Available at: http://www.cdc.gov/mmwr/preview/mmwrhtml/rr5511a1.htm

Yu DT. Reactive arthritis (formerly Reiter syndrome): definition, diagnosis, and management. UpToDate Online 15.3. 2008. Available for subscription at: http://www.utdol.com/utd/content/topic.do?topicKey=spondylo/7349

Page 319: AAP PREP 2009

2009 PREP SA on CD-ROM

page 319

Question: 89

You are evaluating an 18-month-old girl for vomiting. She has a history of febrile seizures and recurrent ear infections. She is receiving no medications. Over the past several weeks, her parents have noticed that she has been "increasingly clumsy." She has vomited each of the last three mornings but has had no diarrhea or fever. Physical examination findings are normal except for an ataxic gait and hyperreflexia.

Of the following, the MOST appropriate next step is

A. administration of an antiemetic

B. computed tomography scan of the head

C. electroencephalography

D. lumbar puncture

E. reassurance and re-evaluation in 3 to 5 days

Page 320: AAP PREP 2009

2009 PREP SA on CD-ROM

page 320

Critique: 89 Preferred Response: B

Initial symptoms of increased intracranial pressure often consist of headaches and confusion that may be accompanied by lethargy. The child described in the vignette exhibits signs of a progressive increase in pressure, such as vomiting (especially in the morning) and changes in motor tone. Physical examination findings include a full or bulging fontanelle and widened sutures. These signs, as well as pupillary changes and papilledema, should prompt rapid evaluation to prevent permanent neurologic injury and progression of the increased pressure with potential neurologic catastrophe. Intracranial pressure is maintained by the balance of the contents of the cranial vault, which includes brain, blood, and cerebrospinal fluid. Increases in intracranial pressure can occur with a wide variety of disease processes, such as brain tumors, hydrocephalus, infections, head trauma, and hypoxic-ischemic injury. Intracranial pressure increases when the compensatory mechanisms of the cranial vault are exceeded and produce numerous symptoms, depending on the age of the patient and the underlying pathology. Computed tomography scan or magnetic resonance imaging of the head is the first priority in evaluating suspected increased intracranial pressure. Meningitis is unlikely in this patient due to the chronicity of symptoms and absence of fever. Accordingly, lumbar puncture is not indicated at this time. Electroencephalography might be indicated if atypical migraines or seizures were suspected, but the initial priority is to evaluate the patient for potential life-threatening disease processes. The child has no evidence of viral infection, and reassurance or administration of antiemetics is not appropriate.

References:

Frankel LR. Neurological emergencies and stabilization. In: Kliegman RM, Behrman RE, Jenson HB, Stanton BF, eds. Nelson Textbook of Pediatrics. 18th ed. Philadelphia, Pa: Saunders Elsevier; 2007:405-412

Larsen GY, Goldstein B. Consultation with the specialist: increased intracranial pressure. Pediatr Rev. 1999;20:234-239. Available at: http://pedsinreview.aappublications.org/cgi/content/full/20/7/234

Page 321: AAP PREP 2009

2009 PREP SA on CD-ROM

page 321

Question: 90

A 16-year-old girl comes to your office complaining that her menstrual periods have been irregular and scanty. Her last period was 3 months ago and lasted for only 2 days. Among the findings on physical examination are fine, moist skin; firm, palpable thyroid gland (Item Q90); and finger tremor. Results of laboratory studies include a thyroid-stimulating hormone value of less than 0.05 mIU/L (normal, 0.5 to 5.0 mIU/L) and free thyroxine value of 1.9 ng/dL (24.5 pmol/L) (normal, 0.6 to 1.3 ng/dL [7.7 to 16.8 pmol/L]).

Of the following, the additional physical examination finding that BEST supports the diagnosis of hyperthyroidism is

A. abdominal obesity

B. atrophy of lingual papillae

C. hepatomegaly

D. hirsutism

E. muscle weakness

Page 322: AAP PREP 2009

2009 PREP SA on CD-ROM

page 322

Question: 90

(Courtesy of M. Rimsza)

Page 323: AAP PREP 2009

2009 PREP SA on CD-ROM

page 323

Critique: 90 Preferred Response: E

The clinical findings of hyperthyroidism in children usually are obvious, but the diagnosis can be subtle in mild disease. Classic clinical findings include weight loss, increased appetite, decreased strength and sports performance, hyperactivity, tremors, sweating, nocturnal sleeplessness (sometimes with daytime somnolence), irritability, decreased school performance, pruritus, and nocturia. Menarcheal girls may have scant, infrequent menses, as described for the girl in the vignette. On physical examination, most children have a palpable, firm thyroid gland (Item C90A) that has an audible bruit. Some children may have fine, moist skin; a visible tremor; slight skin darkening; fine scalp hair, with some hair loss at the temples; muscle weakness; and some loss of muscle mass that can be identified by examining the thenar and hypothenar eminences. A "thyrotoxic stare" accompanies hyperthyroidism. Exophthalmos (Item C90B) may be found in thyrotoxicosis due to Graves disease. Hirsutism, hepatomegaly, abdominal obesity, and changes in the lingual papillae are not findings of hyperthyroidism. Other laboratory studies that are of use in caring for the girl in the vignette include a measurement of triiodothyronine, which often is substantially elevated in hyperthyroidism, and measures of thyroid-stimulating immunoglobulins, which are elevated in Graves disease. Radioactive iodine or technetium uptake imaging can distinguish between subacute thyroiditis (low uptake) and Graves disease (high uptake). This distinction is important because treatments for Graves disease, including use of antithyroid drugs, radioactive iodine, and surgery, are not appropriate for transient subacute thyroiditis.

References:

Fenton CL, Gold JG. Hyperthyroidism. eMedicine Specialties, Pediatrics, Endocrinology. 2006. Available at: http://www.emedicine.com/ped/topic1099.htm

Ferry RJ Jr, Levitsky LL. Graves disease. eMedicine Specialties, Pediatrics, Endocrinology. 2006. Available at: http://www.emedicine.com/ped/topic899.htm

LaFranchi S. Clinical manifestations and diagnosis of hyperthyroidism in children and adolescents. UpToDate Online 15.3. 2008. Available for subscription at: http://www.uptodateonline.com/utd/content/topic.do?topicKey=pediendo/5570

Page 324: AAP PREP 2009

2009 PREP SA on CD-ROM

page 324

Critique: 90

Most children who have hyperthyroidism have a goiter. (Courtesy of M. Rimsza)

Page 325: AAP PREP 2009

2009 PREP SA on CD-ROM

page 325

Critique: 90

Exophthalmos in an adolescent who has thyrotoxicosis. (Courtesy of M. Rimsza)

Page 326: AAP PREP 2009

2009 PREP SA on CD-ROM

page 326

Question: 91

An infant in the newborn nursery does not appear to respond to visual or auditory input. On physical examination, he shows evidence of intrauterine growth restriction (IUGR), absent red reflexes, and numerous bluish papules (Item Q91). The mother, who immigrated to the United States during her third trimester, did not receive prenatal care. She denies use of alcohol, drugs, or tobacco products during pregnancy. She reports that she had a low-grade fever and rash during the second month of the pregnancy.

Of the following, the MOST likely infectious cause of the findings in this infant is

A. cytomegalovirus

B. human immunodeficiency virus

C. rubella virus

D. Toxoplasma gondii

E. varicella-zoster virus

Page 327: AAP PREP 2009

2009 PREP SA on CD-ROM

page 327

Question: 91

Blue papules, as exhibited by the infant in the vignette. (Courtesy of M. Rimsza)

Page 328: AAP PREP 2009

2009 PREP SA on CD-ROM

page 328

Critique: 91 Preferred Response: C

Congenital infections may lead to developmental sequelae in infancy and childhood, including visual impairment, hearing loss, and intellectual disabilities (Item C91A). The infant described in the vignette has clinical findings most consistent with congenital rubella syndrome (CRS). These findings include intrauterine growth restriction (IUGR), absent red reflexes (due to cataracts), and bluish papules (Item C91B). Other clinical findings associated with CRS include nerve deafness, microphthalmia, cardiac defects, meningoencephalitis, hepatomegaly, and microcephaly. Maternal cytomegalovirus (CMV) infection is common, but 90% of infants who have CMV infection are asymptomatic at birth. Maternal symptoms include a flulike illness that may involve fever, but a rash is not seen, as reported by the mother in the vignette. Among the clinical findings of congenital CMV infection are hepatomegaly, splenomegaly, jaundice, petechiae, chorioretinitis, IUGR, purpura, and microcephaly. CMV infection is the leading nongenetic reason for sensorineural hearing loss and the most common congenital infection to cause intellectual disability. Congenital varicella infection may result in zigzag scarring of the skin, limb deformities, or cataracts. Occasionally, severely affected infants may have central nervous system involvement with necrotizing cerebral lesions or microcephaly. Congenital toxoplasmosis presents with IUGR, anemia, jaundice, hepatosplenomegaly, intracranial calcifications, hydrocephalus, microcephaly, and chorioretinitis, but not skin lesions. A newborn infected perinatally with human immunodeficiency virus exhibits no symptoms or signs. Later, the infant may develop subtle clinical findings, such as lymphadenopathy and hepatosplenomegaly, or nonspecific symptoms, such as failure to thrive, chronic or recurrent diarrhea, interstitial pneumonia, or oral thrush. Central nervous system involvement is variable, ranging from mild learning disabilities to severe mental retardation.

References:

Adler SP, Marshall B. Cytomegalovirus infections. Pediatr Rev. 2007;28:92-100. Available at: http://pedsinreview.aappublications.org/cgi/content/full/28/3/92

Mason W. Rubella. In: Kliegman RM, Behrman RE, Jenson HB, Stanton, BF, eds. Nelson Textbook of Pediatrics. 18th ed. Philadelphia, Pa: Saunders Elsevier; 2007:1337-1341

McLeod R, Remington JS. Toxoplasmosis (Toxoplasma gondii). In: Kliegman RM, Behrman RE, Jenson HB, Stanton BF, eds. Nelson Textbook of Pediatrics. 18th ed. Philadelphia, Pa: Saunders Elsevier; 2007:1486-1495

Myers MG, Seward J, La Russa P. Varicella-zoster virus. In: Kliegman RM, Behrman RE, Jenson HB, Stanton BF, eds. Nelson Textbook of Pediatrics. 18th ed. Philadelphia, Pa: Saunders Elsevier; 2007:1366-1372

Stagno S. Cytomegalovirus. In: Kliegman RM, Behrman RE, Jenson HB, Stanton BF, eds. Nelson Textbook of Pediatrics. 18th ed. Philadelphia, Pa: Saunders Elsevier; 2007:1377-1379

Yogev R, Gould Chadwick E. Acquired immunodeficiency syndrome (human immunodeficiency virus). In: Kliegman RM, Behrman RE, Jenson HB, Stanton BF, eds. Nelson Textbook of Pediatrics. 18th ed. Philadelphia, Pa: Saunders Elsevier; 2007:1427-1442

Page 329: AAP PREP 2009

2009 PREP SA on CD-ROM

page 329

Critique: 91

Page 330: AAP PREP 2009

2009 PREP SA on CD-ROM

page 330

Critique: 91

Purpuric macules and papules may be present in congenital rubella, creating a "blueberry muffin" appearance. (Courtesy of M. Rimsza)

Page 331: AAP PREP 2009

2009 PREP SA on CD-ROM

page 331

Question: 92

A 6-month-old boy presents to the emergency department with a 2-day history of fever and a 1-day history of left cheek swelling. You discover that his parents do not believe in providing their children with immunizations. Despite this, the boy has never been ill. He has two older siblings, and nobody is sick at home. The mother denies any recent bug bites or trauma to the area on his cheek. Physical examination reveals a mildly toxic-appearing child who has a temperature of 103.0°F (39.4°C), heart rate of 145 beats/min, respiratory rate of 26 breaths/min, and blood pressure of 80/45 mm Hg. His anterior fontanelle is slightly bulging, his tympanic membranes are erythematous, his left cheek is indurated and appears erythematous to slightly violaceous (Item Q92), and he is irritable.

Of the following, the MOST likely organism to cause this child’s illness is

A. Haemophilus influenzae type b

B. Neisseria meningitidis

C. Staphylococcus aureus

D. Streptococcus pneumoniae

E. Streptococcus pyogenes

Page 332: AAP PREP 2009

2009 PREP SA on CD-ROM

page 332

Question: 92

(Courtesy of D. Krowchuk)

Page 333: AAP PREP 2009

2009 PREP SA on CD-ROM

page 333

Critique: 92 Preferred Response: A

Children who have not received the Haemophilus influenzae type b (Hib) vaccine are at risk for illnesses commonly caused by this organism, including buccal and periorbital cellulitis (as described for the boy in the vignette) (Item C92A), pyogenic arthritis, epiglottitis (Item C92B), and bacterial meningitis. Neisseria meningitidis usually does not cause a facial cellulitis, and Staphylococcus aureus and Streptococcus pyogenes are less likely pathogens in the absence of a history of a break in the skin. S pneumoniae can be the cause of a nontraumatic facial cellulitis in Hib-vaccinated children, but in an unvaccinated child, Hib would be the most likely pathogen. Hib disease can be verified by recovery of the organism from a sterile site (eg, blood, cerebrospinal fluid, joint fluid) or by urine antigen testing. Once the organism is isolated, antimicrobial susceptibility testing is important because approximately 30% to 40% of Hib isolates produce beta-lactamase, making these organisms resistant to ampicillin.

References:

American Academy of Pediatrics. Haemophilus influenzae infections. In: Pickering LK, Baker CJ, Long SS, McMillan JA, eds. Red Book: 2006 Report of the Committee on Infectious Diseases. 27th ed. Elk Grove Village, Ill: American Academy of Pediatrics; 2006:310-318

Tzanakaki G, Mastrantonio P. Aetiology of bacterial meningitis and resistance to antibiotics of causative pathogens in Europe and in the Mediterranean region. Int J Antimicrob Agents. 2007;29:621-629. Abstract available at: http://www.ncbi.nlm.nih.gov/pubmed/17368858

Page 334: AAP PREP 2009

2009 PREP SA on CD-ROM

page 334

Critique: 92

In buccal cellulitis caused by Haemophilus influenzae type b infection, the affected area is indurated and has an erythematous-to-violaceous color. (Courtesy of D. Krowchuk)

Page 335: AAP PREP 2009

2009 PREP SA on CD-ROM

page 335

Critique: 92

Epiglottitis: Swelling and erythema of the epiglottis caused by Haemophilus influenzae type b infection. (Courtesy of K. Woodin)

Page 336: AAP PREP 2009

2009 PREP SA on CD-ROM

page 336

Question: 93

You are seeing a 5-year-old boy who has developed diplopia, dysphagia, dry mouth, diarrhea, weakness in his arms, and shortness of breath over the past 18 hours. According to his records, he received his diphtheria, tetanus, acellular pertussis (DTaP), poliovirus inactivated (IPV), measle-mumps-rubella (MMR), and varicella booster immunizations about 1 month ago. He attended a class picnic 3 weeks ago that was held in a state park. He has no history of unusual exposures or ill contacts, and except for falling off his bike 5 days ago and scraping his arm, he has had no other trauma. Physical examination reveals an awake and alert boy who complains of "seeing double" and of pain with swallowing. His pupils are 3 mm bilaterally and sluggish, and his mucous membranes are dry. He takes shallow breaths, but his lungs are clear, and his abdomen is mildly distended. His left arm has a 4x4-cm abrasion that is mildly swollen, erythematous, and tender, with some serosanguineous drainage. His left arm has 2/5 strength and decreased tone. He has 1+ reflexes in the upper and lower extremities.

Of the following, the MOST likely cause of this patient’s condition is

A. botulism

B. cerebral vascular accident

C. Guillain-Barré syndrome

D. tetanus

E. tick paralysis

Page 337: AAP PREP 2009

2009 PREP SA on CD-ROM

page 337

Critique: 93 Preferred Response: A

Clostridium botulinum spores are found worldwide in soil and marine sediments. Botulism results from the absorption of botulinum toxins, most commonly produced by C botulinum (a large, gram-positive, anaerobic bacillus that has a subterminal spore), into the circulation from a wound or mucosal surface. Of the seven antigenic toxin types that the organism produces, human botulism is caused by neurotoxins A, B, E, and occasionally F. The clinical forms of botulism include: foodborne, infant, wound, and that of undetermined cause, which is rare. Almost all cases of infant botulism and wound botulism are caused by types A and B toxin. The onset of symptoms may be abrupt (within hours) or evolve gradually over several days. Foodborne botulism occurs with the ingestion of food that is contaminated with spores of C botulinum and has been stored improperly under anaerobic conditions, allowing for toxin production. In wound botulism, the spores are introduced into the wound at the time of trauma, where they germinate and produce toxin. Infant botulism results from ingestion of C botulinum spores that germinate, multiply, and produce toxin in the intestine. Most cases of infant disease occur in breastfed infants at the time of introduction of nonhuman milk substances, and the source of the spores usually is not identified. The classic presentation of botulism is acute, bilateral cranial nerve palsies associated with symmetric descending weakness. Fever is absent, sensory deficits do not occur, and the patient remains responsive, with a normal level of consciousness. Foodborne disease usually develops between 12 and 36 hours after toxin ingestion. The initial symptoms include nausea, dry mouth, and diarrhea. Disease progresses to cranial nerve dysfunction, most commonly starting with the eyes (diplopia, blurry vision), and descending to include dysphagia, upper extremity weakness, respiratory dysfunction, and lower extremity weakness. The symptoms of wound botulism are similar to that of food-borne disease, although the incubation period is 4 to 14 days from the time of injury until the onset of symptoms. Also, in many cases, the wound may not appear to be healing, as described for the boy in the vignette. Infant botulism develops 3 to 30 days from the time of exposure to the spore-containing material. Infants present with constipation, which is followed by feeding difficulties, hypotonia, increased drooling, a weak cry, diminished gag reflex, truncal weakness, cranial nerve palsies, and generalized weakness with ventilatory failure. Progression occurs for 1 to 2 weeks, followed by stabilization for another 2 to 3 weeks before recovery starts. An important element in the treatment of all forms of botulism is meticulous supportive care. In addition, patients who have suspected foodborne and wound botulism should be treated with equine trivalent antitoxin (types A, B, and E) available from the Centers for Disease Control and Prevention through state health departments. Immediate administration of antitoxin is critical for successful therapy because it arrests the progression of paralysis but does not reverse it. Patients who have wound botulism also should undergo wound debridement, even if the wound appears to be healing. The role of antibiotic therapy is unknown, but penicillin G or metronidazole frequently is recommended. Human-derived botulinum antitoxin (botulism immune globulin intravenous) (BIGIV) is used in the treatment of infant botulism caused by type A or type B C botulinum toxin. BIGIV therapy should be initiated as early in the illness as possible. Antibiotic therapy is not recommended for infant botulism. Patients who have cerebral vascular accidents usually do not present with bilateral symmetric weakness or the other symptoms demonstrated by the patient in the vignette. Patients who have tetanus have persistent, painful tonic spasms of the muscles of the neck, jaw, and trunk and are very rigid. The paralysis of tick paralysis is ascending, beginning in the lower extremities and ascending symmetrically to involve the trunk, upper extremities, and head within a few hours. Patients who have Guillain-Barré syndrome typically present with numbness and paresthesias of the hands and feet, followed by progressive weakness involving all four extremities. Motor impairment begins in the lower extremities and progresses in an ascending pattern to involve the upper extremities, trunk, and cranial nerves.

Page 338: AAP PREP 2009

2009 PREP SA on CD-ROM

page 338

References:

American Academy of Pediatrics. Botulism and infant botulism (Clostridium botulinum). In: Pickering LK, Baker CJ, Long SS, McMillan JA, eds. Red Book: 2006 Report of the Committee on Infectious Diseases. 27th ed. Elk Grove Village, Ill: American Academy of Pediatrics; 2006:257-260

American Academy of Pediatrics. Tetanus (lockjaw). In: Pickering LK, Baker CJ, Long SS, McMillan JA, eds. Red Book: 2006 Report of the Committee on Infectious Diseases. 27th ed. Elk Grove Village, Ill: American Academy of Pediatrics; 2006:648-653

Bleck TP. Clostridium botulinum (botulism). In: Mandell GL, Bennett JE, Dolin R, eds. Mandell, Douglas, and Bennett's Principles and Practice of Infectious Diseases. 6th ed. Philadelphia, Pa: Elsevier Churchill Livingstone; 2005:2822-2828

Mathieu ME, Wilson BB. Ticks (including tick paralysis). In: Mandell GL, Bennett JE, Dolin R, eds. Mandell, Douglas, and Bennett's Principles and Practice of Infectious Diseases. 6th ed. Philadelphia, Pa: Churchill Livingstone; 2005:3312-3315

Parke JT. Peripheral neuropathies. In: McMillan JA, Feigin RD, DeAngelis CD, Jones MD Jr, eds. Oski's Pediatrics Principles and Practice. 4th ed. Philadelphia, Pa: Lippincott Williams & Wilkins; 2006:2310-2316

Schlagger B, Kornberg AJ, Prensky AL. Cerebrovascular disease in childhood. In: McMillan JA, Feigin RD, DeAngelis CD, Jones MD Jr, eds. Oski's Pediatrics Principles and Practice. 4th ed. Philadelphia, Pa: Lippincott Williams & Wilkins; 2006:2270-2279

Page 339: AAP PREP 2009

2009 PREP SA on CD-ROM

page 339

Question: 94

An 8-month-old girl who has a history of cardiomyopathy following viral myocarditis presents with poor weight gain. She is receiving a 20-kcal/oz milk-based formula and has no history of vomiting or diarrhea. Her only medication is furosemide. Physical examination findings include a heart rate of 130 beats/min, respiratory rate of 60 breaths/min, and blood pressure of 88/44 mm Hg.

Of the following, the MOST appropriate initial strategy to increase weight gain for this girl is to

A. change to a 24-kcal/oz formula

B. discontinue furosemide therapy

C. increase the volume of 20-kcal/oz formula

D. place a gastrostomy feeding tube

E. start parenteral nutrition

Page 340: AAP PREP 2009

2009 PREP SA on CD-ROM

page 340

Critique: 94 Preferred Response: A

Young infants who have cardiac dysfunction, such as the girl described in the vignette, often have difficulty ingesting sufficient calories for growth because they frequently have increased caloric expenditure with feeding. Increased caloric intake may place the infants at risk of fluid overload, necessitating the use of diuretics. Because an increased volume of feedings with a 20-kcal/oz formula may place the infant at risk of fluid overload, use of a more concentrated formula is required. Accordingly, the infant described in the vignette should be changed to a 24-kcal/oz formula. Increasing the caloric density of feedings can meet the goals of increased calories and "relative" fluid restriction. Discontinuation of furosemide likely would lead to weight gain from fluid retention, not a true weight gain. A gastrostomy tube provides a conduit for feeding and probably plays a role in a child incapable of taking in sufficient calories (eg, chronic renal failure or severe developmental delay), but the child in the vignette deserves a trial of high-calorie feedings before subjecting her to this surgical procedure. Finally, parenteral nutrition is not a suitable option because this child has a functional gastrointestinal tract and can tolerate enteral nutrition.

References:

Kelleher DK, Laussen P, Teixeira-Pinto A, Duggan C. Growth and correlates of nutritional status among infants with hypoplastic left heart syndrome (HLHS) after stage 1 Norwood procedure. Nutrition. 2006; 22:237-244. Abstract available at: http://www.ncbi.nlm.nih.gov/pubmed/16500550

Pillo-Blocka F, Adatia I, Sharieff W, McCrindle BW, Zlotkin S. Rapid advancement to more concentrated formula in infants after surgery for congenital heart disease reduces duration of hospital stay: a randomized clinical trial. J Pediatr. 2004;145:761-766. Abstract available at: http://www.ncbi.nlm.nih.gov/pubmed/15580197

Yahav J, Avigad S, Frand M, et al. Assessment of intestinal and cardiorespiratory function in children with congenital heart disease on high-caloric formulas. J Pediatr Gastroenterol Nutr. 1985;4:778-785. Abstract available at: http://www.ncbi.nlm.nih.gov/pubmed/4045636

Page 341: AAP PREP 2009

2009 PREP SA on CD-ROM

page 341

Question: 95

An 18-month-old girl has been having an intermittent nonproductive cough for the past 6 months. Her parents state that the cough awakens the toddler at night a few times a month and occurs when playing vigorously. During a recent upper respiratory tract illness, her cough worsened and occurred daily for 3 weeks. On physical examination, there is no nasal discharge, and the toddler appears healthy.

Of the following, the MOST likely diagnosis is

A. asthma

B. atypical pneumonia

C. gastroesophageal reflux

D. sinusitis

E. upper airway cough syndrome

Page 342: AAP PREP 2009

2009 PREP SA on CD-ROM

page 342

Critique: 95 Preferred Response: A

The chronic cough that is exacerbated during the night, with activity, and during an upper respiratory tract infection described for the child in the vignette most likely represents asthma. Chronic cough typically is defined as one that persists for more than 8 weeks. When the patient's chest radiograph appears normal, three causes account for 95% of chronic coughs: asthma, gastroesophageal reflux (GER), and upper airway cough syndrome (UACS) (previously termed postnasal drip syndrome) (Item C95). Asthma usually develops in early childhood, with 80% of patients reporting symptoms prior to age 6 years. Symptoms may include cough, wheezing, shortness of breath, and chest tightness. The most common trigger for infants and toddlers is a viral upper respiratory tract infection (URI). Fortunately, URI-induced wheezing resolves in most infants by age 6 years (so-called "transient wheezers"). Those who continue to have asthma symptoms after age 6 are at greater risk for persistent asthma. UACS encompasses allergic rhinitis, nonallergic rhinitis, and sinusitis. Allergic rhinitis typically occurs in children older than 3 years of age and is associated with other ocular and nasal symptoms, such as pruritus, sneezing, and rhinorrhea. Sinusitis also is characterized by rhinorrhea and postnasal symptoms. Atypical pneumonia caused by Mycoplasma pneumoniae and Chlamydophila pneumoniae (previously termed Chlamydia pneumoniae ) may present at any age, although it is unusual prior to age 3 years. Characteristic constitutional symptoms include fever, malaise, and headache. Cough can represent the sole manifestation of GER, but GER usually becomes symptomatic during the first few postnatal months, improving by 12 months of age. GER may worsen at night during supine positioning, but exercise and URIs are uncommon precipitating factors for GER symptoms.

References:

Liu AH, Covar RA, Spahn JD, Leung DYM. Childhood asthma. In: Kleigman RM, Behrman RE, Jenson HB, Stanton BF, eds. Nelson Textbook of Pediatrics. 18th ed. Philadelphia, Pa: Saunders Elsevier; 2007:953-969

Weinberger M, Abu-Hasan M. Pseudo-asthma: when cough, wheezing, and dyspnea are not asthma. Pediatrics. 2007;120:855-864. Available at: http://pediatrics.aappublications.org/cgi/content/full/120/4/855

Page 343: AAP PREP 2009

2009 PREP SA on CD-ROM

page 343

Critique: 95

Page 344: AAP PREP 2009

2009 PREP SA on CD-ROM

page 344

Question: 96

A 10-year-old boy comes to the office 2 days after falling off of his bicycle and injuring his forehead. He denies vomiting or headache but complains of a runny nose. Physical examination reveals a well-appearing boy who has a large ecchymotic swelling over the central portion of his forehead with an overlying abrasion. The area is diffusely tender to palpation, and there is a depression over the right lateral aspect of the swelling. Erythema around the abrasion is minimal, and no purulent drainage is present. Clear fluid is draining from his right naris. The remainder of his physical examination findings are normal. You order a computed tomography scan (Item Q96).

Of the following, the MOST appropriate treatment of this boy’s injury should include

A. decongestants

B. nasal packing

C. no specific treatment

D. prophylactic antibiotics

E. surgical repair

Page 345: AAP PREP 2009

2009 PREP SA on CD-ROM

page 345

Question: 96

(Courtesy of D. Mulvihill)

Page 346: AAP PREP 2009

2009 PREP SA on CD-ROM

page 346

Critique: 96 Preferred Response: E

The history of trauma and the physical findings of forehead ecchymosis with clear, persistent rhinorrhea described for the boy in the vignette should raise concern for a cerebrospinal fluid (CSF) leak. Prompt radiologic imaging can aid in determining the cause. In most cases, a frontal sinus fracture is the cause, as exhibited by the boy in the vignette, and head computed tomography (CT) scan usually is adequate to identify the injury (Item C96). If an obvious fracture is not seen on CT scan, further imaging with magnetic resonance imaging or radionucleotide cisternography may be necessary to localize the source. Testing of the fluid for glucose or protein is not diagnostically sensitive and should not be used to determine further evaluation and treatment. The incidence of CSF leak following frontal sinus fracture is nearly 20% in children, twice the rate seen in adults. The primary concern in patients who have CSF rhinorrhea is the development of central nervous system (CNS) infections such as meningitis or brain abscess. Such complications are seen most commonly after trauma and in patients whose leaks do not resolve spontaneously within 7 days. Anterior wall frontal sinus fractures are repaired primarily for cosmetic reasons, but those involving the posterior wall (which occurs in 70% of these injuries in children) require open reconstruction to ensure that the communication between the sinus and brain is eliminated. Decongestants and nasal packing are not indicated in the treatment of CSF rhinorrhea. The use of prophylactic antibiotics for the prevention of CNS infection is controversial, although most studies suggest that the risk of developing antibiotic resistance is greater than the potential prevention of meningitis.

References:

Kellman RM. Maxillofacial trauma. In: Cummings CW, Flint PW, Haughey BH, Robbins KT, Thomas JR eds. Cummings Otolaryngology: Head & Neck Surgery. 4th ed. Philadelphia, Pa: Mosby Elsevier; 2005:chap 26

Kerr JT, Chu FW, Bayles SW. Cerebrospinal fluid rhinorrhea: diagnosis and management. Otolaryngol Clin North Am. 2005;38:597-611. Abstract available at: http://www.ncbi.nlm.nih.gov/pubmed/16005720

Kravitz PR, Koltai PJ. Pediatric facial fractures. In: Cummings CW, Flint PW, Haughey BH, Robbins KT, Thomas JR eds. Cummings Otolaryngology: Head & Neck Surgery. 4th ed. Philadelphia, Pa: Mosby Elsevier; 2005:chap 202

Page 347: AAP PREP 2009

2009 PREP SA on CD-ROM

page 347

Critique: 96

Axial computed tomography scan of the head, as described for the child in the vignette, shows fractures through the anterior and posterior walls of the right frontal sinus and air within the cranium. (Courtesy of D. Mulvihill)

Page 348: AAP PREP 2009

2009 PREP SA on CD-ROM

page 348

Question: 97

A 16-year-old boy in your practice has cystic fibrosis. As a complication of his illness, he has developed cirrhosis and cholestasis. He now complains of shaky hands. Neurologic examination demonstrates hyporeflexia and tremor with hands outstretched.

Of the following, the patient’s symptoms are MOST consistent with deficiency of

A. vitamin A

B. vitamin B1 (thiamine)

C. vitamin C

D. vitamin D

E. vitamin E

Page 349: AAP PREP 2009

2009 PREP SA on CD-ROM

page 349

Critique: 97 Preferred Response: E

Because the young man described in the vignette has chronic cholestasis, he is at risk for developing deficiency of any of the fat-soluble vitamins, including vitamins A, D, E, and K. His neurologic symptoms of tremor and hyporeflexia most strongly suggest vitamin E deficiency. Vitamin E (tocopherol) is an important factor in stabilizing the lipid membrane of the red blood cell and the lipids in the myelin sheath of neurons. Therefore, the most common presenting features of hypovitaminosis E are hemolysis (primarily reported in preterm infants) and peripheral neuropathy (identified in infants and children who have chronic cholestasis, pancreatic insufficiency, or malabsorption). Supplementation of formulas and parenteral nutrition with vitamin E has reduced substantially the incidence of hemolysis in the vitamin E-deficient preterm infant. However, patients who have cystic fibrosis or cholestatic liver disease require both monitoring of vitamin E concentrations and supplementation with vitamin E. Because vitamin E is a fat-soluble vitamin, those who have cholestasis may have difficulty absorbing alpha-tocopherol, the form of vitamin E available in most dietary supplements. For this reason, d-alpha-tocopheryl polyethylene glycol 1,000 succinate, a water-soluble form of vitamin E, should be given to patients who have significant cholestatic liver disease. The recommended dose for a patient who has cholestatic liver disease is 15 to 25 IU/kg per day. Deficiency of vitamin A, B1, C, or D would not be expected to cause such a clinical presentation. Vitamin A deficiency causes impaired vision ("night blindness") and corneal ulcers; vitamin B1 deficiency can cause myopathy and heart failure ("beriberi"); vitamin C deficiency causes irritability, bone lesions, and bruising (scurvy); and vitamin D deficiency causes osteopenia or rickets.

References:

Harmatz P, Burensky E, Lubin B. Nutritional anemias. In: Walker WA, Watkins JB, Duggan C, eds. Nutrition in Pediatrics. 3rd ed. Hamilton, Ontario, Canada: BC Decker; 2003:830-847

Spinozzi NS. Hepatobiliary diseases. In: Hendricks KM, Duggan C. Manual of Pediatric Nutrition. 4th ed. Hamilton, Ontario, Canada: BC Decker; 2005:586-592

Page 350: AAP PREP 2009

2009 PREP SA on CD-ROM

page 350

Question: 98

You are making rounds with medical students in the neonatal intensive care unit and examining a 2-kg, 34 weeks’ gestation newborn whose mother had gestational diabetes mellitus. The infant has no respiratory distress. A medical student asks how to test for fetal lung maturity to predict the risk of neonatal respiratory distress syndrome in the offspring of a pregnant woman who has diabetes mellitus.

Of the following, the MOST appropriate test is

A. disaturated lecithin presence

B. lecithin:sphingomyelin ratio

C. phosphatidylglycerol presence

D. phosphatidylinositol presence

E. total surfactant activity

Page 351: AAP PREP 2009

2009 PREP SA on CD-ROM

page 351

Critique: 98 Preferred Response: E

The human fetus continues to develop lung maturity up until a term gestation, but the lungs generally are sufficiently mature to maintain extrauterine respiration by 36 weeks' gestation. Infants delivered prior to this time have variable degrees of pulmonary maturity as surfactant composition, synthesis, and storage change with advancing fetal development. Tests for determining fetal lung maturation depend on amniotic fluid analysis for surfactant presence and composition because fetal surfactant leaves the lung as an effluent that ebbs and flows with fetal breathing movement and is balanced by fetal swallowing. In general, in the healthy fetus, growth, size, and maturation typically are linked with gestational age. Fetal lung maturity has been assessed using the lecithin-to-sphingomyelin ratio (L:S) for more than 30 years. This test is time-consuming and requires thin-layer chromatography. It depends on fetal lung fluid flowing into the amniotic fluid and altering the amniotic fluid phospholipid composition, with the results expressed as the ratio of a lecithin (phosphatidylcholine) to sphingomyelin per milliliter of amniotic fluid. The sphingomyelin content of amniotic fluid decreases after 32 weeks' gestational age, while the lecithin content, including that portion that is disaturated, increases. The L:S ratio is designed to account for varying amniotic fluid volumes that generally cannot be measured precisely. An L:S ratio of 2.0 typically is achieved by 35 weeks' gestation. Clinically, respiratory distress syndrome (RDS) due to surfactant deficiency is very unlikely if the L:S ratio is 2.0 or greater, it is indeterminate if the L:S ratio is 1.5 to 2.0, and the incidence of RDS is high if the L:S ratio is less than 1.0. Of note, the L:S ratio can increase over a period of several days or can be induced by the administration of glucocorticoids to the mother. Phosphatidylinositol (PI) is a pulmonary phospholipid that increases throughout gestation, as does disaturated lecithin. PI concentrations decrease after 35 weeks' gestation. The mere presence of PI or disaturated lecithin, therefore, does not indicate lung maturity. Phosphatidylglycerol (PG) generally can be detected in the amniotic fluid at 36 weeks' gestation. Due to its appearance relatively late in gestation, it has been used as an indicator of pulmonary surfactant maturity. In pregnancies complicated by maternal diabetes, a mature L:S ratio (>2.0) or the presence of PG may not indicate pulmonary surfactant maturity. In this setting, fetal lung maturity is determined best using a quantitative analysis of total surfactant activity. This rapidly determined, automated fluorescence polarization assay measures all surfactant phospholipids in amniotic fluid referenced as milligrams of surfactant per gram of albumin present. Recent data suggest that a total surfactant activity of greater than 45 mg surfactant phospholipids per gram of albumin is as good as, or better than, an L:S ratio of 2.0 in predicting fetal lung maturity, especially in the presence of maternal diabetes.

References:

Jobe AH. Lung development and maturation. In: Martin RJ, Fanaroff AA, Walsh MC, eds. Fanaroff and Martin's Neonatal-Perinatal Medicine. 8th ed. Philadelphia, Pa: Mosby Elsevier; 2006:1069-1086

Grenache DG, Gronowski AM. Fetal lung maturity. Clin Biochem. 2006;39:1-10. Abstract available at: http://www.ncbi.nlm.nih.gov/pubmed/16303123

Winn-McMillan T, Karon BS. Comparison of the TDx-FLM II and lecithin to sphingomyelin ratio assays in predicting fetal lung maturity. Am J Obstet Gynecol. 2005;193:778-782. Abstract available at: http://www.ncbi.nlm.nih.gov/pubmed/16150274

Page 352: AAP PREP 2009

2009 PREP SA on CD-ROM

page 352

Question: 99

At the end of the summer, you notice an increase in the number of preparticipation sports examinations you are performing. You are pleased at the number of your patients who are involved in sports activities but are reminded that many medical conditions preclude sports participation and must be screened for during the preparticipation visit.

Of the following, the medical condition that is considered a CONTRAINDICATION for sports participation is

A. a boy who has chronic leukemia and splenomegaly wishing to play golf

B. a boy who has insulin-dependent diabetes wishing to play tennis

C. a boy who has well-controlled seizures wishing to participate in basketball

D. a girl who has Marfan syndrome wishing to participate in gymnastics

E. a girl who has one ovary wishing to participate in softball

Page 353: AAP PREP 2009

2009 PREP SA on CD-ROM

page 353

Critique: 99 Preferred Response: D

Sports participation can play an important role in the social development of children and adolescents by teaching leadership and team-building skills and encouraging physical fitness. Most children can be cleared easily for such participation, but not all sports are safe for children and adolescents, and some medical conditions warrant special consideration with regard to participation in some sports. In all cases, the clinician should perform a complete preparticipation sports physical examination and discuss the implications of the sports the patient plans to pursue. Sports can be classified by contact level (Item C99A). Children who have splenomegaly, acute hepatomegaly, and contagious skin lesions should avoid contact sports but can participate in noncontact sports. Children who have a single kidney should be advised to avoid high-contact sports, but those who have other single organs, such as ovaries and testes, can be cleared because the risk of injury is low or protective gear can be worn. Protective eyewear can reduce the risk of injury in the case of a single functional eye. It is prudent for the clinician to advise such patients of the risk to the remaining organ during sports participation, and some advocate having the parents and the athlete sign a document that indicates their understanding and acceptance of such risk. Sports also may be classified by intensity (dynamic and static demand) (Item C99B). This classification is especially important for children and adolescents who have cardiovascular diseases. For example, children who have ventricular dysfunction should avoid high dynamic sports; those who have significant essential hypertension, left heart obstructive disease, or an increased risk of aortic dissection should avoid high static sports. Accordingly, a girl who has Marfan syndrome should avoid gymnastics, a sport that has potentially high static intensity. Several cardiac diseases, such as acute pericarditis or myocarditis, cardiomyopathy, severe pulmonary hypertension, and right-to-left shunting, are absolute contraindications to participation in sports. A full listing of these conditions is outlined in the 36th Bethesda Conference guidelines. Patients who have well-controlled epilepsy can be cleared to participate in most sports, but those whose seizures are poorly controlled should be advised to avoid sports in which they could sustain significant injury should a seizure occur during participation, such as swimming, diving, and riflery. Patients who have diabetes should be encouraged to participate in sports, but they should pay attention to hydration and insulin therapy, especially with sports that last longer than 30 minutes.

References:

36th Bethesda conference: eligibility recommendations for competitive athletes with cardiovascular abnormalities. J Am Coll Cardiol. 2005;45:1312-1375. Available at: http://content.onlinejacc.org/cgi/reprint/45/8/1312

American Academy of Pediatrics Committee on Sports Medicine and Fitness. Medical conditions affecting sports participation. Pediatrics. 2001;107:1205-1209. Available at: http://pediatrics.aappublications.org/cgi/content/full/107/5/1205

Metzl JD. Preparticipation examination of the adolescent athlete: part 1. Pediatr Rev. 2001;22:199-204. Available at: http://pedsinreview.aappublications.org/cgi/content/full/22/6/199

Metzl JD. Preparticipation examination of the adolescent athlete: part 2. Pediatr Rev. 2001;22:227-239. Available at: http://pedsinreview.aappublications.org/cgi/content/full/22/7/227

Singh A, Silberbach M. Consultation with the specialist: cardiovascular preparticipation sports screening. Pediatr Rev. 2006;27:418-424. Available at:

Page 354: AAP PREP 2009

2009 PREP SA on CD-ROM

page 354

http://pedsinreview.aappublications.org/cgi/content/full/27/11/418

Page 355: AAP PREP 2009

2009 PREP SA on CD-ROM

page 355

Critique: 99

Page 356: AAP PREP 2009

2009 PREP SA on CD-ROM

page 356

Critique: 99

Page 357: AAP PREP 2009

2009 PREP SA on CD-ROM

page 357

Question: 100

A child presents to a clinic associated with a disaster relief shelter after a hurricane destroys the community. The mother states that they have run out of the creams prescribed for her daughter’s eczema before the storm, and the child is itchy. She is concerned because there are some blisters and crusting in the antecubital fossae and popliteal fossa where the itching is worst. Physical examination reveals erosions (Item Q100) and erythema surrounding areas of lichenification, with a few vesicles both in clusters and scattered.

Of the following, the MOST likely pathogen involved in this pattern of infection is

A. herpes simplex virus

B. human papillomavirus

C. Sporothrix sp

D. varicella-zoster virus

E. viridans streptococci

Page 358: AAP PREP 2009

2009 PREP SA on CD-ROM

page 358

Question: 100

Erosions that have a "punched-out" appearance, as described for the child in the vignette. (Reprinted with permission from Stricker T, Lips U, Sennhauser FH. Visual diagnosis: an 8-month-old who has an erupting rash. Pediatr Rev. 2007;28:231-234.)

Page 359: AAP PREP 2009

2009 PREP SA on CD-ROM

page 359

Critique: 100 Preferred Response: A

Children who have atopic dermatitis are prone to recurrent skin infections, particularly with Staphylococcus aureus and herpes simplex virus (HSV), for several reasons. Exacerbations of eczema disrupt the skin's protective barrier. The failure to produce endogenous antimicrobial peptides has been offered as a reason for an increase risk for infection with S aureus. Dissemination of cutaneous viral infections (eg, HSV and molluscum contagiosum) is believed to be due to defects in cytokine production and T-cell function. Secondary infection with S aureus may result in "weepy" or crusted erosions (Item C100A). Use of topical and systemic antistaphylococcal antibiotics, with attention to the emergence of community-acquired antibiotic-resistant strains, is indicated. Use of topical immune modulators (eg, tacrolimus) should be interrupted during the infection because of the potential for inhibiting local immune response and the development of irritation, including burning and stinging. Eczema herpeticum results when areas of active dermatitis are infected by HSV. Initially, there may be an increase in complaints of itching and scratching, with a noticeable increase in erythema; fever may be present. As described for the child in the vignette, clustered, often umbilicated vesicles appear; these rupture, forming ulcers that have a "punched-out" appearance (Item C100B). Infection may become disseminated and sometimes is fatal. For those who have extensive involvement, recommended treatment is parenteral acyclovir. Sporotrichosis is a dimorphic fungus that can be isolated from soil and plants such as roses; it also may be found in hay, straw, and decaying vegetation. It may cause infection at a site of minor trauma that appears as a solitary violaceous nodule (Item C100C). Although infections caused by streptococci (viridans and enterococci) may be observed in immunocompromised hosts and in neonates, they do not cause the type of disease described for the child in the vignette. Papules, not vesicles, are characteristic of human papillomavirus infection of the skin. Primary infection with varicella-zoster virus results in varicella, with individual, not grouped, vesicles that rupture, leaving shallow erosions, not "punched-out" ulcers.

References:

American Academy of Pediatrics. Non-group A or B streptococcal and enterococcal infections. In: Pickering LK, Baker CJ, Long SS, McMillan JA, eds. Red Book: 2006 Report of the Committee on Infectious Diseases. 27th ed. Elk Grove Village, Ill: American Academy of Pediatrics; 2006:627-629

American Academy of Pediatrics. Sporotrichosis. In: Pickering LK, Baker CJ, Long SS, McMillan JA, eds. Red Book: 2006 Report of the Committee on Infectious Diseases. 27th ed. Elk Grove Village, Ill: American Academy of Pediatrics; 2006:595-597

Bunikowski R, Mielke M, Skarabis H, et al. Prevalence and role of serum IgE antibodies to the Staphylococcus aureus-derived superantigens SEA and SEB in children with atopic dermatitis. J Allergy Clin Immunol. 1999;103:119-124. Abstract available at: http://www.ncbi.nlm.nih.gov/pubmed/9893195

Horii KA, Simon SD, Liu DY, Sharma V. Atopic dermatitis in children in the United States, 1997-2004: visit trends, patient and provider characteristics, and prescribing patterns. Pediatrics. 2007;120:e527-e534. Available at: http://pediatrics.aappublications.org/cgi/content/full/120/3/e527

Knoell KA, Greer KE. Atopic dermatitis. Pediatr Rev. 1999;20:46-52. Available at: http://pedsinreview.aappublications.org/cgi/content/full/20/2/46

Page 360: AAP PREP 2009

2009 PREP SA on CD-ROM

page 360

Stanbury LR. Herpes simplex virus. In: Kliegman RM, Behrman RE, Jenson HB, Stanton BF, eds. Nelson's Textbook of Pediatrics. Philadelphia, Pa: Saunders Elsevier; 2007:1360-1365

Waggoner-Fountain LA, Grossman LB. Herpes simplex virus. Pediatr Rev. 2004;25:86-93. Available at: http://pedsinreview.aappublications.org/cgi/content/full/25/3/86

Page 361: AAP PREP 2009

2009 PREP SA on CD-ROM

page 361

Critique: 100

When infected with S aureus, lesions of atopic dermatitis become moist and crusted. (Courtesy of D. Krowchuk)

Page 362: AAP PREP 2009

2009 PREP SA on CD-ROM

page 362

Critique: 100

In eczema herpeticum, erosions may have a "punched-out" appearance. (Reprinted with permission from Stricker T, Lips U, Sennhauser FH. Visual diagnosis: an 8-month-old who has an erupting rash. Pediatr Rev. 2007;28:231-234.)

Page 363: AAP PREP 2009

2009 PREP SA on CD-ROM

page 363

Critique: 100

Sporotrichosis produces erythematous or violaceous nodules at the site(s) of inoculation. (Courtesy of Peter Somers, MD, PhD, DermAtlas; www.dermatlas.org)

Page 364: AAP PREP 2009

2009 PREP SA on CD-ROM

page 364

Question: 101

A 4-week-old infant who was born at term without any complications ate well and gained weight for the first 3 weeks after birth. Over the last week, however, his mother reports that he appears hungry but fatigues with feeding and now takes twice as long to complete his feeding as he did 1 week ago. He also breathes fast during his feedings and stops frequently to "catch his breath."

Of the following, the MOST likely explanation for the symptoms in this infant is

A. aspiration syndrome

B. congestive heart failure

C. gastroesophageal reflux disease

D. inborn error of metabolism

E. pneumonia

Page 365: AAP PREP 2009

2009 PREP SA on CD-ROM

page 365

Critique: 101 Preferred Response: B

The symptoms described for the child in the vignette are typical of progressive congestive heart failure (CHF). The "hunger" described by the mother suggests that the problem results from the infant's inability to take enough calories for satiation and growth. Such so-called poor feeding is due to his inability to generate a prolonged suck while maintaining nasal breathing because of the tachypnea that is caused by pulmonary congestion. Thus, the infant seems to stop sucking and "catch his breath." Decreased caloric intake coupled with increased caloric expenditure caused by tachypnea and tachycardia makes it difficult for the infant to gain weight. Indeed, weight loss is common in infants who have CHF. CHF is a clinical syndrome that reflects the inability of the myocardium to meet the metabolic requirements of the body, including those for growth. Congenital heart defects are the most common reason for pediatric heart failure, and the failure develops most frequently during early infancy. CHF results from excessive workload imposed on the cardiac muscle that usually is caused by the structural defects. The cardiac defects may impose an excessive volume load on the left ventricle (eg, large ventricular septal defect, atrioventricular septal defect), an excessive pressure load on the ventricle (eg, aortic stenosis), or a combination of volume and pressure load (eg, ventricular septal defect with coarctation). Less commonly, CHF may be caused by an intrinsic alteration in myocardial performance, which could result from an inflammatory or an infectious process that directly affects the myocardium and depresses its contractile function. Aspiration syndrome typically is an acute event resulting from the passage of gastrointestinal contents (including food) into the lungs. The symptoms usually are acute and not associated with poor feeding in spite of hunger. Similarly, gastrointestinal reflux presents more acutely with signs of discomfort that might include arching during feedings. Pneumonia is associated with other signs of infection, including tachypnea when at rest, fever, or change in behavior. Inborn errors of metabolism, although less common, are important to consider in the neonate and infant who is not feeding well. Most commonly, they are associated with symptoms that are present both at rest and during feeding as well as other systemic signs.

References:

Balfour I. Management of chronic congestive heart failure in children. Curr Treat Options Cardiovasc Med. 2004;6:407-416. Abstract available at: http://www.ncbi.nlm.nih.gov/pubmed/15324616

Dreyer WJ, Fisher DJ. Clinical recognition and management of chronic congestive cardiac failure. In: Garson A Jr, Bricker JT, Fisher DJ, Neish SR, eds. The Science and Practice of Pediatric Cardiology. 2nd ed. Baltimore, Md: Williams & Wilkins, 1998:2309-2325

Silberbach M, Hannon D. Presentation of congenital heart disease in the neonate and young infant. Pediatr Rev. 2007;28:123-131. Available at: http://pedsinreview.aappublications.org/cgi/content/full/28/4/123

Talner NS, McGovern JJ, Carboni MP. Congestive heart failure. In: Moller JH, Hoffman JIE, eds. Pediatric Cardiovascular Medicine. Philadelphia, Pa: Churchill Livingstone; 2000:817-829

Page 366: AAP PREP 2009

2009 PREP SA on CD-ROM

page 366

Question: 102

A 10-year-old boy has double vision and drooping eyelids. On physical examination, he is afebrile and has normal mentation. Pupillary responses are normal, but he has bilateral ptosis. He cannot fully adduct his right eye. You note that his ptosis increases with sustained upward gaze (Item Q102). Bedside forced vital capacity is normal.

Of the following, the test MOST likely to confirm the diagnosis is

A. brainstem auditory evoked potentials

B. cold caloric testing

C. edrophonium test

D. lumbar puncture

E. visual evoked potentials

Page 367: AAP PREP 2009

2009 PREP SA on CD-ROM

page 367

Critique: 102 Preferred Response: C

The child described in the vignette presents with progressive fatigable weakness affecting his ocular muscles and bilateral eyelids. Fatigable weakness means that at the onset of a voluntary movement, or after a period of rest, muscle strength is normal. However, after a period of sustained use, muscle strength diminishes. Such a pattern is characteristic of weakness due to pathology at the neuromuscular junction. New-onset weakness mandates urgent evaluation and consideration of problems involving airway protection and respiratory insufficiency, particularly if there is evidence on physical examination of generalized weakness. If the pharyngeal muscles are affected, speech sounds hyponasal. Negative inspiratory force or forced vital capacity should be documented promptly because these findings determine whether the child should be transferred to an intensive care unit for ventilatory monitoring and support. The first step in the diagnostic process is to localize the problem to the proper level of the nervous system: brain, brainstem/cerebellum, spinal cord, nerve, junction, or muscle. Such a determination not only allows for urgent appropriate diagnosis and treatment but can reduce unnecessary discomfort, risks, and costs of inappropriate diagnostic testing. Consultation with a neurologist can be useful before ordering testing. In this case, bilateral weakness in the eyelids and normal mentation make a cerebral or midbrain cause unlikely. Lateral gaze palsy from a pontine lesion also should affect the full face (as occurs in Bell palsy), which is not present. A multifocal cranial neuropathy is rare and unlikely. The ocular fatigability suggests a neuromuscular junction problem such as myasthenia gravis. The diagnosis of myasthenia gravis can be confirmed with the edrophonium test ("Tensilon® test"), a specialized assessment using the acetylcholinesterase inhibitor edrophonium. Because the test can pose a risk of life-threatening bradycardia, it should be undertaken by an experienced physician with atropine at the bedside. Neurologic examination before and after administration ideally should be documented on video (Item C102). Lumbar puncture is the appropriate test for suspected Guillain-Barré syndrome (GBS), an acute inflammatory demyelinating polyneuropathy. Typically, affected children present with weakness beginning in the proximal legs, pain, and absent reflexes. A variant of GBS can involve predominantly the brainstem and cerebellum, but usually weakness is more extensive than described for this child, and the weakness is not fatigable. Further, at this early stage, the characteristic cerebrospinal fluid findings in GBS, normal cells and high protein, often have not emerged. Therefore, lumbar puncture is not the preferred study in this setting. Cold caloric testing evaluates vestibular function. This patient has no nystagmus and does not require this uncomfortable test. Neurophysiologic testing with auditory or visual evoked potentials is not needed when no clinical evidence suggests that these systems are affected. Treatment of myasthenia gravis is complex and is managed best by an experienced neurologist. Treatment may include immunomodulation (eg, intravenous immunoglobulin or plasmapheresis) and acetylcholinesterase inhibitors. Imaging to assess for the presence of a thymoma also is standard practice in a child. The patient and family also should be educated about disease management and early recognition of impending myasthenic crisis.

References:

Andrews PI. Autoimmune myasthenia gravis in childhood. Semin Neurol. 2004;24:101-110. Abstract available at: http://www.ncbi.nlm.nih.gov/pubmed/15229797

Mehta S. Neuromuscular disease causing acute respiratory failure. Respir Care. 2006;51:1016-1023. Available at: http://www.rcjournal.com/contents/09.06/09.06.1016.pdf

Page 368: AAP PREP 2009

2009 PREP SA on CD-ROM

page 368

Parr JR, Jayawant S. Childhood myasthenia: clinical subtypes and practical management. Dev Med Child Neurol. 2007;49:629-635. Available at: http://www.blackwell-synergy.com/doi/abs/10.1111/j.1469-8749.2007.00629.x

Sarnat HB. Disorders of neuromuscular transmission and of motor neurons. In: Kliegman RM, Behrman RE, Jenson HB, Stanton BF, eds. Nelson Textbook of Pediatrics. 18th ed. Philadelphia, Pa: Saunders Elsevier; 2007:2554-2558

Page 369: AAP PREP 2009

2009 PREP SA on CD-ROM

page 369

Question: 103

You are called to the newborn nursery to evaluate an infant who has a limb anomaly. The infant is normally grown and vigorous. On physical examination, you note a terminal transverse limb defect at the distal aspect of the right forearm, resulting in absence of the hand on that side (Item Q103).

Of the following, these findings are MOST likely related to prenatal exposure to

A. alcohol

B. cocaine

C. marijuana

D. methamphetamine

E. tobacco

Page 370: AAP PREP 2009

2009 PREP SA on CD-ROM

page 370

Question: 103

Terminal transverse limb defect, as exhibited by the infant described in the vignette. (Courtesy of V. Shashi)

Page 371: AAP PREP 2009

2009 PREP SA on CD-ROM

page 371

Critique: 103 Preferred Response: B

Despite extensive medical literature on the teratogenic effects of cocaine, the degree to which maternal cocaine use affects the developing embryo/fetus remains largely uncertain. Many studies on this subject are fraught with design flaws, and a bias toward publishing adverse outcomes has been suggested. Adding to this confusion is the fact that women who abuse cocaine during pregnancy typically do not use the drug in isolation; they often drink alcohol, smoke cigarettes, and use other recreational drugs as well. Even so, prenatal cocaine exposure generally is believed to be associated with vascular disruptive events in the embryo/fetus due to its potent vasoconstrictive and hypertensive effects. Such events include cerebral infarction, urogenital anomalies, and limb reduction defects, as described for the infant in the vignette (Item C103A). In addition, the incidence of placental abruption appears to be increased in cocaine-exposed pregnancies. Although neurobehavioral alterations have been reported in neonates who were exposed to cocaine prenatally, it is not clear that these are due solely to cocaine exposure. Prenatal alcohol exposure is associated with a spectrum of abnormalities termed fetal alcohol spectrum disorder. Individuals who have fetal alcohol syndrome, which is the most severe end of the spectrum, typically exhibit intrauterine growth restriction, microcephaly, neurodevelopmental impairment, and a characteristic pattern of dysmorphic features (Item C103B). Although they may have hypoplasia of the distal fingers and toes, they typically do not have more severe limb deficiencies. The effects of cigarette smoking on pregnancy outcome have been, and continue to be, studied extensively. Cigarette smoking is associated with an increased risk for miscarriage, reduced fetal growth, and abnormal placentation. Generally, cigarette smoking is not associated with major congenital anomalies, although there may be an increase in facial clefting. An association with congenital limb deficiencies has been reported in some studies, but it has not been borne out in others. No increased incidence of birth defects appears to be associated with marijuana or methamphetamine use during pregnancy.

References:

Cigarette smoking, methamphetamine, alcohol, cocaine, marijuana. Reprotox. Available for subscription at: http://www.reprotox.org

Cigarette smoking, methamphetamine, alcohol, cocaine, marijuana. Teris. Available for subscription at http://depts.washington.edu/terisweb/teris/

Hoyme HE, May PA, Kalberg WO, et al. A practical clinical approach to diagnosis of fetal alcohol spectrum disorders: clarification of the 1996 Institute of Medicine criteria. Pediatrics. 2005;115:39-47. Available at: http://pediatrics.aappublications.org/cgi/content/full/115/1/39

Page 372: AAP PREP 2009

2009 PREP SA on CD-ROM

page 372

Critique: 103

Limb reduction defects may result from vascular disruptive events. (Courtesy of V. Shashi)

Page 373: AAP PREP 2009

2009 PREP SA on CD-ROM

page 373

Critique: 103

Characteristics of fetal alcohol syndrome include midface hypoplasia, a relatively smooth philtrum, and a narrow upper lip with a poorly defined "cupid's bow." (Courtesy of M. Rimsza)

Page 374: AAP PREP 2009

2009 PREP SA on CD-ROM

page 374

Question: 104

A 16-year-old girl who attends boarding school in your community comes to your office because she is feeling depressed. You see her alone for the visit, and she relates that she feels suicidal at this time and has a plan to kill herself.

Of the following, the BEST description of your obligation to alert her parents to her situation is that

A. no parental notification is necessary because she is a mature minor

B. no parental notification is necessary because she is an emancipated minor

C. parental notification is necessary due to billing issues

D. parental notification is necessary due to her serious threats of self-harm

E. parental notification is prohibited by the Health Insurance Portability and Accountability Act

Page 375: AAP PREP 2009

2009 PREP SA on CD-ROM

page 375

Critique: 104 Preferred Response: D

Confidentiality and consent issues related to adolescents are often complex and problematic. Confidentiality is defined as an agreement between the patient and clinician that medical information will not be shared without the patient's explicit permission. For the girl described in the vignette, confidentiality must be breached with parental notification due to the serious nature of her threat to herself. Such breaching of confidentiality also would be necessary if she posed a threat to others. Parental notification is not necessary for billing purposes. However, confidentiality may be breached inadvertently by billing procedures, such as parents receiving an explanation of benefits document. The physician should explain such possible breaches and work with the adolescent to avoid them. Informed consent has the vital components of being given voluntarily, the patient having all of the information necessary to make an informed decision, and the individual being capable of making a decision based on the information provided. All states require parental consent for medical care for minors, with some notable exceptions, including emergency care; care for the "emancipated minor"; and care related to certain conditions such as pregnancy, sexually transmitted infections, contraception, substance abuse, and mental illness. The exceptions vary by state. An emancipated minor is a legal status existing for minors who are living apart from parents and are financially independent, but this designation varies among states, and some states do not have this legal designation. Marriage, parenthood, military service, and living independently are common criteria for the emancipated status. Generally, this status allows a minor to consent for all health care. A "mature minor" is one who is deemed by a physician as capable of giving the same degree of consent for treatment as an adult, provided the treatment is low-risk. The mature minor rule may be used in states where no state law regarding minor consent exists. The girl described in the vignette is not financially independent from her parents and, as a result, cannot be considered emancipated. Because her condition and treatment are not low-risk, the mature minor principle would not apply. The Health Insurance Portability and Accountability Act (HIPAA) of 1996 protects the confidentiality of adolescents who are considered minors under some circumstances. Parents and guardians have control over health information access for nonemancipated minors, except in situations in which minors are able to consent to their own health care. HIPAA defers to state laws that allow or prohibit disclosure of confidential information to parents. HIPAA allows a physician to disclose information to a parent if a state law requires the physician to do so. If the state law permits, but does not require, disclosure to a parent, HIPAA allows the physician discretion to disclose. If state law prohibits the disclosure of information to a parent, disclosure must not be made without the minor's permission. If there is no state law in place, the physician has discretion to disclose or not to a parent. For the girl described in the vignette, disclosure is necessary due to the health threat, not by any HIPAA regulation. The Center for Adolescent Health and the Law has published a compendium of state laws that addresses confidentiality and consent and allows clinicians to determine the specifics of the law in their own state of practice.

References:

English A, Kenney KE. State Minor Consent Laws: A Summary. 2nd ed. Chapel Hill, NC: Center for Adolescent Health & the Law; 2003

Joffe A. Legal and ethical issues in adolescent health care. In: Osborn LM, DeWitt TG, First LR, Zenel JA eds. Pediatrics. Philadelphia, Pa: Elsevier Mosby; 2005:1428-1430.

Weddle M, Kokotailo P. Adolescent substance abuse: confidentiality and consent. Pediatr Clin North Am. 2002;49:301-315. Abstract available at: http://www.ncbi.nlm.nih.gov/pubmed/11993284

Page 376: AAP PREP 2009

2009 PREP SA on CD-ROM

page 376

Weddle M, Kokotailo PK. Confidentiality and consent in adolescent substance abuse: an update. Virtual Mentor: American Medical Association Journal of Ethics. 2005;7(3). Available at: http://virtualmentor.ama-assn.org/2005/03/pfor1-0503.html

Page 377: AAP PREP 2009

2009 PREP SA on CD-ROM

page 377

Question: 105

You are assisting a pediatric resident in evaluating a 12-year-old girl who has type 1 diabetes and has been vomiting for the past 12 hours. Initial laboratory results include: · Blood glucose, 630.0 mg/dL (35.0 mmol/L)· Serum sodium, 150.0 mEg/L (150.0 mmol/L) · Serum potassium, 6.0 mEq/L (6.0 mmol/L)· Serum chloride, 90.0 mEq/L (90.0 mmol/L)· Serum bicarbonate, 10.0 mEq/L (10.0 mmol/L) The anion gap for this child is CLOSEST to

A. 4

B. 10

C. 50

D. 80

E. 323

Page 378: AAP PREP 2009

2009 PREP SA on CD-ROM

page 378

Critique: 105 Preferred Response: C

The anion gap is the difference between routinely measured cations and anions in the blood. Clinically, this is estimated by using the following equation: [Na+]-[Cl-]-[HCO3-]. Thus, the anion gap for the child described in the vignette is 150-90-10=50. A normal anion gap is 8 to 16 and reflects unmeasured anions such as proteins, sulfates, phosphates, and organic acids. Calculation of the anion gap assists the clinician in both classifying metabolic acidosis as normal anion gap and elevated anion gap and directing further evaluation and treatment. Metabolic acidosis with an increased anion gap occurs when bicarbonate is lost and replaced by unmeasured anions such as ketones, lactate, phosphate, or urate. Common causes of increased anion gap acidosis in children include ketoacidosis, as seen in diabetes mellitus or starvation; lactic acidosis, as seen in shock or with inborn errors of metabolism; and renal failure. Increased anion gap acidosis also is seen with the accumulation of toxic metabolites from ingestions such as methanol, ethylene glycol, and salicylate. Normal anion gap metabolic acidosis is characterized by loss of bicarbonate with an equal increase in chloride that preserves the normal anion difference. Common causes of normal anion gap acidosis in children include diarrhea, renal tubular acidosis, administration of large volumes of normal saline, and hypoaldosteronism.

References:

Greenbaum LA. Electrolyte and acid-base disorders. In; Kliegman RM, Behrman RE, Jenson HB, Stanton BF, eds. Nelson Textbook of Pediatrics. 18th ed. Philadelphia, Pa: Saunders Elsevier; 2007:267-308

Schwaderer AL, Schwartz GJ. Back to basics: acidosis and alkalosis. Pediatr Rev. 2004;25:350-357. Available at: http://pedsinreview.aappublications.org/cgi/content/full/25/10/350

Page 379: AAP PREP 2009

2009 PREP SA on CD-ROM

page 379

Question: 106

The parents of a 12-year-old girl in whom you recently diagnosed type 1 diabetes mellitus ask you about potential long-term complications. In your discussion, you stress the importance of blood glucose control to prevent complications and review risk factors for diabetes complications, including hyperglycemia and tobacco smoking.

Of the following, the MOST important additional risk factor for diabetes complications is

A. celiac disease

B. hypertension

C. hypothyroidism

D. lack of regular exercise

E. undernutrition

Page 380: AAP PREP 2009

2009 PREP SA on CD-ROM

page 380

Critique: 106 Preferred Response: B

The Diabetes Control and Complications Trial (DCCT) results, published in 1994, demonstrated unambiguously that glycemic control directly correlates with the long-term prevention of complications of diabetes mellitus type 1 (DM1). However, additional risk factors, such as hypertension and cigarette smoking, are almost as important as hyperglycemia in the development of diabetes complications. Celiac disease occurs in approximately 6% of individuals who have DM1 in North America and may hamper diabetes control because of malabsorption of nutrients. It is also a risk factor for poor bone mineralization in individuals who have DM1. However, it does not alter the risk of long-term complications directly. Hypothyroidism due to chronic lymphocytic thyroiditis may develop in 5% or more of people who have DM1, but it is not an independent risk factor for cardiovascular or other DM1 complications unless it is chronically untreated and affects lipid metabolism. Lack of regular exercise has been associated with an increase in cardiovascular risk factors for children who have DM1, but it has not yet been correlated directly with the development of long-term complications. Undernutrition, unless it is the result of poor diabetes control, has not been correlated with long-term complications of diabetes.

References:

Freemark M, Levitsky LL. Screening for celiac disease in children with type 1 diabetes: two views of the controversy. Diabetes Care. 2003;26:1932-1939. Available at: http://care.diabetesjournals.org/cgi/content/full/26/6/1932

Gallego PH, Wiltshire E, Donaghue KC. Identifying children at particular risk of long-term diabetes complications. Pediatr Diabetes. 2007;8(suppl 6):40-48. Abstract available at: http://www.ncbi.nlm.nih.gov/pubmed/17727384

Glastras SJ, Mohsin F, Donaghue KC. Complications of diabetes mellitus in childhood. Pediatr Clin North Am. 2005;52:1735-1753. Abstract available at: http://www.ncbi.nlm.nih.gov/pubmed/16301091

Herbst A, Kordonouri O, Schwab KO, Schmidt F, Holl RW, on behalf of the DPV Initiative of the German Working Group for Pediatric Diabetology Germany. Impact of physical activity on cardiovascular risk factors in children with type 1 diabetes. Diabetes Care. 2007;30:2098-2100. Available at: http://care.diabetesjournals.org/cgi/content/full/30/8/2098

Levitsky LL, Misra M. Complications and screening in children and adolescents with type 1 diabetes mellitus. UpToDate Online 15.3. 2008. Available for subscription at:http://www.uptodateonline.com/utd/content/topic.do?topicKey=pediendo/17677

Raile K, Galler A, Hofer S, et al. Diabetic nephropathy in 27,805 children, adolescents, and adults with type 1 diabetes: effect of diabetes duration, A1C, hypertension, dyslipidemia, diabetes onset, and sex. Diabetes Care. 2007;30:2523-2528. Available at: http://care.diabetesjournals.org/cgi/content/full/30/10/2523

Page 381: AAP PREP 2009

2009 PREP SA on CD-ROM

page 381

Question: 107

A 12-year-old boy recently took a standardized achievement test at school. His score dropped from 105 on last year’s achievement test to 95 on the most recent test. Last season the boy played hockey and fell down, hitting his head, although he did not lose consciousness. He had no previous head injury. He was evaluated in the emergency department and had normal findings on computed tomography scan. The mother asks whether the boy had suffered brain injury due to his fall that caused him to lose academic skills.

Of the following, the MOST appropriate response is to

A. explain that it is normal to have a small variation between scores

B. recommend neurorehabilitation due to loss of his academic skills

C. restrict the child’s contact sports activity for the next season

D. send the child for comprehensive neuropsychological testing

E. send the child for head magnetic resonance imaging

Page 382: AAP PREP 2009

2009 PREP SA on CD-ROM

page 382

Critique: 107 Preferred Response: A

The small difference in scores on standardized achievement tests described for the boy in the vignette is normal. To be significant, the change would need to be greater than 1 standard deviation (15 points). Because the values reported for the boy are within that expected for test-retest variation, brain magnetic resonance imaging, contact sports restriction, neuropsychological testing, and neurorehabilitation are not needed. Standardized scores measure performance for a group of students, such as all the students in one school, at a certain grade, or in a single school district. Standardized scores employ a procedure in which scores are reported in terms of standard deviation units from the mean. When scores are standardized to the normal bell curve, two thirds of the students in the sample fall between -1 and +1 standard deviations from the mean. Scores also may be expressed as percentiles or, in some cases, grade equivalents. Standardized scores are less useful for evaluating change in an individual student. A single student may be learning more every day and growing in ability or skill, but still be at the same relative rank compared with his or her peers. Achievement tests measure what a child has learned in specific subject areas such as reading or mathematics. Criterion-based testing evaluates how much a student knows or can do using a specific set of standards, not by comparison with other students, which probably makes it more useful for identifying whether a student is improving or meeting objectives. A child who has a significant discrepancy (determined by each state) between an expected achievement score based on an intelligence quotient test and the actual score may have a specific learning disability and be eligible for specialized educational services.

References:

Ewen JB, Shapiro BK. Specific learning disabilities. In: Accardo PJ. Capute & Accardo's Neurodevelopmental Disabilities in Infancy and Childhood. Volume II: The Spectrum of Neurodevelomental Disabilities. 3rd ed. Baltimore, Md: Paul H. Brookes Publishing Co; 2008:553-577

Mahone EM. Psychological assessment. In: Accardo PJ, ed. Capute & Accardo's Neurodevelopmental Disabilities in Infancy and Childhood. Volume II: The Spectrum of Neurodevelopmental Disabilities. 3rd ed. Baltimore, MD: Paul H. Brookes Publishing Co; 2008:261-281

Page 383: AAP PREP 2009

2009 PREP SA on CD-ROM

page 383

Question: 108

The microbiology laboratory called your junior partner today to tell her that the blood culture from a patient she admitted 2 days ago is growing Haemophilus influenzae type b. Because she has never treated an infection caused by this organism, she wants to know what antimicrobial agent would be best to use for her patient.

Of the following, the MOST appropriate antimicrobial agent to treat this infection is

A. ampicillin

B. cefotaxime

C. clindamyin

D. gentamicin

E. vancomycin

Page 384: AAP PREP 2009

2009 PREP SA on CD-ROM

page 384

Critique: 108 Preferred Response: B

Haemophilus influenzae type b (Hib) is a pleomorphic gram-negative coccobacillus. These types of organisms are not susceptible to antimicrobial agents used against gram-positive pathogens, such as clindamycin or vancomycin, or some agents that are effective against other gram-negative organisms, such as the aminoglycosides (eg, gentamicin). Because approximately 30% to 40% of Hib isolates produce beta-lactamase, ampicillin should not be used empirically until susceptibility testing supports its potential efficacy. For patients who have potentially life-threatening illness (eg, bacteremia) from Hib, cefotaxime or ceftriaxone has become the antimicrobial agent of choice, with meropenem or chloramphenicol serving as alternative agents. References:

American Academy of Pediatrics. Haemophilus influenzae infections. In: Pickering LK, Baker CJ, Long SS, McMillan JA, eds. Red Book: 2006 Report of the Committee on Infectious Diseases. 27th ed. Elk Grove Village, Ill: American Academy of Pediatrics; 2006:310-318

Tzanakaki G, Mastrantonio P. Aetiology of bacterial meningitis and resistance to antibiotics of causative pathogens in Europe and in the Mediterranean region. Int J Antimicrob Agents. 2007;29:621-629. Abstract available at: http://www.ncbi.nlm.nih.gov/pubmed/17368858

Page 385: AAP PREP 2009

2009 PREP SA on CD-ROM

page 385

Question: 109

A 9-year-old previously healthy boy presents for evaluation of a progressively worsening cellulitis of his left leg. Two days ago, he sustained an abrasion to his shin after falling off his bicycle onto a gravel road. Over the last 12 hours, he has developed a temperature of 102.0°F (38.9°C), and the wound has become very erythematous, swollen, and tender, with some red streaking. On physical examination, the boy has a temperature of 101.5°F (38.6°C) and a 5x6-cm abrasion of the anterior lateral surface of his left shin that is draining a serosanguineous discharge. The abrasion is surrounded by an 8-cm area of erythema, swelling, and induration, with a red streak extending up toward his knee. The area is tender to palpation, and he limps when walking. There is some shotty left inguinal adenopathy. A complete blood count shows a peripheral white blood cell count of 16.0x103/mcL (16.0x109/L) with a differential count of 65% neutrophils, 5% band forms, 25% lymphocytes, and 5% monocytes.

Of the following, the MOST likely pathogen causing this patient’s condition is

A. Staphylococcus aureus (methicillin-resistant)

B. Staphylococcus aureus (methicillin-sensitive)

C. Staphylococcus epidermidis

D. Streptococcus pyogenes

E. Streptococcus pneumoniae

Page 386: AAP PREP 2009

2009 PREP SA on CD-ROM

page 386

Critique: 109 Preferred Response: D

Streptococcus pyogenes (group A Streptococcus) is a ubiquitous organism that is the most common bacterial cause of pharyngitis and can be the source of a variety of other cutaneous and systemic infections. Streptococcal cellulitis is an acute, spreading inflammation of the skin and subcutaneous tissues. Clinical findings include local pain, tenderness, swelling, and erythema that progresses rapidly and may involve large areas of skin. Systemic manifestations include fever, chills, and malaise with associated lymphangitis (red, tender, linear streaks directed toward enlarged, tender regional lymph nodes) or bacteremia. The patient described in the vignette has the classic clinical picture for a cellulitis caused by group A Streptococcus. Streptococcal pharyngitis primarily occurs among children 5 to 15 years of age, with the peak incidence seen during the first few years of school. However, all age groups are susceptible to infection. Symptoms consist of the abrupt onset of severe sore throat, malaise, fever, headache, nausea, vomiting, and abdominal pain. Coryza, cough, and conjunctivitis are absent. Prominent physical findings include: erythema and edema of the posterior pharynx, with enlarged hyperemic tonsils; patchy discrete tonsillopharyngeal exudates (Item C109A); and tender, enlarged, submandibular lymph nodes. The cutaneous clinical manifestations of a group A streptococcal infection include scarlet fever, impetigo, erysipelas, streptoccocal cellulitis, and necrotizing fasciitis. Scarlet fever results from infection with a streptococcal strain that produces streptococcal pyrogenic exotoxins. This disease usually is associated with pharyngeal infections but also may follow streptococcal infections at other sites. Clinical findings include a diffuse erythematous blush that blanches with pressure and spares the face, palms, and soles. Skin folds in the neck, axillae, groin, elbows, and knees appear as lines of deeper red (Pastia lines) (Item C109B). The skin has a sandpaper texture (Item C109C), and there is usually circumoral pallor. Impetigo (pyoderma) consists of discrete purulent lesions representing primary infection of the skin. The lesion begins as a papule that rapidly evolves into a vesicle surrounded by an area of erythema. The vesicle gives rise to pustules that enlarge and subsequently break down to form thick crusts (Item C109D). The lesions heal slowly and may leave areas of hypopigmentation. Erysipelas is a superficial cutaneous process that has prominent lymphatic involvement. Its distinguishing features are lesions that are raised and have a clear line of demarcation between involved and uninvolved tissue. The cutaneous lesion begins as a localized area of erythema and swelling and subsequently spreads rapidly, with advancing red margins that are raised and well-demarcated from adjacent normal tissue. There is marked edema, often with bleb formation (Item C109E). Necrotizing fasciitis is an infection of the deeper subcutaneous tissues and fascia characterized by extensive and rapidly spreading necrosis and gangrene of the skin and underlying tissue. Typically, necrotizing fasciitis begins at a site of minor or inapparent trauma. The initial lesion is an area of mild erythema that rapidly evolves to severe extensive inflammation; the skin becomes dusky and purplish, and bullae containing yellow or hemorrhagic fluid appear (Item C109F). Systemically, patients are ill and have a high temperature and toxic appearance. The systemic clinical manifestations of group A streptococcal infection include toxic shock syndrome (TSS), pneumonia, septic arthritis and osteomyelitis, and meningitis. TSS is defined as any streptococcal infection associated with the sudden onset of shock and organ failure. The first phase of TSS begins with an influenza-like prodrome characterized by fever, chills, myalgias, nausea, vomiting, and diarrhea that precedes the development of hypotension by 24 to 48 hours. A defined or superficial portal of entry or evidence of infection also may be present during this stage. Phase 2 of TSS is characterized by tachycardia, tachypnea, persistent fever, and if present, increasingly severe pain at a site of infection. Phase 3 of TSS is characterized by the previously noted symptoms accompanied by the sudden onset of shock and organ failure. Pneumonia due to group A Streptococcus frequently is

Page 387: AAP PREP 2009

2009 PREP SA on CD-ROM

page 387

associated with a preceding viral infection. The onset of disease usually is abrupt and is characterized by fever, dyspnea, cough, and pleuritic chest pain. Chest radiography shows a consolidative lobar pneumonia that has moderate-to-large pleural effusion or empyema. Meningitis due to group A Streptococcus is relatively rare and typically follows an upper respiratory tract infection (otitis and sinusitis) or neurosurgical conditions. Staphylococcus epidermidis does not cause cellulitis. S pneumoniae is a cause of preseptal cellulitis but is not associated with other forms of cellulitis. The cellulitis caused by S aureus (methicillin-sensitive or -resistant) is not associated with lymphangitis, and progression is not as rapid as with group A streptococcal cellulitis.

References:

Bisno AL, Stevens DL. Streptococcal infections of skin and soft tissues. N Engl J Med. 1996;334:240-246. Extract available at: http://content.nejm.org/cgi/content/extract/334/4/240

Bisno AL, Stevens DL. Streptococcus pyogenes. In: Mandell GL, Bennett JE, Dolin R, eds. Mandell, Douglas, and Bennett's Principles and Practice of Infectious Diseases. 6th ed. Philadelphia, Pa: Elsevier Churchill Livingstone; 2005:2362-2379

Sellers BJ, Woods ML, Morris SE, Saffle JR. Necrotizing group A streptococcal infections associated with streptococcal toxic shock syndrome. Am J Surg. 1996;172:523-528. Abstract available at: http://www.ncbi.nlm.nih.gov/pubmed/8942557

Stevens DL. Dilemmas in the treatment of invasive Streptococcus pyogenes infections. Clin Infect Dis. 2003;37:341-343. Available at: http://www.journals.uchicago.edu/doi/full/10.1086/376652

Stevens DL. Streptococcal toxic-shock syndrome: spectrum of disease, pathogenesis, and new concepts in treatment. Emerg Infect Dis. 1995;1:69-78. Abstract available at: http://www.ncbi.nlm.nih.gov/pubmed/8903167

Page 388: AAP PREP 2009

2009 PREP SA on CD-ROM

page 388

Critique: 109

Pharyngitis caused by Streptococcus pyogenes is characterized by enlarged tonsils and exudates. (Courtesy of M. Rimsza)

Page 389: AAP PREP 2009

2009 PREP SA on CD-ROM

page 389

Critique: 109

In patients who have scarlet fever, erythema or petechiae may be concentrated in skin folds in the antecubital fossae (ie, Pastia lines). (Courtesy of D. Krowchuk)

Page 390: AAP PREP 2009

2009 PREP SA on CD-ROM

page 390

Critique: 109

The rash of scarlet fever is comprised of fine erythematous papules that have a rough or "sandpaper" feel. (Courtesy of D. Krowchuk)

Page 391: AAP PREP 2009

2009 PREP SA on CD-ROM

page 391

Critique: 109

In nonbullous impetigo, vesicles rupture and serous fluid dries, leaving a yellow or "honey-colored" crust. (Courtesy of D. Krowchuk)

Page 392: AAP PREP 2009

2009 PREP SA on CD-ROM

page 392

Critique: 109

Erysipelas represents a form of cellulititis characterized by a well-defined and often elevated border and vesicle formation. (Courtesy of Red Book® Online)

Page 393: AAP PREP 2009

2009 PREP SA on CD-ROM

page 393

Critique: 109

In necrotizing fasciitis, affected areas of the skin become dusky or violaceous and bullae appear. (Courtesy of Bernard Cohen, MD, DermAtlas; www.dermatlas.org)

Page 394: AAP PREP 2009

2009 PREP SA on CD-ROM

page 394

Question: 110

A 6-year-old boy presents to an urgent care center with the complaint of bright red blood and clots in the urine. There is no history of trauma, and the boy has no dysuria, frequency, urgency, abdominal pain, or back pain. On physical examination, his temperature is 98.6°F (37°C), heart rate is 76 beats/min, respiratory rate is 14 breaths/min, and blood pressure is 110/68 mm Hg. He has no abdominal tenderness, flank tenderness, or edema. Urinalysis reveals a specific gravity of 1.025, pH of 6.5, 3+ blood, trace protein, and negative leukocyte esterase and nitrite. Microscopy shows more than 100 red blood cells/high-power field (HPF), less than 5 white blood cells/HPF, and no casts. Electrolyte values are normal, blood urea nitrogen is 14.0 mg/dL (5.0 mmol/L), and creatinine is 0.5 mg/dL (44.2 mcmol/L).

Of the following, the MOST appropriate next test is

A. antinuclear antibody titer

B. complement component 3 (C3) measurement

C. computed tomography scan of the abdomen

D. renal biopsy

E. renal/bladder ultrasonography

Page 395: AAP PREP 2009

2009 PREP SA on CD-ROM

page 395

Critique: 110 Preferred Response: E

A good first step in the evaluation of gross hematuria in a child who is otherwise well is to characterize the urine color to aid in localizing the lesion to a particular anatomic region. Patients who have cola- or tea-colored urine often have painless hematuria without clots, as is seen classically in glomerulonephritis. Bright red urine (often with clots), as described for the boy in the vignette, may have an underlying nonglomerular renal cause, a lower urinary tract cause (ureter or bladder), or rarely be due to a hematologic disorder. Patients who have bright red urine with clots need to be evaluated for possible structural causes for hematuria. Such nonglomerular renal causes include a ruptured cyst in cases of cystic kidney disease (as in autosomal dominant and recessive forms of polycystic kidney disease), a renal mass (Wilms tumor) (Item C110), or a renal stone (nephrolithiasis or urolithiasis). Additional causes include renal vein thrombosis, papillary necrosis, or hypercalciuria. Hematologic disorders such as sickle trait/disease or a bleeding disorder (von Willebrand disease) can result in a similar clinical picture and should be considered in the investigation. Lower urinary tract causes include hemorrhagic cystitis, bladder calculi, or rarely, a tumor of the lower urinary tract (hemangioma of the bladder or rhabdomyosarcoma of the bladder). The evaluation for possible cysts, stones, or tumors in all children who have gross hematuria involves renal/bladder ultrasonography to look for a structural abnormality. In rare cases, additional testing is required, such as an abdominal computed tomography (CT) scan to look for renal calculi if the clinical suspicion is high and the ultrasonography result is negative. However, routine use of the CT scan is strongly discouraged due to long-term concerns about the ionizing radiation exposure (estimated to be equivalent to the radiation dose of 250 two-view chest radiographs). Patients who have cola-colored urine require monitoring of blood pressure and renal function while looking for an underlying cause for the suspected glomerulonephritis. The best single diagnostic test in the evaluation of acute glomerulonephritis is measurement of complement component 3 (C3). Additional testing usually performed at the outset includes measurement of C4, antinuclear antibody, antidouble-stranded DNA antibody, and serum albumin. These tests help the clinician to categorize the glomerulonephritis into hypocomplementemic or normocomplementemic types and narrow the differential diagnosis accordingly. A renal biopsy usually is considered in patients who have acute glomerulonephritis accompanied by a rising serum creatinine value or in cases of normocomplementemic glomerulonephritis or suspected lupus nephritis. Because the patient in the vignette is unlikely to have glomerulonephritis, he does not require testing for C3 or ANA or a renal biopsy.

References:

Brody AS, Frush DP, Huda W, Brent RL, and the Section on Radiology. Radiation risk to children from computed tomography. Pediatrics. 2007;120:677-682. Available at: http://pediatrics.aappublications.org/cgi/content/full/120/3/677

Moxey-Mims M. Hematuria and proteinuria. In: Kher KK, Schnaper HW, Makker SP, eds. Clinical Pediatric Nephrology. 2nd ed. London, England: Informa Healthcare; 2007:129-141

Pan CG. Evaluation of gross hematuria. Pediatr Clin North Am. 2006;53:401-412. Abstract available at: http://www.ncbi.nlm.nih.gov/pubmed/16716787

Page 396: AAP PREP 2009

2009 PREP SA on CD-ROM

page 396

Critique: 110

Wilms tumor: A transverse image from a computed tomography scan demonstrates a normal right kidney and a very large tumor originating from the displaced left kidney. (Courtesy of D. Mulvihill)

Page 397: AAP PREP 2009

2009 PREP SA on CD-ROM

page 397

Question: 111

A 10-year-old boy presents with a 2-month history of chronic cough. His parents are unsure of a specific preceding trigger. They are concerned because the school nurse has called on multiple occasions requesting that the boy be taken home due to his persistent cough. The boy denies any chest pain, dyspnea, or syncope. Use of a sedating antihistamine and over-the-counter cold and cough liquid has not alleviated his symptoms. On physical examination, the boy has vital signs within the normal range and appears healthy. A thorough examination reveals no abnormalities. During the encounter, the boy repeatedly exhibits a harsh, "barky" cough that resolves when you leave the examination room, only to recur when you return. You suspect he has a psychogenic cough.

Of the following, the MOST accurate statement regarding psychogenic cough is that

A. most cases are associated with underlying psychological illness

B. most cases are preceded by an upper respiratory tract infection

C. resolution of the cough often is followed by recurrent wheezing

D. symptoms persist during the day and while the child is asleep

E. the cough noise often is dramatically different from the postnasal drip syndrome cough

Page 398: AAP PREP 2009

2009 PREP SA on CD-ROM

page 398

Critique: 111 Preferred Response: B

Psychogenic cough, sometimes called habit-cough syndrome, is an uncommon cause of chronic cough in children and adolescents. Many experts discourage using the label "psychogenic" because underlying psychological or psychosomatic concerns are uncommon in affected children. Most cases of psychogenic cough begin with an uncomplicated upper respiratory tract viral illness, but the cough lingers for months to years. The classic triad of symptoms includes a repetitive cough, cessation during sleep, and the characteristic hacking, barky cough, as described for the boy in the vignette. Although most affected children undergo multiple therapeutic interventions and diagnostic tests, the characteristic presentation may allow clinicians to avoid expensive or invasive tests to rule out other causes. Successful therapies for habit-cough include behavioral therapy, suggestion therapy, and self-hypnosis. Once resolved, most cases of habit-cough do not recur. Unlike some cases of viral upper respiratory tract infections, habit-cough resolution is not followed by recurrent wheezing.

References:

Linz AJ. The relationship between psychogenic cough and the diagnosis and misdiagnosis of asthma: a review. J Asthma. 2007;44:347-355. Abstract available at: http://www.ncbi.nlm.nih.gov/pubmed/17613629

Weinberger M, Abu-Hasan M. Pseudo-asthma: when cough, wheezing, and dyspnea are not asthma. Pediatrics. 2007;120:855-864. Available at: http://pediatrics.aappublications.org/cgi/content/full/120/4/855

Page 399: AAP PREP 2009

2009 PREP SA on CD-ROM

page 399

Question: 112

A 6-year-old girl fell onto her outstretched right arm while roller skating yesterday. She continued to skate, but on returning home, she noticed that her right forearm was swollen and painful. Her mother applied ice and gave her ibuprofen, but the swelling is worse today. On physical examination, the girl has moderate swelling over the right distal radius with minimal pain on palpation. She has full range of motion of her wrist and hand. Radiographs are obtained (Item Q112).

Of the following, a TRUE statement regarding this patient’s fracture is that

A. median nerve injury often is seen in association with this fracture

B. reduction of the fracture is not necessary if there is less than 15 degrees of angulation

C. the fracture frequently is associated with growth arrest

D. the fracture is likely to produce permanent deformity

E. the patient should be immobilized in a cast for 8 to 10 weeks

Page 400: AAP PREP 2009

2009 PREP SA on CD-ROM

page 400

Question: 112

Radiograph of the forearm, as described for the child in the vignette. (Courtesy of E. Anthony)

Page 401: AAP PREP 2009

2009 PREP SA on CD-ROM

page 401

Critique: 112 Preferred Response: B

Pediatric fracture patterns differ from those in adults for a variety of biomechanical, physiologic, and anatomic reasons. Compared to adult bone, pediatric bone is highly porous; contains lower amounts of mineral and higher amounts of cartilage; has thicker, stronger, and more metabolically active periosteum; and has relatively stronger ligaments and open growth plates. These differences lead to fracture types such as greenstick fractures, plastic deformation, torus fractures, and physeal or growth plate fractures that are unique to children. Greenstick fractures are incomplete fractures characterized by a cortical break on one side of the bone with intact periosteum on the opposing side (Item C112A), as seen in the radiograph obtained for the girl in the vignette. In some cases, the bone may bend without breaking (Item C112B). The strong periosteum and decreased resistance to bending forces of pediatric bones are responsible for this phenomenon. Torus fractures result from compressive forces on the bone that lead to a periosteal "buckle" (Item C112C). Children who have open growth plates are at risk for physeal fractures that involve the relatively weak, cartilage-rich growth plates. Treatment for the greenstick fracture with less than 15 degrees of angulation, as described for the child in the vignette, involves cast immobilization and close orthopedic follow-up, but no reduction of the fracture is necessary. In older children who have fractures with greater than 15 degrees of angulation, reduction often is necessary to decrease the risk of permanent deformity. Greenstick fractures typically are not associated with nerve entrapment or growth arrest (if the physis is not involved) and heal without residual sequelae following cast immobilization for 4 to 6 weeks. Patients should be cautioned to avoid reinjury following cast removal. Refracture is seen in 7% to 17% of forearm fractures and occurs most commonly following greenstick fractures.

References:

Carson S, Woolridge DP, Colletti J, Kilgore K. Pediatric upper extremity injuries. Pediatr Clin North Am. 2006;53:41-67. Abstract available at: http://www.ncbi.nlm.nih.gov/pubmed/16487784

Gholve PA, Hosalkar HS, Wells L. Common fractures. In: Kleigman RM, Behrman RE, Jenson HB, Stanton BF, eds. Nelson Textbook of Pediatrics. 18th ed. Philadelphia, Pa: Saunders Elsevier; 2007:2834-2841

Greenstick fracture of forearm. In: Wheeless' Textbook of Orthopaedics. Available at: http://wheelessonline.com/ortho/green_stick_frx_of_forearm

Page 402: AAP PREP 2009

2009 PREP SA on CD-ROM

page 402

Critique: 112

Greenstick fractures are incomplete fractures characterized by a cortical break on one side of the bone (area of lucency, yellow arrow) and intact periosteum on the opposite side. This patient also has a greenstick fracture of the ulna (red arrow). (Courtesy of E. Anthony)

Page 403: AAP PREP 2009

2009 PREP SA on CD-ROM

page 403

Critique: 112

In some greenstick fractures, the bone may bend but not break (arrow). (Courtesy of D. Krowchuk)

Page 404: AAP PREP 2009

2009 PREP SA on CD-ROM

page 404

Critique: 112

Torus (buckle) fracture of the left distal radius (yellow arrows) and the radial side of the ulna (red arrow). The term "torus" is used because of the similarity of appearance of the fracture with the convex molding often used at the base of columns. (Courtesy of E. Anthony)

Page 405: AAP PREP 2009

2009 PREP SA on CD-ROM

page 405

Question: 113

A 5-year-old child is admitted to the hospital with epigastric pain and vomiting. On physical examination, she has a tender epigastrium, but no peritoneal signs. Her amylase is 400 U/L and lipase is 670 U/L. Abdominal ultrasonography demonstrates a prominent pancreatic head, but no gallstones or biliary tract dilation. Review of her chart demonstrates two prior hospitalizations over the past 3 years due to pancreatitis. She has no other significant findings in her medical history and no history of trauma preceding any of these episodes.

Of the following, the condition that BEST explains the patient’s history is

A. alpha-1-antitrypsin deficiency

B. colipase deficiency

C. hereditary pancreatitis

D. primary sclerosing cholangitis

E. systemic lupus erythematosus

Page 406: AAP PREP 2009

2009 PREP SA on CD-ROM

page 406

Critique: 113 Preferred Response: C

The child described in the vignette has a history of at least three episodes of pancreatitis before age 5 years, and gallstones have been excluded by ultrasonography. Children who have hereditary pancreatitis (HP) can present with recurrent episodes of pancreatitis and often are well between episodes. In adulthood, affected individuals are at risk for developing pancreatic insufficiency and cancer. Systemic lupus erythematosus (SLE) may be associated with pancreatitis in the context of severe vasculitis, but SLE is rare before 5 years of age, and the child has no other signs of the condition. Alpha-1-antitrypsin deficiency and primary sclerosing cholangitis may cause chronic liver disease, but they rarely affect the pancreas. Colipase deficiency may result in pancreatic insufficiency and fat malabsorption, but it usually does not cause pancreatic inflammation. Pancreatitis generally is less severe in children than in adults. A child who has acute pancreatitis typically presents with epigastric pain and vomiting, and epigastric tenderness can be appreciated on palpation. More severe pancreatitis can cause intravascular volume depletion, third spacing, oliguria, pulmonary edema, and pleural effusions. The most common laboratory abnormalities in mild pancreatitis are elevated amylase and lipase values. Lipase is a more specific enzyme for pancreatitis; hyperamylasemia also can be seen with salivary gland inflammation, ovarian tumors, renal insufficiency, diabetic ketoacidosis, and ruptured ectopic pregnancy. Laboratory abnormalities associated with more severe pancreatitis include hyperglycemia, hypocalcemia, high blood urea nitrogen and creatinine, low hematocrit, and coagulopathy. In severe cases of pancreatitis, abdominal computed tomography scan can identify the extent and severity of pancreatic injury. Some children experience recurrent episodes of pancreatitis without an obvious cause (ie, no history of medications that can cause pancreatitis, gallstones, trauma, or systemic vasculitis). Anatomic and genetic causes should be considered in such children. Anatomic causes such as choledochocele, pancreas divisum, or a long common channel connecting the pancreas and bile ducts can be identified by either magnetic resonance imaging or endoscopic retrograde cholangiopancreatography. The most common genetic causes of recurrent pancreatitis are cystic fibrosis (CF) and HP. Patients who have CF and present with pancreatitis often have minimal pulmonary disease and no clinical signs of pancreatic insufficiency or malabsorption. In some cases, a single mutation in the CFTR gene may cause a heterozygous carrier for CF to present with recurrent pancreatitis. HP is caused most commonly by a mutation in the pancreatic cationic trypsinogen gene and is inherited in an autosomal dominant pattern. Although there often is a family history, some patients have no family history because a de novo gene mutation has developed.

References:

Pietzak MM, Thomas DW. Pancreatitis in childhood. Pediatr Rev. 2000;21: 406-412. Available at: http://pedsinreview.aappublications.org/cgi/content/full/21/12/406

The National Pancreas Foundation web site. Available at: www.pancreasfoundation.org

Page 407: AAP PREP 2009

2009 PREP SA on CD-ROM

page 407

Question: 114

You admit a term newborn to the neonatal intensive care unit because of noisy breathing. Findings on physical examination include mild micrognathia, an intact palate, and inspiratory stridor with suprasternal retractions when the infant is in the supine position that diminish but do not disappear when the infant is prone. Stridor becomes more audible when the infant cries. When the infant is asleep and prone, the breath sounds are clear and equal bilaterally, with no stridor or wheezing. There is no heart murmur. Pulse oximetry is 94% on room air.

Of the following, the MOST likely cause of this infant’s stridor is

A. cleft lip

B. laryngomalacia

C. tracheal hemangioma

D. tracheomalacia

E. vocal cord polyp

Page 408: AAP PREP 2009

2009 PREP SA on CD-ROM

page 408

Critique: 114 Preferred Response: B

The infant described in the vignette has congenital stridor (noisy inspiratory breathing) and exhibits only mild increased work of breathing when supine and when distressed. Although no immediate intervention is required to maintain airway patency, the clinician examining the patient needs to determine the cause of the stridor. Some have reported a characteristic coarseness of the stridor when due to a supraglottic (laryngeal) disorder, a musical quality associated with glottic (vocal cord) disorders, and more of an expiratory noise or prolonged expiratory phase with a wheeze in subglottic disorders of airway obstruction (Item C114A). The most common cause of congenital stridor is laryngomalacia, a congenital weakness, or floppiness, in airway laryngeal cartilages (epiglottis, arytenoid cartilages, larynx) (Item C114B). A cleft lip does not cause stridor. A tracheal hemangioma (Item C114C) may cause airway obstruction, but usually this is associated with expiratory wheezing. Tracheomalacia may complicate prolonged intubation and positive pressure ventilation in preterm infants and typically is associated with expiratory wheezing or inspiratory cessation of airflow with airway collapse unless positive distending pressure is applied (Item C114D). A vocal cord polyp is a rare congenital condition that may be associated with human papillomavirus infection that can cause stridor, which would not be expected to change with position.

References:

Aly H. Respiratory disorders in the newborn: identification and diagnosis. Pediatr Rev. 2004;25:201-208. Available at: http://pedsinreview.aappublications.org/cgi/content/full/25/6/201

Brodsky L. Consultation with the specialist: congenital stridor. Pediatr Rev. 1996;17:408-411. Available at: http://pedsinreview.aappublications.org/cgi/reprint/17/11/408

Sprecher RC, Arnold JE. Upper airway lesions. In: Martin RJ, Fanaroff AA, Walsh MC, eds. Fanaroff and Martin's Neonatal-Perinatal Medicine. 8th ed. Philadelphia, Pa: Mosby Elsevier; 2006:1146-1154

Vicencio AG, Parikh S, Adam HM. In brief: laryngomalacia and tracheomalacia: common dynamic airway lesions. Pediatr Rev. 2006;27:e33-e35. Available at: http://pedsinreview.aappublications.org/cgi/content/full/27/4/e33

Page 409: AAP PREP 2009

2009 PREP SA on CD-ROM

page 409

Critique: 114

Page 410: AAP PREP 2009

2009 PREP SA on CD-ROM

page 410

Question: 115

A 13-year-old boy who plays baseball comes to your office for a preparticipation sports physical examination. He always has been an average player and is interested in a preseason conditioning program to improve his strength and agility because he wants to play on his school team. His mother is concerned about the program because it involves weight training, and she asks for your advice.

Of the following, a TRUE statement about conditioning programs for young athletes is that these programs

A. have been shown to decrease ultimate linear growth if begun before puberty

B. prevent future catastrophic sports-related injuries

C. should begin with low-resistance exercise, with weight added in small increments as tolerated

D. should involve high-weight, low-repetition activities

E. should involve weight training only without aerobic activities

Page 411: AAP PREP 2009

2009 PREP SA on CD-ROM

page 411

Critique: 115 Preferred Response: C

Preventive conditioning is an important part of sports programs for preadolescents and adolescents because participation can help to improve strength and overall performance. Programs should include both aerobic conditioning and strength training. The most significant risks to the athlete involve strength, or resistance, training, but such risks can be minimized by adhering to a few guidelines. Weight training programs that consist of high-repetition activities with low resistance are safe for both preadolescents and adolescents. Training should begin by teaching proper technique with no weight, adding small increments of weight as strength increases. Sessions lasting 20 to 30 minutes undertaken 2 to 3 days a week are sufficient to improve strength. Power lifting programs, which involve low-repetition, high-resistance exercises, should not be undertaken by preadolescents because of the risk of injury. In general, such programs should be avoided until skeletal maturity is reached. Injuries reported in studies evaluating strength training programs include muscle strain, epiphyseal injuries in the wrist, and apophyseal injuries, but most of these can be avoided by using proper technique. Participation in these programs does not decrease linear growth. Although preventive conditioning has been shown to improve strength and performance, there is no evidence that participation in such programs can prevent catastrophic injury. However, athletes who have particular recurrent injuries, such as to the shoulder or ankle, can reduce their risk of further injury by participating in specific strengthening exercises focusing on those areas.

References:

American Academy of Pediatrics Committee on Sports Medicine and Fitness. Strength training by children and adolescents. Pediatrics. 2001;107:1470-1472. Available at: http://pediatrics.aappublications.org/cgi/content/full/107/6/1470

Metzl JD. Preparticipation examination of the adolescent athlete: part 2. Pediatr Rev. 2001;22:227-239. Available at: http://pedsinreview.aappublications.org/cgi/content/full/22/7/227

Page 412: AAP PREP 2009

2009 PREP SA on CD-ROM

page 412

Question: 116

A 10-year-old boy who recently emigrated from Central America is referred by the school nurse for evaluation of obesity.Physical examination reveals an obese but generally healthy boy who has acanthosis nigricans (Item Q116).He has had limited access to medical care in the past.

Of the following, the physical finding MOST likely to suggest an underlying cause for the child's obesity is

A. a normal blood pressure

B. abdominal striae

C. penile length at 1 standard deviation below the mean

D. small hands and feet

E. stature greater than the 95th percentile

Page 413: AAP PREP 2009

2009 PREP SA on CD-ROM

page 413

Question: 116

(Courtesy of M. Rimsza)

Page 414: AAP PREP 2009

2009 PREP SA on CD-ROM

page 414

Critique: 116 Preferred Response: D

The incidence of obesity continues to increase among children in industrialized nations. In general, the causes in most children relate to increased caloric intake and decreased physical activity. However, the pediatrician must exclude genetic, metabolic, or other underlying causes of obesity in children for two reasons: 1) some underlying causes may require treatment and 2) the focus on changing the child's lifestyle must involve parents who are reassured that hormonal or other abnormalities are not the cause. A child who exhibits normal linear growth, has normal developmental milestones, and has normal findings on physical examination is unlikely to have an underlying cause for his or her obesity. The most common metabolic cause of obesity is hypothyroidism, but routine testing for thyroid function in the overweight child who has normal linear growth velocity for age and no clinical signs of hypothyroidism is unwarranted. A stature greater than the 95th percentile should be interpreted with caution because one measurement is not sufficient to determine growth velocity. However, this finding is reassuring because normal to above-average height is unlikely in a child who has hypothyroidism and common in children who have exogenous obesity. Children who have small hands and feet, hypogonadism, learning disabilities, or mental retardation should be evaluated for Prader- Willi and Bardet-Biedl syndromes. These syndromes often present in infancy or early childhood with hypotonia and developmental delay. Small hands and feet are common and strongly suggest an underlying anomaly for the child described in the vignette (Item C116A). Genetic testing for these conditions is available. The normal blood pressure reported for the boy in the vignette is reassuring because hypertension often complicates obesity. Penile length is normal at 1 standard deviation below the mean, but it is important to remember that measurement of penile length can be a challenge in obese children due to interference from the pubic fat pad. Striae may be seen in many overweight children simply due to rapid weight gain. Although this finding also occurs with cortisol excess (Item C116B), conditions associated with hypercortisolism usually are associated with other signs and symptoms. Thus, the finding of striae in an obese child is not an indication that additional testing is necessary. Treatment of childhood obesity is difficult, although there is some evidence for a variety of interventions that may prove successful in some populations. Numerous diets, including low-carbohydrate diets, appear to have some impact on obesity in children. For morbidly obese adolescents who have comorbidities unresponsive to diet and exercise, some centers are moving toward bariatric surgery. Barriers to healthy lifestyles (lack of availability in some communities of safe outdoor play areas, specific exercise programs geared to very obese children, and school lunch programs) remain topics of scientific and political investigation.

References:

Arterburn DE. Obesity in children. BMJ Clinical Evidence. 2007. Available for subscription at:http://clinicalevidence.bmj.com/ceweb/conditions/chd/0325/0325.jsp

Rodearmel SJ Wyatt HR, Stroebele N, Smith SM, Ogden LG, Hill JO. Small changes in dietary sugar and physical activity as an approach to preventing excessive weight gain: the America on the Move Family Study. Pediatrics. 2007;120:e869-e879 Available at: http://pediatrics.aappublications.org/cgi/content/full/120/4/e869

Schneider MB, Brill SR. Obesity in children and adolescents. Pediatr Rev. 2005;26:155-162. Available at: http://pedsinreview.aappublications.org/cgi/content/full/26/5/155

Page 415: AAP PREP 2009

2009 PREP SA on CD-ROM

page 415

Shaw K, Gennat H, O'Rourke P, Del Mar C. Exercise for overweight or obesity. Cochrane Database Syst Rev. 2006;4:CD003817. Available at: http://www.mrw.interscience.wiley.com/cochrane/clsysrev/articles/CD003817/frame.html

Summerbell CD, Waters E, Edmunds LD, Kelly S, Brown T, Campbell KJ. Interventions for preventing obesity in children. Cochrane Database Syst Rev. 2005;3:CD001871. Available at: http://www.mrw.interscience.wiley.com/cochrane/clsysrev/articles/CD001871/frame.html

Thomas DE, Elliott EJ, Baur L. Low glycaemic index or low glycaemic load diets for overweight and obesity. Cochrane Database Syst Rev. 2007;3:CD005105. Available at: http://www.mrw.interscience.wiley.com/cochrane/clsysrev/articles/CD005105/frame.html

Page 416: AAP PREP 2009

2009 PREP SA on CD-ROM

page 416

Critique: 116

Prader-Willi syndrome is characterized by short stature, obesity, hypogonadism, and small hands and feet. (Courtesy of Y. Lacassie)

Page 417: AAP PREP 2009

2009 PREP SA on CD-ROM

page 417

Critique: 116

The striae observed in patients who have Cushing syndrome often have a violaceous color. (Courtesy of M. Rimsza)

Page 418: AAP PREP 2009

2009 PREP SA on CD-ROM

page 418

Question: 117

You are evaluating a 14-year-old girl in the clinic. She has had a fever for nearly 2 weeks, which she has attributed to a "cold," although she has not had cough or upper respiratory tract symptoms. She is concerned about some "spots" that she has noticed on her palms and soles. On physical examination, you note splenomegaly and erythematous, nontender macules on her fingers, palms (Item Q117), and soles of her feet. Additionally, she has lost 8 lb since her visit 6 months ago.

Of the following, the MOST appropriate next study for evaluation of this patient is

A. antinuclear antibody

B. echocardiography

C. Lyme titers

D. ophthalmologic examination

E. tuberculin skin test

Page 419: AAP PREP 2009

2009 PREP SA on CD-ROM

page 419

Question: 117

(Courtesy of M. Rimsza)

Page 420: AAP PREP 2009

2009 PREP SA on CD-ROM

page 420

Critique: 117 Preferred Response: B

The fever, Janeway lesions (erythematous, nontender macular lesions on the fingers and soles), splenomegaly, and weight loss reported for the girl in the vignette strongly suggest infective endocarditis. Infective endocarditis describes an infection involving the endocardial surface of the heart that can occur in individuals who have structurally normal hearts, although it is more common in children who have congenital heart disease. The most common sites are the cardiac valves, but infection also may occur on the margins of a ventricular septal defect, along the chordae supporting the atrioventricular valves, or along vascular grafts or foreign material such as a prosthetic valve. Although a relatively rare occurrence in the pediatric population, infective endocarditis continues to be a major source of morbidity and even mortality. Viridans streptococci (eg, S mitis, S bovis) as well as Staphylococcus aureus are the most common bacterial pathogens causing endocarditis in children. The clinical manifestations of infective endocarditis are many and can present variably in affected individuals. The most common is fever, which may be associated with shaking chills. Constitutional nonspecific manifestations include anorexia, weight loss, malaise, night sweats, arthralgias, myalgias, and splenomegaly. In addition, a number of extracardiac manifestations are associated with infective endocarditis, including petechiae and splinter hemorrhages seen under the nails, hematuria, and glomerulonephritis. Roth spots are retinal hemorrhages that have a clear center and can be seen on ophthalmologic examination (Item C117A). Janeway lesions may be evident on the fingers, palms, or soles (Item C117B). Osler nodes are small, raised erythematous or purple nodules on the pads of the digits that typically are very painful. Among the cardiac manifestations of infective endocarditis is a murmur, which is heard in nearly 50% of affected patients. Such murmurs typically result from valvular insufficiency, and the left heart valves are affected far more commonly than the right heart valves. Regurgitation of the mitral valve produces a holosystolic murmur typically heard best at the cardiac apex, with radiation to the left axilla (Item C117C). Regurgitation of the aortic valve produces a diastolic murmur that generally is heard best at the mid-left or right sternal border with the patient in the sitting position, leaning forward (Item C117D). If the regurgitation is severe, congestive heart failure also can be present. The causative pathogen of infective endocarditis can be identified with blood cultures. Imaging of cardiac valves and evaluation of myocardial function by echocardiography are important parts of an assessment that can help guide management of valvular regurgitation and congestive heart failure, if present. An ophthalmologic examination is an important component of the evaluation, but it is not diagnostic and will not affect treatment. With a presentation such as that of the girl described in the vignette, it is important to rule out other possible causes of systemic disease such as lupus erythematosus, Lyme disease, and tuberculosis, but such diagnostic tests do not have the same urgency or impact on management as echocardiography in a patient who clearly has infective endocarditis.

References:

Baltimore RS. Infective endocarditis in children. Pediatr Infect Dis J. 1992;11:907-912. Abstract available at: http://www.ncbi.nlm.nih.gov/pubmed/1454430

Hoyer A, Silberbach M. Infective endocarditis. Pediatr Rev. 2005;26:394-400. Available at: http://pedsinreview.aappublications.org/cgi/content/full/26/11/394

Taubert KA, Dajani AS. Infective endocarditis. In: Moller JH, Hoffman JIE, eds. Pediatric Cardiovascular Medicine. Philadelphia, Pa: Churchill Livingstone; 2000:768-779

Page 421: AAP PREP 2009

2009 PREP SA on CD-ROM

page 421

Critique: 117

Roth spots are retinal hemorrhages that have a pale center (arrow). (Courtesy of SY Lesnik Oberstein and eyetext.net)

Page 422: AAP PREP 2009

2009 PREP SA on CD-ROM

page 422

Critique: 117

Janeway lesions are painless erythematous or hemorrhagic macules on the fingers, palms, or soles. (Courtesy of M. Rimsza)

Page 423: AAP PREP 2009

2009 PREP SA on CD-ROM

page 423

Question: 118

A mother brings her 8-year-old daughter to your office after the girl experiences a first unprovoked generalized tonic-clonic seizure at school. The child had been seen in an emergency department, and results of a head computed tomography scan performed there were normal. Her development, school performance, and results of physical examination are normal. You review safety concerns (no unsupervised time in bathtub or pools, wearing a bicycle helmet) and seizure first aid with the mother. Following published guidelines, you obtain routine electroencephalography (EEG), which a neurologist interprets as normal. The mother asks you about anticonvulsant therapy to prevent further seizures.

Of the following, you are MOST likely to advise her that

A. repeat sleep-deprived EEG is needed to select medication

B. treatment with carbamazepine should be initiated

C. treatment with phenobarbital should be initiated

D. treatment with phenytoin should be initiated

E. treatment with seizure medication should be deferred

Page 424: AAP PREP 2009

2009 PREP SA on CD-ROM

page 424

Critique: 118 Preferred Response: E

For a child who has a first unprovoked seizure, as described in the vignette, safety concerns (no unsupervised time in bathtub or pools, wearing a helmet while on a bike or scooter) and seizure first aid should be reviewed with the parents and documented in the chart. Treatment after a single seizure in childhood is not recommended, no matter what EEG shows. In some cases, retrospective evaluation of children suggests that they have had subtle complex partial seizures or previously have been found after an unwitnessed seizure. Therefore, it is critical for the physician seeing the child after an apparent first seizure to obtain a very careful history to be certain that the seizure is, indeed, the first. The most important diagnostic decision after a first unprovoked seizure is whether to obtain neuroimaging to diagnose a focal, treatable cause for seizures. Neuroimaging should be obtained after a first unprovoked seizure when the seizure occurs in the first year after birth, is focal/asymmetric, or begins with a stare and subsequently generalizes. Neuroimaging also should be obtained after a first unprovoked seizure when the history or physical examination results indicate probable focal central nervous system pathology, such as if the child has unilateral weakness or when postictal confusion is prolonged, particularly if the child is febrile (to rule out herpes encephalitis). Obtaining an EEG is recommended by published practice guidelines. There are two primary problems to keep in mind when deciding whether to order an EEG: 1) Inter-reader agreement in the interpretation of EEGs is low. The level of training of neurologists reading pediatric EEGs may be low in some communities, which is a particular problem for neonatal EEGs, and even experienced EEG readers frequently differ in their visual interpretations. 2) Positive and negative predictive values are not very informative. The sensitivity of EEG varies widely, depending on who reads it, but at best is approximately 50%; that is, about 50% of children who have epilepsy and 50% of children who have a first unprovoked seizure that eventually will recur have epileptiform EEGs, but the rest have normal EEGs. Accordingly, a normal EEG reading does not rule out recurrence or epilepsy and only means that a recurrent seizure is somewhat less likely. The specificity after first seizure is, at best, about 70%; that is, about 30% of children who do not have a seizure recurrence still have epileptiform discharges on the EEG. Thus, a positive EEG also does not predict epilepsy accurately. Repeat EEGs with sleep deprivation have been advocated by neurology practice parameters for many years in cases where the first EEG is read as normal and sleep was not present. The best available evidence suggests, at most, modest benefit to this practice if used routinely in children. Any benefits of the higher yield should be weighed against the potential for additional false-positive results, the time for the additional trip to the EEG laboratory, missed school and work, cost, and the burden of sleep deprivation. After a first seizure, the repeat EEG will not result in any additional information that changes acute management. Regardless of EEG results, two or more unprovoked seizures in a child are diagnostic of epilepsy and warrant treatment and consultation with a neurologist for selection of medication. Because phenobarbital causes sedation and cognitive and behavioral changes in young children, it would not be used, except in countries where resources for medical care are very limited. Phenytoin has somewhat erratic absorption and can cause gingival hyperplasia in children, so it is not first-line therapy for a new diagnosis of epilepsy. Carbamazepine is a good choice for a child diagnosed with partial epilepsy, but not for generalized epilepsy. Valproic acid is a good choice for both partial and generalized epilepsy in children older than age 2 years. Each of these generic choices, along with the plethora of new medications, has unique adverse event profiles, which physicians must understand.

References:

Page 425: AAP PREP 2009

2009 PREP SA on CD-ROM

page 425

Gilbert DL, Buncher CR. An EEG should not be obtained routinely after first unprovoked seizure in childhood. Neurology. 2000;54:635-641. Abstract available at: http://www.ncbi.nlm.nih.gov/pubmed/10680796

Gilbert DL, DeRoos S, Bare MA. Does sleep or sleep deprivation increase epileptiform discharges in pediatric electroencephalograms? Pediatrics. 2004;114:658-662. Available at: http://pediatrics.aappublications.org/cgi/content/full/114/3/658

Gilbert DL, Sethuraman G, Kotagal U, Buncher CR. Meta-analysis of EEG test performance shows wide variation among studies. Neurology. 2003;60:564-570. Abstract available at: http://www.ncbi.nlm.nih.gov/pubmed/12601093

Hirtz D, Ashwal S, Berg A, et al. Practice parameter: evaluating a first nonfebrile seizure in children: report of the Quality Standards Subcommittee of the American Academy of Neurology, the Child Neurology Society, and the American Epilepsy Society. Neurology. 2000;55:616-623. Available at: http://www.neurology.org/cgi/content/full/55/5/616

Hirtz D, Berg A, Bettis D, et al. Practice parameter: treatment of the child with a first unprovoked seizure: report of the Quality Standards Subcommittee of the American Academy of Neurology and the Practice Committee of the Child Neurology Society. Neurology. 2003;60:166-175. Available at: http://www.neurology.org/cgi/content/full/60/2/166

Stroink H, van Donselaar CA, Geerts AT, Peters AC, Brouwer OF, Arts WF. The accuracy of the diagnosis of paroxysmal events in children. Neurology. 2003;60:979-982. Abstract available at: http://www.ncbi.nlm.nih.gov/pubmed/12654963

Page 426: AAP PREP 2009

2009 PREP SA on CD-ROM

page 426

Question: 119

You receive a call from a local neonatologist because the mother of a newly admitted baby has identified you as her pediatrician. The baby was born at term and had Apgar scores of 1, 2, and 4 at 1, 5, and 10 minutes, respectively. She is requiring significant ventilatory and fluid support, and she has multiple congenital anomalies, including bilateral microtia, depressed nasal bridge, ocular hypertelorism, cleft palate, and macrocephaly. Echocardiography shows truncus arteriosus, and head ultrasonography reveals dilated ventricles consistent with hydrocephalus.

Of the following, the teratogenic agent that is MOST likely responsible for this infant’s features is

A. alcohol

B. isotretinoin

C. phenobarbital

D. phenytoin

E. thalidomide

Page 427: AAP PREP 2009

2009 PREP SA on CD-ROM

page 427

Critique: 119 Preferred Response: B

The newborn described in the vignette has features consistent with retinoic acid embryopathy, also called "Accutane embryopathy" (Accutane® is one of the brand names for isotretinoin [13-cis-retinoic acid]). Soon after the drug was licensed for use in the United States, Lammer and associates published an article describing 21 infants who had histories of prenatal isotretinoin exposure and negative outcomes. It now is known that many retinoids are teratogenic in a number of species, with malformations largely due to inhibited migration of cranial neural crest cells during embryogenesis. The craniofacial, cardiovascular, and central nervous systems sustain the greatest deleterious effects. Currently, a 35% risk for retinoic acid embryopathy is believed to exist in the infants of women who take isotretinoin beyond 15 days after conception. Maternal use of isotretinoin prior to conception does not appear to be associated with an increased risk for fetal anomalies. Paternal use of isotretinoin does not cause birth defects. Prenatal alcohol exposure is associated with fetal alcohol spectrum disorder. The most severe end of the spectrum is fetal alcohol syndrome (FAS); affected infants typically exhibit intrauterine growth restriction; microcephaly; and dysmorphisms that include short palpebral fissures, smooth philtrum, and a narrow upper vermilion (Item C119A). Congenital heart defects are seen in some infants who have FAS. Phenobarbital and phenytoin use during pregnancy can result in a fetal anticonvulsant syndrome in the exposed infant. Affected neonates also may have intrauterine growth restriction, ocular hypertelorism, midface hypoplasia (Item C119B), cleft lip+/-cleft palate, and heart defects. Thalidomide is a sedative that was marketed in the 1950s in Europe and was prescribed to pregnant women to ameliorate morning sickness. It is probably the most notorious of all teratogens and was taken off the market for many years but was reapproved by the United States Food and Drug Administration in 1998 for use in the treatment of erythema nodosum leprosum and multiple myeloma. The prescribing of thalidomide is strictly controlled, and physicians who do so must be registered in a special safety program. Thalidomide exposure during pregnancy is associated with a variety of anomalies, the most striking of which is severe limb reduction defects.

References:

Isotretinoin, alcohol, phenytoin, phenobarbital, thalidomide. Reprotox. Available for subscription at: http://www.reprotox.org

Isotretinoin, alcohol, phenytoin, phenobarbital, thalidomide. Teris. Available for subscription at: http://depts.washington.edu/terisweb/teris/

Jones KL. Retinoic acid embryopathy. In: Smith's Recognizable Patterns of Human Malformation. 6th ed. Philadelphia, Pa; Elsevier Saunders; 2006:660-661

Lammer EJ, Chen DT, Hoar RM, et al. Retinoic acid embryopathy. N Engl J Med. 1985;313:837-841. Abstract available at: http://www.ncbi.nlm.nih.gov/pubmed/3162101

Page 428: AAP PREP 2009

2009 PREP SA on CD-ROM

page 428

Critique: 119

Characteristic facial appearance of a newborn who has fetal alcohol syndrome includes midface hypoplasia, a relatively smooth philtrum, and a narrow upper lip with a poorly defined "cupid's bow." (Courtesy of M. Rimsza)

Page 429: AAP PREP 2009

2009 PREP SA on CD-ROM

page 429

Critique: 119

Arched eyebrows, wide-spaced eyes, and a short upturned nose are characteristic of infants exposed to phenytoin during pregnancy. (Courtesy of M. Rimsza)

Page 430: AAP PREP 2009

2009 PREP SA on CD-ROM

page 430

Question: 120

You are evaluating a 14-year-old boy for his preparticipation sports physical examination before he tries out for the freshman football team. He has no chronic health problems and no previous history of head injuries. His mother expresses concern about recent reports of professional football players sustaining cognitive damage due to repeated concussions.

Of the following, the MOST appropriate statement to include in your counseling regarding head injuries in contact sports is that

A. baseline, detailed neuropsychological testing has been well established as a tool to use in the management of head injuries in pediatric athletes

B. most athletic participants who sustain concussions lose consciousness

C. return-to-play guidelines for pediatric athletes are well established and evidence-based

D. seasonal and lifetime sports exclusion guidelines for pediatric athletes after head injury are well established and evidence-based

E. the preparticipation visit history of an athlete should include the number of prior concussions, timing and severity of each, and description of resulting symptoms

Page 431: AAP PREP 2009

2009 PREP SA on CD-ROM

page 431

Critique: 120 Preferred Response: E

Although sports participation can have many physical and social benefits for children and adolescents, there are risks, including athletic concussion, which is described as mild traumatic brain injury that is caused by an impact to the head. Numerous guidelines and literature reviews have examined athletic concussion, but few have focused specifically on the pediatric athlete. Medical examination frequently is mandated by schools and organizations before children can participate in sports, and preparticipation assessment of an athlete is an opportune time to focus on how to prevent head trauma as well recognition of concussions. Loss of consciousness is not the only indication that an athlete has suffered a concussion; in fact, most sports participants who sustain concussions do not lose consciousness. Documenting the history of the number of prior concussions, timing and severity of each, and the description of resulting symptoms can help establish a baseline for medical care. The initial "grade" of the concussion generally is accepted as less important than the systematic tracking of each athlete's recovery course over time. Although conventional neuropsychological assessment is a well-established and recommended methodology for the evaluation of children and adults who have sustained neurologic injury or who have neurodevelopmental problems, the use of comprehensive neuropsychological evaluations at a preparticipation visit is impractical and expensive because of the extensive time needed to complete an evaluation. Abbreviated "baseline" neuropsychological testing for athletic purposes has been introduced, but sufficient data regarding the reliability, validity, and clinical utility of neuropsychological instruments used in this model are lacking for pediatric athletes. Similarly, the value of such baseline testing in making individual decisions about return to play after a head injury has not been demonstrated. However, neuropsychological testing still can play a valuable role in evaluating and managing certain athletes after injury, especially those who experience poor recovery and multiple injuries, and in developing appropriate posttraumatic educational management plans if cognitive difficulties exist. Return to play after concussion is a controversial area, and none of the current guidelines have garnered sufficient scientific support to be recognized as the "gold standard." Experts agree that the earliest an athlete should return to play is when no signs or symptoms of any type are apparent at rest or exertion; neurologic examination results are normal; and neuroimaging, when performed, provides unremarkable results. Some advocate for even more conservative standards with pediatric athletes, including always removing them from the day's competition after a concussion and not allowing return to play unless cleared medically. Seasonal and lifetime sports exclusion guidelines are not well-established or evidence-based for pediatric athletes, but a conservative approach is inarguably appropriate.

References:

Guskiewicz KM, Weaver NL, Padua DA, Garrett WE Jr. Epidemiology of concussion in collegiate and high school football players. Am J Sports Med. 2000;28:643-650. Abstract available at: http://www.ncbi.nlm.nih.gov/pubmed/11032218

Kirkwood MW, Yeates KO, Wilson PE. Pediatrics sport-related concussion: a review of the clinical management of an oft-neglected population. Pediatrics. 2007;117:1359-1371. Available at: http://pediatrics.aappublications.org/cgi/content/full/117/4/1359

Page 432: AAP PREP 2009

2009 PREP SA on CD-ROM

page 432

Question: 121

You are evaluating a 1-year-old girl who was admitted to the pediatric intensive care unit following 3 days of diarrhea and decreased oral intake. Her heart rate is 160 beats/min, respiratory rate is 30 breaths/min, blood pressure is 70/40 mm Hg, and she has weak peripheral pulses. An arterial blood gas evaluation on room air reveals pH of 7.08, Paco2 of 25 mm Hg, Pao2 of 100 mm Hg, and HCO3 of 5.0 mEq/L (5.0 mmol/L). Initial electrolyte values are: sodium of 130.0 mEq/L (130.0 mmol/l), potassium of 4.0 mEq/L (4.0 mmol/L), chloride of 95.0 mEq/L (95.0 mmol/L), bicarbonate of 6.0 mEq/L (6.0 mmol/L), and glucose of 100.0 mg/dL (5.6 mmol/L).

Of the following, the MOST appropriate initial therapy is

A. 5 mcg/kg per minute dopamine infusion

B. 1 mEq/kg intravenous potassium chloride

C. 1 mEq/kg intravenous sodium bicarbonate

D. 20 mL/kg intravenous 0.9% sodium chloride

E. 1 unit/kg intravenous insulin

Page 433: AAP PREP 2009

2009 PREP SA on CD-ROM

page 433

Critique: 121 Preferred Response: D

The child described in the vignette demonstrates severe metabolic acidosis, normally defined as a pH of less than 7.10 or serum bicarbonate concentration of less than 8.0 mEq/L (8.0 mmol/L), with partial respiratory compensation. Severe metabolic acidosis is a medical emergency that requires urgent intervention via an organized approach. Because this patient's respiratory drive, oxygenation, and ventilation are adequate, therapy to correct the metabolic acidosis and its underlying cause should be the first priority. Volume repletion using isotonic fluids (such as normal saline) at an initial dose of 20 mL/kg is the first-line therapy to correct metabolic acidosis. Administration of 1 mEq/kg sodium bicarbonate to raise the pH to above 7.2 may be indicated to correct the adverse effects of severe acidemia, such as decreased myocardial contractility, increased pulmonary vascular resistance, sensitization to cardiac arrhythmias, and decreased responsiveness to catecholamine administration, but volume administration remains the initial therapy of choice. Adequate ventilation is imperative before administering sodium bicarbonate to ensure that the additionally generated carbon dioxide is eliminated and not retained, which would decrease cellular pH further. Children who have severe shock may require vasopressor (eg, dopamine) administration if they do not respond to initial volume resuscitation. Because metabolic acidosis produces an extracellular shift of potassium and subsequent increase in serum potassium concentration, administration of potassium should be deferred until adequate urine output is assured and correction of the underlying acidosis has begun. Insulin is the cornerstone of treatment of the metabolic acidosis seen in diabetic ketoacidosis, but the child in the vignette has no evidence of diabetes mellitus.

References:

Greenbaum LA. Electrolyte and acid-base disorders. In: Kliegman RM, Behrman RE, Jenson HB, Stanton BF, eds. Nelson Textbook of Pediatrics. 18th ed. Philadelphia, Pa: Saunders Elsevier; 2007:267-308

Schwaderer AL, Schwartz GJ. Back to basics: acidosis and alkalosis. Pediatr Rev. 2004;25:350-357. Available at: http://pedsinreview.aappublications.org/cgi/content/full/25/10/350

Page 434: AAP PREP 2009

2009 PREP SA on CD-ROM

page 434

Question: 122

A 3-year-old girl presents to the emergency department in an almost unresponsive state. Her parents say that she has become increasingly ill over the past 5 days and has been very thirsty, with increased urination. This morning she began to vomit and could not keep down fluids. Findings on physical examination in addition to unresponsiveness include rapid, sighing respirations and flushed cheeks. You estimate that she is 10% dehydrated. Initial laboratory studies reveal a blood glucose concentration of 700.0 mg/dL (38.9 mmol/L), sodium of 130.0 mEq/L (130.0 mmol/L), potassium of 4.6 mEq/L (4.6 mmol/L), chloride of 96.0 mEq/L (96.0 mmol/L), bicarbonate of 8.0 mEq/L (8.0 mmol/L), and a venous pH of 7.0.

Of the following, the MOST appropriate action to decrease this child’s risk for cerebral edema during treatment is to

A. avoid potassium replacement until the serum potassium value is less than 4.0 mEq/L (4.0 mmol/L)

B. correct acidosis rapidly with sodium bicarbonate

C. rehydrate initially with 3% saline

D. rehydrate slowly using 0.45% to 0.9% saline

E. replace continuing urinary fluid losses with 0.2% saline

Page 435: AAP PREP 2009

2009 PREP SA on CD-ROM

page 435

Critique: 122 Preferred Response: D

Treatment of diabetic ketoacidosis (DKA) requires supplementation with fluid, electrolytes, insulin, and carbohydrate to replenish losses of fluid and electrolytes and supply insulin for proper carbohydrate, protein, and lipid metabolism and to repair acidosis. The major life-threatening complication of the treatment of DKA in children is cerebral edema, which occurs in 1 in 100 to 1 in 400 episodes of DKA. Slow rehydration with fluid containing adequate electrolytes may decrease the risk of cerebral edema. The rate of replacement still is argued, but calculated steady replacement of fluid losses as 0.45% to 0.9% saline, with glucose and potassium added as necessary over 36 to 48 hours, generally is considered appropriate. Large urinary potassium losses occur during the development of DKA. Even children who present with hyperkalemia have had potassium losses. Insulin and glucose drive potassium into cells and lower circulating potassium concentrations relatively rapidly during treatment. Therefore, potassium replacement should be started as soon as the child is urinating and there is no worrisome hyperkalemia. Avoiding potassium replacement until the potassium is less than 4.0 mEq/L (4.0 mmol/L) is unnecessary. Peripheral acidosis is corrected rapidly by sodium bicarbonate, but bicarbonate dissociates in blood into bicarbonate ion and carbon dioxide, and the carbon dioxide crosses cellular and blood-brain barriers more rapidly than bicarbonate ion. Therefore, the net biologic intracellular effect of bicarbonate administration is an increase in intracellular and central nervous system (as measured in cerebrospinal fluid) acidosis. Several studies suggest that administration of bicarbonate may increase the chance for an adverse outcome, including the increased risk of cerebral edema. On the other hand, it is possible that only those who have the most severe acidosis receive bicarbonate and that it is disease severity rather than treatment that worsens outcome. Three percent saline is hypertonic and would worsen renal fluid losses. This is not a good maintenance fluid for treatment of DKA, although some evidence suggests that hypertonic saline is as effective as mannitol in reducing symptomatic cerebral swelling, should it occur. Urine sodium losses in DKA usually are about 75.0 mEq/L (75.0 mmol/L). Therefore, 0.2% saline is a hypotonic replacement solution whose use is ill advised.

References:

Dunger DB, Sperling MA, Acerini CL, et al. European Society for Paediatric Endocrinology/Lawson Wilkins Pediatric Endocrine Society consensus statement on diabetic ketoacidosis in children and adolescents. Pediatrics. 2004;113:e133-e140. Available at: http://pediatrics.aappublications.org/cgi/content/full/113/2/e133

Edge JA, Jakes RW, Roy Y, et al. The UK case-control study of cerebral oedema complicating diabetic ketoacidosis in children. Diabetologia. 2006;49:2002-2009. Abstract available at: http://www.ncbi.nlm.nih.gov/pubmed/16847700

Wolfsdorf J, Craig ME, Daneman D, et al; International Society for Pediatriac and Adolescent Diabetes. Diabetic ketoacidosis. Pediatr Diabetes. 2007;8:28-43

Page 436: AAP PREP 2009

2009 PREP SA on CD-ROM

page 436

Question: 123

A 14-year-old boy has been receiving occupational therapy due to weakness in his graphomotor (eg, handwriting) skills. During the school annual Individualized Education Plan (IEP) meeting, his mother asks about alternative strategies that could be used to help him compensate for his area of weakness.

Of the following, the BEST alternative strategy is to

A. allow him to use print rather than cursive writing for his notes

B. have a class scribe write notes for him

C. have him use a word processor/laptop computer

D. have him use audio books

E. provide preferential seating near the blackboard

Page 437: AAP PREP 2009

2009 PREP SA on CD-ROM

page 437

Critique: 123 Preferred Response: C

As students who have learning difficulties progress in their education, the emphasis changes from remediation to accommodation. Accommodation encompasses modifications in how tasks are given to students so that affected children may complete the same school work as other students. Such accommodations allow students who have learning issues to present their knowledge without being affected adversely by their disabilities. The boy in the vignette, who has weakness in his graphomotor skills, should be allowed access to electronic devices such as a laptop computer to help accommodate his area of weakness. The use of a class scribe will not foster his independence in taking his own notes. Allowing him to print instead of writing in script may make his notes neater to read, but it will slow his ability to take notes efficiently. Preferential seating near the blackboard may be helpful for a student who has a visual acuity problem, but it will not help this student's handwriting. Audio texts are useful for students who read slowly. Other potential accommodations allow a change in timing, formatting, setting, scheduling, response, or presentation of the assignment or test. Such accommodations do not alter what the test or assignment measures in any significant manner. Examples of such accommodations include a Braille version of a test for a student who is blind or taking a test alone in a quiet room for one who has attention-deficit/hyperactivity disorder.

References:

Thorne G. Graphomotor Skills: Why Some Kids Hate to Write. Center for Development and Learning. Covington, La. 2006. Available at: http://www.cdl.org/resource-library/articles/graphomotor.php

Shaywitz SE, Shaywitz BA. Dyslexia (specific reading disability) Pediatr Rev. 2003;24:147-153. Available at: http://pedsinreview.aappublications.org/cgi/content/full/24/5/147

Page 438: AAP PREP 2009

2009 PREP SA on CD-ROM

page 438

Question: 124

During hospital rounds, you are evaluating a 7-year-old boy who has been hospitalized for 1 month after developing a perforated appendix. He recently started eating solid foods, but today he developed loose stools and a mild amount of abdominal discomfort. He has had four bowel movements within the past 6 hours that are described as watery and nonbloody. His parents are very concerned that this is another complication of his perforated appendix. On your physical examination, the boy is afebrile and has active bowel sounds and only minor discomfort on palpation to his abdomen.

Of the following, the MOST likely organism to be causing this patient’s diarrhea is

A. Campylobacter jejuni

B. Clostridium difficile

C. Salmonella enteritidis

D. Shigella sonnei

E. Yersinia enterocolitica

Page 439: AAP PREP 2009

2009 PREP SA on CD-ROM

page 439

Critique: 124 Preferred Response: B

Although any gastrointestinal pathogen, including Campylobacter jejuni, Salmonella enteritidis, Shigella sonnei, or Yersinia enterocolitica, can be acquired in a nosocomial outbreak, hospitalized patients are most likely to have gastrointestinal problems due to Clostridium difficile. Infection with this organism usually occurs in the setting of altered bowel flora, such as in children who have a history of receiving antimicrobial therapy or other medications (eg, chemotherapeutic agents), those who have required repeated enemas, and those who have had prolonged nasal gastric tube insertions. Although penicillins, clindamycin, and cephalosporins commonly are associated with C difficile infection, almost every antimicrobial agent has been described as a potential cause. In fact, many clinical microbiology laboratories will not accept a stool culture for the other pathogens from a patient hospitalized for longer than 3 days unless testing for C difficile is negative. Clinical symptoms of C difficile infection vary widely, and asymptomatic colonization is common in infants and young children. Infants become colonized during the first postnatal week, with up to 50% of infants younger than 1 year of age possibly being colonized. Carriage rates decrease to 1% to 5% by 2 years of age. Risk factors for becoming colonized in the hospital include having an infected roommate and having symptomatic patients on the same ward. Symptomatic disease is due to the action of the toxins (A and B) that are produced by the organism. The clinical manifestations of a C difficile infection include diarrhea; abdominal cramps; abdominal tenderness; and in some instances, fever, systemic toxicity, and bloody stools. Severe or even fatal cases have been described, especially in patients who have problems with neutropenia or those who have other gastrointestinal problems (eg, Hirschsprung disease). The diagnosis of disease due to C difficile is made by documenting the presence of toxin A or B with a commercially available kit or by endoscopic findings of pseudomembranous enterocolitis (Item C124). Symptomatic infants younger than 1 year of age should be investigated for other causes of diarrhea because the carriage rate is so high, and the presence of toxin may not be responsible for the patient's clinical signs or symptoms.

References:

American Academy of Pediatrics. Clostridium difficile. In: Pickering LK, Baker CJ, Long SS, McMillan JA, eds. Red Book: 2006 Report of the Committee on Infectious Diseases. 27th ed. Elk Grove Village, Ill: American Academy of Pediatrics; 2006:261-263

Brook I. Pseudomembranous colitis in children. J Gastroenterol Hepatol. 2005;20:182-186. Abstract available at: http://www.ncbi.nlm.nih.gov/pubmed/15683418

Page 440: AAP PREP 2009

2009 PREP SA on CD-ROM

page 440

Critique: 124

Clostridium difficile pseudomembranous colitis. The pseudomebranes are shown by arrows. (Courtesy of A. Bousvaros)

Page 441: AAP PREP 2009

2009 PREP SA on CD-ROM

page 441

Question: 125

You are evaluating a 7-year-old boy who has human immunodeficiency virus (HIV) infection and failure to thrive. Over the last 2 months, he has had intermittent temperatures to 102.5°F (39.2°C), anorexia, abdominal pain, diarrhea, and a 6-lb weight loss. His HIV infection has been poorly controlled because of noncompliance with medications. His viral load is 150,000 copies, and his CD4 count is 40 cells/mm3. Physical examination shows a thin, small-for-age boy who is playing quietly. He weighs 15.8 kg (<5th percentile). He has some oral thrush, cervical and inguinal adenopathy, and mild diffuse abdominal tenderness to palpation.

Of the following, the MOST likely pathogen causing this patient’s illness is

A. Cryptosporidium

B. cytomegalovirus

C. Mycobacterium avium-intracellulare complex

D. Mycobacterium tuberculosis complex

E. Salmonella sp

Page 442: AAP PREP 2009

2009 PREP SA on CD-ROM

page 442

Critique: 125 Preferred Response: C

The child described in the vignette displays clinical signs and symptoms that are classic for disseminated Mycobacterium avium-intracellulare complex (MAC) disease. This infection occurs in patients who have impaired cell-mediated immunity (eg, congenital immune defects or human immunodeficiency virus [HIV] infection). The greatest risk factor for MAC in patients who have HIV infection is a severe depression of the CD4 cell count. Patients whose CD4 counts are less than 100 cells/mm3 are at the highest risk for disease. Clinical manifestations of disseminated nontuberculous mycobacterial infection may include high fever, night sweats, weight loss, lymphadenopathy, abdominal pain, fatigue, hepatosplenomegaly, diarrhea, and anemia. Diagnosis is established by isolation of the organism from blood or infected tissues. The nontuberculous mycobacteria that most commonly cause disease in children are: MAC (eg, M avium and M intracellulare), M fortuitum, M abscessus, M marinum, M chelonae, and M kansasii. These organisms may cause a variety of clinical syndromes, including cervical lymphadenitis (MAC), cutaneous infections (M fortuitum, M abscessus, M marinum, M chelonae), otitis media (M abscessus), osteomyelitis (MAC, M fortuitum, M kansasii), central catheter infections (M chelonae, M fortuitum), pulmonary disease (MAC, M kansasii, M fortuitum), and disseminated infection (MAC). In the immunocompromised host, the most common manifestations of cytomegalovirus disease are pneumonia, colitis, and retinitis, none of which are manifested in this patient. Disease caused by Cryptosporidium is associated with a chronic diarrhea; fever usually is not seen. M tuberculosis generally is associated with pulmonary manifestations. Salmonella sp is associated with acute diarrhea.

References:

American Academy of Pediatrics. Diseases caused by nontuberculous mycobacteria (atypical mycobacteria, mycobacteria other than Mycobacterium tuberculous). In: Pickering LK, Baker CJ, Long SS, McMillan JA, eds. Red Book: 2006 Report of the Committee on Infectious Diseases. 27th ed. Elk Grove Village, Ill: American Academy of Pediatrics; 2006:698-704

Gordin FM, Horsburgh CR Jr. Mycobacterium avium complex. In: Mandell GL, Bennett JE, Dolin R, eds. Mandell, Douglas, and Bennett's Principles and Practice of Infectious Diseases. 6th ed. Philadelphia, Pa: Elsevier Churchill Livingstone; 2005:2897-2909

Horsburgh CR Jr, Caldwell MB, Simonds RJ. Epidemiology of disseminated nontuberculous mycobacterial disease in children with acquired immunodeficiency syndrome. Pediatr Infect Dis J. 1993;12:219-222. Abstract available at: http://www.ncbi.nlm.nih.gov/pubmed/8095716

Page 443: AAP PREP 2009

2009 PREP SA on CD-ROM

page 443

Question: 126

You are seeing a 7-year-old boy for occasional nocturnal enuresis. His weight and height are at the 50th percentile for age, his blood pressure is 110/66 mm Hg, and there are no unusual findings on physical examination. Urinalysis shows a specific gravity of 1.030, pH of 6.5, 2+ blood, and no protein. Urine microscopy reveals 10 to 20 red blood cells/high-power field and no casts or crystals. Results of a repeat urine sample 3 weeks later are unchanged. Laboratory findings include:· Blood urea nitrogen, 12.0 mg/dL (4.3 mmol/L)· Creatinine, 0.4 mg/dL (35.4 mcmol/L)· Complement component 3 (C3), 110.0 mg/dL (normal, 86.0 to 166.0 mg/dL) · Complement component 4 (C4), 22.0 mg/dL (normal, 13.0 to 32.0 mg/dL) · Antinuclear antibody, negative· Erythrocyte sedimentation rate, 6 mm/hr

Of the following, the MOST appropriate next step is

A. abdominal computed tomography scan

B. referral for cystoscopy

C. referral for renal biopsy

D. renal/bladder ultrasonography

E. repeat urinalysis in 1 month

Page 444: AAP PREP 2009

2009 PREP SA on CD-ROM

page 444

Critique: 126 Preferred Response: D

Microscopic hematuria is defined as a positive dipstick test for blood and more than 5 red blood cells/high-power field on microscopy (Item C126). Persistent microscopic hematuria is defined as blood on urinalysis detected on repeat samples over a 1-month period. Children who have persistent microscopic hematuria, such as the boy described in the vignette, require investigation for an underlying cause. The urgency of such an evaluation in the absence of symptoms is predicated on whether the patient has accompanying proteinuria (>1+ on a dipstick). Those who have proteinuria require urgent evaluation to look for an underlying glomerulopathy in which disruption of the glomerular capillary barrier (as occurs in glomerulonephritis) results in red blood cells and albumin gaining access to the urinary space. Up to 4% of all children may have microscopic hematuria on a routine screening urinalysis. Based on this high incidence, the initial recommendation for a child who has isolated, asymptomatic microscopic hematuria is to undergo repeat urinalysis in 2 to 3 weeks time. If the hematuria persists, as occurred in the patient in the vignette, the clinician should evaluate the patient for an underlying genitourinary problem. This evaluation typically includes renal function tests (serum creatinine) and serologic testing for an underlying immune complex-mediated glomerulonephritis (complement components 3 and 4, antinuclear antibody). In addition, renal imaging (renal/bladder ultrasonography) to screen for cysts, stones, and tumors is indicated. Abdominal computed tomography scan is not indicated prior to a renal/bladder ultrasonography in this clinical setting. More invasive testing, such as cystoscopy, should not be undertaken unless there is a strong suspicion of bladder pathology, especially prior to screening with ultrasonography. Similarly, renal biopsy is not indicated in the absence of proteinuria, hypertension, azotemia, or gross hematuria. Repeat urinalysis is redundant because persistent hematuria already has been substantiated.

References:

Moxey-Mims M. Hematuria and proteinuria. In: Kher KK, Schnaper HW, Makker SP, eds. Clinical Pediatric Nephrology. 2nd ed. London, England: Informa Healthcare; 2007:129-141

Patel HP, Bissler JJ. Hematuria in children. Pediatr Clin North Am. 2001;48:1519-1537. Abstract available at: http://www.ncbi.nlm.nih.gov/pubmed/11732128

Vehaskari VM, Rapola J, Koskimies O, Savilahti E, Vilska J, Hallman N. Microscopic hematuria in school children: epidemiology and clinicopathologic evaluation. J Pediatr. 1979;95:676-684. Abstract available at: http://www.ncbi.nlm.nih.gov/pubmed/490233

Page 445: AAP PREP 2009

2009 PREP SA on CD-ROM

page 445

Critique: 126

Numerous red blood cells (arrows) on microscopic examination of urine. (Reprinted with permission from Schumann GB, Friedman SK. Wet Urinalysis. Chicago, Ill: ASCP Press; 2007. ©2007, American Society for Clinical Pathology.)

Page 446: AAP PREP 2009

2009 PREP SA on CD-ROM

page 446

Question: 127

An 8-year-old girl presents with multiple episodes of "bronchitis." For the past 2 years, she has had problems with coughing, wheezing, and difficulty catching her breath during vigorous exercise. Treatment with a metered dose beta2 agonist inhaler has improved her symptoms. In your office, you discuss the different tests to assess lung function.

Of the following, the BEST test to measure lung function for this girl is

A. arterial blood gas

B. exhaled breath condensate

C. exhaled nitric oxide

D. pulse oximetry

E. spirometry

Page 447: AAP PREP 2009

2009 PREP SA on CD-ROM

page 447

Critique: 127 Preferred Response: E

Spirometry, also referred to as a pulmonary function test (PFT), measures inspiratory and expiratory respiratory effort. Established pretest norms are based on the patient's height and ethnicity. Measurements of lung function that can be obtained with spirometry include the forced vital capacity (FVC), volume of air exhaled during the first second (FEV1), FEV1/FVC ratio, peak expiratory flow (PEF), and airflow during the middle half of the effort (forced expiratory flow) (FEF25-75). Because ideal test conditions require a patient to exhale for 6 seconds, children younger than 6 to 7 years of age often are unable to complete the test. Also, spirometry does not measure total lung capacity (TLC) or residual volume (RV). An arterial blood gas is a gas diffusion measurement that provides insight into oxygenation (PO2) and ventilation (PCO2), but it does not measure lung function. Pulse oximetry is a simple, noninvasive method to measure oxygenation, but it also does not measure ventilation or lung function. Exhaled nitric oxide (eNO) measurement was approved by the United States Food and Drug Administration in 2003 for children ages 4 years and older. Although not a measure of lung function, eNO is a useful noninvasive tool to measure nitric oxide, a marker of airway inflammation. Exhaled breath condensate is a noninvasive method to measure the pH of the airway, another marker of inflammation. Similar to eNO, exhaled breath condensate does not measure lung function but is being developed as another tool to assess airway inflammation and assist with asthma management.

References:

Guill MF. Asthma update: clinical aspects and management. Pediatr Rev. 2004;10:335-344. Available at: http://pedsinreview.aappublications.org/cgi/content/full/25/10/335

Liu AH, Covar RA, Spahn JD, Leung DYM. Childhood asthma. In: Kliegman RM, Behrman RE, Jenson HB, Stanton BF, eds. Nelson Textbook of Pediatrics. 18th ed. Philadelphia, Pa: Saunders Elsevier; 2007:953-969

Silkoff PE, Carlson M, Bourke T, Katial R, Ogren E, Szefler SJ. The Aerocrine exhaled nitric oxide monitoring system NIOX is cleared by the US Food and Drug Administration for monitoring therapy in asthma. J Allergy Clin Immunol. 2004;114:1241-1256. Abstract available at: http://www.ncbi.nlm.nih.gov/pubmed/15536442

Page 448: AAP PREP 2009

2009 PREP SA on CD-ROM

page 448

Question: 128

A 4-year-old girl is brought to the emergency department after sticking a hair pin in a household electrical outlet. The mother reports that she heard the child scream, and when she investigated, smoke was coming from the outlet and the child was crying, holding her right hand. There was a black imprint on her fingers in the shape of the hair pin. She washed the fingers with soap and water and drove the child to the emergency department for further evaluation. On physical examination, the child is tearful but awake and alert. Her right index finger and thumb have erythematous burn imprints with small blisters surrounded by soot. She has no other burns or other findings of note on the remainder of her examination.

Of the following, it is MOST important to evaluate this patient for

A. arrhythmias

B. compartment syndrome

C. immunization status

D. myoglobinuria

E. skin grafting

Page 449: AAP PREP 2009

2009 PREP SA on CD-ROM

page 449

Critique: 128 Preferred Response: C

Electrical injuries account for approximately 5% of admissions to burn units in the United States annually. Most injuries occur in adults and are related to high-voltage sources in the workplace. Pediatric injuries typically are related to contact with household, low-voltage sources such as electric cords or wall outlets. The primary determinant of tissue damage following electrical exposure is current strength, which is directly proportional to voltage and inversely related to tissue resistance. Because voltage is the only variable commonly known, electrical exposures are classified as low- (<1,000 V) and high- (>1,000 V) voltage. Standard household electricity in the United States is 110 V; utility power lines carry in excess of 14,000 V. The most common injury following contact with electricity is burns. Excluding lightening injuries, electrical burns may occur from any of the following mechanisms: electrothermal heating from direct contact with the electrical source, arc exposure in which the body becomes part of the electrical circuit, flash contact in which the current arc strikes the skin but does not enter the body, and flame burns that result when clothing or other objects combust in the presence of electrical current. Oral burns due to chewing on an electrical cord (Item C128) usually are electrothermal. Arc exposure can be associated with deep tissue burns and internal organ involvement as the current flows through body; the extent of injury may be underestimated by the appearance of the skin wounds. Flash contact is characterized by surface burns accompanied by soot, as described for the girl in the vignette. Although both high- and low-voltage exposures can lead to injuries in a variety of organ systems, high-voltage exposures are responsible for most significant injuries. The most commonly affected organ is the skin, with burns resulting as described previously. Arrhythmias, including asystole and ventricular fibrillation, may occur, usually at the time of contact. Deep thermal injuries to bone and muscle may occur after arc exposure and lead to compartment syndrome or rhabdomyolysis with subsequent renal damage. Risk factors for these more serious injuries include extensive full-thickness burns, cardiac arrest, or high-voltage exposure. It is rare for children exposed to household current to sustain injuries other than burns. The girl in the vignette has a surface thermal burn related to contact with the heated hair pin, and the presence of soot on her finger is evidence of flash contact. She has no clinical features or risk factors for rhabdomyolysis, myoglobinuria, compartment syndrome, or arrhythmias. The wound is small and should not require skin grafting. Therefore, other than ascertaining her tetanus immunization status and providing wound care, no further evaluation is indicated.

References:

Chen EH, Sareen A. Do children require ECG evaluation and inpatient telemetry after household electrical exposures? Ann Emerg Med. 2007;49:64-67

Pinto DS, Clardy PF. Environmental electrical injuries. UpTo Date Online 15.3. 2008 Available for subscription at: http://www.utdol.com/utd/content/topic.do?topicKey=ad_emerg/2283&selectedTitle=1~150&source=search_result

Price TG, Cooper MA. Electrical and lightning injuries. In Marx JA, ed. Rosen's Emergency Medicine: Concepts and Clinical Practice. 6th ed. Philadelphia, Pa: Mosby Elsevier; 2006:chap 140

Rosen CL, Adler JN, Rabban JT, et al. Early predictors of myoglobinuria and acute renal failure following electrical injury. J Emerg Med. 1999;17:783-789. Abstract available at: http://www.ncbi.nlm.nih.gov/pubmed/10499690

Page 450: AAP PREP 2009

2009 PREP SA on CD-ROM

page 450

Critique: 128

Oral burns may result from chewing on an electrical cord. (Courtesy of M. Wright)

Page 451: AAP PREP 2009

2009 PREP SA on CD-ROM

page 451

Question: 129

A 12-year-old girl has had intermittent periumbilical abdominal pain for the past 4 years. Sometimes the pain worsens when she drinks a glass of milk. A lactose breath hydrogen test demonstrates a breath hydrogen of 40 ppm after 1 hour (normal, <20 ppm).

Of the following, the food that this girl is MOST likely to tolerate is

A. buttermilk

B. cheddar cheese

C. ice cream

D. skim milk

E. whole milk

Page 452: AAP PREP 2009

2009 PREP SA on CD-ROM

page 452

Critique: 129 Preferred Response: B

Lactose intolerance (lactose maldigestion) is a common cause of pediatric recurrent abdominal pain. Lactase is a digestive enzyme located on the intestinal villi that converts the disaccharide lactose to the monosaccharides glucose and galactose. The monosaccharides can be absorbed across the intestinal epithelial cells. In an individual who has low intestinal lactase (lactose intolerance), the lactose cannot be broken down and passes into the lower intestine and colon. The malabsorbed lactose can cause osmotic diarrhea or be fermented by bacteria, resulting in pain, gas, and bloating. Lactose intolerance can either be primary (lactase activity that gradually declines with aging) or secondary (an infection or enteropathy damages the villi, resulting in lactose intolerance). Primary lactose intolerance (adult-type hypolactasia) is extremely common, occurring in approximately 20% of white adults, 80% of African American adults, and 90% of Asian adults. It is uncommon in children younger 5 years of age, but as children grow, those who are genetically predisposed to lactose intolerance may develop the condition gradually during their school age years. Lactose intolerance may be diagnosed clinically by elimination diet and rechallenge or through lactose breath hydrogen testing. Breath testing offers the advantage of providing a more definitive diagnosis, so milk is not withdrawn from the diet unnecessarily. In a breath hydrogen test, an adult is given approximately 50 g of lactose (the equivalent of 4 cups of milk) as a single dose. Patients who cannot digest and absorb this amount of lactose have the lactose fermented to hydrogen by intestinal bacteria, which can be measured in the breath. Because intestinal bacterial fermentation is essential for an accurate test, patients should not receive antibiotics for at least 2 weeks prior to the test. Many patients who have lactose intolerance diagnosed on breath test can tolerate small amounts of lactose in their diet. Specifically, hard cheeses, such as cheddar, which often have very little lactose (<0.5 g/serving), can be eaten by many patients who have lactose intolerance. Other products, such as whole or skim milk, ice cream, or buttermilk, have more lactose (approximately 10 g/serving) and are less likely to be tolerated (Item C129).

References:

Heyman MB for American Academy of Pediatrics Committee on Nutrition. Lactose intolerance in infants, children, and adolescents. Pediatrics. 2006;118:1279-1286. Available at: http://pediatrics.aappublications.org/cgi/content/full/118/3/1279

Thiessen PN. Recurrent abdominal pain. Pediatr Rev. 2002;23:39-46. Available at: http://pedsinreview.aappublications.org/cgi/content/full/23/2/39

Page 453: AAP PREP 2009

2009 PREP SA on CD-ROM

page 453

Critique: 129

Page 454: AAP PREP 2009

2009 PREP SA on CD-ROM

page 454

Question: 130

You are treating a former extremely low-gestational age newborn (ELGAN) who was born at 26 weeks’ gestation weighing 700 g. She is now 4 weeks old. Her nurse asks when the eye examination for retinopathy of prematurity (ROP) will be performed and what risk for significant visual impairment exists in this infant.

Of the following, the BEST time to obtain the first ROP screening eye examination in this infant is

A. 4 weeks after discharge from the hospital

B. 4 weeks after weaning from oxygen

C. 5 weeks after birth

D. 5 weeks after the expected delivery date

E. 5 weeks after weaning from the ventilator

Page 455: AAP PREP 2009

2009 PREP SA on CD-ROM

page 455

Critique: 130 Preferred Response: C

Retinopathy of prematurity (ROP) is a multifactorial disorder that has been linked historically and epidemiologically to the survival of low-birthweight preterm infants. As the survival of smaller and more immature infants has increased throughout the past 5 decades, the incidence of ROP has shifted toward the smallest, most immature, and sickest babies in neonatal intensive care units (NICUs) in the United States and worldwide. Prospective studies and critical reviews and analyses of extensive data confirm that the greatest known risks for ROP among NICU patients include: very low birthweight (<1,500 g), severity of illness, gestational immaturity (especially <28 weeks' gestational age), intrauterine growth restriction, male sex, and systemic fungal infection. Among the additional possible risks are antenatal dexamethasone exposure (compared to betamethasone) and late treatment with dexamethasone (treatment of infants more than 3 weeks of age for 2 or more weeks). The American Academy of Pediatrics recommends a specific timetable for testing at-risk preterm infants (Item C130). In general, infants should be tested between 31 and 34 weeks gestational age. Accordingly, the infant described in the vignette should be tested 5 weeks after birth. Because most 26 weeks' gestation infants are discharged from the hospital at a gestational equivalent of 34 to 36 weeks, or 8 to 10 postnatal weeks of age, this would be too late for an initial examination, as would one occurring 4 weeks later. Many 26 weeks' gestation infants may have chronic lung disease and be receiving assisted ventilation for days to weeks or be discharged from the hospital on oxygen, so screening is not based on the time of weaning from assisted ventilation or oxygen. Five weeks after the expected delivery date is too late for initial screening.

References:

Bharwani SK, Dhanireddy R. Systemic fungal infection is associated with the development of retinopathy of prematurity in very low birth weight infants: a meta-review. J Perinatol. 2007;28:61-66. Abstract available at: http://www.ncbi.nlm.nih.gov/pubmed/18046338

Darlow BA, Hutchinson JL, Henderson-Smart DJ, Donoghue DA, Simpson JM, Evans NJ on behalf of the Australian and New Zealand Neonatal Network. Prenatal risk factors for severe retinopathy of prematurity among very preterm infants of the Australian and New Zealand Neonatal Network. Pediatrics. 2005;115:990-996. Available at: http://pediatrics.aappublications.org/cgi/content/full/115/4/990

Hagadorn JI, Richardson DK, Schmid CH, Cole CH. Cumulative illness severity and progression from moderate to severe retinopathy of prematurity. J Perinatol. 2007;27:502-509. Abstract available at: http://www.ncbi.nlm.nih.gov/pubmed/17568754

Karlowicz MG, Giannone PJ, Pestian J, Morrow AL, Shults J. Does candidemia predict threshold retinopathy of prematurity in extremely low birth weight (<1000 g) neonates? Pediatrics. 2000;105:1036-1040. Available at: http://pediatrics.aappublications.org/cgi/content/full/105/5/1036

Lee BH, Stoll BJ, McDonald SA, Higgins RD for the National Institute of Child Health and Human Development Neonatal Research Network. Adverse neonatal outcomes associated with antenatal dexamethasone versus antenatal betamethasone. Pediatrics. 2006;117:1503-1510. Available at: http://pediatrics.aappublications.org/cgi/content/full/117/5/1503

Markestad T, Kaaresen PI, Rønnestad A, et al on behalf of the Norwegian Extreme Prematurity Study

Page 456: AAP PREP 2009

2009 PREP SA on CD-ROM

page 456

Group. Early death, morbidity, and need of treatment among extremely premature infants. Pediatrics. 2005;115:1289-1298. Available at: http://pediatrics.aappublications.org/cgi/content/full/115/5/1289

Section on Ophthalmology, American Academy of Pediatrics, American Academy of Ophthalmology, American Association for Pediatric Ophthalmology and Strabismus. Screening examination of premature infants for retinopathy of prematurity. Pediatrics. 2006;117:572-576. Available at: http://pediatrics.aappublications.org/cgi/content/full/117/2/572

Page 457: AAP PREP 2009

2009 PREP SA on CD-ROM

page 457

Critique: 130

Reprinted with permission from the Section on Ophthalmology, American Academy of Pediatrics, American Academy of Ophthalmology, American Association for Pediatric Ophthalmology and Strabismus. Screening examination of premature infants for retinopathy of prematurity. Pediatrics. 2006; 117:572-576.

Page 458: AAP PREP 2009

2009 PREP SA on CD-ROM

page 458

Question: 131

A 15-year-old boy who has mild persistent asthma is brought to the emergency department because of increased work of breathing of 1 day’s duration. He reports a low-grade fever and nonproductive cough for the past 4 days, but this morning he developed difficulty breathing and a cough that produced a small amount of yellowish sputum. His respiratory rate is 24 breaths/min, heart rate is 80 beats/min, and temperature is 99.0°F (37.3°C). He appears in no respiratory distress, but his lung examination reveals bilateral rales and occasional wheezes. A chest radiograph shows bilateral diffuse infiltrates with no effusions.

Of the following, the MOST likely etiologic agent causing his symptoms is

A. Haemophilus influenzae

B. Mycobacterium tuberculosis

C. Mycoplasma pneumoniae

D. Staphylococcus aureus

E. Streptococcus pneumoniae

Page 459: AAP PREP 2009

2009 PREP SA on CD-ROM

page 459

Critique: 131 Preferred Response: C

Mycoplasma pneumoniae is a common cause of community-acquired pneumonia in school-age children and adolescents, but it is rare in infants and toddlers. Other clinical manifestations of M pneumoniae infection include upper respiratory tract infection, pharyngitis, bullous myringitis, and bronchitis. Occasionally, systemic symptoms such as malaise and fever are present. Among the rarer clinical presentations are hemolytic anemia, aseptic meningitis, and Stevens-Johnson syndrome. M pneumoniae is responsible for many cases of pneumonia and acute chest syndrome in children who have sickle cell disease. Infection also may precipitate an exacerbation of asthma, as described for the boy in the vignette. A recent study showed that children of different ages may respond differently to infection with M pneumoniae. Children younger than 5 years of age were more likely to experience coryza, vomiting and diarrhea, and tachypnea than were older children. Typical radiographic findings are bilateral, diffuse infiltrates (Item C131A), but occasionally a lobar infiltrate or pleural effusion may be seen. The diagnosis frequently can be made clinically, but serologic testing is available. The complement fixation test and, more recently, immunofluorescence and enzyme immunoassay can be used to measure levels of immunoglobulin M (IgM) and IgG antibodies to M pneumoniae. High concentrations of IgM may be present for months after acute infection. Comparing acute and convalescent IgG titers allows for more accurate diagnosis, although this may be impractical. Serum cold hemagglutinin titers also may be positive in patients who have acute infection but have limited sensitivity and specificity. Direct polymerase chain reaction is both sensitive and specific for the diagnosis but is not widely available. Macrolide antibiotics generally are used for treatment, although many infections are self-limited, and a recent review found no conclusive evidence that antibiotics are effective in reducing symptoms. Haemophilus influenzae and Streptococcus pneumoniae also cause community-acquired pneumonia, but fever generally is present with these infections, and radiographs typically reveal lobar infiltrates (Item C131B) with or without pleural effusions. Lobar infiltrates also are typical of pneumonia caused by Staphylococcus aureus, and pneumatoceles (Item C131C) or lung abscesses are common with this infection. The radiologic features of M tuberculosis infection are varied and include lobar infiltrates, hilar adenopathy, and cavitary lesions (Item C131D).

References:

American Academy of Pediatrics. Mycoplasma pneumoniae infections. In: Pickering LK, Baker CJ, Long SS, McMillan JA, eds. Red Book: 2006 Report of the Committee on Infectious Diseases. 27th ed. Elk Grove Village, Ill: American Academy of Pediatrics; 2006:468-470

Gavranich JB, Chang AB. Antibiotics for community acquiredlower respiratory tract infections (LRTI) secondary to Mycoplasma pneumoniae in children. Cochrane Database Syst Rev. 2005;3:CD004875. Available at: http://www.mrw.interscience.wiley.com/cochrane/clsysrev/articles/CD004875/frame.html

Othman N, Isaacs D, Kesson A. Mycoplasma pneumoniae infections in Australian children. J Paediatr Child Health. 2005;41:671-676. Abstract available at: http://www.ncbi.nlm.nih.gov/pubmed/16398873

Page 460: AAP PREP 2009

2009 PREP SA on CD-ROM

page 460

Critique: 131

Anteroposterior radiograph of the chest in Mycoplasma pneumoniae pneumonia shows widespread streaky lung opacities consistent with areas of atelectasis and infiltrates. (Courtesy of D. Mulvihill)

Page 461: AAP PREP 2009

2009 PREP SA on CD-ROM

page 461

Critique: 131

Lingular infiltrate in an 8-month-old infant who has bacterial pneumonia. (Courtesy of D. Krowchuk)

Page 462: AAP PREP 2009

2009 PREP SA on CD-ROM

page 462

Critique: 131

Anteroposterior radiograph of the chest demonstrates consolidation in the right upper lobe and focal air collection within lung parenchyma consistent with a pneumatocele (arrow). (Courtesy of D. Mulvihill)

Page 463: AAP PREP 2009

2009 PREP SA on CD-ROM

page 463

Critique: 131

Anteroposterior chest radiograph in tuberculosis shows extensive mediastinal, paratracheal (red arrow), and hilar adenopathy with cavitary changes in the right lung (yellow arrow). (Courtesy of D. Krowchuk)

Page 464: AAP PREP 2009

2009 PREP SA on CD-ROM

page 464

Question: 132

A resident in continuity clinic approaches you to review the laboratory values obtained at a patient’s 12-month health supervision visit. The fingerstick hemoglobin measurement was 10.5 g/dL (105.0 g/L), and the lead concentration was 11.0 mcg/dL (0.53 mcmol/L).

Of the following, the next BEST step for this boy is to

A. admit him to the hospital for chelation therapy

B. call child protective services to move him to a shelter

C. call the health department to arrange for an environmental investigation

D. measure the venous lead concentration

E. refer him for formal developmental evaluation and neuropsychometric testing

Page 465: AAP PREP 2009

2009 PREP SA on CD-ROM

page 465

Critique: 132 Preferred Response: D

Routine screening of children for exposure to environmental lead combined with environmental abatement has resulted in an overall decrease in lead toxicity among children living in the United States. However, the laboratory findings of mild anemia and a capillary blood lead concentration that is marginally elevated, as described for the child in the vignette, are still common. Repeat measurement of the blood lead by venipuncture to avoid skin contamination as well as further evaluation of the child's mild anemia are appropriate next steps. In 1997, as a result of the decrease in lead exposure and prevalence of lead poisoning, the Centers for Disease Control and Prevention (CDC) updated its screening guidelines and incorporated, for the first time, screening policies that use local blood lead concentration data or housing data collected by the United States Bureau of Census to determine risk. In geographic areas where limited exposure to lead and limited prevalence of elevated blood lead concentrations are documented, targeted (selective) screening may be more appropriate than universal screening. However, one of the goals of Healthy People 2010 is the elimination of childhood lead poisoning as a public health problem. Current guidelines state that health-care professionals should use blood lead tests to screen children at ages 1 and 2 years and children 36 to 72 months of age who previously had not been screened if they meet specific criteria. The CDC and state and local health departments continue to develop surveillance and intervention tools to determine the best practices for targeted screening. Pediatricians may consult the CDC Web site and their local health departments for zip code-related risk levels as well as questionnaires to assess individual children's risk factors based on history of housing or other exposures. Screening of toddlers for lead exposure continues to be recommended and even mandated by health-care organizations. The American Academy of Pediatrics policy statement recommends the use of venous samples for initial screening whenever possible. If capillary testing is performed and the lead concentration is greater than 10.0 mcg/dL (0.5 mcmol/L), the lead concentration must be confirmed by a venous sample because microlead sampling is more likely to yield false-positive results due to contamination from environmental lead. Lead ingestion may cause a microcytic anemia by interfering with iron absorption and utilization in heme production and can inhibit enzymes required for heme synthesis directly. Children who have lead poisoning may have pica either as a cause or symptom of lead poisoning. In these children, iron supplementation should be initiated until the presence or absence of iron deficiency is determined. In many communities, formal environmental evaluation of the child's home or removal of the child and family from that home is not conducted until the blood lead value has been confirmed and is higher than 20.0 mg/dL (1.0 mcmol/L). Although the child's developmental status should be monitored carefully in the future as a part of recommended surveillance and screening, formal neurodiagnostic testing is not indicated at this time unless there are other risk factors for developmental delay. Counseling the family about potential sources of lead in the environment (eg, dirt, cosmetics, or household items) and lead solder in old pipes may be beneficial. Most asymptomatic children who have mildly elevated blood lead concentrations are not candidates for chelation therapy with the currently available drugs because the toxicity of these drugs outweighs the potential benefit of treatment, and chelation is unlikely to increase lead excretion significantly. Chelation therapy should be considered, however, if lead concentrations are higher than 44.0 mcg/dL (2.12 mcmol/L). The role of chelation is not clearly defined for children whose blood lead concentrations range from 20.0 to 45.0 mcg/dL (0.97 to 2.17 mcmol/L). In this range, the clinician may choose to pursue further environmental screening, attempt to eradicate lead from the child's environment, and measure blood lead concentrations monthly. If the concentration remains in this range, despite successful eradication of the lead source, the physician should institute behavior modification, nutritional sufficiency, or chelation treatment.

Page 466: AAP PREP 2009

2009 PREP SA on CD-ROM

page 466

Succimer is the drug of choice for children whose blood lead concentrations are 45.0 to 100.0 mcg/dL (2.17 to 4.8 mcmol/L). At values higher than 69.0 mcg/dL (3.3 mcmol/L), a second drug, CaNa2EDTA, is added. For children who require two-drug treatment, the first dose always is succimer, followed 4 hours later by CaNa2EDTA, because children who present with lead encephalopathy may deteriorate when treated with CaNa2EDTA alone. D-penicillamine is not recommended as first-line therapy because adverse effects and allergy are common (33%). Dimercaprol is not a first-line drug because the rate of adverse effects approaches 50%. Serious adverse effects of chelation therapy may occur due to chelation of other electrolytes. NaEDTA has been associated with fatal hypocalcemia and, therefore, is contraindicated. The clinician should be careful not to confuse CaNa2EDTA with NaEDTA. Chelation therapy should be conducted in conjunction with a pediatric toxicologist or pharmacist under very close supervision. Oral agents may be used, but succimer is not palatable and must be emptied from a capsule onto food. Once lead has been ingested, the percentage absorbed may be modified by essential nutrients. A healthy diet can be recommended for both lead-exposed and nonexposed children. Particular attention must be given to calcium and iron intake. If iron deficiency is diagnosed, supplemental iron should be prescribed. For children who are not iron-deficient, a multivitamin with iron can be recommended, but its efficacy is unproven. Similarly, no published data support a role for therapeutic administration of calcium or iron as treatment for lead poisoning in the absence of deficiency. Such studies are being conducted. Blood lead concentrations fall precipitously after completion of chelation, but rebound within weeks, even if there is no further exposure to lead, due to release of lead from bone stores. In general, the concentrations do not return to the high values seen before chelation, and a second course of chelation rarely is indicated.

References:

Binns,HJ, Campbell,C, Brown,MJ for the Advisory Committee on Childhood Lead Poisoning Prevention. Interpreting and managing blood lead levels of less than 10 mcg/dL in children and reducing childhood exposure to lead: recommendations of the Centers for Disease Control and Prevention Advisory Committee on Childhood Lead Poisoning Prevention. Pediatrics. 2007;120:e1285-e1298. Available at: http://pediatrics.aappublications.org/cgi/content/full/120/5/e1285

Centers for Disease Control and Prevention. Lead program: state and local programs. Available at: http://www.cdc.gov/nceh/lead/grants/contacts/CLPPP%20Map.htm

Laraque D, Trasande L. Lead poisoning: successes and 21st century challenges. Pediatr Rev. 2005;26:435-443. Available at: http://pedsinreview.aappublications.org/cgi/content/full/26/12/435

Markowitz M. Lead poisoning. Pediatr Rev. 2000;21:327-335. Available at: http://pedsinreview.aappublications.org/cgi/content/full/21/10/327

Richardson M. Microcytic anemia. Pediatr Rev. 2007;28:5-14.

Rischitelli G, Nygren P, Bougatsos C, Freeman M, Helfand M. Screening for Elevated Lead Levels in Childhood and Pregnancy: An Update of a 1996 U.S. Preventive Services Task Force Review. Rockville, Md: Agency for Healthcare Research and Quality, U.S. Department of Health and Human Services; 2006. Available at: http://www.ahrq.gov/clinic/uspstf06/lead/leadsum.htm

Page 467: AAP PREP 2009

2009 PREP SA on CD-ROM

page 467

Yuan W, Holland SK, Cecil KM, et al. The impact of early childhood lead exposure on brain organization: a functional magnetic resonance imaging study of language function. Pediatrics. 2006;118:971-977. Available at: http://pediatrics.aappublications.org/cgi/content/full/118/3/971

Page 468: AAP PREP 2009

2009 PREP SA on CD-ROM

page 468

Question: 133

You are evaluating a 15-year-old boy in the emergency department who presents with fever, chills, malaise, and blood in his urine. On physical examination, he appears comfortable and alert and has a temperature of 102.7°F (39.3°C), a blood pressure of 110/40 mm Hg, no rashes, and clear breath sounds. He has a diastolic murmur heard best in the sitting position (Item Q133). You elicit no abdominal or flank tenderness.

Of the following, the BEST next step in the management of this patient is

A. administration of broad-spectrum antibiotics

B. blood cultures

C. renal ultrasonography

D. transesophageal echocardiography

E. urine culture

Page 469: AAP PREP 2009

2009 PREP SA on CD-ROM

page 469

Critique: 133 Preferred Response: B

The patient described in the vignette has history and physical examination findings that are highly suggestive of infective endocarditis. These include symptoms of chills and malaise; a history of fever; and the findings of hematuria, a new murmur, and fever. Typically, the diagnosis is confirmed by isolation of the offending organism from blood cultures. Blood cultures from three to five sites should be obtained prior to initiation of antibiotic therapy. Because the bacterial shedding is constant, the practitioner should not wait until the patient is febrile to obtain blood cultures. Viridans streptococci (eg, S bovis, S mitis) as well as Staphylococcus aureus are the most common bacterial pathogens causing endocarditis in children. However, clinicians must be concerned about organisms such as Enterococcus, coagulase-negative Staphylococcus, fungi, and a group of bacteria referred to as the HACEK organisms (Haemophilus sp, Actinobacillus actinomycetemcomitans, Cardiobacterium hominis, Eikenella corrodens, and Kingella kingae). The HACEK organisms are gram-negative oral and pharyngeal flora that are fastidious and slow-growing, often requiring growth factors and carbon dioxide to be isolated in cultures. Treatment of endocarditis depends on the isolated organism. In general, long-term antibiotic treatment (4 to 6 weeks) is undertaken in an effort to eradicate completely the bacteria that have been sequestered in a nonvascular vegetation. Surgery is reserved for patients who develop severe congestive heart failure from severe valve regurgitation or deterioration. The boy in the vignette requires intravenous antibiotic treatment, but blood cultures should be obtained before therapy is begun. He also should undergo echocardiography, which may be performed from the transesophageal approach to improve the sensitivity, but similar to renal ultrasonography, such a study is performed after blood cultures have been obtained. The absence of vegetation at the time of echocardiography does not rule out a diagnosis of infective endocarditis. Patients who have infective endocarditis may exhibit hematuria from the deposition of immune complexes resulting in glomerulonephritis. Although fever and hematuria may be associated with urinary tract infection, the presence of a diastolic murmur and absence of urinary symptoms make such a diagnosis unlikely.

References:

Baltimore RS. Infective endocarditis in children. Pediatr Infect Dis J. 1992;11:907-912. Abstract available at: http://www.ncbi.nlm.nih.gov/pubmed/1454430

Hoyer A, Silberbach M. Infective endocarditis. Pediatr Rev. 2005;26:394-400. Available at: http://pedsinreview.aappublications.org/cgi/content/full/26/11/394

Taubert KA, Dajani AS. Infective endocarditis. In: Moller JH, Hoffman JIE, eds. Pediatric Cardiovascular Medicine. Philadelphia, Pa: Churchill Livingstone; 2000:768-779

Page 470: AAP PREP 2009

2009 PREP SA on CD-ROM

page 470

Question: 134

A 6-year-old boy has had difficulty walking and lower leg pain for 2 days. Five days ago, he had fever and cough that had lasted for 3 days. On physical examination, the child has no fever, and vital signs are normal, as are cranial nerves, speech, and language. Muscle bulk, tone, and reflexes are normal and symmetric, but his lower legs are painful to palpation. Serum creatine kinase is 2,000 U/L, and urine is negative for myoglobin.

Of the following, the MOST likely diagnosis is

A. dermatomyositis

B. Duchenne muscular dystrophy

C. Guillain-Barré syndrome

D. metabolic myopathy

E. viral myositis

Page 471: AAP PREP 2009

2009 PREP SA on CD-ROM

page 471

Critique: 134 Preferred Response: E

A gait disturbance, such as described for the boy in the vignette, can result from a variety of potentially serious disease processes and, therefore, requires urgent evaluation. The muscle pain and otherwise normal neurologic findings prompted measurement of serum creatine kinase, which can help to localize the problem rapidly. The prodrome of an upper respiratory tract illness and rapid onset of symptoms is suggestive of viral myositis. Dermatomyositis is a more indolent, chronic process and should not present acutely with muscle pain. Moreover, dermatomyositis is characterized by specific skin findings such as the heliotrope rash over the eyelids (Item C134A) and Gottron papules (Item C134B). Duchenne muscular dystrophy presents with more chronic weakness. Although Guillain-Barré syndrome can present with pain and weakness, the preserved reflexes, focal pain over the leg muscles, and elevated creatine kinase value are not consistent with that diagnosis. Metabolic myopathies due to mitochondrial dysfunction can present with acute pain, weakness, tender muscles, and rhabdomyolysis. However, the prevalence is much lower than viral myositis. Influenza A and B and enteroviruses may cause viral myositis. Other causes are in Item C134C.

References:

Moughan B. Musculoskeletal symptom complexes. In: Long SS, Pickering LK, Prober CG, eds. Principles and Practice of Pediatric Infectious Diseases. New York, NY: Churchill Livingstone; 2003:150-159

Pasternack MS, Swartz MN. Myositis. In: Mandell GL, Bennett JE, Dolin R, eds. Mandell, Douglas, and Bennett's Principals and Practice of Infectious Diseases. 6th ed. Philadelphia, Pa: Elsevier Churchill Livingstone; 2005:1194-1203

Roos KL. Viral infections. In: Goetz CG, ed. Textbook of Clinical Neurology. 3rd ed. Saunders Elsevier; 2007:chapt 41

Page 472: AAP PREP 2009

2009 PREP SA on CD-ROM

page 472

Critique: 134

Violaceous discoloration of the cheeks and eyelids (ie, heliotrope) in a child who has dermatomyositis. (Courtesy of J. Jorizzo)

Page 473: AAP PREP 2009

2009 PREP SA on CD-ROM

page 473

Critique: 134

Gottron papules: Erythematous scaling papules overlying the knuckles in a child who has dermatomyositis. (Courtesy of D. Krowchuk)

Page 474: AAP PREP 2009

2009 PREP SA on CD-ROM

page 474

Critique: 134

Page 475: AAP PREP 2009

2009 PREP SA on CD-ROM

page 475

Question: 135

The pregnant mother of a child in your practice recently learned that her grandmother had a child who died of "probable metabolic disease" at 2 days of age. She does not know details, and medical records on that child no longer are available. The mother asks if her pregnancy can be tested to see if the fetus could be affected with the same disorder.

Of the following, the MOST accurate statement regarding metabolic disease in the prenatal setting is that

A. fetuses affected with metabolic diseases are unlikely to come to term

B. knowing the parents’ ethnic backgrounds aids in determining which tests should be offered

C. level 2 ultrasonography during the second trimester is likely to be helpful in detecting metabolic disease

D. poor fetal growth is common in metabolic diseases

E. prenatal metabolic screening panels are widely available

Page 476: AAP PREP 2009

2009 PREP SA on CD-ROM

page 476

Critique: 135 Preferred Response: B

Most inborn errors of metabolism do not become evident until after birth. Typically, affected fetuses are normally formed and normally grown, and no abnormalities are detectable on prenatal ultrasonography. With rare exception, maternal serum screening does not detect metabolic abnormalities in the fetus. Usually, affected individuals are born at term. Although most of the United States has extended newborn screening panels to detect a variety of inborn errors, no such panel exists for routine screening during pregnancy. The vignette highlights the importance of accessing as much information as possible regarding family members who die from metabolic causes. Even if a diagnosis cannot be made in a particular instance, medical records may be helpful in offering the expectant couple guidance or in making a diagnosis posthumously. Taking a careful family history, with attention to familial ethnic, religious, and geographic origins, may bring to light conditions that are likely to run in families. For example, because of the high carrier rate for some metabolic diseases, parents of Ashkenazi Jewish descent, whose ancestors originated from Eastern Europe, are at increased risk for having children affected by Tay-Sachs disease (Item C135) and people of French-Canadian heritage are at increased risk for having children affected by tyrosinemia. More than 100 metabolic disorders can be diagnosed prenatally if clinicians have sufficient information available to know which disorder may occur in a family.

References:

Driscoll DA, Sehdev HM, Marchiano DA. Prenatal carrier screening for genetic conditions. NeoReviews. 2004;5:e290-e295. Available for subscription at: http://neoreviews.aappublications.org/cgi/content/full/5/7/e290

Prenatal diagnosis: emerging technologies for prenatal diagnosis. In: Nussbaum RL, McInnes RR, Willard HF, eds. Thompson & Thompson Genetics in Medicine. 7th ed. Philadelphia, Pa: Elsevier Saunders; 2007:456

Page 477: AAP PREP 2009

2009 PREP SA on CD-ROM

page 477

Critique: 135

Page 478: AAP PREP 2009

2009 PREP SA on CD-ROM

page 478

Question: 136

A 17-year-old young man comes to your office for a preparticipation sports physical examination for high school wrestling. He reports that his coach would like him to wrestle in a weight category that is 10 lb less that his current weight. After you determine that his desired weight is in the range of healthy weight for his height, you counsel him regarding safe weight loss.

Of the following, the MOST appropriate statement regarding healthy weight control practices for young athletes is that

A. an appropriate diet for most athletes consists of a minimum of 1,500 kcal/day

B. dehydration causes greater body heat storage, reduces blood volume, and results in increased exercise tolerance

C. most high school boys who participate in "weight-sensitive" sports practice unhealthy weight loss behaviors

D. optimal values for body composition have been established for all sports

E. weight loss beyond 1.5% of body weight per week results in muscle weakness

Page 479: AAP PREP 2009

2009 PREP SA on CD-ROM

page 479

Critique: 136 Preferred Response: E

Sports in which a lean body mass is desirable or competing at the lowest possible weight is seen as an advantage include distance running, cross country skiing, swimming and diving, dance, figure skating, gymnastics, and weight-class football and wrestling. Other sports, such as football, rugby, basketball, and power lifting, emphasize weight gain in the form of lean body mass. Whether attempting to lose weight and body fat or gain weight and muscle mass, some youth may resort to unhealthy dieting and exercise practices, supplement use, and drug ingestion. Some practices, including rapid weight loss with voluntary dehydration, the use of anabolic steroids, and inappropriate use of stimulants or insulin, may be fatal. Inappropriate weight loss also may result in medical complications such as delayed physical maturation and potential growth impairment; oligomenorrhea and amenorrhea; development of eating disorders; depression; an increased incidence of infectious diseases; and renal, endocrine, gastrointestinal, and thermoregulatory disturbances. Disordered eating behaviors are prevalent in both female and male athletes. It is reported that 10% to 15% of high school boys in "weight-sensitive sports" practice unhealthy weight loss behaviors, with numerous studies reporting such behaviors in wrestlers. Voluntary dehydration frequently is used to lose weight, and techniques can include fluid restriction, spitting, use of laxatives and diuretics, wearing of rubber suits during activities to cause sweating, and excessive sweating in saunas and steam baths. Dehydration results in greater body heat storage, causing an excessive increase in the core body temperature and a decreased blood volume, and decreased exercise tolerance. It also increases the risk of heat-related illness. No optimal values for body composition have been established for any sport. Rather than a specific percentage of body fat for an individual athlete, a range that is realistic and appropriate should be recommended. If it is appropriate that an adolescent loses weight, healthy weight loss should not exceed 1.5% of the total body weight per week or 1 to 2 lb/wk. Weight loss beyond these guidelines results in the breakdown and metabolism of muscle, causing muscular weakness. An appropriate diet for most athletes consists of a minimum of 2,000 kcal/day. The ideal method of losing weight is to consume 1,750 fewer kcal per week and expend 1,750 kcal more per week by exercising. A well-balanced diet that contains approximately 55% to 65% of calories from carbohydrates, 15% to 20% of calories from protein, and 20% to 30% of calories from fat is recommended.

References:

American Academy of Pediatrics Council on Sports Medicine and Fitness and Council on School Health. Active healthy living: prevention of childhood obesity through increased physical activity. Pediatrics. 2006;117:1834-1842. http://pediatrics.aappublications.org/cgi/content/full/117/5/1834

American Academy of Pediatrics Committee on Sports Medicine and Fitness. Promotion of healthy weight-control practices in young athletes. Pediatrics. 2005;116:1557-1564. Available at: http://pediatrics.aappublications.org/cgi/content/full/116/6/1557

McCrory P, Johnston K, Meeuwisse W, et al. Summary and agreement statement of the 2nd International Conference on Concussion in Sport, Prague 2004. Br J Sports Med. 2005;39:196-204. Abstract available at: http://www.ncbi.nlm.nih.gov/pubmed/15793085

Page 480: AAP PREP 2009

2009 PREP SA on CD-ROM

page 480

Question: 137

You are evaluating a 2-week-old breastfed infant who is 15% below his birthweight and has been lethargic and fed poorly over the past 4 days. You administer a normal saline fluid bolus. Laboratory results include: · Blood glucose of 126.0 mg/dl (7.0 mmol/L)· Serum sodium of 170.0 mEq/L (170.0 mmol/L)· Serum potassium of 5.0 mEq/L (5.0 mmol/L)· Blood urea nitrogen of 31.0 mg/dL (11.1 mmol/L)· Serum creatinine of 2.9 mg/dL (256.4 mcmol/L)

Of the following, the MOST appropriate initial fluid for correction is a solution containing 5% dextrose and

A. NaCl (%): 0.20

KCl (mEq/L): 40Duration of Infusion(hr): 12 to 24B. NaCl (%): 0.45

KCl (mEq/L): 0Duration of Infusion(hr): 48 to 72C. NaCl (%): 0.45

KCl (mEq/L): 40Duration of Infusion(hr): 12 to 24D. NaCl (%): 0.9

KCl (mEq/L): 0Duration of Infusion(hr): 12 to 24E. NaCl (%): 0.9

KCl (mEq/L): 40Duration of Infusion(hr): 48 to 72

Page 481: AAP PREP 2009

2009 PREP SA on CD-ROM

page 481

Critique: 137 Preferred Response: B

Hypernatremia (serum sodium concentration greater than 145.0 mEq/L [145.0 mmol/L]) is common in hospitalized children. Hypernatremia results from excessive sodium administration (incorrectly mixed formula, sodium bicarbonate, hypertonic saline) or a deficit of water in relation to sodium. Such a deficit may occur from decreased water intake (inadequate breastfeeding, fluid restriction, lack of access to fluids), fluid losses (nephrogenic and central diabetes insipidus, increased insensible losses), or fluid losses in greater proportion than sodium losses (diarrhea, vomiting, diuretic use, burns). Treatment of hypernatremia is directed at correction of both the serum sodium concentration and the circulatory volume. Initially, some children may require isotonic fluid boluses to restore circulation. It is important to note that children who have hypernatremia often appear less dehydrated than they actually are due to preservation of the extracellular volume. The following equation can be used to estimate the free water deficit: Water deficit (mL)= 4 mL x ideal body weight (kg) x desired change in serum sodium concentration Hypernatremia, especially if chronic, should be corrected slowly, with a desired goal of decreasing the serum sodium by 0.5 mEq/L per hour to avoid cerebral edema. Severe hypernatremia (serum sodium >170.0 mEq/L [170.0 mmol/L]), as described for the child in the vignette, should be corrected over 48 to 72 hours. Fluid administration generally consists of 1/4 to 1/2 normal saline solutions. Symptoms of overcorrection, such as changes in mental status or onset of seizures, suggest the development of cerebral edema and should be treated with hypertonic saline and slowing of the sodium correction. In general, potassium administration should be withheld in cases of severe hypernatremic dehydration until adequate urine output is assured.

References:

Greenbaum LA. Electrolyte and acid-base disorders. In: Kliegman RM, Behrman RE, Jenson HB, Stanton BF, eds. Nelson Textbook of Pediatrics. 18th ed. Philadelphia, Pa: Saunders Elsevier; 2007:267-308

Moritz ML, Ayus JC. Disorders of water metabolism in children: hyponatremia and hypernatremia. Pediatr Rev. 2002:23:371-380. Available at: http://pedsinreview.aappublications.org/cgi/content/full/23/11/371

Page 482: AAP PREP 2009

2009 PREP SA on CD-ROM

page 482

Question: 138

You are asked to see a 7-year-boy in whom medulloblastoma (primitive neuroectodermal tumor) was diagnosed at age 3 years. Treatment at that time consisted of chemotherapy and craniospinal irradiation. During the past year, he grew 2 cm, although he is eating normally, and his weight is appropriate for height. Despite spinal irradiation, the upper-to-lower segment ratio is normal for his age.

Of the following, the MOST likely diagnosis is

A. acquired growth hormone deficiency

B. chemotherapy-induced renal failure

C. Cushing syndrome

D. irradiation-induced epiphyseal fusion

E. tumor recurrence

Page 483: AAP PREP 2009

2009 PREP SA on CD-ROM

page 483

Critique: 138 Preferred Response: A

Cranial irradiation has gradual deleterious effects on pituitary hormone secretion due to damage to hypothalamic releasing factors. Growth hormone, thyroid-stimulating hormone (TSH), and the gonadotropins tend to be affected most, with adrenocorticotropic hormone (ACTH) relatively protected, although all neuroendocrine function can be disturbed by cranial irradiation. The boy described in the vignette has a normal upper-to-lower segment ratio, indicating that his spine still is growing despite radiation. This suggests that his growth attenuation is due to an acquired endocrine deficiency of either growth hormone or thyroid hormone. Early in the deficiency process, if this child were tested with standard stimuli to growth hormone release, he might be able to release growth hormone, but if continuous nocturnal growth hormone sampling were performed, diminished and disorderly spontaneous growth hormone release might be documented. With time, secretory function to stimuli also would be lost. Renal failure from chemotherapy would be expected to have manifested earlier and been recognized by this time. Cushing syndrome is a possible but rare cause of growth attenuation caused by adrenal or pituitary oversecretion of cortisol or ACTH, respectively, and is not associated with cranial irradiation. Irradiation-induced epiphyseal fusion at the level of the spine is common following spinal irradiation. The epiphyseal fusion of the spine results in an increased lower segment (measured from the pubic symphysis to the heel) compared with the upper segment because the legs continue to grow while the spine does not. This results in an increased lower segment-to-upper segment ratio. Tumor recurrence has more obvious manifestations than statural growth attenuation.

References:

Ballonoff A, Kavanagh B. Complications of cranial irradiation. UpToDate Online 15.3. 2008. Available for subscription at: http://www.uptodateonline.com/utd/content/topic.do?topicKey=rad_ther/2462

Darzy KH, Pezzoli SS, Thorner MO, Shalet SM. The dynamics of growth hormone (GH) secretion in adult cancer survivors with severe GH deficiency acquired after brain irradiation in childhood for nonpituitary brain tumors: evidence for preserved pulsatility and diurnal variation with increased secretory disorderliness. J Clin Endocrinol Metab. 2005;90:2794-2803. Available at: http://jcem.endojournals.org/cgi/content/full/90/5/2794

Rose SR. Growth failure after childhood cancer: role of growth hormone deficiency. UpToDate online 15.3. 2008. Available for subscription at: http://www.uptodateonline.com/utd/content/topic.do?topicKey=ped_onco/2817

Page 484: AAP PREP 2009

2009 PREP SA on CD-ROM

page 484

Question: 139

A 24-month-old child has been evaluated and found eligible for early intervention services because of language delay. His mother is reluctant to pursue therapy because she feels that his language will improve without intervention.

Of the following, the MOST appropriate response is to

A. agree with the mother that his speech probably will improve without therapy

B. explain that therapy at this age is parent-based training to promote appropriate development in the home setting

C. explain that therapy is critical for the child or the child will have regression in her language development

D. postpone therapy pending re-evaluation in 6 months

E. warn the mother that if she does not enroll the child in therapy, she can be reported for child neglect

Page 485: AAP PREP 2009

2009 PREP SA on CD-ROM

page 485

Critique: 139 Preferred Response: B

Federal legislation for the provision of services to infants, toddlers, and preschool children who have disabilities has evolved since 1986. From its inception as part of PL99-457 (the Education of the Handicapped Amendments) through the most recent changes defined in PL 105-17 (Individuals with Disabilities Education Act, Part C), legislative efforts uphold the rights of students and parents to the key components of a free and appropriate public education. Federal regulations define early intervention services as services that "are designed to meet the developmental needs of each child eligible under this part and the needs of the family related to enhancing the child's development." The child described in the vignette will benefit from early intervention therapy, the goal of which is to enhance the development of infants and toddlers who have disabilities and minimize their potential for developmental delays. The services are designed to meet the needs of the child and family and promote the child's development in natural environments. Therapy at this stage may be selected by the parents, and parent-based training in the home is most appropriate for this child. Because language delays have been identified for this boy, his parents should be encouraged to initiate services without delay. His speech probably will improve more slowly without therapy, but it will not regress without therapy. Participation in the Part C early intervention system for infants and toddlers is voluntary for the family; they have the right to accept or decline specific early intervention services. Thus, declining such services does not constitute child neglect. Early intervention services were established to allow children who have developmental problems to reach their potential. Research studies have identified specific times in which a child's brain is especially efficient at learning specific information. Coordinated, community-based multidisciplinary programs for early intervention represent effective public policy because they not only help to improve some children's cognitive outcome, but they also aid in family functioning.

References:

American Academy of Pediatrics Committee on Children With Disabilities. Role of the pediatrician in family-centered early intervention services. Pediatrics. 2001;107:1155-1157. Available at: http://pediatrics.aappublications.org/cgi/content/full/107/5/1155

American Academy of Pediatrics Committee on Children With Disabilities. The pediatrician's role in development and implementation of an Individual Education Plan and/or and Individual Family Service Plan. Pediatrics. 1999;104:124-127. Available at: http://pediatrics.aappublications.org/cgi/content/full/104/1/124 Policy reaffirmed. Pediatrics. 2006;117:1846-1847. Available at: http://pediatrics.aappublications.org/cgi/content/full/117/5/1846

Page 486: AAP PREP 2009

2009 PREP SA on CD-ROM

page 486

Question: 140

A 10-year-old boy presents with a 1-day history of fever and a swollen leg. According to his mother, the boy developed a small abrasion on his leg while playing outside 3 days ago. Last night he began to complain of pain in the area and had a low-grade fever. This morning his temperature was 102.4°F (39.1°C) and the area around the abrasion looked very red and was tender to palpation. About 2 hours later, the swelling had increased. Physical examination reveals a boy in no apparent distress who has a temperature of 101.4°F (38.6°C), a heart rate of 93 beats/min, a respiratory rate of 23 breaths/min, and a blood pressure of 95/65 mm Hg. All other findings are normal, except for a small erythematous abrasion just above the medial malleolus that has no discharge. Erythema from this area extends to a well-demarcated region of the mid-calf and is tender to touch (Item Q140).

Of the following, the MOST likely pathogen is

A. Pseudomonas aeruginosa

B. Staphylococcus aureus

C. Streptococcus pneumoniae

D. Streptococcus pyogenes

E. Vibrio vulnificans

Page 487: AAP PREP 2009

2009 PREP SA on CD-ROM

page 487

Question: 140

Erythema, as described for the boy in the vignette. (Courtesy of D. Krowchuk)

Page 488: AAP PREP 2009

2009 PREP SA on CD-ROM

page 488

Critique: 140 Preferred Response: D

Cellulitis is an infection and inflammation of the connective tissue that involves the dermis and often is preceded by a break in the skin. Cellulitis also can be seen in patients who have an underlying condition that predisposes them to such an illness (eg, lymphatic stasis, diabetes mellitus). Streptococcus pyogenes and Staphylococcus aureus are the two most common bacterial agents responsible for the development of this infection. Although distinction between the two pathogens is difficult, the rapidly spreading erythema without purulence described for patient in the vignette is most consistent with S pyogenes infection (Item C140A). Infections caused by S aureus tend to be more localized and produce purulent material. Confirmation of the etiologic agent requires recovery of the organism from an aspirate of the most erythematous area or a culture from purulent exudates or from the blood. For most patients, initial empiric therapy is with an antimicrobial agent that has activity against both of these gram-positive cocci. Empiric treatment courses have changed recently, with the increased prevalence of methicillin-resistant S aureus (MRSA). Mild forms of cellulitis may be treated with a topical antimicrobial ointment (eg, mupirocin). More advanced cases require the addition of an oral antimicrobial agent such as clindamycin, trimethoprim-sulfamethoxazole, or doxycycline in geographic areas that are highly endemic for MRSA. Cephalexin or amoxicillin-clavulanate is an adequate initial empiric agent in a community that has less than a 5% prevalence of MRSA. Severe forms of cellulitis require intravenous antimicrobial agents and possibly even surgical debridement. Pseudomonas aeruginosa can cause cellulitis but is more common when the patient has a history of a penetrating injury to the skin that involves water (eg, cut on the leg from a stick in a creek), is immunocompromised, or has diabetes mellitus. S pneumoniae occasionally causes cellulitis but is not a common cause of this infection. Vibrio vulnificans can cause a life-threatening form of cellulitis termed "necrotizing fasciitis," but such infections are associated with injuries from fish or objects in salt water in an immunocompromised patient (Item C140B).

References:

Jaggi P, Shulman ST. Group A streptococcal infections. Pediatr Rev. 2006;27:99-105. Available at: http://pedsinreview.aappublications.org/cgi/content/full/27/3/99

Todd JK. Staphylococcal infections. Pediatr Rev. 2005;26:444-450. Available at: http://pedsinreview.aappublications.org/cgi/content/full/26/12/444

Page 489: AAP PREP 2009

2009 PREP SA on CD-ROM

page 489

Critique: 140

Cellulitis caused by Streptococcus pyogenes is characterized by rapidly spreading erythema. (Courtesy of D. Krowchuk)

Page 490: AAP PREP 2009

2009 PREP SA on CD-ROM

page 490

Critique: 140

Necrotizing fasciitis is characterized by pain, erythema, and swelling that progress to necrosis and bullae formation. It may be caused by infection with a number of agents. (Courtesy of Bernard Cohen, MD; DermAtlas; www.dermatlas.org)

Page 491: AAP PREP 2009

2009 PREP SA on CD-ROM

page 491

Question: 141

You are evaluating a previously healthy 3-year-old boy for a 3-day history of nausea; vomiting; and profuse watery, nonbloody diarrhea that has worsened over the last 24 hours. He can keep down water and an oral electrolyte maintenance solution but has no interest in eating solid food. The family returned 5 days ago from a 1-week vacation at a resort in Acapulco, Mexico. On physical examination, the tired-appearing little boy has a temperature of 100.8°F (38.3°C), moist mucous membranes, and a soft abdomen with mild diffuse tenderness to palpation. Laboratory tests document a peripheral white blood cell count of 6.7x103/mcL (6.7x109/L); hemoglobin of 12.0 g/dL (120.0 g/L); platelet count of 230.0x103/mcL (230.0x109/L); and a differential count of 50% neutrophils, 40% lymphocytes, and 10% monocytes.

Of the following, the MOST likely pathogen causing this patient’s condition is

A. Campylobacter jejuni

B. Escherichia coli

C. Giardia lamblia

D. Salmonella sp

E. Shigella sp

Page 492: AAP PREP 2009

2009 PREP SA on CD-ROM

page 492

Critique: 141 Preferred Response: B

The profuse watery, nonbloody diarrhea and history of vacationing in Mexico described for the child in the vignette are consistent with traveler's diarrhea. By far, the most common cause of traveler's diarrhea is enterotoxigenic Escherichia coli (ETEC), accounting for up to 75% of cases. There are at least five types of diarrhea-producing E coli strains, which differ in epidemiologic features and diarrhea type (Item C141). The primary treatment of all diarrheal disease is to provide adequate fluid replacement by the oral or parenteral routes. The use of additional antibiotic therapy depends on the type of E coli that is suspected of causing the illness. Treatment of EHEC infections is entirely supportive. Antibiotics are contraindicated because they can induce the expression and release of Shiga toxins, and their use has been associated with a higher risk of hemolytic-uremic syndrome in children. If a patient who has E coli diarrhea does not improve after several days of supportive care and assays for Shiga toxin are negative, antimicrobial therapy can be considered. Antimicrobial agents that may be used in the treatment of severe watery ETEC diarrhea in a traveler or for dysentery caused by EIEC strains include trimethoprim-sulfamethoxazole, azithromycin, or ciprofloxacin. Selection of the agent is based on patient age and antimicrobial susceptibility of the organism. Data regarding the treatment of EAEC infections with antibiotics are sparse, and optimal therapy is unknown. Shigella, Salmonella, and Campylobacter sp are much less common causes of traveler's diarrhea, but may be responsible for up to 30% of cases, 15% of cases, and 15% of cases, respectively. Giardia lamblia is not recognized as a cause of acute traveler's diarrhea.

References:

American Academy of Pediatrics. Escherichia coli diarrhea (including hemolytic-uremic syndrome). In: Pickering LK, Baker CJ, Long SS, McMillan JA, eds. Red Book: 2006 Report of the Committee on Infectious Diseases. 27th ed. Elk Grove Village, Ill: American Academy of Pediatrics; 2006:291-296

Donnenberg MS. Enterobacteriaceae. In: Mandell GL, Bennett JE, Dolin R, eds. Mandell, Douglas, and Bennett's Principles and Practice of Infectious Diseases. 6th ed. Philadelphia, Pa: Elsevier Churchill Livingstone; 2005:2567-2586

Ericsson CD, DuPont HL. Travelers' diarrhea: approaches to prevention and treatment. Clin Infect Dis. 1993;16:616-626

Guerrant RL, Bobak DA. Nausea, vomiting, and noninflammatory diarrhea. In: Mandell GL, Bennett JE, Dolin R, eds. Mandell, Douglas, and Bennett's Principles and Practice of Infectious Diseases. 6th ed. Philadelphia, Pa: Elsevier Churchill Livingstone; 2005:1236-1249

Nataro JP, Kaper JB. Diarrheogenic Escherichia coli. Clin Microbiol Rev. 1998;11:142-201. Available at: http://cmr.asm.org/cgi/content/full/11/1/142?view=long&pmid=9457432

Page 493: AAP PREP 2009

2009 PREP SA on CD-ROM

page 493

Critique: 141

Page 494: AAP PREP 2009

2009 PREP SA on CD-ROM

page 494

Question: 142

A 5-year-old boy presents with dark red urine, fever, and rhinorrhea. He was well until 2 days ago, when he developed rhinorrhea and mild cough. He denies urgency, frequency, dysuria, back pain, or musculoskeletal complaints. On physical examination, the slightly ill-appearing boy has a temperature of 99.5°F (37.5°C), heart rate of 130 beats/min, respiratory rate of 18 breaths/min, and blood pressure of 104/58 mm Hg. He has pale conjunctivae, mild scleral icterus, a hyperdynamic precordium, and a I/VI systolic murmur at the left upper sternal border. There is no edema, and musculoskeletal and neurologic examination results are normal. Urinalysis results include: red appearance, a specific gravity of 1.030, pH of 6.5, 3+ blood, and no protein. Microscopy reveals fewer than 5 red blood cells/high-power field (HPF), fewer than 5 white blood cells/HPF, and no casts.

Of the following, the BEST initial test to obtain is

A. complete blood count with manual differential count

B. creatine phosphokinase measurement

C. renal/bladder ultrasonography

D. urine culture

E. viral culture of urine for adenovirus

Page 495: AAP PREP 2009

2009 PREP SA on CD-ROM

page 495

Critique: 142 Preferred Response: A

The differential diagnosis for causes of red urine includes hematuria, hemoglobinuria, myoglobinuria, and porphyrinuria. Some urinary tract pathogens (eg. Serratia marcescens) also can be associated with red urine. In addition, red urine can result from ingestion of certain foods (beets, blackberries, or food dyes) and medications (deferoxamine, rifampin, and phenolphthalein). The initial test for evaluating a child who has red urine is the urinalysis to look for the presence or absence of blood. The 3+ blood on a dipstick described for the child in the vignette narrows the differential diagnosis to hematuria, hemoglobinuria, or myoglobinuria. The dipstick test incorporates an indicator impregnated into the paper strip that, upon contact with free hemoglobin, myoglobin, or intact red blood cells, oxidizes the indicator to result in a blue color change. This dipstick test is exquisitely sensitive to blood to the level of 5 red blood cells/high-power field. The fewer than 5 red blood cells/high power field on microscopy described for the patient in the vignette eliminates hematuria. The presence of tachycardia, pale conjunctivae, and a systolic murmur is suggestive of anemia, and scleral icterus supports a possible hemolytic anemia. Accordingly, a complete blood count to assess for anemia is the most appropriate next laboratory test to obtain for this boy. The absence of musculoskeletal complaints and normal musculoskeletal and neurologic examination results make myoglobinuria (usually related to rhabdomyolyis) unlikely. If myoglobinuria was suspected, measurement of creatine phosphokinase would be useful. Renal/bladder ultrasonography is helpful in evaluating patients who have bright red urine, with clots and red blood cells noted on urinalysis. Such findings are suggestive of a structural cause for the red urine, such as renal or bladder mass or stones. Red urine caused by bacterial or viral infections is due to hematuria. Thus, culture of the urine for bacteria or viral culture for adenovirus is inappropriate because this boy does not have hematuria.

References:

Kalia A, Travis LB. Hematuria, leukocyturia, and cylindruria. In: Edelmann CM Jr, Bernstein J, Meadow SR, Spitzer A, Travis LB, eds. Pediatric Kidney Disease. 2nd ed. Boston, Ma: Little, Brown and Company; 1992:553-563

Moxey-Mims M. Hematuria and proteinuria. In: Kher KK, Schnaper HW, Makker SP, eds. Clinical Pediatric Nephrology. 2nd ed. London, England: Informa Healthcare; 2007:129-141

Schwartz G. Clinical assessment of renal function. In: Kher KK, Schnaper HW, Makker SP, eds. Clinical Pediatric Nephrology. 2nd ed. London, England: Informa Healthcare; 2007:71-93

Page 496: AAP PREP 2009

2009 PREP SA on CD-ROM

page 496

Question: 143

You are evaluating a 16-year-old girl during her biannual asthma follow-up visit. She has a history of mild persistent asthma that is well-controlled on a low-dose inhaled corticosteroid. You review the asthma guidelines and recommend that she receive the influenza vaccine. Her mother immediately replies, "Oh no, my daughter has a severe egg allergy and was told to never get the influenza vaccine."

Of the following, the vaccine that is contraindicated in a patient who has a severe immunoglobulin E-mediated egg allergy is

A. human papillomavirus vaccine

B. measles-mumps-rubella

C. tetanus diphtheria

D. varicella

E. yellow fever

Page 497: AAP PREP 2009

2009 PREP SA on CD-ROM

page 497

Critique: 143 Preferred Response: E

Of currently available vaccines, the measles-mumps-rubella (MMR) vaccine, trivalent influenza vaccine (TIV), live attenuated intranasal influenza vaccine (LAIV), and yellow fever vaccine all contain small amounts of egg protein. All of these vaccines are contraindicated in patients who have egg allergies except the MMR. Despite being developed in chick embryos, the MMR vaccine contains only 0.5 to 1 ng of ovalbumin per dose, a quantity far lower than that required to cause an immunoglobulin (Ig) E-mediated reaction. Patients who have experienced severe anaphylaxis to egg in the past generally should not receive the TIV, LAIV, or yellow fever vaccine. Children who have mild egg allergies (eg, atopic dermatitis, cutaneous-only symptoms) and who are at high risk for complications from influenza should be offered the TIV. Protocols for skin testing, two-step dose challenge, single-dose challenge, and desensitization are available for such patients. Clinicians should remember that many patients who have experienced adverse food reactions in the past are not truly allergic to that food. Pursuing skin or serum allergy testing to egg, particularly if negative, will aid in counseling the families on the risks of future influenza vaccination and therapeutic options. Although diphtheria and tetanus toxoids and acellular pertussis, human papillomavirus, and varicella vaccines do not contain egg protein, other vaccine components may result in a reaction.

References:

American Academy of Pediatrics. Active immunization. In: Pickering LK, Baker CJ, Long SS, McMillan JA, eds. Red Book: 2006 Report of the Committee on Infectious Diseases. 27th ed. Elk Grove Village, Ill: American Academy of Pediatrics; 2006:9-10

American Academy of Pediatrics. Reporting of adverse events. Reporting of vaccine-preventable diseases. In: Pickering LK, Baker CJ, Long SS, McMillan JA, eds. Red Book: 2006 Report of the Committee on Infectious Diseases. 27th ed. Elk Grove Village, Ill: American Academy of Pediatrics; 2006:41-50

Cerecedo Carballo I, Dieguez Pastor MC, Bartolomé Zavala B, Sánchez Cano M, de la Hoz Caballer B. Safety of measles-mumps-rubella vaccine (MMR) in patients allergic to eggs. Allergol Immunopathol (Madr). 2007;35:105-109. Available at: http://db.doyma.es/cgi-bin/wdbcgi.exe/doyma/mrevista.pubmed_full?inctrl=05ZI0102&rev=105&vol=35&num=3&pag=105

Cox JE, Cheng TL. In brief: egg-based vaccines. Pediatr Rev. 2006;27:118-119. Available at: http://pedsinreview.aappublications.org/cgi/content/full/27/3/118

Piquer-Gibert M, Plaza-Martín A, Martorell-Aragonés A, et al. Recommendations for administering the triple viral vaccine and antiinfluenza vaccine in patients with egg allergy. Allergol Immunopathol (Madr). 2007;35:209-212. Available at: http://db.doyma.es/cgi-bin/wdbcgi.exe/doyma/mrevista.pubmed_full?inctrl=05ZI0102&rev=105&vol=35&num=5&pag=209

Page 498: AAP PREP 2009

2009 PREP SA on CD-ROM

page 498

Question: 144

A 3-month-old infant is brought to the office for fussiness, increased sleeping, and poor feeding. According to his mother, he was doing well until 4 days ago, when his formula intake decreased from 6 oz every 3 to 4 hours to 1 to 2 oz every 4 hours and she had to awaken him to feed. He has had no vomiting, diarrhea, or fever. He was born at term, and the mother had no antenatal infections. On physical examination, the infant is difficult to console and has a high-pitched cry. His temperature is 98.2°F (36.8°C), heart rate is 160 beats/min, and respiratory rate is 30 breaths/min. His anterior fontanelle is flat, pupils are 4 mm and equally reactive, and there is no evidence of corneal abrasions. His lungs are clear, heart sounds are normal, and abdominal evaluation findings are benign. His extremities are warm, well-perfused, and have normal tone. Results of the initial laboratory evaluation, including a complete blood count with differential count, electrolytes, and urinalysis, are normal. The fecal occult blood test result is negative.

Of the following, the MOST appropriate next study is

A. abdominal ultrasonography

B. chest radiography

C. computed tomography scan of the brain

D. serum ammonia determination

E. urine organic acid screen

Page 499: AAP PREP 2009

2009 PREP SA on CD-ROM

page 499

Critique: 144 Preferred Response: C

The differential diagnosis of the irritable infant is extensive and includes conditions that affect all organ systems (Item C144A). The evaluation should be based on a complete history and physical examination as well as a high index of suspicion for serious occult causes. For the patient described in the vignette, the concern should be high for nonaccidental head injury, despite the lack of trauma history or external or cutaneous findings on physical examination, and should prompt the physician to obtain neuroimaging (eg, computed tomography scan or magnetic resonance imaging). Of note, recent studies have reconfirmed the incidence of occult head injuries and the importance of neuroimaging in the evaluation of suspected child abuse. Almost 30% of children undergoing child abuse evaluation in one study had occult brain injury despite the absence of neurologic symptoms, and as many as 10% of brain injuries may be missed if only skeletal surveys and ophthalmologic examinations are performed. The presenting signs and symptoms of nonaccidental head trauma due to inflicted traumatic brain injury (also known as "shaken baby syndrome") often are nonspecific. It is estimated that as many of 30% of cases are not diagnosed initially, in part because the findings may be attributed to other conditions, such as viral syndrome, colic, or formula intolerance. The history also may be misleading, with most caretakers reporting no trauma. In some instances, the sole finding may be a disproportionately large head circumference. The absence of external findings is related largely to the biomechanics of the injurious event. Vigorous shaking, with or without impact, leads to traction on the dural bridging veins. Shearing of these veins causes bleeding into the subdural space (Item C144B). Especially in those cases where there was no impact of the infant's head during the shaking episode, bruising or swelling is likely to be absent. Abdominal ultrasonography to evaluate for intussusception or hydronephrosis, serum ammonia and urine organic acid determinations to evaluate for metabolic errors, or chest radiography to look for cardiomegaly or pulmonary infiltrates may be indicated in the evaluation of the irritable infant. However, in contrast to abusive head trauma, it is likely that signs, symptoms, or abnormalities on screening laboratory evaluations would provide clues to these other diagnoses.

References:

Herman M, Le A. The crying infant. Emerg Med Clin North Am. 2007;25:1137-1159. Abstract available at: http://www.ncbi.nlm.nih.gov/pubmed/17950139

Keenan HT, Runyan DK, Marshall SW, Nocerna MA, Merten DF. A population-based comparison of clinical and outcome characteristics of young children with serious inflicted and noninflicted traumatic brain injury. Pediatrics. 2004;114:633-639. Available at: http://pediatrics.aappublications.org/cgi/content/full/114/3/633

Laskey AL, Holsti M, Runyan DK, Socolar RRS. Occult head trauma in young suspected victims of physical abuse. J Pediatr. 2004;144:719-722. Abstract available at: http://www.ncbi.nlm.nih.gov/pubmed/15192615

Newton AW, Vandeven AM. Update on child maltreatment with a special focus on shaken baby syndrome. Curr Opin Pediatr. 2005;17:246-251. Abstract available at: http://www.ncbi.nlm.nih.gov/pubmed/15800421

Sirotnak AP, Grigsby T, Krugman R. Physical abuse of children. Pediatr Rev. 2004;25:264-277. Available at: http://pedsinreview.aappublications.org/cgi/content/full/25/8/264

Page 500: AAP PREP 2009

2009 PREP SA on CD-ROM

page 500

Critique: 144

Page 501: AAP PREP 2009

2009 PREP SA on CD-ROM

page 501

Critique: 144

Computed tomography scan without contrast in a 3-year-old who was the victim of nonaccidental trauma shows a large left subdural hematoma (arrows) with 4-mm left-to-right midline shift. (Courtesy of D. Krowchuk)

Page 502: AAP PREP 2009

2009 PREP SA on CD-ROM

page 502

Question: 145

A 13-year-old boy who has a 1-year history of abdominal pain in the epigastric and periumbilical regions presents for further evaluation. According to his history, the pain occurs one to three times per week and sometimes interferes with school attendance and physical activity. Findings on physical examination are normal. You review the diagnostic studies that have been performed in the past year.

Of the following, the finding that MOST warrants referral for upper endoscopy is

A. abnormal lactose breath hydrogen test result

B. elevated serum alkaline phosphatase value

C. elevated serum amylase value

D. mild anemia (hematocrit, 33% [0.33])

E. positive tissue transglutaminase antibody

Page 503: AAP PREP 2009

2009 PREP SA on CD-ROM

page 503

Critique: 145 Preferred Response: E

Most children who have recurrent abdominal pain (RAP) have functional abdominal pain. Therefore, most affected children do not require gastrointestinal endoscopy. However, if certain conditions are strongly suspected, endoscopy can be an essential part of the diagnostic evaluation. Such conditions include peptic ulcer disease, bleeding esophageal varices, reflux esophagitis, eosinophilic esophagitis, inflammatory bowel disease, and celiac disease. Most of these conditions present with symptoms considered "red flags," including chronic diarrhea, weight loss, unexplained fever, hematemesis, or lower gastrointestinal tract bleeding. The choice of either upper gastrointestinal endoscopy or lower gastrointestinal endoscopy (colonoscopy) is based on the patient's clinical symptoms. Tissue forceps biopsies can be taken through the endoscope and tissue sent to the pathologist for identification of subtle lesions (eg, esophagitis, celiac disease, giardiasis, Helicobacter pylori gastritis). The strongest indication for upper endoscopy for the patient described in the vignette is a positive tissue transglutaminase antibody test. This antibody is a sensitive and specific test for celiac disease, although some false-positive results can be seen. Because a lifelong gluten-free diet is the only treatment for celiac disease, it is still recommended that patients undergo endoscopy with duodenal biopsies to establish the diagnosis. The endoscopic visual appearance may be normal in patients who have celiac disease, but in severe cases, "scalloped" duodenal folds may be seen. Generally, three to four biopsies of the duodenum are needed to evaluate for celiac disease; histologic features include intraepithelial lymphocytosis and villous atrophy. Elevated alkaline phosphatase values suggest normal adolescent bone growth, bone inflammation, osteopenia, or hepatobiliary disease. Elevated serum amylase values suggest pancreatitis, and routine upper endoscopy is rarely helpful in affected patients (although endoscopic retrograde cholangiopancreatography may be useful in evaluating recurrent pancreatitis). Mild anemia is a nonspecific finding that can be seen for many different reasons. An abnormal lactose breath test result suggests lactose intolerance, which does not require endoscopic confirmation.

References:

Fox VL. Gastrointestinal endoscopy: patient preparation and surgical considerations. In: Walker WA, Goulet O, Kleinman RE, Sherman PM, Shneider BL, Sanderson IR, eds. Pediatric Gastrointestinal Disease. 4th ed. Hamilton, Ontario, Canada: BC Decker; 2004:1666-1673

Thakkar K, Gilger MA, Shulman RJ, El Serag H. Esophagogastroduodenoscopy in children with abdominal pain: a systematic review. Am J Gastroenterol. 2007;102:654-661. Abstract available at: http://www.ncbi.nlm.nih.gov/pubmed/17222318

Page 504: AAP PREP 2009

2009 PREP SA on CD-ROM

page 504

Question: 146

You are called to the newborn nursery to see a 2.1-kg term infant whose bedside glucose screening test value is 30 mg/dL (1.7 mmol/L). The nurse describes the baby as being generally lethargic, jittery with stimulation, and intolerant of oral feeding attempts at 4 hours of age (poor oral suckling and emesis of the small volumes of formula taken). He was born at 41 weeks’ gestation to a mother who had poor weight gain, smoked cigarettes, and had hypertension. The Apgar scores following a vaginal delivery were 6 and 8 at 1 and 5 minutes, respectively. There is no history of maternal diabetes, illicit drug use, or intrapartum difficulties. On physical examination, the baby’s vital signs are normal except for tachypnea (respiratory rate of 80 breaths/min), with pulse oximetry of 90% on room air. The infant has plethora, acrocyanosis, and generalized low tone. He exhibits rapid, shallow tachypnea, with clear lungs bilaterally on auscultation. There is a soft I/VI systolic murmur along the lower left sternal border and no gallop. Upon stimulation, he has jittery hand movements. Laboratory findings include: · Serum glucose, 45.0 mg/dL (2.5 mmol/L) · White blood cell count, 7.0x103/mcL (7.0x109/L) with a normal differential count · Platelet count, 150.0x103/mcL (150.0x109/L) · Hematocrit, 70% (0.70)An arterial blood gas reveals a pH of 7.40, Pao2 of 75 mm Hg, Paco2 of 30 mm Hg, and base excess of -7 mEq/L.

Of the following, the MOST appropriate treatment for this infant’s underlying problem is

A. administration of amphotericin B

B. double-volume exchange transfusion

C. intubation and assisted ventilation

D. partial exchange transfusion

E. phototherapy

Page 505: AAP PREP 2009

2009 PREP SA on CD-ROM

page 505

Critique: 146 Preferred Response: D

By definition, polycythemia exists when the hematocrit (HCT) is greater than 65% (0.65). This condition occurs in newborns who are small for gestational age, infants of diabetic mothers, the recipient twin in a twin-twin transfusion syndrome-affected pregnancy, or infants who have delayed clamping of the umbilical cord after delivery. The decision to treat polycythemia is contingent upon the presence of symptoms associated with the state of hyperviscosity of the circulating blood conferred upon it by the increased HCT or an HCT of greater than 70% (0.70). Such symptoms include those described for the infant in the vignette as well as those described in the Table (Item C146). Although blood hyperviscosity also may occur due to markedly elevated white blood cell numbers (generally, >100.0x103/mcL [100.0x109/L]) or with elevations of certain plasma protein fractions, in the newborn, it is almost exclusively related to an increase in the red blood cell mass, as reflected in the HCT. The treatment of choice for symptomatic polycythemia, as seen in this infant, is a partial exchange transfusion, which reduces the red blood cell mass and maintains a euvolemic state by the administration of crystalloid (eg, normal saline). The partial exchange transfusion is partial in that it removes only a portion of the circulating volume of blood (as opposed to a complete or double-volume exchange transfusion) and replaces (exchanges) it with crystalloid. The volume of such an exchange is based on the following formula: Volume of exchange (mL) = [Infant's blood volume] x [Observed HCT-Desired HCT]/Observed HCT The blood volume of an infant who has polycythemia is 100 mL/kg. For a 3-kg infant who has an observed HCT of 70% (0.70), the volume of exchange is: = [3 kg x 100 mL/kg] x [0.70-0.55]/0.70 = 300 mL x 0.15/0.70 = 300 mL x 0.214 = 64 mL Polycythemia cannot be treated solely with intravenous crystalloid because this fluid leaves the circulatory compartment easily. Because the patient does not have evidence of systemic fungal infection, amphotericin B is not indicated and would not treat polycythemia. The infant in the vignette does not have hypoxemia or hypercarbia that warrants intubation and assisted ventilation. Phototherapy does not treat polycythemia, only the hyperbilirubinemia that follows. A double-volume exchange transfusion is used to treat severe hyperbilirubinemia.

References:

Ceriani Cernadas JM, Carroli G, Pellegrini L, et al. The effect of timing of cord clamping on neonatal venous hematocrit values and clinical outcome at term: a randomized, controlled trial. Pediatrics. 2006;117:e779-e786. Available at: http://pediatrics.aappublications.org/cgi/content/full/117/4/e779

Linderkamp O. Blood viscosity of the neonate. NeoReviews. 2004;5:e406-e416. Available for subscription at: http://neoreviews.aappublications.org/cgi/content/full/5/10/e406

Luchtman-Jones L, Schwartz AL, Wilson DB. Blood component therapy for the neonate. In: Martin RJ, Fanaroff AA, Walsh MC, eds. Fanaroff and Martin's Neonatal-Perinatal Medicine. 8th ed. Philadelphia, Pa: Mosby Elsevier; 2006:1344-1356

Page 506: AAP PREP 2009

2009 PREP SA on CD-ROM

page 506

Philip AGS, Saigal S. When should we clamp the umbilical cord? NeoReviews. 2004;5:e142-e154. Available for subscription at: http://neoreviews.aappublications.org/cgi/content/full/5/4/e142

Page 507: AAP PREP 2009

2009 PREP SA on CD-ROM

page 507

Critique: 146

Page 508: AAP PREP 2009

2009 PREP SA on CD-ROM

page 508

Question: 147

A worried mother brings her 4-year-old son to your office because his right eye has been red for 3 days. She assumed it was pink eye that he contracted at child care, but she now is concerned because he has developed swelling in front of his right ear, and his eye has become redder. They live in a wooded area and got a new kitten 6 weeks ago, but there is no history of the kitten scratching the child. Physical examination reveals a well-appearing child who has obvious conjunctival injection (Item Q147A) of the right eye but no discharge or pain. You palpate a 2x2-cm tender, mobile preauricular lymph node (Item Q147B) and a 2x3-cm anterior cervical lymph node on the right. The remainder of the physical examination findings are normal.

Of the following, the MOST likely pathogen causing this boy’s symptoms is

A. Bartonella henselae

B. Francisella tularensis

C. Haemophilus influenzae

D. Pasteurella multocida

E. Staphylococcus aureus

Page 509: AAP PREP 2009

2009 PREP SA on CD-ROM

page 509

Question: 147

(Courtesy of M. Rimsza)

Page 510: AAP PREP 2009

2009 PREP SA on CD-ROM

page 510

Question: 147

(Courtesy of M. Rimsza)

Page 511: AAP PREP 2009

2009 PREP SA on CD-ROM

page 511

Critique: 147 Preferred Response: A

The boy described in the vignette has signs of Parinaud oculoglandular syndrome, with painless, nonpurulent conjunctivitis and ipsilateral preauricular lymphadenopathy (Item C147A). These findings, combined with the cervical lymphadenopathy and his exposure to a kitten, make cat-scratch disease, or Bartonella henselae infection, the most likely possibility. Although most patients who have cat-scratch disease report some contact with kittens or cats, many do not recall being scratched. Cats younger than 1 year of age are most likely to transmit the organism to humans; human-to-human transmission does not occur. The most common clinical manifestation of cat-scratch disease is regional lymphadenopathy, with cervical and axillary areas affected most (Item C147B). A papule (Item C147C) at the site of the scratch often precedes the development of lymphadenopathy. Lymph nodes may enlarge considerably and may be tender, warm, and erythematous; suppuration can occur in up to 30% of patients. Nodes may remain enlarged for several months. Other clinical manifestations of cat-scratch disease include oculoglandular syndrome, fever of unknown origin, painful osteolytic lesions, and hepatic or splenic microabscesses. Among the less common manifestations are encephalopathy, thrombocytopenic purpura, and neuroretinitis. Cat-scratch disease can be diagnosed by obtaining an indirect immunofluorescent antibody assay for B henselae. Biopsy of an affected node also may show the bacilli or granulomas but should be reserved for atypical cases. The course of cat-scratch disease is self-limited. The efficacy of antibiotic therapy is controversial, but some studies have shown improvement in those who have severe systemic symptoms and those who are immunocompromised. Several oral agents have shown efficacy: azithromycin, erythromycin, ciprofloxacin, trimethoprim-sulfamethoxazole, and rifampin. Needle aspiration of suppurative lymph nodes may hasten recovery and reduce pain, but incision and drainage is not recommended because of the risk of sinus tract formation and persistent drainage. Tularemia, infection with Francisella tularensis, also can cause an oculoglandular syndrome, but the conjunctivitis typically is painful. A history of exposure to ticks or wild rabbits would make this diagnosis more likely. Haemophilus influenzae can cause conjunctivitis with ipsilateral preauricular lymphadenopathy, but the conjunctivitis typically is purulent, and ipsilateral otitis media might be expected. Pasteurella multocida is the most common organism causing infection after a cat bite. Typical findings include cellulitis and suppuration at the area of the bite; conjunctivitis is rare. Staphylococcus aureus also can cause purulent conjunctivitis, but the history of kitten exposure for the patient in the vignette makes this a less likely causative pathogen.

References:

American Academy of Pediatrics. Cat-scratch disease. In: Pickering LK, Baker CJ, Long SS, McMillan JA, eds. Red Book: 2006 Report of the Committee on Infectious Diseases. 27th ed. Elk Grove Village, Ill: American Academy of Pediatrics; 2006:246-248.

English R. Cat-scratch disease. Pediatr Rev. 2006;27:123-128. Available at: http://pedsinreview.aappublications.org/cgi/content/full/27/4/123

Page 512: AAP PREP 2009

2009 PREP SA on CD-ROM

page 512

Critique: 147

Parinaud oculoglandular syndrome (inoculation of the conjunctiva with ipsilateral preauricular adenopathy) in a 6-year-old boy. (Courtesy of the Red Book® Online)

Page 513: AAP PREP 2009

2009 PREP SA on CD-ROM

page 513

Critique: 147

Axillary lymphadenopathy in cat-scratch disease. (Courtesy of M. Rimsza)

Page 514: AAP PREP 2009

2009 PREP SA on CD-ROM

page 514

Critique: 147

A papule at the site of inoculation may be observed and precedes the development of lymphadenopathy by 1 to 2 weeks. (Courtesy of M. Rimsza)

Page 515: AAP PREP 2009

2009 PREP SA on CD-ROM

page 515

Question: 148

A mother brings in her 3-month-old boy because he has had a worsening cough over the past 2 days. She reports that he has been afebrile but not feeding as well as he normally does. You note a few coarse breath sounds, rare wheezing, and intermittent subcostal retractions on physical examination. His respiratory rate is 56 breaths/min and temperature is 101.5°F (38.6°C). There is a family history of asthma. A trial of albuterol results in minimal improvement in his chest findings. His oxygen saturation is 89% on room air.

Of the following, the next BEST course of action is to

A. arrange for home administration of albuterol

B. hospitalize the boy for intravenous steroid and antibiotic administration

C. hospitalize the boy for observation and supportive care

D. order a complete blood count

E. refer the boy to a pulmonologist

Page 516: AAP PREP 2009

2009 PREP SA on CD-ROM

page 516

Critique: 148 Preferred Response: C

The child described in the vignette clearly exhibits tachypnea for age and has clinical examination findings consistent with respiratory distress. The indications for hospital admission for a child who has bronchiolitis are similar to those for any child who has any lower respiratory tract disorder. Respiratory distress that is indicated by tachypnea or hypoxemia; signs of accessory muscle use; poor feeding or inability to maintain the airway; signs of respiratory failure such as cyanosis, listlessness, or fatigue; and dehydration necessitate immediate hospitalization. At the age of 3 months, the boy is at risk for dehydration and poor feeding due to his inability to suck and swallow from breast and bottle in conjunction with the reported signs and symptoms. A trial of bronchodilator did not improve his condition, which is not unusual in acute bronchiolitis. For the minority of children (<35%) who do respond to albuterol, a trial in the home with very close follow-up and revisit might be considered. Although nebulized racemic epinephrine has been found to be effective in some children who have bronchiolitis, home use never has been shown to be safe or effective. Viral bronchiolitis has a natural course of worsening over the first 3 to 5 days, which represents an additional concern for the child in the vignette. The use of antibiotics has not been proven to be helpful in viral bronchiolitis, and the use of steroids is controversial and not clearly efficacious. For children who have classic signs and symptoms of bronchiolitis, such as described for this child, a complete blood count is not indicated. Most cases of viral bronchiolitis resolve without sequelae, although there is some suggestion of an increased risk for future development of recurrent wheezing in affected children. Bronchiolitis alone is not an indication for a pediatric pulmonary consultation unless the child fails to improve or exhibits other signs of underlying pulmonary disease. Once a decision has been made to admit a child who has bronchiolitis, whether the child should be admitted to an observation, inpatient, or intensive care unit depends on local or practice preference. There is evidence that the sequential use of respiratory scoring systems may allow for early discharge or the use of observation units.

References:

Gadomski AM, Bhasale AL. Bronchodilators for bronchiolitis. Cochrane Database Syst Rev. 2006;3:CD001266. Available at: http://www.mrw.interscience.wiley.com/cochrane/clsysrev/articles/CD001266/frame.html

Lozan JM. Bronchiolitis (updated). BMJ Clinical Evidence. 2007. Available for subscription at: http://clinicalevidence.bmj.com/ceweb/conditions/chd/0308/0308.jsp

Pelletier AJ, Mansbach JM, Camargo CA Jr. Direct medical costs of bronchiolitis hospitalizations in the United States.Pediatrics 2006;118:2418-2423. Available at: http://pediatrics.aappublications.org/cgi/content/full/118/6/2418

Subcommittee on Diagnosis and Management of Bronchiolitis. Clinical practice guideline:diagnosis and management of bronchiolitis. Pediatrics. 2006;118:1774-1793. Available at: http://pediatrics.aappublications.org/cgi/content/full/118/4/1774

Willson DF, Horn SD, Hendley JO, Smout R, Gassaway J. Effect of practice variation on resource utilization in infants hospitalized for viral lower respiratory illness. Pediatrics. 2001;108;851-855. Available at: http://pediatrics.aappublications.org/cgi/content/full/108/4/851

Page 517: AAP PREP 2009

2009 PREP SA on CD-ROM

page 517

Question: 149

A 15-year-old patient is brought to your office with a complaint of chest pain. She had been healthy until 3 days ago, when she developed a fever. The pain is precordial, referred to the epigastrium, and exacerbated by deep breathing and coughing. She refuses to lie down and prefers to sit leaning forward.

Of the following, the MOST likely expected finding on electrocardiography is

A. elevation of the S-T segments

B. first-degree heart block

C. pre-excitation with a delta wave

D. tall peaked T waves

E. T-wave flattening

Page 518: AAP PREP 2009

2009 PREP SA on CD-ROM

page 518

Critique: 149 Preferred Response: A

Chest pain in children and adolescents is a common problem for which patients and parents frequently seek medical care. The causes of chest pain in the pediatric population are varied and can be considered by organ systems: musculoskeletal, respiratory, gastrointestinal, psychological, and cardiac. Among the musculoskeletal causes are chest wall strain, trauma, costochondritis, and the precordial catch syndrome. Respiratory causes include asthma, pneumonia, pneumothorax, pneumomediastinum, and chronic cough. Chest pain may result from gastritis, esophagitis, or indigestion. Psychogenic processes, including anxiety, fear, and attention-seeking behaviors, may cause or exacerbate chest pain. Perhaps the most common causes of chest pain in pediatrics are those referred to as idiopathic. Such a diagnosis often is given to the patient who presents with a 1- to 2-week history of intermittent, brief, sharp, or stabbing pain that is not associated with exercise or exertion. The cardiac causes of chest pain are important to recognize because they can be associated with significant morbidity and mortality. Pericardial pain (resulting from inflammation and often associated with pericarditis), angina and myocardial ischemia, arrhythmias, or aortic dissection may cause chest pain. The common causes of pericarditis are viral, inflammatory, and rheumatologic. The typical pain of pericarditis frequently is substernal, positional, and can be severe. Patients often prefer to sit leaning forward, as described for the girl in the vignette, and may refuse to lie down. The pain worsens with deep inspiration, coughing, or movement of the upper torso. Because the pericardium is inflamed, pericardial effusion may occur in affected patients, which may lead to pericardial tamponade. Some, but not all, pericardial effusions in pericarditis have the associated finding of a friction rub noted on auscultation. The absence of a rub does not exclude pericardial effusion or pericarditis. Electrocardiographic findings can include S-T segment elevation, low voltage, or in cases of large pericardial effusion, electrical alternans. The latter finding is a beat-to-beat variation in voltage that likely results from the pendular motion of the heart in the effusion. First-degree heart block (Item C149A) is not likely in pericarditis. Pre-excitation is seen in patients who have Wolff-Parkinson-White syndrome (Item C149B) rather than pericarditis. Abnormalities of the T waves usually implicate an electrolyte abnormality. Tall, peaked T waves are seen with hyperkalemia (Item C149C), and flattened T waves are a nonspecific finding that may be seen with hypokalemia (Item C149D) or certain ventricular strain patterns.

References:

Cava J, Sayger PL. Chest pain in children and adolescents. Pediatr Clin North Am. 2004;51:1553-1568. Abstract available at: http://www.ncbi.nlm.nih.gov/pubmed/15561173

Demmler GJ. Infectious pericarditis in children. Pediatr Infect Dis J. 2006;25:165-166

Fahey J. Chest pain. In: Rudolph C, Rudolph A, eds. Rudolph's Pediatrics. 21st ed. New York, NY: McGraw Hill Medical Publishing Division; 2003:1894-1897

Nowlen TT, Bricker JT. Pericardial disease. In: Moller JH, Hoffman JIE, eds. Pediatric Cardiovascular Medicine. Philadelphia, Pa: Churchill Livingstone; 2000:780-792

Page 519: AAP PREP 2009

2009 PREP SA on CD-ROM

page 519

Critique: 149

First-degree heart block represents a prolongation of the PR interval; it is not a feature of pericarditis. (Courtesy of A. Friedman)

Page 520: AAP PREP 2009

2009 PREP SA on CD-ROM

page 520

Critique: 149

Pre-excitation (early depolarization of the QRS complex) manifests as delta waves (arrows) and is a feature of Wolff-Parkinson-White syndrome. (Courtesy of A. Friedman)

Page 521: AAP PREP 2009

2009 PREP SA on CD-ROM

page 521

Critique: 149

The electrocardiogram in hyperkalemia shows peaked T waves. (Courtesy of A. Friedman)

Page 522: AAP PREP 2009

2009 PREP SA on CD-ROM

page 522

Critique: 149

In hypokalemia, the electrocardiogram shows ST segment depression, flattening of T waves, and the appearance of a U wave (arrow). (Courtesy of A. Friedman)

Page 523: AAP PREP 2009

2009 PREP SA on CD-ROM

page 523

Question: 150

A 14-year-old boy who has epilepsy presents to the emergency department after a generalized tonic-clonic seizure that began on the playground at school. He continued to convulse en route in the ambulance, where he received 15 mg diazepam rectally and intravenous access was achieved. In the emergency department, he continues to be unresponsive, exhibiting tachycardia and nonsuppressable bilateral synchronous rhythmic clonic jerks.

Of the following, the MOST appropriate medication to administer next is

A. fosphenytoin 20 mg/kg intravenously

B. pentobarbital 5 mg/kg intravenously

C. phenobarbital 20 mg/kg intravenously

D. phenytoin 7 mg/kg orally

E. valproic acid 15 mg/kg intravenously

Page 524: AAP PREP 2009

2009 PREP SA on CD-ROM

page 524

Critique: 150 Preferred Response: A

Standard recommendations for pharmacologic treatment of status epilepticus and refractory status epilepticus in adults were established in the 1990s. They also represent the standard of care for children and remain valid at the time of this writing (Item C150). Because the boy described in the vignette continues to have seizures after administration of a reasonable dose of diazepam (step 2), he now should receive either phenytoin 20 mg/kg intravenously or fosphenytoin 20 mg phenytoin equivalents/kg intravenously. Use of phenobarbital before phenytoin is not standard. Pentobarbital is reserved for use when other treatments have failed to induce a medical coma, a state of complete unresponsiveness manifested by a characteristic electroencephalographic pattern of "burst suppression" (bursts interspersed with flat or nearly flat tracing). Intravenous valproic acid and levetiracetam are new options for use in selected cases, but they are not standard at this time.

References:

Johnston MV. Seizures in childhood. In: Kliegman RM, Behrman RE, Jenson HB, Stanton BF, eds. Nelson Textbook of Pediatrics. 18th ed. Philadelphia, Pa: Saunders Elsevier; 2007:2457-2475

Riviello JJ Jr, Ashwal S, Hirtz D, et al. Practice parameter: diagnostic assessment of the child with status epilepticus (an evidence-based review): report of the Quality Standards Subcommittee of the American Academy of Neurology and the Practice Committee of the Child Neurology Society. Neurology. 2006;67:1542-1550. Available at: http://www.neurology.org/cgi/content/full/67/9/1542

Riviello JJ Jr, Holmes GL. The treatment of status epilepticus. Semin Pediatr Neurol. 2004;11:129-138. Abstract available at: http://www.ncbi.nlm.nih.gov/pubmed/15259866

Page 525: AAP PREP 2009

2009 PREP SA on CD-ROM

page 525

Critique: 150

Page 526: AAP PREP 2009

2009 PREP SA on CD-ROM

page 526

Question: 151

A 2-day-old male is approaching hospital discharge from the regular nursery. You receive an urgent call from the nurse caring for him, who says that he would not awaken for his last feeding and is now difficult to arouse. A blood glucose determination is normal. You arrange for laboratory tests and call the neonatologist to evaluate the baby while you make plans to leave your office. When you arrive at the hospital 45 minutes later, the baby has been transferred to the neonatal intensive care unit, is now comatose, and has irregular breathing. Results of a complete blood count with differential count, platelets, and a chemistry panel, including renal and liver function tests, are normal.

Of the following, the MOST appropriate laboratory test for diagnosis and formulating a treatment plan for this baby is

A. acylcarnitine profile

B. serum ammonia

C. total and free carnitine

D. urine organic acids

E. very long-chain fatty acids

Page 527: AAP PREP 2009

2009 PREP SA on CD-ROM

page 527

Critique: 151 Preferred Response: B

The differential diagnosis for any infant who has reduced arousability or coma must include inborn errors of metabolism, particularly those that cause elevations of serum ammonia. This is especially true when the infant has previously been alert and feeding well because the change in mental status suggests the possible accumulation of a substance that impairs consciousness. In addition to evaluation for sepsis, initial laboratory tests should include measurement of serum ammonia, electrolytes, and urine ketones. The anion gap also must be calculated. Elevated ammonia concentrations are associated with a variety of inborn errors, including urea cycle defects, transient hyperammonemia, maple syrup urine disease, organic acidopathies, fatty acid oxidation defects, and nonketotic hyperglycinemia (Item C151). Normally, ammonia values are below 110.0 mcmol/L in newborns. Because elevated ammonia concentrations can cause irreversible brain damage, the measurement of serum ammonia and initiation of treatment for hyperammonemia must be the first steps in evaluating the infant in a coma. In a metabolic emergency involving an unknown diagnosis and a serum ammonia concentration of 200.0 mcmol/L or greater, benzoate and phenylacetate (or one of the recently marketed preparations for this purpose) should be administered intravenously to lower the concentration. If the ammonia concentration is 600.0 mcmol/L or greater, hemodialysis is recommended. The infant described in this vignette has laboratory values suggestive of a urea cycle defect, such as ornithine transcarbamylase deficiency. If he had a low glucose value or acidosis, evaluation for other causes of hyperammonemia would be appropriate and should include measurement of total and free carnitine, urine organic acids, and very long-chain fatty acids as well as acylcarnitine profile.

References:

Burton BK. Inborn errors of metabolism in infancy: a guide to diagnosis. Pediatrics. 1998;102:e69-e78. Available at: http://pediatrics.aappublications.org/cgi/content/full/102/6/e69

Hoffmann GF, Nyhan WL, Zschocke J, Kahler SG, Mayatepek E. Approach to the patient with metabolic disease. In: Inherited Metabolic Diseases. Philadelphia, Pa: Lippincott Williams & Wilkins; 2002:19-94

Niemi A-K, Enns GM. Pharmacology review: sodium phenylacetate and sodium benzoate in the treatment of neonatal hyperammonemia. NeoReviews. 2006;7:e486-e495. Available for subscription at: http://neoreviews.aappublications.org/cgi/content/full/7/9/e486

Nyhan WL, Barshop NA, Ozand PT. Hyperammonemia and disorders of the urea cycle: introduction to hyperammonemia and disorders. In: Atlas of Metabolic Diseases. 2nd ed. London, England: Hodder Arnold; 2005:191-198

Page 528: AAP PREP 2009

2009 PREP SA on CD-ROM

page 528

Critique: 151

Page 529: AAP PREP 2009

2009 PREP SA on CD-ROM

page 529

Question: 152

During the annual health supervision visit of a 16-year-old patient, he reports smoking a pack of cigarettes daily. He plans to become a vocal music major in college and is concerned that smoking may affect his voice, but he is uncertain if he wants to stop smoking at this time.

Of the following, the MOST appropriate statement to include in your counseling regarding smoking cessation is that

A. chronic obstructive lung disease is the first pulmonary problem to arise in cigarette smokers

B. cigarettes contain few other toxic chemicals aside from nicotine

C. erectile dysfunction is not associated with cigarette smoking

D. initial symptoms of nicotine dependence occur in some teens after only a few cigarettes

E. lung cancer is the only cancer associated with cigarette smoking

Page 530: AAP PREP 2009

2009 PREP SA on CD-ROM

page 530

Critique: 152 Preferred Response: D

Tobacco use has decreased for adolescents in the United States since peaks in the late 1990s, but it still remains prevalent, with 23% of 9th through 12th graders reporting cigarette use on one or more of the past 30 days preceding the Centers for Disease Control and Prevention (CDC) National Youth Risk Behavior Survey. Although relatively few studies have examined smoking cessation techniques specific to adolescents, brief counseling interventions modeled after similar interventions demonstrated to be effective with adults are recommended. The "5 A's" (Ask, Advise, Assess, Assist, and Arrange) are advocated for use by the United States Public Health Service Task Force practice guidelines in counseling adolescents to stop tobacco use. Among the physiologic consequences of tobacco use are ischemic heart disease; numerous cancers, including lung, head and neck, esophageal, gastric, colorectal, bladder, renal, prostate, and cervical; premature wrinkling of the skin; low-birthweight infants associated with maternal smoking in pregnancy; adverse effects on natural killer cells and other immune modulators; and chronic obstructive pulmonary disease, with small airway disease present before evidence of chronic disease is demonstrable. Cigarettes usually contain thousands of chemicals aside from nicotine, including cyanide, carbon monoxide, formaldehyde, and tar, that may have long-term detrimental effects on health. Emphasizing the immediate and short-term effects of tobacco use rather than the long-term effects is believed to be a better counseling technique for teens. Discussing the effects on athletic performance and on vocal and dramatic performance where voice is important, the potential for erectile dysfunction, and the possibility of symptoms of nicotine dependence occurring in some teens after only a few cigarettes may be more salient to a young person and more important in making a decision to stop tobacco use.

References:

Ammerman SD. Tobacco. In: Neinstein LS, ed. Adolescent Health Care: A Practical Guide. 5th ed. Philadelphia, Pa: Lippincott Williams & Wilkins;, 2008:888-907

Centers for Disease Control and Prevention. Youth Risk Behavior Survey: 2007. Available at: http://www.cdc.gov/Features/RiskBehavior/

Centers for Disease Control and Prevention. Youth risk behavior surveillance-United States, 2005. MMWR Surv Summ. 2006;55(No.SS-5). Available at: http://www.cdc.gov/mmwr/PDF/SS/SS5505.pdf

Klein JD, Camenga DR. Tobacco prevention and cessation in pediatric patients. Pediatr Rev. 2004;25:17-26. Available at: http://pedsinreview.aappublications.org/cgi/content/full/25/1/17

Page 531: AAP PREP 2009

2009 PREP SA on CD-ROM

page 531

Question: 153

You are evaluating a 2-month-old girl who has suspected infantile botulism. On physical examination, she has a weak cry, poor head control, dilated pupils, and a markedly decreased gag reflex. Her respiratory rate is 30 breaths/min, and she has decreased breath sounds bilaterally at her lung bases. Her oxygen saturation on room air is 85%, but has increased to 90% on 3 L/min of oxygen administered by nasal cannula. An arterial blood gas evaluation on 3 L/min oxygen shows a pH of 7.24, a Paco2 of 60 mm Hg, and a Pao2 of 70 mm Hg.

Of the following, the MOST appropriate immediate step is

A. intubation with mechanical ventilation

B. oxygen administration via a nonrebreathing face mask

C. oxygen administration via a high-flow nasal cannula

D. placement of a nasal trumpet

E. serial measurements of negative inspiratory force

Page 532: AAP PREP 2009

2009 PREP SA on CD-ROM

page 532

Critique: 153 Preferred Response: A

The patient described in the vignette demonstrates respiratory failure, which is defined as the inability to maintain adequate oxygenation (hypoxemia, as defined by a PaO2 less than 60 mm Hg) and ventilation (hypercarbia, as defined by a PaCO2 greater than 50 mm Hg). Although her oxygen saturation has improved with oxygen administration, findings of her neurologic examination include decreased tone, impaired airway protection, and decreased aeration in the setting of a progressive neurologic disease. In addition, her elevated PaCO2 reflects her inadequate respiratory effort with subsequent impaired ventilation. Impending respiratory failure in pediatric patients typically is characterized by tachypnea, increased use of accessory muscles, or retractions. Hypoxemia, grunting, and fatigue are late signs and worrisome for respiratory collapse. Oxygen should be administered to patients who have evidence of impending respiratory failure in their position of comfort with a goal of maintaining the oxygen saturation at greater than 90%. A variety of factors, including the cause of the respiratory failure, speed of progression, patient age, associated morbidities, and anticipated procedures such as the need for imaging studies or transport, can guide the clinician in managing respiratory failure and deciding whether to proceed with intubation. The impending respiratory failure seen for the patient in the vignette mandates placement of an endotracheal tube and assistance with ventilation. Careful monitoring is imperative for patients experiencing respiratory failure, especially those who may have chronic respiratory failure and an impaired respiratory drive, although the first priority is correction of the hypoxemia. Administration of oxygen to patients who have chronic respiratory failure should be at the lowest concentration needed to maintain an oxygen saturation of 90%, with frequent arterial blood gas monitoring to ensure that their PaCO2 is not rising. Failure to respond to oxygen administration indicates the need for more aggressive intervention. Bag-valve-mask ventilation can support the oxygenation and ventilation needs of most pediatric patients while appropriate personnel trained in endotracheal intubation are assembled. Patients who have underlying or acute neurologic or neuromuscular processes that interfere with their ability to protect their airways or generate adequate tidal volumes are candidates for early intubation. Serial negative inspiratory force measurements are an excellent method of following respiratory effort in patients who have underlying neuromuscular disorders, but they normally require a cooperative patient of at least 5 years of age. Administration of higher concentrations of oxygen (eg, by high-flow nasal cannula or nonrebreathing face mask) has the potential of increasing the patient's oxygenation but would not improve the ventilation. Placement of a nasal trumpet is indicated for patients who have an intact respiratory drive and evidence of upper airway obstruction.

References:

Frankel LR. Respiratory distress and failure. In: Kliegman RM, Behrman RE, Jenson HB, Stanton, BF, eds. Nelson Textbook of Pediatrics. 18th ed. Philadelphia, Pa: Saunders Elsevier; 2007:421-423

Pope J, McBride J. Consultation with the specialist: respiratory failure in children. Pediatr Rev. 2004;25:160-167. Available at: http://pedsinreview.aappublications.org/cgi/content/full/25/5/160

Page 533: AAP PREP 2009

2009 PREP SA on CD-ROM

page 533

Question: 154

A 12-year-old girl who developed type 1 diabetes at age 3 years comes in with her parents for a health supervision visit. Her diabetes control has been excellent. Physical examination reveals Sexual Maturity Rating 2 pubic hair and breast development and a palpable and somewhat firm thyroid gland.

Of the following, the MOST likely cause of the thyroid enlargement is

A. Graves disease

B. Hashimoto thyroiditis

C. iodine deficiency

D. multinodular goiter

E. pubertal thyroid enlargement

Page 534: AAP PREP 2009

2009 PREP SA on CD-ROM

page 534

Critique: 154 Preferred Response: B

Approximately 10% of people who have type 1 diabetes develop autoantibodies against the thyroid or chronic lymphocytic thyroiditis, sometimes referred to as Hashimoto thyroiditis. This is more common in children in whom the onset of type 1 diabetes occurs before the age of 10 years, such as the girl described in the vignette. A smaller percentage, perhaps 50% or more of individuals who have chronic lymphocytic thyroiditis, eventually develop hypothyroidism, and a very small number may develop other autoimmune thyroid disorders, including Graves disease. Iodine deficiency is not very common in the United States because iodine is freely available from sources such as iodized salt, seafood, iodophors used to clean stainless steel milk storage containers, and sodium alginate thickeners derived from seaweed that are found in many fast foods. Multinodular goiter would be unusual in an iodine-sufficient child of this age without pre-existing thyroid problems. On physical examination, a multinodular goiter has a bumpy or irregular firm surface. Although the thyroid becomes somewhat more palpable during puberty, perhaps because of increased need for thyroid hormone during rapid growth, it is soft and not firm. Thyroid autoimmunity may be associated with other autoimmune disorders. Approximately 3% to 4% of individuals who have chronic lymphocytic thyroiditis develop celiac disease. Vitiligo also can be found. Less frequently, adrenal insufficiency, pernicious anemia, ovarian failure, type 1 diabetes, or other autoimmune disorders can develop in an individual who initially has chronic lymphocytic thyroiditis. Some have classified these disorders as polyglandular autoimmune syndromes type 1, 2, or 3. Type 1 is caused by a known gene defect in AIRE, an immune regulator. Individuals who have this rare severe autosomal recessive disorder can exhibit candidiasis, hypoparathyroidism, Addison disease, and pernicious anemia in addition to type 1 diabetes, vitiligo, and other autoimmunities. A specific underlying gene defect has not been identified in those who have type 2 and type 3 autoimmune endocrinopathies, but these conditions have linkages to human lymphocyte antigen types related to the autoimmune response. Type 2 autoimmune endocrinopathy includes autoimmune thyroid disease or diabetes with adrenal insufficiency. The type 3 disorder usually occurs in adults and is comprised of variable combinations of autoimmune endocrine disorders, but not adrenal insufficiency.

References:

Aldasouqi SA, Akinsoto OPA, Jabbour SA. Polyglandular autoimmune syndrome type 1. eMedicine Specialties, Endocrinology, Multiple Endocrine Disease and Miscellaneous Endocrine Disease. 2006. Available at: http://www.emedicine.com/med/topic1867.htm

Kaloumenou I, Alevizaki M, Ladopoulos C, et al. Thyroid volume and echostructure in schoolchildren living in an iodine-replete area: relation to age, pubertal stage, and body mass index. Thyroid. 2007;17:875-881. Abstract available at: http://www.ncbi.nlm.nih.gov/pubmed/17956161

Kordonouri O, Hartmann R, Deiss D, Wilms M, Grüters-Kieslich A. Natural course of autoimmune thyroiditis in type 1 diabetes: association with gender, age, diabetes duration, and puberty. Arch Dis Child. 2005;90:411-414. Abstract available at: http://www.ncbi.nlm.nih.gov/pubmed/15781936

LaFranchi S. Thyroiditis. In: Kliegman RM, Behrman RE, Jenson HB, Stanton BF, eds. Nelson Textbook of Pediatrics. 18th ed. Philadelphia, Pa: Saunders Elsevier; 2007:2327-2329

Page 535: AAP PREP 2009

2009 PREP SA on CD-ROM

page 535

Question: 155

A 9-year-old child has been struggling in his regular third-grade classroom and has not yet received additional educational support. A comprehensive psychoeducational evaluation reveals a significant discrepancy between cognitive testing scores and academic performance for reading and writing.

Of the following, the BEST educational intervention for this boy is

A. after-school private tutoring for language arts

B. mainstream classroom with preferential seating

C. resource services for language arts and reading

D. self-contained classroom for all subjects

E. summer school so he can catch up

Page 536: AAP PREP 2009

2009 PREP SA on CD-ROM

page 536

Critique: 155 Preferred Response: C

The Individuals with Disabilities Education Act (IDEA) of 1990 (PL 101-476) defines the guidelines for education of children in the United States who have specific learning disabilities. According to the "least restrictive clause" in IDEA, children who have learning disorders should be integrated into the mainstream classroom as much as possible. The child described in the vignette is having learning issues in the area of reading and writing. The most appropriate setting for him is in the mainstream class for all of his subjects except language arts and reading. For these skills, he should receive extra educational support. Typically, this is accomplished by having the student go to a specialized classroom (resource room) staffed by a special education teacher. Preferential seating close to the teacher in a regular classroom will not address his areas of academic weakness. Although he may benefit from additional tutoring and summer school, these strategies could be offered in addition to resource services. Children who have more significant learning issues (eg, autism spectrum disorder, cognitive impairment) may require a self-contained classroom that provides more individualized and intensive educational support.

References:

American Academy of Pediatrics Committee on Children With Disabilities. The pediatrician's role in development and implementation of an Individual Education Plan and/or an Individual Family Service Plan. Pediatrics. 1999;104:124-127. Available at: http://pediatrics.aappublications.org/cgi/content/reprint/104/1/124

Council on Children With Disabilities. Policy statement: provision of educationally related services for children and adolescents with chronic diseases and disabling conditions. Pediatrics. 2007;119:1218-1223. Available at: http://pediatrics.aappublications.org/cgi/content/full/119/6/1218

Ewen JB, Shapiro BK. Specific learning disabilities. In: Accardo PJ. Capute & Accardo's Neurodevelopmental Disabilities in Infancy and Childhood. Volume II: The Spectrum of Neurodevelomental Disabilities. 3rd ed. Baltimore, Md: Paul H. Brookes Publishing Co; 2008:553-577

Fessler MA, Plourde PA. Psychoeducational assessment. In: Accardo PJ. Capute & Accardo's Neurodevelopmental Disabilities in Infancy and Childhood. Volume II: The Spectrum of Neurodevelomental Disabilities. 3rd ed. Baltimore, Md: Paul H. Brookes Publishing Co; 2008:591-610

Page 537: AAP PREP 2009

2009 PREP SA on CD-ROM

page 537

Question: 156

You are the physician for the child care center that your child attends. When an outbreak of diarrhea occurs at the center, the director calls you. She knows infections due to Giardia lamblia are common in child care centers, but asks if she needs to be worried about anything else.

Of the following, the organism MOST likely to cause a child care outbreak is

A. Aeromonas hydrophila

B. Campylobacter jejuni

C. Salmonella sp

D. Shigella sp

E. Yersinia enterocolitica

Page 538: AAP PREP 2009

2009 PREP SA on CD-ROM

page 538

Critique: 156 Preferred Response: D

Outbreaks of diarrheal illness can occur in child care facilities when caretakers do not adhere to hand hygiene policies and the pathogen requires a small inoculum for disease production. Giardia lamblia and Shigella sp commonly cause child care center outbreaks. Shigella are gram-negative bacilli that are spread readily person-to-person through fecal-oral and oral-anal contamination or by houseflies or other contaminated fomites. Among the common bacterial causes of gastroenteritis, Shigella is unique because ingestion of as few as 10 organisms can cause human disease. There are four primary Shigella pathogens: S sonnei, S flexneri, S boydii, and S dysenteriae. The typical incubation period is 2 to 4 days, and the illness caused by Shigella can range from a mild diarrhea to life-threatening dysentery. Fever and abdominal cramps with high-volume, watery stools followed 24 to 48 hours later by small-volume, bloody mucoid stools with tenesmus is a common presentation. Bloody mucoid stools are more common with diarrhea due to S dysenteriae and S flexneri; S boydii and S sonnei usually cause only watery diarrhea. Seizures can occur, with an incidence that is higher than would be expected from febrile seizures alone. Other complications can include Reiter syndrome, hemolytic-uremic syndrome, colonic perforation, and toxic encephalopathy. Stool microscopy revealing a large number of neutrophils in a patient who has clinical findings suggestive of shigellosis supports the diagnosis but is not specific for any species of Shigella. Aeromonas hydrophila, Campylobacter jejuni, Salmonella sp, and Yersinia enterocolitica are not commonly involved in outbreaks of gastrointestinal disease in child care centers due to their large inoculum requirements.

References:

American Academy of Pediatrics. Children in out-of-home child care. In: Pickering LK, Baker CJ, Long SS, McMillan JA, eds. Red Book: 2006 Report of the Committee on Infectious Diseases. 27th ed. Elk Grove Village, Ill: American Academy of Pediatrics; 2006:130-145.

American Academy of Pediatrics. Shigella infections. In: Pickering LK, Baker CJ, Long SS, McMillan JA, eds. Red Book: 2006 Report of the Committee on Infectious Diseases. 27th ed. Elk Grove Village, Ill: American Academy of Pediatrics; 2006:589-591

Spence JT, Cheng TL. In brief: Shigella species. Pediatr Rev. 2004;25:329-330. Available at: http://pedsinreview.aappublications.org/cgi/content/full/25/9/329

Page 539: AAP PREP 2009

2009 PREP SA on CD-ROM

page 539

Question: 157

A frantic mother brings in her three children, all of whom suddenly developed fevers, red and sore eyes, headaches, and sore throats 2 days after attending a swimming party at a country club. The mother states that five other children and 10 adults who attended the party have similar symptoms. On physical examination, all of the children have temperatures higher than 102.0°F (38.9°C), bilateral conjunctivitis, nasal congestion, and exudative pharyngitis.

Of the following, the MOST likely cause of these findings is

A. adenovirus

B. Chlamydia trachomatis

C. Epstein-Barr virus

D. Leptospira sp

E. Mycoplasma pneumoniae

Page 540: AAP PREP 2009

2009 PREP SA on CD-ROM

page 540

Critique: 157 Preferred Response: A

The respiratory tract is the most common site of adenoviral infections. Clinical presentations of infection include symptoms of the common cold (cough, fever, sore throat, and rhinorrhea), pharyngitis, tonsillitis, otitis media, pharyngoconjunctival fever, acute hemorrhagic conjunctivitis, croup, bronchiolitis, and a pertussislike syndrome. Pharyngoconjunctival fever is characterized by conjunctivitis (Item C157), pharyngitis, rhinitis, cervical adenitis, and fever. Community outbreaks of pharyngoconjunctival fever have been attributed to exposure to water from contaminated swimming pools and fomites from shared towels. The onset is acute, and fever and symptoms last for 3 to 5 days. The children described in the vignette have physical findings consistent with adenovirus-associated pharyngoconjunctival fever. Conjunctivitis is not seen with disease caused by Mycoplasma pneumoniae. Conjunctivitis is one of the most distinct findings with Leptospira infection, but pharyngitis usually is not seen, and patients are much more ill, exhibiting chills, myalgias, uveitis, rash, and aseptic meningitis. Conjunctivitis is seen in a small percentage of persons infected with Epstein-Barr virus, but community outbreaks associated with contaminated swimming pools do not occur and do not represent a mode of disease transmission. Chlamydia trachomatis is associated with neonatal conjunctivitis and trachoma, but trachoma is rare in the United States.

References:

American Academy of Pediatrics. Adenovirus infections. In: Pickering LK, Baker CJ, Long SS, McMillan JA, eds. Red Book: 2006 Report of the Committee on Infectious Diseases. 27th ed. Elk Grove Village, Ill: American Academy of Pediatrics; 2006:202-204

American Academy of Pediatrics. Chlamydia trachomatis. In: Pickering LK, Baker CJ, Long SS, McMillan JA, eds. Red Book: 2006 Report of the Committee on Infectious Diseases. 27th ed. Elk Grove Village, Ill: American Academy of Pediatrics; 2006:252-257

American Academy of Pediatrics. Leptospirosis. In: Pickering LK, Baker CJ, Long SS, McMillan JA, eds. Red Book: 2006 Report of the Committee on Infectious Diseases. 27th ed. Elk Grove Village, Ill: American Academy of Pediatrics; 2006:424-426

American Academy of Pediatrics. Mycoplasma pneumoniae infections. In: Pickering LK, Baker CJ, Long SS, McMillan JA, eds. Red Book: 2006 Report of the Committee on Infectious Diseases. 27th ed. Elk Grove Village, Ill: American Academy of Pediatrics; 2006:468-470

Singh-Naz N, Rodriguez W. Adenoviral infections in children. Adv Pediatr Infect Dis. 1996;11:365-388

Page 541: AAP PREP 2009

2009 PREP SA on CD-ROM

page 541

Critique: 157

Adenovirus infection may produce conjunctivitis. (Courtesy of Red Book® Online)

Page 542: AAP PREP 2009

2009 PREP SA on CD-ROM

page 542

Question: 158

A 16-year-old girl presents with symptoms of burning with micturition and back pain. Her temperature is 101.3°F (38.5°C), heart rate is 88 beats/min, respiratory rate is 14 breaths/min, and blood pressure is 108/64 mm Hg. You can elicit costovertebral angle tenderness on the left side and suprapubic tenderness. Her urinalysis demonstrates a urine specific gravity of 1.025, pH of 6.5, 2+ blood, 1+ protein, 3+ leukocyte esterase, and positive for nitrite. Microscopy reveals 5 to 10 red blood cells/high-power field (HPF), 50 to 100 white blood cells/HPF, 4+ bacteria, and occasional squamous epithelial cells.

Of the following, the MOST likely diagnosis is

A. acute bacterial cystitis

B. acute glomerulonephritis

C. acute pyelonephritis

D. bacterial urethritis

E. nephrolithiasis

Page 543: AAP PREP 2009

2009 PREP SA on CD-ROM

page 543

Critique: 158 Preferred Response: C

The adolescent described in the vignette presents with classic symptoms of acute pyelonephritis (upper urinary tract infection [UTI]): fever, back pain, and dysuria associated with physical findings of costovertebral angle and suprapubic tenderness. UTIs typically are classified as lower urinary tract (involving the bladder and urethra) and upper urinary tract (involving the ureters, renal pelvis, and kidneys). In an adolescent, a bladder infection (cystitis) is not accompanied by flank pain or fever. Urethritis, which may be caused by a sexually transmitted infection, is accompanied by symptoms of dysuria, but not symptoms of flank pain or fever. Distinguishing between upper and lower UTI may be more difficult in younger children. In neonates and infants, the presence of fever in the setting of a UTI characterizes an upper tract infection (pyelonephritis). In contrast, the absence of fever suggests lower tract infection. The diagnosis often is made during the evaluation of an infant for fever and irritability. A study from Taiwan revealed that a UTI was present in 13.6% of infants younger than 8 weeks of age who presented with fever. Urine white blood cell counts greater than 10/high-power field (pyuria) were the best indicators of UTI in this setting. Bacteremia accompanies UTI in 31% of pediatric patients younger than 1 month of age and 21% of those 1 to 2 months of age. Febrile infants who have no apparent source of fever had UTI in 7.5% of cases in another study. Older infants present with fever and vomiting, and the diagnosis is based on a positive urine culture collected by transurethral or suprapubic catheterization. The incidence of bacteremia falls to 3% in children 2 to 36 months of age. As children develop verbal skills, it becomes easier to distinguish between upper and lower tract infection because they can express their symptoms of dysuria and frequency. Acute glomerulonephritis does not present with dysuria and flank pain. Nephrolithiasis can present with flank pain and possibly dysuria (if accompanied by crystalluria), but the presence of suprapubic pain, fever, and disproportionate pyuria to hematuria in the setting of a positive nitrite result make this diagnosis unlikely.

References:

Ginsburg CM, McCracken GH Jr. Urinary tract infections in young infants. Pediatrics. 1982;69;409-412. Available at: http://pediatrics.aappublications.org/cgi/content/abstract/69/4/409

Hoberman A, Chao HP, Keller DM, Hickey R, Davis HW, Ellis D. Prevalence of urinary tract infection in febrile infants. J Pediatr. 1993;123:17-23. Abstract available at: http://www.ncbi.nlm.nih.gov/pubmed/8320616

Johnson CE. New advances in childhood urinary tract infections. Pediatr Rev. 1999:20:335-342. Available at: http://pedsinreview.aappublications.org/cgi/content/full/20/10/335

Raszka WV Jr, Khan O. Pyelonephritis. Pediatr Rev. 2005;26:364-370. Available at: http://pedsinreview.aappublications.org/cgi/content/full/26/10/364

Page 544: AAP PREP 2009

2009 PREP SA on CD-ROM

page 544

Question: 159

A 17-year-old boy presents to the emergency department with respiratory distress and hypoxia (room air Po2, 86%). His parents called 911 after their son started "gasping for air." The boy has a history of moderate persistent asthma that recently worsened after a viral infection. He uses a daily inhaled corticosteroid and an as-needed beta2 agonist inhaler. During the past 2 days, he has not been able to go to school because of his breathing problems, and his parents have used his nebulizer every 2 hours over the past 12 hours. On physical examination, the boy is awake and responsive to questions, but his respiratory rate is 34 breaths/min, and he has nasal flaring and intercostal retractions. Lung examination demonstrates equal breath sounds bilaterally but obvious expiratory wheezing with a prolonged expiratory phase. The results of an arterial blood gas on a nonrebreather with 100% oxygen are: pH of 7.35 (normal, 7.35 to 7.45), Pco2 of 45 mm Hg (normal, 35 to 45 mm Hg), bicarbonate of 24.0 mEq/L (24.0 mmol/L)(normal, 22.0 to 26.0 mEq/L [22.0 to 26.0 mmol/L]), and Po2 of 90 mm Hg (normal, 75 to 100 mm Hg).

Of the following, a TRUE statement regarding the management of this boy’s condition is that

A. a normal carbon dioxide value on an arterial blood gas measurement makes an asthma exacerbation unlikely

B. if assisted ventilation is required, the ventilator rate should be set at 34 breaths/min

C. initial corticosteroid therapy can be administered either orally or intravenouslyD. the parents should have doubled his inhaled corticosteroid dose when his beta2 agonist was not working

E. the patient should blow into a paper bag to help calm him

Page 545: AAP PREP 2009

2009 PREP SA on CD-ROM

page 545

Critique: 159 Preferred Response: C

Regardless of the level of asthma severity, affected patients can experience exacerbations. The National Heart, Lung, and Blood Institute 2007 Expert Panel 3 Asthma Guidelines have outlined in detail the importance of recognition and treatment of asthma exacerbations both at home and in the emergency department (available at http://www.nhlbi.nih.gov/guidelines/asthma/asthgdln.pdf). The teenager described in the vignette is having a severe asthma exacerbation that has not improved with multiple short-acting beta2 agonist (SABA) nebulization treatments administered at home. Although he is able to answer questions and is alert, he exhibits hypoxia, tachypnea, and accessory muscle use. Additional signs of severe obstruction include peak expiratory flow rates less than 50% of predicted, quiet breath sounds, an inability to speak whole phrases, and cyanosis. He should receive supplemental oxygen, an arterial blood gas, a SABA nebulizer treatment, and corticosteroids as soon as possible. Most patients, including this boy, should be able to tolerate oral corticosteroids, which should be continued for 3 to 10 days. Intravenous corticosteroids typically are reserved for life-threatening asthma exacerbations. Most experts and published experience support oral and intravenous corticosteroids as being equally effective. One commonly used technique during an asthma exacerbation is to double the inhaled steroid dose at the onset of the asthma exacerbation. However, this recommendation is not supported by results of a controlled trial or the 2007 Expert Panel 3 asthma guidelines. One of the primary concerns during an acute exacerbation is whether the patient has impending respiratory failure. A patient who is too dyspneic to speak, perspiring, or has a peak expiratory flow of less than 25% may need to be intubated. An initial blood gas measurement can be helpful in assessing the patient's ventilatory status. However, a normal or elevated PCO2 in the context of significant tachypnea is worrisome and requires aggressive treatment. Indeed, a patient who has a respiratory rate of 34 breaths/min should have a low PCO2 (respiratory alkalosis) unless there is airway obstruction. Patients who have respiratory failure due to asthma and require intubation should be treated differently from other patients who have respiratory failure requiring intubation. Because the expiratory phase during an asthma exacerbation is prolonged, setting the ventilatory rate to match the patient's tachypnea does not allow sufficient time to expel carbon dioxide. An inappropriately high ventilator rate also can lead to increased thoracic pressure, increased positive end-expiratory pressure, and possibly a pneumothorax. A slower rate (8 to 12 breaths/min) is recommended if such a patient is intubated. Blowing in a paper bag has no role in the treatment of asthma. An asthma action plan should be developed for patients and their families to prevent asthma exacerbations and emergency department visits. The action plan can help patients understand the symptoms associated with asthma exacerbation, the role of peak expiratory flows in determining the severity of an exacerbation, and specific action(s) to take during an exacerbation.

References:

Harrison TW, Oborne J, Newton S, Tattersfield AE. Doubling the dose of inhaled corticosteroid to prevent asthma exacerbations: randomised controlled trial. Lancet. 2004;363:271-275. Abstract available at: http://www.ncbi.nlm.nih.gov/pubmed/14751699

Keeley D, McKean M. Asthma and other wheezing disorders in children. BMJ Clinical Evidence. 2006. Available for subscription at: http://clinicalevidence.bmj.com/ceweb/conditions/chd/0302/0302.jsp#Q1

National Asthma Education and Prevention Program. Expert Panel Report 3 (EPR-3): Guidelines for the Diagnosis and Management of Asthma-Summary Report 2007. J Allergy Clin Immunol. 2007;120:S94-S138. Abstract available at: http://www.ncbi.nlm.nih.gov/pubmed/17983880

Page 546: AAP PREP 2009

2009 PREP SA on CD-ROM

page 546

Question: 160

You are camping with a group of boys at a rural campground in the southeastern United States when one of the campers is bitten by a snake. His tent mates kill the snake (Item Q160). The victim is crying and guarding his right hand. On examination of the boy’s hand, you note several small, erythematous abrasions but no swelling or ecchymosis.

Of the following, the MOST appropriate course of action is to

A. apply a tourniquet above the bite

B. apply ice to the wound

C. incise and suction the wound

D. provide local wound care

E. transport the boy to the hospital for antivenom

Page 547: AAP PREP 2009

2009 PREP SA on CD-ROM

page 547

Question: 160

Snake, as described in the vignette. (Courtesy of D. Krowchuk, wih special thanks to Sci Works, Winston-Salem, NC)

Page 548: AAP PREP 2009

2009 PREP SA on CD-ROM

page 548

Critique: 160 Preferred Response: D

Each year approximately 7,000 to 8,000 venomous snake bites occur in the United States that result in five to six deaths and significant morbidity. Although most bites occur in young adult males, children are disproportionately represented among the fatalities because their smaller size leads to a higher venom dose per kilogram. In the United States, almost all venomous snakes are pit vipers (cottonmouths, rattlesnakes [Item C160A], copperheads [Item C160B]). Of these, eastern and western diamondback rattlesnakes are responsible for most fatal bites. Besides pit vipers, the only other venomous snake in the United States is the coral snake, which is found in the southern United States. Management of a snake bite is predicated on whether the bite was caused by a venomous or nonvenomous snake, which can be determined either by examining the (preferably dead) snake or by the appearance of the wound. A pit viper can be identified by its triangular-shaped head, elliptical pupils, and presence of fangs; a nonvenomous snake has a round head and pupils and no fangs or rattle (Item C160C). Although the coral snake also has a round head, it is recognizable by its unique striped coloration in which red and yellow bands are adjacent ("red on yellow, kill a fellow") (Item C160D and Item C160E). Because the snake often is not available for inspection, examination of the wound should yield findings that can guide further treatment. A pit viper bite generally has visible fang marks; a nonvenomous snake typically leaves an abrasion or scratches at the site, such as described in the vignette. If the wound has been envenomated (approximately 25% of pit viper bites are "dry" or nonenvenomated), pain at the site is immediate, followed by rapid development of erythema, swelling, and regional spread of the pain. Over the next several hours, hemorrhagic bulla and lymphangitis may be noted (Item C160F). Early systemic symptoms include nausea, vomiting, hypotension, and tachycardia. In contrast to pit viper venom, which contains a mixture of hemotoxic, cardiotoxic, myotoxic, and proteolytic enzymes, coral snake venom is primarily a neurotoxin. The bite typically is painless, but neurologic and autonomic symptoms, including hypersalivation, weakness, altered mental status, cranial nerve palsies, and respiratory paralysis, may develop within 12 hours. Both the snake and wound appearance for the patient described in the vignette indicate that he was bitten by a nonvenomous snake. Local wound care and tetanus immunization update are indicated. Had the wound been envenomated, field treatment would include maintenance of the victim's respiratory and cardiovascular functions; immobilization and dependent positioning of the extremity; and removal of watches, rings, and tight clothing. Use of tourniquets, wound incision and suction, or application of ice is not recommended and may lead to increased tissue injury. Rapid transport to the nearest appropriate facility where antivenom can be given is critical. The currently available pit viper antivenom is the sheep-derived Fab antibody-segment-based product (crotaline Fab). In six small studies performed to evaluate safety and efficacy, crotaline Fab controlled both local and systemic symptoms effectively and caused fewer cases of acute allergic reactions or serum sickness than the previously available but no longer manufactured horse-derived crotaline antivenom. Although the studies included few children, the limited data support its use.

References:

Gold BS, Dart RC, Barish RA. Bites of venomous snakes. N Engl J Med. 2002;347:347-356. Extract available at: http://content.nejm.org/cgi/content/extract/347/5/347

Schmidt JM. Antivenom therapy for snakebites in children: is there evidence? Curr Opin Pediatr. 2005;17:234-238. Abstract available at: http://www.ncbi.nlm.nih.gov/pubmed/15800419

Singletary EM, Rochman AS, Bodmer JCA, Holstege CP. Envenomations. Med Clin North Am. 2005;89:1195-1224. Abstract available at: http://www.ncbi.nlm.nih.gov/pubmed/16227060

Page 549: AAP PREP 2009

2009 PREP SA on CD-ROM

page 549

Critique: 160

Blacktailed rattlesnake. Note the triangular-shaped head characteristic of venomous snakes. (Courtesy of J. Brashears and M. Feldner)

Page 550: AAP PREP 2009

2009 PREP SA on CD-ROM

page 550

Critique: 160

Southern copperhead. (Courtesy of the Centers for Disease Control and Prevention, Public Health Image Library, Edward J. Wozniak)

Page 551: AAP PREP 2009

2009 PREP SA on CD-ROM

page 551

Critique: 160

Comparison of venomous snakes (pit vipers) and nonvenomous snakes in the United States. (Reprinted with permission from Gold BS, Dart RC, Barish RA. Bites of venoumous snakes. New Engl J Med. 2002;347:347-356 Copyright © 2002 Massachusetts Medical Society. All rights reserved.

Page 552: AAP PREP 2009

2009 PREP SA on CD-ROM

page 552

Critique: 160

Arizona coral snake: Note the wide red and black rings separated by narrow pale yellow rings. The coral snake can be distinguished from similarly colored harmless snakes by the adjacent red and yellow bands ("red on yellow, kill a fellow"). See item C160E. (Courtesy of J Brashears and M Feldner)

Page 553: AAP PREP 2009

2009 PREP SA on CD-ROM

page 553

Critique: 160

Nonvenomous milk snake: Although the markings are similar to those of a coral snake, red and black (not yellow) bands are adjacent ("red on black, friend of Jack"). (Courtesy of the Centers for Disease Control and Prevention, Public Health Image Library, Edward J. Wozniak)

Page 554: AAP PREP 2009

2009 PREP SA on CD-ROM

page 554

Critique: 160

Ecchymosis, swelling, and ruptured hemorrhagic bullae may be observed following the bite of a venomous snake. (Courtesy of M. Rimsza)

Page 555: AAP PREP 2009

2009 PREP SA on CD-ROM

page 555

Question: 161

An 8-year-old girl presents to the emergency department with a history of recurrent severe vomiting. According to the family, she has had four similar episodes in the past 6 months. Each time, the child awakens from sleep, vomits every 20 minutes for 6 hours, and then goes back to bed. Between episodes, which occur approximately every 4 to 6 weeks, the child is happy and playful. Normal results have been found on prior head magnetic resonance imaging, upper gastrointestinal radiograph series, and renal ultrasonography. In the emergency department, the girl is quiet, somewhat listless, and prefers to be in a dark room with an emesis basin. Results of physical examination are unremarkable, and optic discs are sharp. After 8 hours of intravenous hydration, the symptoms resolve, and she is discharged.

Of the following, the MOST appropriate medication to treat her underlying condition is

A. amitriptyline

B. lubiprostone

C. omeprazole

D. ranitidine

E. sucralfate

Page 556: AAP PREP 2009

2009 PREP SA on CD-ROM

page 556

Critique: 161 Preferred Response: A

The history described for the girl in the vignette is most consistent with cyclic vomiting syndrome (CVS), a symptom complex usually considered a migraine variant. The hallmarks of this syndrome are intense periods of vomiting that usually last between 8 and 48 hours. The typically affected patient has an aura of intense nausea or malaise that is followed by vomiting of undigested food, gastric fluid, and ultimately bile. During the peak of the attack, patients may vomit as frequently as three to six times per hour. Physical examination findings usually are benign, but patients characteristically are listless and in a state characterized as "conscious coma." Abdominal pain, dehydration, headache, and electrolyte abnormalities can accompany the episode. Most episodes are self-limited and resolve spontaneously. In between episodes, the patient usually feels completely well and has no significant symptoms of nausea or vomiting. Precipitating factors for episodes include infection, psychological stress, and menstrual periods (catamenial migraine), but stressors cannot be identified in most episodes. Although most patients who have CVS have a migraine variant, other conditions can present with recurrent episodic vomiting, including pancreatitis, ureteropelvic junction obstruction, malrotation, eosinophilic gastroenteritis, and even intracranial lesions. Most notably, certain metabolic defects (urea cycle defects, fatty acid oxidation defects) can present with recurrent vomiting. Therefore, a thorough evaluation should exclude other causes. Helpful diagnostic tests include upper gastrointestinal radiographic series (to exclude malrotation), abdominal ultrasonography (to exclude ureteropelvic junction obstruction), and magnetic resonance imaging of the head (to exclude intracranial lesions). In addition, in the middle of a CVS episode, electrolytes should be measured and metabolic studies (ammonia, urinary organic acids and acylcarnitine, plasma amino acids, pyruvate, and lactate) obtained. Supportive care during a vomiting episode of CVS includes intravenous fluids, antiemetics (ondansetron), and antianxiety medications (lorazepam). If the episodes are frequent, prophylactic antimigraine therapy is effective in reducing their frequency and severity. This therapy should be instituted in consultation with a specialist (pediatric neurologist or gastroenterologist). Daily amitriptyline is used commonly as first-line prophylactic therapy for children older than age 5 years; cyproheptadine often is used in younger children. Lubiprostone is used in the treatment of refractory constipation. Omeprazole, ranitidine, and sucralfate are used to treat reflux or gastritis and have not been shown to be useful in treating CVS.

References:

Cyclic Vomiting Syndrome Association of America web site. Available at: http://www.cvsaonline.org

Li BU, Misiewicz L. Cyclic vomiting syndrome: a brain-gut disorder. Gastroenterol Clin North Am. 2003;32:997-1019. Abstract available at: http://www.ncbi.nlm.nih.gov/pubmed/14562585

Page 557: AAP PREP 2009

2009 PREP SA on CD-ROM

page 557

Question: 162

You are examining a preterm infant who was delivered at 34 weeks’ gestation due to premature labor and a maternal urinary tract infection. His mother is 23 years old and has had insulin-dependent diabetes mellitus for many years. Her diabetes reportedly was well managed during the pregnancy, and a recent hemoglobin A1c measurement was 7.0%. On physical examination, the infant is appropriately grown for gestational age, weighs 2 kg, and has a gestational age assessment that equates to 34 weeks. He requires supplemental oxygen with an FiO2 of 0.40 administered by continuous nasal positive airway pressure.

Of the following, the GREATEST concerns for this infant related to his mother’s diabetes are

A. anemia, hypotension, and hypercalcemia

B. apnea, bradycardia, and hypercalcemia

C. hyperglycemia, hyperacusis, and aortic stenosis

D. hyperglycemia, hypocalcemia, and sepsis

E. hypoglycemia, hypocalcemia, and polycythemia

Page 558: AAP PREP 2009

2009 PREP SA on CD-ROM

page 558

Critique: 162 Preferred Response: E

The infant described in the vignette is an appropriately grown infant of a diabetic mother (IDM). In this instance, the mother has longstanding insulin insufficiency (type 1 diabetes mellitus). Other classifications of diabetes in pregnancy include previously existing insulin resistance (type 2 diabetes mellitus) and gestational diabetes (insulin resistance occurring only in pregnancy). Common sequelae of maternal diabetes seen in the fetus include fetal macrosomia, polyhydramnios, and congenital anomalies. Neonatal problems requiring evaluation and management in the IDM include: * Birth defects (congenital anomalies) * Septal hypertrophy of the myocardium and possible heart failure * Hypoglycemia (not hyperglycemia) * Hypocalcemia (not hypercalcemia) * Hyperbilirubinemia * Macrosomia with or without visceromegaly * Polycythemia (not anemia) * Polyhydramnios * Renal vein thrombosis * Neonatal small left colon Sepsis is not more common in IDMs, nor is hypotension, except in the presence of septal hypertrophy. Apnea and bradycardia may occur in a preterm 34-week gestation infant, but it is not related to maternal diabetes. Aortic stenosis is not a common cardiac sequela in IDMs. Hyperacusis is not related to maternal diabetes.

References:

Cowett RM. Neonatal care of the infant of the diabetic mother. NeoReviews. 2002;3:e190-e196. Available for subscription at: http://neoreviews.aappublications.org/cgi/content/full/3/9/e190

Cowett RM. The infant of the diabetic mother. NeoReviews. 2002;3:e173-e189. Available for subscription at: http://neoreviews.aappublications.org/cgi/content/full/3/9/e173

Kalhan SC, Parimi PS. Disorders of carbohydrate metabolism. In: Martin RJ, Fanaroff AA, Walsh MC, eds. Fanaroff and Martin's Neonatal-Perinatal Medicine. 8th ed. Philadelphia, Pa: Mosby Elsevier; 2006:1467-1490

Page 559: AAP PREP 2009

2009 PREP SA on CD-ROM

page 559

Question: 163

You are discussing the pharmacokinetics and potential interactions of drugs used in the pediatric population with a group of medical students. One of them asks you if medications should be taken with food, and you respond that interactions between food and drugs may either reduce or increase the drug absorption, depending on the type of medication.

Of the following, the MOST accurate advice regarding taking medications with food is that

A. amoxicillin should not be given with meals

B. ciprofloxacin should be taken with dairy products

C. griseofulvin should be taken with high-protein meals

D. isoniazid should be taken with food

E. tetracycline should not be taken with dairy products

Page 560: AAP PREP 2009

2009 PREP SA on CD-ROM

page 560

Critique: 163 Preferred Response: E

It is important for the clinician to recognize that the concomitant use of foods and medications may result in an undesired response to the drug, either through decreased efficacy or increased risk of toxicity. Such interactions may result from a feature of the drug or food. For example, some medications, such as isotretinoin or griseofulvin, are more bioavailable when taken with fatty foods because they are lipophilic agents. Others, such as digoxin, have lower bioavailability with high-fiber meals because the fiber binds the drug. In fact, most food-drug interactions occur because some characteristic of the food affects the bioavailability of the drug. Components in some foods and dairy products chelate a portion of the drug and reduce bioavailability, which occurs with many antibiotics. Other antibiotics are acid-labile agents, so the increased gastric acid secretion that occurs with food intake may reduce their bioavailability. For some medications, the effect is minimal unless the dose is inadequate, but for others, the risk is higher and warrants careful attention to avoid treatment failure. Item C163 lists some antibiotics commonly used in the pediatric population that have potential food-drug interactions. In general, for medications that should not be taken with food, the drug should be given either 1 to 2 hours before or 2 hours after the ingestion of the food in question. Many other medications should be administered with caution to ensure proper bioavailability and efficacy. Patients should be advised to pay close attention to any instructions offered by their pharmacist to avoid adverse reactions or treatment failures.

References:

Gal P, Reed M. Principles of drug therapy. In: Kliegman RM, Behrman RE, Jenson HB, Stanton BF, eds. Nelson Textbook of Pediatrics. 18th ed. Philadelphia, Pa: Saunders Elsevier; 2007:331-338

Schmidt LE, Dalhoff K. Food-drug interactions. Drugs. 2002;62:1481-1502. Abstract available at: http://www.ncbi.nlm.nih.gov/pubmed/12093316

Page 561: AAP PREP 2009

2009 PREP SA on CD-ROM

page 561

Critique: 163

Page 562: AAP PREP 2009

2009 PREP SA on CD-ROM

page 562

Question: 164

During the health supervision visit of a term newborn boy, his mother relates that a cousin's child died at age 4 months from sudden infant death syndrome. She asks what she can do to prevent such an occurrence in her son.

Of the following, the single MOST important preventive measure is to

A. avoid use of a pacifier

B. cosleep in the parental bed for close observation

C. ensure side sleep positioning to prevent aspiration

D. ensure supine sleep positioning

E. use home apnea and bradycardia monitoring

Page 563: AAP PREP 2009

2009 PREP SA on CD-ROM

page 563

Critique: 164 Preferred Response: D

Sudden infant death syndrome (SIDS) remains a significant cause of death in the first postnatal year. However, the rate of death has diminished by more than 50% since the recommendation was made more than a decade ago for supine rather than prone sleep position for children younger than 1 year of age. In the United States, infant cosleeping with parents increases the risk of SIDS, possibly related to frank suffocation. However, the recommendation to avoid cosleeping remains controversial when providing support for breastfeeding and considering the prevalence of cosleeping worldwide. Further risk factors associated with cosleeping include parental obesity and sleeping on a very soft surface, such as a waterbed. Tobacco smoke exposure increases the risk of SIDS, especially in the setting of cosleeping. Although initial recommendations suggested placing the child either supine or on the side to prevent SIDS, subsequent research has shown that the side position also can increase the risk of SIDS, possibly due to the propensity for a bundled infant placed on his or her side to roll forward into the prone position. Therefore, parents should be urged to place the infant supine instead of on the side for sleep. Home apnea and bradycardia monitoring has not been shown to reduce the risk of SIDS, even in families where a previous child has died from SIDS. Its use is limited to preterm infants who have apnea of prematurity and infants in whom central apnea, known cardiac arrhythmia, or other identifiable cause of events that may respond to monitoring and cardiorespiratory resuscitation is a distinct possibility. Pacifier use may reduce the incidence of SIDS but is not without controversy due to possible interference with breastfeeding in the early weeks after birth. However, the peak incidence of SIDS occurs between 2 and 6 months, which is primarily after the establishment of successful breastfeeding during the first 2 postnatal weeks. Modeling of appropriate infant sleep position and good patient education in the hospital may be contributing factors to adoption of these practices.

References:

Colson ER, Levenson S, Rybin D, et al. Barriers to following the supine sleep recommendation among mothers at four centers for the Women, Infants, and Children Program. Pediatrics. 2006;118:e243-e250. Available at: http://pediatrics.aappublications.org/cgi/content/full/118/2/e243

Creery D, Mikrogianakis A. Sudden infant death syndrome. BMJ Clinical Evidence. 2006. Available for subscription at: http://clinicalevidence.bmj.com/ceweb/conditions/chd/0315/0315.jsp

Farrell PA, Weiner GM, Lemons JA. SIDS, ALTE, apnea, and the use of home monitors. Pediatr Rev. 2002;23:3-9. Available at: http://pedsinreview.aappublications.org/cgi/content/full/23/1/3

Fu LY, Moon RY. Apparent life-threatening events (ALTES) and the role of home monitors. Pediatr Rev. 2007;28:203-208. Available at: http://pedsinreview.aappublications.org/cgi/content/full/28/6/203

Hauck FR, Omojokun OO, Siadaty MS. Do pacifiers reduce the risk of sudden infant death syndrome? A meta-analysis. Pediatrics. 2005;116:e716-e723. Available at: http://pediatrics.aappublications.org/cgi/content/full/116/5/e716

Hein HA, Pettit SF. Back to Sleep: good advice for parents but not for hospitals? Pediatrics.

Page 564: AAP PREP 2009

2009 PREP SA on CD-ROM

page 564

2001;107:537-539. Available at: http://pediatrics.aappublications.org/cgi/content/full/107/3/537

Moon RY, Fu LY. Sudden infant death syndrome. Pediatr Rev. 2007;28:209-214. Available at: http://pedsinreview.aappublications.org/cgi/content/full/28/6/209

Page 565: AAP PREP 2009

2009 PREP SA on CD-ROM

page 565

Question: 165

You are leading teaching rounds with the residents at the hospital. They present an 18-month-old boy who has had 6 days of a temperature to at least 102.3°F (39.1°C). He also has nonexudative conjunctivitis, a polymorphous rash, erythema of his lips, and swelling of his hands and feet. The residents ask you to comment on the use of echocardiography in this condition.

Of the following, the MOST accurate statement about echocardiography in this disease is that

A. abnormal results at diagnosis suggest a poor outcome

B. it should be performed only if C-reactive protein concentrations are elevated

C. it should be performed to confirm the diagnosis

D. normal results at diagnosis obviate the need to repeat the study

E. the study may be useful in confirming atypical cases

Page 566: AAP PREP 2009

2009 PREP SA on CD-ROM

page 566

Critique: 165 Preferred Response: E

The diagnostic criteria for Kawasaki disease (KD) are fever for at least 5 days and at least four of the following five features: * Changes in the extremities (edema, erythema, desquamation) * Polymorphous exanthema * Conjunctival injection * Erythema and/or fissuring of the lips and oral cavity * Cervical lymphadenopathy In addition, to meet the diagnosis of KD, such findings should not be explainable by another disease process. The patient described in the vignette has had fever for more than 5 days and has four of the criteria of KD. He requires echocardiography for assessment and to obtain baseline measurements of the coronary arteries. Echocardiography is an important component of the evaluation of the patient who is suspected to have KD because it is a noninvasive, portable, and virtually risk-free method of assessing cardiac structure and function. The complete echocardiographic assessment of the patient who has acute KD includes an evaluation of cardiac function, determination of the presence of valvular dysfunction, assessment for pericardial effusion, and baseline measurement of the coronary arteries. Myocarditis, with or without valvular dysfunction and with or without pericardial effusion, can occur in those who have KD and usually is not significant. However, myocarditis must be suspected in the patient who has abnormal hemodynamic findings, and echocardiography should be pursued. By the end of the first week of the illness, as many as 50% of children who have KD exhibit echocardiographic evidence of coronary arteritis. This usually manifests as a "brightness" of the perivascular component of the coronary artery and is the result of edema and thickening of the vascular wall. In addition, there may be dilation, also referred to as ectasia, of the coronary arteries during this phase of the illness. Such dilation, typically transient and asymptomatic, is distinct from the coronary artery aneurysms that can develop in the second stage (2 to 4 weeks) of the illness (Item C165). Echocardiography also can play a role diagnosing KD that presents in an incomplete or atypical pattern (with fewer of the findings described previously at presentation). For example, a child who has had a fever of unknown origin for 5 or more days but only two or three of the clinical criteria of KD may have atypical KD. Newburger and colleagues have published an extensive protocol to facilitate the evaluation of such patients. For those who have suspected atypical KD, further laboratory evaluation, including measurement of C-reactive protein (CRP) and erythrocyte sedimentation rate (ESR), can be helpful. If the CRP is 3.0 mg/dL or greater and the ESR is 40 mm/hr or greater, supplemental laboratory tests should include echocardiography and assessment of serum albumin, hemoglobin, alanine aminotransferase, white blood cell count, and platelet count. If the CRP is less than 3.0 mg/dL and the ESR is less than 40 mm/hr, the child should be followed daily; if the fever persists for at least 7 days, echocardiography should be performed. Echocardiography also should be obtained, even if the fever resolves, if typical skin peeling occurs under the nail beds of the fingers and toes. Abnormal findings, such as coronary arteritis, in patients who have KD do not portend a poor outcome; rather, they are consistent with the diagnosis of KD. For this patient, who meets the criteria for the diagnosis, echocardiography is performed regardless of the findings on laboratory tests. Echocardiography neither confirms nor dispels the diagnosis for patients who meet the clinical criteria of KD, and normal results do not obviate the need to repeat the study to assess disease progression.

References:

American Academy of Pediatrics. Kawasaki disease. In: Pickering LK, Baker CJ, Long SS, McMillan JA, eds. Red Book: 2006 Report of the Committee on Infectious Disease. 27th ed. Elk Grove Village, Ill: American Academy of Pediatrics; 2006:412-415

Page 567: AAP PREP 2009

2009 PREP SA on CD-ROM

page 567

Miyashita M, Karasawa K, Taniguchi K, et al. Usefulness of real-time 3-dimensional echocardiography for the evaluation of coronary artery morphology in patients with Kawasaki disease. J Am Soc Echocardiogr. 2007;20:930-933. Abstract available at: http://www.ncbi.nlm.nih.gov/pubmed/17555938

Newburger JW, Takahashi M, Gerber MA, et al. AHA scientific statement. Diagnosis, treatment, and long-term management of Kawasaki disease. A statement for health professionals from the Committee on Rheumatic Fever, Endocarditis and Kawasaki Disease, Council on Cardiovascular Disease in the Young, American Heart Association. Circulation. 2004;110:2747-2771. Available at: http://circ.ahajournals.org/cgi/content/full/110/17/2747

Satou GM, Giamelli J, Gewitz MH. Kawasaki disease: diagnosis, management, and long-term implications. Cardiol Rev. 2007;15:163-169. Abstract available at: http://www.ncbi.nlm.nih.gov/pubmed/17575479

Page 568: AAP PREP 2009

2009 PREP SA on CD-ROM

page 568

Critique: 165

Computed tomography angiography of the heart with three-dimensional reconstruction shows a large aneurysm of the left coronary artery in a patient who has Kawasaki disease. (Courtesy of D. Mulvihill)

Page 569: AAP PREP 2009

2009 PREP SA on CD-ROM

page 569

Question: 166

A 6-year-old boy presents in late summer to the emergency department with a severe headache and muscle pains. He recently returned from a camping trip. On physical examination, he is febrile and has no focal weakness, but he suffers a prolonged tonic-clonic seizure and becomes unresponsive. Head computed tomography scan reveals no abnormalities. Acyclovir and fosphenytoin are administered. Magnetic resonance imaging shows subtle, diffuse signal change and thickening in the cerebral cortex, no signal changes in temporal lobes, and no meningeal enhancement.

Of the following, the MOST likely cause of the boy’s symptoms is

A. arbovirus

B. Borrelia burgdorferi

C. herpes simplex virus 1

D. Listeria monocytogenes

E. Taenia solium

Page 570: AAP PREP 2009

2009 PREP SA on CD-ROM

page 570

Critique: 166 Preferred Response: A

The child described in the vignette has symptoms of an acute viral infection, with fever, muscle pains, and a prolonged seizure. Involvement of the central nervous system should be suspected because of the seizure. The magnetic resonance imaging (MRI) findings suggest focal encephalitis that could be due to an arbovirus or herpes simplex infection. Arbovirus infection is more likely to occur in late summer because arbovirus is transmitted to humans by mosquitoes. In addition, herpes simplex infection results in necrotizing focal encephalitis in the temporal lobes (Item C166A) rather than the subtle thickening described in the vignette. In both cases, focal infection and inflammation in the brain's cortex can lead to seizures. In the case of herpes, outside the neonatal period, the temporal lobe infection occurs because of reactivation of the herpes simplex virus (HSV) in the trigeminal nerve and spread directly into adjacent brain tissue. Virus also can enter the brain via the blood, in which case generalized encephalitis is more likely. Examples include HSV in the neonatal period or measles, although this is much rarer because of the measles-mumps-rubella vaccine. Other viruses prone to cause encephalitis include human immunodeficiency virus, cytomegalovirus, and rabies. The other infectious etiologies are less likely, given the details of this case. Ingestion of the eggs of Taenia solium (pork tapeworm) can lead to neurocysticercosis. This can present with focal-onset seizures, but affected patients usually do not appear ill, and computed tomography scan or MRI typically reveal isolated or multiple cysts (Item C166B). Borrelia burgdorferi is transmitted by ticks, and early infection presents with the characteristic rash (erythema migrans). Late-stage disease can present with central nervous system findings, the most common of which is facial nerve palsies. Meningitis is less likely but can occur. Focal encephalitis is extremely unlikely. Listeria monocytogenes is not likely to present in a child of this age, and the characteristic central nervous system infection is brainstem encephalitis.

References:

Halstead S. Arborvirus encephalitis in North America. In: Kliegman RM, Behrman RE, Jenson HB, Stanton BF, eds. Nelson Textbook of Pediatrics. 17th ed. Philadelphia, Pa: Saunders Elsevier; 2007:1405-1408

Mazzulli T. Laboratory diagnosis of infection due to viruses, Chlamydia, and Mycoplasma. In: Long SS, Pickering LK, Prober CG, eds. Principles and Practice of Pediatric Infectious Diseases. 2nd ed. New York, NY: Churchill Livingstone; 2003:1392-1408

Willoughby RE Jr. Encephalitis, meningoencephalitis, and postinfectious encephalomyelitis. In: Long SS, Pickering LK, Prober CG, eds. Principles and Practice of Pediatric Infectious Diseases. 2nd ed. New York, NY: Churchill Livingstone; 2003:291-295

Page 571: AAP PREP 2009

2009 PREP SA on CD-ROM

page 571

Critique: 166

Axial computed tomography scan in herpes simplex virus encephalitis shows necrosis in the right temporal lobe (arrow). (Courtesy of D. Gilbert)

Page 572: AAP PREP 2009

2009 PREP SA on CD-ROM

page 572

Critique: 166

Neurocysticercosis: Coronal T1-weighted magnetic resonance imaging following contrast reveals a 1-cm bilobed lesion in the left parietal area with ring enhancement (arrow). (Courtesy of D. Gilbert)

Page 573: AAP PREP 2009

2009 PREP SA on CD-ROM

page 573

Question: 167

A 7-month-old boy presents to the emergency department with vomiting and diarrhea. Findings on physical examination are normal except for dehydration and lethargy. Laboratory tests reveal a serum glucose concentration of 30.0 mg/dL (1.7 mmol/L). The mother tells you that she recently had the flu. Family history is negative for any serious or chronic illnesses. You are considering an inborn error of metabolism.

Of the following, the MOST helpful next laboratory test is measurement of

A. serum calcium

B. serum lipids

C. serum sodium

D. urine ketones

E. urine reducing substances

Page 574: AAP PREP 2009

2009 PREP SA on CD-ROM

page 574

Critique: 167 Preferred Response: D

In the absence of sepsis, hypoglycemia in infancy most commonly is associated with disorders of carbohydrate metabolism, such as glycogen storage diseases (GSDs) or fatty acid oxidation disorders. A disorder of glycogen storage should be suspected for the infant who presents with hypoglycemia, massive hepatomegaly without splenomegaly, and lactic acidosis; in these conditions, glycogen stores in the liver cannot be broken down to supply necessary glucose. Symptoms of tremulousness and irritability with fasting may be present for some time prior to diagnosis. The infant described in the vignette is unlikely to have GSD because he has normal findings on physical examination. His hypoglycemia is in association with symptoms of vomiting and diarrhea and a recent exposure to influenza, raising the question of whether he is unable to create energy from fat stores during this hypermetabolic state. The next step in making a diagnosis is to measure urine ketones. If he is able to break down fatty acids for energy, he should have large ketones in his urine; if this is not the case, he will have little to no ketones in his urine. Should the latter be the case, total and free carnitine concentrations should be measured and an acylcarnitine profile be obtained to determine which type of fatty acid oxidation defect is present. Individuals who have GSD often have marked hyperlipidemia with apparent hyponatremia (correction must be made for the increased serum solids), but the absence of these findings is not very helpful in making a diagnosis for the child described in the vignette. Serum calcium values typically are normal in both fatty acid oxidation and glycogen storage disorders. Urine reducing substances characteristically are elevated in infants who have galactosemia, which presents in the newborn period with hepatomegaly and jaundice.

References:

Burton BK. Inborn errors of metabolism in infancy: a guide to diagnosis. Pediatrics. 1998;102:e69-e78. Available at: http://pediatrics.aappublications.org/cgi/content/full/102/6/e69

Hoffmann GF, Nyhan WL, Zschocke J, Kahler SG, Mayatepek E. Approach to the patient with metabolic disease. In: Inherited Metabolic Diseases. Philadelphia, Pa. Lippincott Williams & Wilkins; 2002:19-94

Nyhan WL, Barshop BA, Ozand PT. Disorders of carbohydrate metabolism. In: Atlas of Metabolic Diseases. 2nd ed. London, England: Hodder Arnold; 2005:371-372

Nyhan WL, Barshop BA, Ozand PT. Hyperammonemia and disorders of the urea cycle: introduction to hyperammonemia and disorders of the urea cycle. In: Atlas of Metabolic Diseases. 2nd ed. London, England: Hodder Arnold; 2005:191-192

Nyhan WL, Barshop BA, Ozand PT. Organic acidemia: introduction. In: Atlas of Metabolic Diseases. 2nd ed. London, England: Hodder Arnold; 2005:1-3, 191-192,371-372

Page 575: AAP PREP 2009

2009 PREP SA on CD-ROM

page 575

Question: 168

A 15-year-old young woman has had joint pain for the past 3 days. She developed fever, chills, and fatigue 4 days ago, but the fever has resolved. In addition, she explains that her left elbow, right knee, and right wrist are all painful, red, and swollen, and she has a rash on her hands and feet that looks like pus-filled bumps. She is sexually active, with inconsistent condom use for contraception. Physical examination reveals an afebrile young woman who has swelling, tenderness, and mild erythema of the left elbow, right knee, and right wrist. She has a few pustules and vesicles on the right palm and bilateral soles (Item Q168). The abdomen is not tender and is without masses.

Of the following, the MOST likely pathogen causing this patient’s symptoms is

A. Chlamydia trachomatis

B. group A beta-hemolytic streptococci

C. Neisseria gonorrhoeae

D. parvovirus B19

E. Treponema pallidum

Page 576: AAP PREP 2009

2009 PREP SA on CD-ROM

page 576

Question: 168

(Courtesy of Bernard Cohen, MD, DermAtlas; www.dermatlas.org)

Page 577: AAP PREP 2009

2009 PREP SA on CD-ROM

page 577

Critique: 168 Preferred Response: C

Although probably most recognizable as causing urethritis or cervicitis, Neisseria gonorrhoeae can be implicated in a number of other infections, including neonatal conjunctivitis, salpingitis, proctitis, pelvic inflammatory disease, and disseminated gonococcal infection (DGI). DGI produces gonococcal bacteremia and may result in petechial or pustular acral skin lesions, asymmetric arthralgia, tenosynovitis, and septic arthritis, as described for the young woman in the vignette. Less frequently, perihepatitis, endocarditis, and meningitis can occur. Patients who have DGI typically present with a triad of tenosynovitis, dermatitis, and polyarthralgias without purulent arthritis or with purulent arthritis without associated skin lesions. There may be crossover between these two syndromes, however. The dermatitis is described as painless lesions that often are few in number and usually pustular or vesiculopustular (Item C168A). Such lesions frequently are transient, lasting for 3 to 4 days. The differential diagnosis of DGI may include group A beta-hemolytic streptococcal infection with acute rheumatic fever, but the rash of this infection is very transient and almost never pustular or vesicopustular. Parvovirus B19 also may cause a rash (fifth disease or erythema infectiosum) and arthritis, but the rash usually is reticular on the extremities (Item C168B) and has a "slapped cheeks" appearance on the face (Item C168C). Chlamydia trachomatis infection is not associated with a rash or arthralgias. Secondary syphilis, caused by Treponema pallidum, is characterized by rash, mucocutaneous lesions, and lymphadenopathy. The rash is described as polymorphic, generalized maculopapular (Item C168D) and typically involves the palms and soles (Item C168E).

References:

American Academy of Pediatrics. Syphilis. In: Pickering LK, Baker CJ, Long SS, McMillan JA, eds. Red Book: 2006 Report of the Committee on Infectious Diseases. 27th ed. Elk Grove Village, Ill: American Academy of Pediatrics; 2006:631-644

Goldenberg DL, Sexton DJ. Disseminated gonococcal infection. UpToDate Online 15.3. 2008. Available for subscription at: http://www.utdol.com/utd/content/topic.do?topicKey=stds/9841

Workowski KA, Berman SM, Centers for Disease Control and Prevention. Sexually transmitted diseases treatment guidelines, 2006. MMWR Recomm Rep. 2006;55(RR11):1-94. Available at: http://www.cdc.gov/mmwr/preview/mmwrhtml/rr5511a1.htm

Page 578: AAP PREP 2009

2009 PREP SA on CD-ROM

page 578

Critique: 168

Patients who have disseminated gonococcal infection often exhibit necrotic pustules that have a gray appearance. (Courttesy of D. Krowchuk)

Page 579: AAP PREP 2009

2009 PREP SA on CD-ROM

page 579

Critique: 168

Fifth disease produces a lacy, reticulated erythema on the extremities. (Courtesy of D. Krowchuk)

Page 580: AAP PREP 2009

2009 PREP SA on CD-ROM

page 580

Critique: 168

Erythematous ("slapped") cheeks are a feature of erythema infectiosum. (Courtesy of D. Krowchuk)

Page 581: AAP PREP 2009

2009 PREP SA on CD-ROM

page 581

Critique: 168

Secondary syphilis produces a generalized eruption composed of erythematous macules and papules. (Courtesy of C. Haverstock)

Page 582: AAP PREP 2009

2009 PREP SA on CD-ROM

page 582

Critique: 168

The eruption of secondary syphilis often involves the palms and soles. (Courtesy of C. Haverstock)

Page 583: AAP PREP 2009

2009 PREP SA on CD-ROM

page 583

Question: 169

A 5-year-old boy has been receiving mechanical ventilation in the pediatric intensive care unit for 1 week due to complicated adenoviral pneumonia. Over the past several days, he has developed markedly increased oxygen requirements and progressive opacification of his bilateral lung fields on chest radiography (Item Q169). He now meets the clinical criteria for acute respiratory distress syndrome (ARDS). His parents ask about the prognosis.

Of the following, the MOST accurate statement regarding the natural history of ARDS is that

A. complete resolution usually occurs by 1 month

B. development of other organ failure significantly increases the mortality rate

C. mortality is unrelated to the underlying cause

D. the major cause of death is unremitting respiratory failure

E. the overall mortality rate in children is 80%

Page 584: AAP PREP 2009

2009 PREP SA on CD-ROM

page 584

Question: 169

Chest radiography findings, as observed for the child in the vignette. (Courtesy of D. Mulvihill)

Page 585: AAP PREP 2009

2009 PREP SA on CD-ROM

page 585

Critique: 169 Preferred Response: B

Acute respiratory distress syndrome (ARDS) represents the severe end of the acute lung injury spectrum. Classically, ARDS is defined as a syndrome of lung inflammation with increased vascular permeability characterized by an acute onset, bilateral infiltrates on chest radiography (Item C169), absence of left atrial hypertension, and a PaO2/FiO2 of less than 200 mm Hg. The exact incidence of ARDS in the pediatric population has been difficult to quantify because of changes in disease definition and limitations of epidemiologic studies (both in methodology and age ranges studied). Most experts agree that ARDS accounts for 2% to 4% of pediatric intensive care unit admissions. It has been estimated that affected patients account for 8% of total patient days and 33% of deaths in the pediatric intensive care unit. Historically, risk factors for development of ARDS included sepsis, trauma, aspiration, pneumonia, near-drowning, and multiorgan dysfunction. ARDS normally progresses through several phases. The initial exudative phase is characterized by pulmonary edema, profound hypoxemia, and atelectasis. Some patients recover in 3 to 7 days, but others progress to the fibroproliferative stage, which is characterized by persistent hypoxemia, decreased lung compliance, and development of pneumothoraces. The final recovery phase consists of resolution of hypoxemia and improved lung compliance. Complete resolution may take 6 to 12 months, and some patients develop lifelong restrictive lung disease, lung cysts, or decreased exercise tolerance. Overall mortality in ARDS has been reported to be 30% to 40%, although exact rates in pediatric patients remain uncertain. Patients rarely die solely from lung failure; multiorgan system failure or withdrawal of life support is the most common cause of death. Mortality rates vary, depending on the underlying disease process, even when corrected for severity of illness. Patients who develop ARDS following trauma, for example, tend to have much better survival rates than those who develop ARDS following bone marrow transplantation or gram-negative sepsis.

References:

Frankel LR. Respiratory distress and failure. In: Kliegman RM, Behrman RE, Jenson HB, Stanton BF, eds. Nelson Textbook of Pediatrics. 18th ed. Philadelphia, Pa: Saunders Elsevier; 2007:421-423

Vish M, Shanley TP. Acute lung injury and acute respiratory distress syndrome. In: Wheeler DS, Wong HR, Shanley TP, eds. Pediatric Critical Care Medicine: Basic Science and Clinical Evidence. New York, NY: Springer-Verlag; 2007:395-411

Page 586: AAP PREP 2009

2009 PREP SA on CD-ROM

page 586

Critique: 169

Anteroposterior chest radiograph in acute respiratory distress syndrome demonstrates diffuse air space disease bilaterally without focal consolidation or effusions. (Courtesy of D. Mulvihill)

Page 587: AAP PREP 2009

2009 PREP SA on CD-ROM

page 587

Question: 170

A 6-month-old baby whose family has just emigrated from Ecuador is brought to your office by the maternal grandmother. Physical examination reveals length at the 5th percentile, weight at the 10th percentile, head circumference at the 25th percentile, a sallow complexion with jaundice, hoarse cry, dry skin, and large tongue. The anterior fontanelle measures 3x4 cm. You diagnose primary hypothyroidism and start appropriate thyroid hormone replacement therapy.

Of the following, the MOST likely long-term outcome in this baby is

A. adrenal insufficiency

B. microcephaly

C. normal adult height

D. normal intellectual function

E. precocious puberty

Page 588: AAP PREP 2009

2009 PREP SA on CD-ROM

page 588

Critique: 170 Preferred Response: C

The baby described in the vignette has congenital hypothyroidism and has not received early treatment with thyroid hormone. Fortunately, congenital hypothyroidism now usually is diagnosed soon after birth by newborn screening programs in the United States, but this screening may not be available in resource-limited countries. Children who have severe hypothyroidism at birth tend to have poorer intellectual outcomes than their siblings, even if therapy is started in the first few days after birth. However, earlier therapy with larger doses of thyroid hormone leads to improved intellectual outcomes, even in the most high-risk athyreotic group of infants. Despite the guarded prognosis for intellectual outcome, with continued and appropriate thyroid hormone therapy, this child should grow normally in height, and adult height should be appropriate for the family. Congenital hypothyroidism is not associated with adrenal insufficiency, although acquired hypothyroidism because of chronic lymphocytic thyroiditis may be associated with the development of autoimmune adrenal insufficiency (Addison disease). Microcephaly is an unusual outcome for children treated appropriately with thyroid hormone. Indeed, brain catch-up growth in the first postnatal year and a somewhat more flattened basal skull can give affected children larger head circumferences, split sutures, and the appearance of macrocephaly with increased intracranial pressure. On the other hand, overtreatment with thyroid hormone can lead to more rapid fusion of the cranial sutures and microcephaly, so treatment must be monitored with careful and frequent laboratory measurements of thyroid-stimulating hormone and thyroid hormone (free thyroxine [T4] or total T4 if free T4 is not available). The potential for normal intellectual function in this child is guarded. At the least, he is likely to have learning disabilities and some cerebellar problems, including clumsiness and decreased fine motor skills. Precocious puberty is not an outcome of congenital hypothyroidism unless a child is overtreated with thyroid hormone and develops premature maturation, which is very unusual. Some children who have severe acquired hypothyroidism have manifested signs of sexual precocity that disappear after treatment of the hypothyroidism (Van Wyk-Grumbach syndrome).

References:

American Academy of Pediatrics, Rose SR and the Section on Endocrinology and Committee on Genetics, American Thyroid Association, Brown RS and the Public Health Committee, Lawson Wilkins Pediatric Endocrine Society. Update of newborn screening and therapy for congenital hypothyroidism. Pediatrics. 2006;117:2290-2303. Available at: http://pediatrics.aappublications.org/cgi/content/full/117/6/2290

Kempers MJ, van der Sluijs Veer L, Nijhuis-van der Sanden RW, et al. Neonatal screening for congenital hypothyroidism in the Netherlands: cognitive and motor outcome at 10 years of age. J Clin Endocrinol Metab. 2007;92:919-924. Abstract available at: http://www.ncbi.nlm.nih.gov/pubmed/17164300

Postellon DJ, Bourgeois MJ, Varma S. Congenital hypothyroidism. eMedicine Specialties, Pediatrics: General Medicine, Endocrinology. 2007. Available at: www.emedicine.com/ped/topic501.htm

Page 589: AAP PREP 2009

2009 PREP SA on CD-ROM

page 589

Question: 171

You are meeting with a family that recently moved to the United States. The two children, ages 11 months and 24 months, show evidence of global developmental delays. The parents ask what services are available for their children. You explain that United States federal law provides children with early intervention services.

Of the following, the BEST explanation about the provisions of the law is that

A. eligibility depends on the socioeconomic level of the family

B. family participation is mandated by law

C. the provision of services requires referral by a physician

D. the services are provided only to children who have autism

E. services for infants and toddlers must be family-based and culturally competent

Page 590: AAP PREP 2009

2009 PREP SA on CD-ROM

page 590

Critique: 171 Preferred Response: E

Public Law 94-142, the Education of the Handicapped Act, passed in 1975, gave children between the ages of 5 and 18 years access to a free, appropriate public education. Services to children between the ages of 3 and 5 years of age were optional. In 1986, the law was amended as 99-457, which established early intervention programs for children from birth to 3 years who had developmental delay. The law also mandated that preschool children receive services. The law was amended again in 1990 with 101-476, the "Individuals with Disabilities Education Act" (IDEA). The portion entitled Part H, the Program for Infants and Toddlers with Disabilities, required states to develop and implement community-based systems of care that are coordinated, family-based, and culturally competent, involving greater interagency collaboration. It mandated early identification and provision of services to infants and toddlers who have developmental delays and established conditions. In 1997, the IDEA amendment, PL 105-17 (Part C, formerly Part H), encouraged the states that did not serve the at-risk population to track and monitor such children so that they could be referred when needed. Eligibility for such programs is dependent on whether a child has any developmental disability, not just autism. Families do not need a physician's referral for participation, they are not required to participate, and such participation is not dependent on income.

References:

American Academy of Pediatrics Committee on Children With Disabilities. Role of the pediatrician in family-centered Early Intervention Services. Pediatrics. 2001;107:1155-1157. Available at: http://pediatrics.aappublications.org/cgi/content/full/107/5/1155

Davidson L. In brief: law and the child. Pediatr Rev. 2003;24:213-214. Available at: http://pedsinreview.aappublications.org/cgi/content/full/24/6/213

Individuals with Disabilities Education Act 1997. Office of Special Education and Rehabilitative Services. U.S. Department of Education. Available at: http://www.ed.gov/about/offices/list/osers/policy.html

Page 591: AAP PREP 2009

2009 PREP SA on CD-ROM

page 591

Question: 172

Results of a stool culture from a 2-year-old boy who has been hospitalized with bloody diarrhea indicate that the causative agent is Shigella sp. The boy is allergic to trimethoprim-sulfamethoxazole.

Of the following, the MOST appropriate antimicrobial agent to use for this patient is

A. amoxicillin

B. azithromycin

C. cefdinir

D. ciprofloxacin

E. linezolid

Page 592: AAP PREP 2009

2009 PREP SA on CD-ROM

page 592

Critique: 172 Preferred Response: B

Shigella are gram-negative bacilli that are readily spread person-to-person through fecal-oral contamination. They usually are recovered from bacterial stool cultures using MacConkey and Hektoen-Enteric agars. The four primary Shigella species (S sonnei, S flexneri, S boydii, and S dysenteriae) cannot be distinguished clinically; they must be identified in the microbiology laboratory. Shigella sp primarily infect the large intestine and may cause a range of manifestations from mild diarrhea to life-threatening dysentery. Unlike the nontyphoidal forms of Salmonella for which the indications for antimicrobial therapy are limited to young (<3 months of age) or immunocompromised patients who have gastroenteritis, all patients who have shigellosis benefit from therapy. Antimicrobial therapy can shorten the duration of diarrhea and eradicate organisms from the stool. Although trimethoprim-sulfamethoxazole is a good agent for empiric therapy of shigellosis, antimicrobial susceptibility testing always should be performed due to increasing problems with resistance. If ampicillin and trimethoprim-sulfamethoxazole resistance is verified, parenteral ceftriaxone, a fluoroquinolone (eg, ciprofloxacin), or azithromycin can be used. Choosing an appropriate antimicrobial agent to treat the boy described in the vignette, who is allergic to trimethoprim-sulfamethoxazole and is 2 years of age, is a challenge. Although ciprofloxacin is effective, it should not be used routinely in children younger than 18 years of age unless no other options are available. Surprisingly, the clinical usefulness of the oral cephalosporins (eg, cefdinir), with the exception of cefixime, have not been demonstrated. Amoxicillin is less effective than ampicillin because of its rapid absorption from the gastrointestinal tract, and linezolid is an antimicrobial agent from a class of drugs called the oxazolidinones that are indicated for the treatment of resistant gram-positive organisms (eg, methicillin-resistant Staphylococcus aureus). Therefore, the agent of choice for this boy is azithromycin. The duration of antibiotic therapy for shigellosis typically is 5 days.

References:

American Academy of Pediatrics. Shigella infections. In: Pickering LK, Baker CJ, Long SS, McMillan JA, eds. Red Book: 2006 Report of the Committee on Infectious Diseases. 27th ed. Elk Grove Village, Ill: American Academy of Pediatrics; 2006:589-591

Niyogi SK. Increasing antimicrobial resistance-an emerging problem in the treatment of shigellosis. Clin Microbiol Infect. 2007;13:1141-1143. Abstract available at: http://www.ncbi.nlm.nih.gov/pubmed/17953700

Spence JT, Cheng TL. In brief: Shigella species. Pediatr Rev. 2004;25:329-330. Available at: http://pedsinreview.aappublications.org/cgi/content/full/25/9/329

Page 593: AAP PREP 2009

2009 PREP SA on CD-ROM

page 593

Question: 173

You are called to the delivery room to evaluate a term female infant born by precipitous normal spontaneous vaginal delivery to an 18-year-old young woman who received no prenatal care. The mother reports using marijuana and alcohol early in her pregnancy and was seen in the emergency room on two occasions for urinary tract infections. She had several "colds" late in her pregnancy. She lives with her boyfriend and has two dogs, a cat, and a turtle as pets. Physical examination of the infant reveals a 2-kg lethargic, jaundiced infant who has a weak cry, microcephaly, and a distended abdomen. Her liver is palpable 6 cm below the right costal margin, and her spleen is palpable 4 cm below the left costal margin. She has a diffuse petechial rash with areas of purpura on her extremities (Item Q173). Laboratory tests show a peripheral white blood cell count of 10.6x103/mcL (10.6x109/L), hemoglobin of 12.0 mg/dL (120.0 g/L), and platelet count of 60.0x103/mcL (60.0x109/L). The alanine aminotransferase measurement is 300 U/L, and the aspartate aminotransferase value is 420 U/L. Head ultrasonography shows scattered intracerebral calcifications.

Of the following, the MOST rapid test for making the diagnosis in this infant is

A. blood culture

B. cerebrospinal fluid polymerase chain reaction

C. nasopharyngeal viral culture

D. serology

E. urine culture

Page 594: AAP PREP 2009

2009 PREP SA on CD-ROM

page 594

Question: 173

Purpura, as exhibited by the infant described in the vignette. (Reprinted with permission from Stehel E, Sanchez PJ. NeoReviews. 2005;6:e38-e45.)

Page 595: AAP PREP 2009

2009 PREP SA on CD-ROM

page 595

Critique: 173 Preferred Response: E

The newborn described in the vignette has signs and symptoms suggestive of congenital cytomegalovirus (CMV) infection. CMV is a ubiquitous DNA virus that may be transmitted vertically from mother to infant in utero by transplacental passage of maternal bloodborne virus, at birth by passage through an infected maternal genital tract, or postnatally by ingestion of CMV-positive human milk. Approximately 1% of all liveborn infants are infected in utero and excrete CMV at birth. The risk for infection is greatest during the first half of gestation. In utero fetal infection can occur after maternal primary infection or after reactivation of infection during pregnancy, but sequelae are much more common in infants exposed to maternal primary infection, with 10% having manifestations evident at birth. Symptomatic congenital CMV disease is characterized by intrauterine growth restriction; jaundice; hepatosplenomegaly; hepatitis; thrombocytopenia with petechiae and purpura (Item C173A); and severe central nervous system involvement that can be characterized by microcephaly, intracerebral calcifications, chorioretinitis (Item C173B), or sensorineural hearing loss. Viral culture is the test of choice for confirming the diagnosis of congenital CMV infection. The diagnosis is established by isolation of the virus from urine, stool, cerebrospinal fluid (CSF), or saliva in the first 1 to 2 postnatal weeks. CSF polymerase chain reaction testing can be used to detect CMV DNA, but it is less sensitive than viral isolation by culture and is not used routinely. Routine blood culture and nasopharyngeal viral culture are not helpful for detecting CMV. Standard serologic testing is a cumbersome approach to diagnosing congenital CMV disease; serial samples need to be obtained to make the diagnosis clearly. The presence of CMV immunoglobulin M antibodies at birth is highly suggestive of a congenital CMV infection, but a confirmatory urine culture for CMV is recommended to establish the diagnosis definitively.

References:

American Academy of Pediatrics. Cytomegalovirus infection. In: Pickering LK, Baker CJ, Long SS, McMillan JA, eds. Red Book: 2006 Report of the Committee on Infectious Diseases. 27th ed. Elk Grove Village, Ill: American Academy of Pediatrics; 2006:273-277

Demmler GJ. Congenital cytomegalovirus infection and disease. Adv Pediatr Infect Dis. 1996;11:135-162

Demmler GJ. Cytomegalovirus. In: Feigin RD, Cherry JD, Demmler GJ, Kaplan SL, eds. Textbook of Pediatric Infectious Diseases. 5th ed. Philadelphia, Pa: Saunders; 2004:1912-1932

Modlin JF, Grant PE, Makar RS, Roberts DJ, Krishnamoorthy KS. Case records of the Massachusetts General Hospital. Weekly clinicopathological exercises. Case 25-2003: a newborn boy with petechiae and thrombocytopenia. N Engl J Med. 2003;349:691-700

Page 596: AAP PREP 2009

2009 PREP SA on CD-ROM

page 596

Critique: 173

Congenital cytomegalovirus infection may produce petechiae and purpura, as seen on the face of this infant. (Reprinted with permission from Stehel E, Sanchez PJ. NeoReviews. 2005;6:e38-e45.)

Page 597: AAP PREP 2009

2009 PREP SA on CD-ROM

page 597

Critique: 173

Cytomegalovirus retinitis: White areas, often arranged along veins, represent edema due to inflammation. Areas of early scarring (hyperpigmentation) are present. (Courtesy of R.G. Weaver, Jr)

Page 598: AAP PREP 2009

2009 PREP SA on CD-ROM

page 598

Question: 174

A 6-year-old girl is experiencing daytime and nighttime enuresis of 1 month’s duration. She achieved daytime continence at age 3 and has been dry at night since age 4. She has no history of fever, but does have some dysuria. The physical examination is remarkable only for suprapubic tenderness. Urinalysis demonstrates a specific gravity of 1.015, pH of 6.5, 1+ blood, trace protein, 3+ leukocyte esterase, and positive for nitrite. Microscopy reveals 2 to 5 red blood cells/high-power field (HPF), 20 to 50 white blood cells/HPF, and 3+ bacteria. Results of a urine culture are pending.

Of the following, the MOST appropriate empiric treatment for this patient is

A. amoxicillin

B. cefixime

C. cephalexin

D. ciprofloxacin

E. trimethoprim-sulfamethoxazole

Page 599: AAP PREP 2009

2009 PREP SA on CD-ROM

page 599

Critique: 174 Preferred Response: E

The child described in the vignette has dysuria, incontinence, and suprapubic tenderness, but she is otherwise well and has no fever or systemic symptoms. Based on the urinary findings of positive leukocyte esterase and nitrite with pyuria on microscopy, she most likely has a lower urinary tract infection (UTI) or cystitis. Cystitis is treated with empiric outpatient oral antibiotic therapy directed at the most likely urinary pathogens until the urine culture and antibiotic susceptibilities are available. Approximately 90% of UTIs are caused by Escherichia coli. Because E coli are usually sensitive to trimethoprim-sulfamethoxazole (TMP-SMX), it is an excellent agent for initial treatment of cystitis. In some regions of the country, bacterial resistance to this agent may be increasing, but it still is considered the best first-line agent because it may be more effective than beta-lactam antibiotics (eg, amoxicillin) in treating UTI, is inexpensive and readily available, and allows the clinician to reserve more broad-spectrum antibiotics for treatment failures. Due to high resistance rates, amoxicillin should not be used as first-line empiric treatment for cystitis. Broader-spectrum antibiotics such as first-generation (cephalexin) and third-generation (cefixime) cephalosporins or fluoroquinolones (ciprofloxacin) should be reserved for second-line treatment unless the child has recurrent UTI and a history of resistance or allergy to TMP-SMX. The standard duration of treatment is 7 to 14 days. A recent meta-analysis supported the use of a short course (2 to 4 days) of antibiotics, which compared favorably with a standard course (7 to 14 days) in terms of treatment failures and recurrence of lower UTI. Single-dose antibiotic regimens have been used in adults, but pediatric studies demonstrate an increased number of treatment failures and UTI recurrences compared with standard 7- to 14-day treatment regimens.

References:

Jantausch B, Kher K. Urinary tract infection. In: Kher KK, Schnaper HW, Makker SP, eds. Clinical Pediatric Nephrology. 2nd ed. London, England: Informa Healthcare; 2007:553-573

Keren R, Chan E. A meta-analysis of randomized, controlled trials comparing short- and long-course antibiotic therapy for urinary tract infections in children. Pediatrics. 2002;109:e70. Available at: http://pediatrics.aappublications.org/cgi/content/full/109/5/e70

Larcombe J. Urinary tract infection in children. BMJ Clinical Evidence. 2007. Available for subscription at: http://clinicalevidence.bmj.com/ceweb/conditions/chd/0306/0306.jsp

Michael M, Hodson EM, Craig JC, Martin S, Moyer VA. Short versus standard duration oral antibiotic therapy for acute urinary tract infection in children. Cochrane Database Syst Rev. 2003;1:CD003966. Available at: http://www.mrw.interscience.wiley.com/cochrane/clsysrev/articles/CD003966/frame.html

Raszka WV Jr, Khan O. Pyelonephritis. Pediatr Rev. 2005;26:364-370. Available at: http://pedsinreview.aappublications.org/cgi/content/full/26/10/364

Page 600: AAP PREP 2009

2009 PREP SA on CD-ROM

page 600

Question: 175

An 8-year-old boy presents with wheezing, coughing, and difficulty breathing of 6 months’ duration. Findings on his history and pulmonary function tests are suggestive of moderate persistent asthma. In preparation for asthma management, you have reviewed the current asthma guidelines, educated the patient on peak flow monitoring, and discussed possible therapeutic options. You decide to start him on a daily inhaled corticosteroid.

Of the following, the MOST likely adverse event he may experience from inhaled corticosteroids is

A. acne

B. decreased adult height

C. mood swings

D. oral candidiasis

E. weight gain

Page 601: AAP PREP 2009

2009 PREP SA on CD-ROM

page 601

Critique: 175 Preferred Response: D

Despite remaining the cornerstone therapy for persistent asthma, many parents are reluctant to have their children begin using inhaled corticosteroids. Addressing parents' concerns and outlining the potential adverse effects may improve understanding and compliance. Local oropharyngeal symptoms, including dysphonia, oral candidiasis, and cough, are commonly encountered adverse effects of inhaled corticosteroids. Local symptoms appear to be dose-related and can be lessened by using a "spacer" and rinsing out the mouth after use. A number of studies have documented a transient decrease in growth velocity during the first year of inhaled corticosteroid therapy, but a daily cumulative dose up to 800 mcg of inhaled budesonide has been shown not to affect predicted adult height. Less common adverse effects of inhaled steroids include acne, mood swings, weight gain, decreased serum immunoglobulin G concentrations, and rarely, posterior subcapsular cataracts and adrenal suppression. Inhaled steroids are used in persistent asthma to help halt lung remodeling by decreasing bronchial inflammation, reducing inflammatory mediators, and decreasing bronchial hyperresponsiveness.

References:

Allen DB, Bielory L, Derendorf H, Dluhy R, Colice GL, Szefler SJ. Inhaled corticosteroids: past lessons and future issues. J Allergy Clin Immunol. 2003;112(3 suppl):S1-S40. Abstract available at: http://www.ncbi.nlm.nih.gov/pubmed/14515117

Keeley D, McKean M. Asthma and other wheezing disorders in children. BMJ Clinical Evidence. 2006. Available for subscription at: http://clinicalevidence.bmj.com/ceweb/conditions/chd/0302/0302.jsp#Q1

Schielmer RP, Spahn JD, Covar R, Szefler SJ. Glucocorticoids. In: Adkinson NF, Jr, Yunginger JW, Busse WW, Bochner BS, Holgate ST, Simons FE, eds. Middleton's Allergy Principles and Practice. 6th ed. Philadelphia, Pa: Mosby Inc; 2003:870-913

Page 602: AAP PREP 2009

2009 PREP SA on CD-ROM

page 602

Question: 176

Your first patient of the day is a 2-year-old girl who is brought in by her mother after a brown spider was found in the child’s bed. The mother has brought the spider for you to inspect (Item Q176A). On physical examination, there is a 2-cm bulla with 4 cm of surrounding erythema on the medial aspect of the girl’s calf (Item Q176B). The child otherwise appears well and occasionally scratches at the lesion.

Of the following, the MOST appropriate course of action for this patient is to

A. begin dapsone therapy

B. begin local wound care

C. prescribe a 5-day course of prednisone

D. refer the child to a surgeon for excision of the bite area

E. transfer the patient to the emergency department for antivenom

Page 603: AAP PREP 2009

2009 PREP SA on CD-ROM

page 603

Question: 176

Type of spider brought by the family for inspection. (Courtesy of the Centers for Disease Control and Prevention, Public Health Image Library)

Page 604: AAP PREP 2009

2009 PREP SA on CD-ROM

page 604

Question: 176

Bulla with surrouding erythema, as described for the child in the vignette. (Courtesy of M. Rimsza)

Page 605: AAP PREP 2009

2009 PREP SA on CD-ROM

page 605

Critique: 176 Preferred Response: B

In the United States, two spider species are responsible for most spider bite-related illness and injury: Lactrodectus sp (black widow spiders) (Item C176A) and Loxosceles sp. Black widow spider bites cause a systemic syndrome characterized by autonomic dysfunction, muscle cramping, and rigidity due to neurotoxins in the venom. Spiders of the genus Loxosceles, of which the brown recluse spider (Loxosceles recluse) is best known, are recognized primarily as a cause of necrotic skin lesions, although systemic symptoms, including a flulike illness, hemolytic anemia, and renal failure, may occur. The spider responsible for the skin lesion described for the child in the vignette is a brown recluse, which can be recognized by its brown color, violin-shaped marking on the thorax (Item C176B), and three sets of eyes. Found throughout the central and southern United States, they live in dark, protected environments such as wood piles or storage sheds but can be found indoors in bedsheets and clothing hampers. They are nocturnal, and human bites typically occur at night. Brown recluse spider venom contains a variety of proteolytic enzymes that can cause extensive tissue damage. The bite itself usually is painless and may go unnoticed until pain, erythema, and pruritus develop at the site. The initial erythematous maculopapular lesion becomes bullous (Item C176C) and increases in size over the subsequent 48 hours. After that time period, an eschar develops (Item C176D) and subsequently separates, leaving a deep ulcer at the site (Item C176E). Systemic symptoms include fever, chills, vomiting, and arthralgias. Because children are smaller and receive a larger per kilogram venom dose, they are affected more commonly by the systemic syndrome, known as loxoscelism. Black widow spider bites also are painless and leave little more than two small red, pinpoint marks on the skin. The venom causes severe muscular cramping, tremors, and autonomic symptoms such as drooling and sweating. The symptoms seen after black widow spider bites frequently are mistaken for a variety of other conditions, including cholinergic crisis and acute abdominal processes. Treatment of brown recluse spider bites, black widow spider bites, and bites of all other spiders indigenous to North America is largely supportive and includes conscientious wound care and tetanus immunization, if indicated. The muscular cramping caused by a black widow spider bite is controlled readily with benzodiazepines, narcotics, and calcium gluconate. An antivenom is available, although its use is indicated only when autonomic symptoms or pain cannot be managed with usual measures. Skin grafting may be necessary if tissue necrosis occurs following a brown recluse spider bite. Many therapeutic modalities have been advocated to limit the extent of the brown recluse bite wound, including dapsone, early wound excision, hyperbaric oxygen, and steroids, but none have proven effective. Loxosceles antivenom is not available in the United States. Skin findings caused by a wide variety of other pathologic conditions often are attributed to brown recluse spider bites. The differential diagnosis is extensive and includes conditions such as staphylococcal and streptococcal skin lesions, cutaneous anthrax, atypical mycobacterial infection, sporotrichosis, ecthyma gangrenosum, herpes simplex and zoster, vasculitic lesions, erythema chronicum migrans, and erythema nodosum. Because necrotic skin lesions can be caused by a number of serious conditions, and documented spider bites are rare, clinicians should consider alternative diagnoses when faced with a necrotic skin lesion, if the spider is not available for inspection.

References:

Singletary EM, Rochman AS, Bodmer JCA, Holstege CP. Envenomations. Med Clin North Am. 2005;89:1195-1224. Abstract available at: http://www.ncbi.nlm.nih.gov/pubmed/16227060

Sjogren R, MacGregor RS, Zenel J. Visual diagnosis: an infant who has a red papule on a swollen, tender arm. Pediatr Rev. 2004;25:182-185. Available at: http://pedsinreview.aappublications.org/cgi/content/full/25/5/182

Page 606: AAP PREP 2009

2009 PREP SA on CD-ROM

page 606

Swanson DL, Vetter RS. Bites of brown recluse spiders and suspected necrotic arachnidism. N Engl J Med. 2005;352:700-707. Extract available at: http://content.nejm.org/cgi/content/extract/352/7/700

Page 607: AAP PREP 2009

2009 PREP SA on CD-ROM

page 607

Critique: 176

Female black widow spider: Note the red hourglass-shaped marking on the ventral abcominal surface (the color of this marking is variable, however, and may be yellow, orange, or white). (Courtesy of the Centers for Disease Control and Prevention, Public Health Image Library, James Gathany)

Page 608: AAP PREP 2009

2009 PREP SA on CD-ROM

page 608

Critique: 176

Brown recluse spider: Note the violin-shaped marking on the thorax (arrow). (Courtesy of the Centers for Disease Control and Prevention, Public Health Image Library)

Page 609: AAP PREP 2009

2009 PREP SA on CD-ROM

page 609

Critique: 176

Following the bite of a brown recluse spider, a bulla with surrounding erythema may develop. (Courtesy of M. Rimsza)

Page 610: AAP PREP 2009

2009 PREP SA on CD-ROM

page 610

Critique: 176

An eschar with surrounding erythema may occur following the bite of a brown recluse spider. (Courtesy of M. Smith)

Page 611: AAP PREP 2009

2009 PREP SA on CD-ROM

page 611

Critique: 176

Folliowing the bite of a brown recluse spider, an eschar may develop and subsequently separate, leaving a deep ulcer. (Courtesy of M. Rimsza)

Page 612: AAP PREP 2009

2009 PREP SA on CD-ROM

page 612

Question: 177

A 12-year-old boy has had intermittent heartburn for the past several years. Results of an upper gastrointestinal radiographic series performed at age 11 years were normal. Over the past year, he has had several episodes of "food getting stuck in his chest." The most common foods that cause him difficulty are hot dogs, steak tips, and chicken strips. Physical examination findings are unremarkable. He has been treated with omeprazole for 3 months, but symptoms persist.

Of the following, the MOST appropriate next diagnostic test is

A. esophageal manometry study

B. esophagogastroduodenoscopy with biopsy

C. radionuclide gastric emptying scan

D. serum immunoglobulin E measurement

E. 24-hour esophageal pH probe study

Page 613: AAP PREP 2009

2009 PREP SA on CD-ROM

page 613

Critique: 177 Preferred Response: B

The chronic heartburn and dysphagia to solid foods, primarily meats, described for the boy in the vignette strongly suggest either reflux esophagitis (possibly with an esophageal stricture) or eosinophilic esophagitis (EE) (Item C177A). The most appropriate next test to evaluate the dysphagia is esophagogastroduodenoscopy with biopsies. The endoscopy can characterize the severity of inflammation, exclude Barrett esophagus (a premalignant complication of gastroesophageal reflux [GER]), and potentially differentiate between GER and EE. Esophageal manometry, radionuclide gastric emptying scan, measurement of serum immunoglobulin E, and 24-hour esophageal pH probe study are potentially useful in the evaluation of GER or allergy, but they are not the appropriate initial tests in evaluating solid food dysphagia. GER is defined as excessive regurgitation of gastric contents (both acid and nonacid) into the esophagus. The most common gastrointestinal symptoms are heartburn (pyrosis), dysphagia (difficulty swallowing), and odynophagia (painful swallowing). GER also may result in extraintestinal symptoms, including hoarseness, chronic cough, exacerbation of asthma, apnea, and aspiration pneumonia. GER is extremely common in childhood. According to one cross-sectional study, 3% to 5% of healthy children report reflux symptoms at any given time. Because GER frequently is mild and self-limited, empiric therapy with an antacid, histamine receptor blocker, or proton pump inhibitor is appropriate and can alleviate symptoms. Prolonged, persistent reflux (ie, more than 3 months) or complicated reflux (ie, associated with hematemesis or respiratory symptoms) warrants further evaluation. Among the useful tests to evaluate GER are upper gastrointestinal radiographic series (which excludes malrotation and hiatal hernia), pH probe study or esophageal impedance (which determine the extent and duration of reflux over a 24-hour period), gastric emptying scan (which rules out gastroparesis), and esophageal motility evaluation (which assesses peristalsis and esophageal sphincter pressure). Upper endoscopy with biopsy generally is the most valuable test because it can determine if chronic GER has caused any esophageal injury (Item C177B). In the past 10 years, researchers have recognized that not all heartburn or dysphagia is caused by GER. EE presents similarly to reflux, but is believed to be an atopic condition. Affected patients tend to have more dysphagia symptoms than heartburn. Often, there is a history of meat impaction in the esophagus. As noted previously, upper endoscopy with biopsies frequently can differentiate between the two conditions. GER usually is characterized by ulceration and injury limited to the distal 5 cm of the esophagus. In contrast, EE is characterized by inflammation of the proximal, middle, and distal esophagus, with large numbers of eosinophils seen on biopsy. Differentiating between these two conditions is important because treatment of GER involves acid blockade. In contrast, the therapies for EE include anti-inflammatory therapy (eg, fluticasone, budesonide), acid blockade, or elimination diets.

References:

Furuta GT, Straumann A. Review article: the pathogenesis and management of eosinophilic oesophagitis. Aliment Pharmacol Ther. 2006;24:173-182. Available at: http://www.blackwell-synergy.com/doi/full/10.1111/j.1365-2036.2006.02984.x

Kumar Y, Sarvananthan R. Gastro-oesophageal reflux in children. BMJ Clinical Evidence. 2007. Available for subscription at: http://clinicalevidence.bmj.com/ceweb/conditions/chd/0310/0310.jsp

Nelson SP, Chen EH, Syniar GM, Kaufer Christoffer K; for the Pediatric Practice Research Group. Prevalence of symptoms of gastroesophageal reflux during childhood. Arch Pediatr Adolesc Med. 2000;154:150-154.

Page 614: AAP PREP 2009

2009 PREP SA on CD-ROM

page 614

Critique: 177

Eosinophilic esophagitis: Findings at endoscopy may include mucosal rings (A), furrows (B), and exudates (C). (Reprinted with permission from Hopp R, Natarajan N. Index of suspicion (case 1). Pediatr Rev. 2007;28:389-394 and courtesy of Doernbecher Children’s Hospital)

Page 615: AAP PREP 2009

2009 PREP SA on CD-ROM

page 615

Critique: 177

In severe esophagitis, the areas of erythema represent inflammation, and white exudates cover erosions. (Courtesy of A. Bousvaros)

Page 616: AAP PREP 2009

2009 PREP SA on CD-ROM

page 616

Question: 178

You are seeing a 12-month-old infant who was born at 26 weeks’ gestation. He receives daily diuretics and nasal cannula oxygen with a baseline flow of 0.1 L/min, but his mother called this morning reporting that he had a temperature of 100.5°F (37.8°C), nasal congestion, increased work of breathing with a rapid respiratory rate, and a "wheezing" cough. You instructed her to increase the oxygen flow rate to 0.5 L/min and come directly to the clinic. Physical examination reveals intercostal and subcostal retractions, a respiratory rate of 80 breaths/min, and a prolonged expiratory phase with audible wheezing. A copious, cloudy, green nasal discharge is present. No heart murmur is audible. Pulse oximetry while receiving 0.5 L/min oxygen reveals an oxygen saturation of 85% at rest.

Of the following, the BEST explanation for this child’s presenting signs of respiratory distress is

A. acute cor pulmonale

B. foreign body aspiration

C. gastroesophageal reflux

D. lower respiratory tract infection

E. upper respiratory tract infection

Page 617: AAP PREP 2009

2009 PREP SA on CD-ROM

page 617

Critique: 178 Preferred Response: D

The infant described in the vignette has symptoms of a lower respiratory tract infection. The presence of nasal discharge suggests a concomitant or preceding upper respiratory tract infection, but an upper respiratory tract infection alone would not result in tachypnea, retractions, or hypoxemia. It is unlikely that the infant has aspirated a foreign body because pulmonary findings are bilateral. There is no history of emesis or prolonged concerns for gastroesophageal reflux. Although chronic hypoxemia may contribute to cor pulmonale, this is not an acute condition and is inconsistent with the findings described in the vignette. Chronic lung disease of prematurity (CLD), also referred to as bronchopulmonary dysplasia, remains a continuing concern for graduates of the neonatal intensive care unit (NICU) who have required intubation, assisted ventilation, and oxygen therapy for extended periods of time. Primarily affecting very low-birthweight (VLBW) infants (<1,500 g), CLD is characterized by a sustained oxygen requirement beyond 36 weeks' gestation corrected age; obstructive small airways disease that may be reversible with bronchodilators; and inflammatory characteristics that include increased airway secretions, smooth muscle bronchoreactivity, and increased interstitial lung water. The infant who has CLD generally has an increased work of breathing, with an associated increased caloric expenditure, and a decreased pulmonary reserve capacity. Hence, he or she has little ability to meet increased metabolic and pulmonary work associated with exertion (eg, physical exercise) or increased respiratory work (associated with lower respiratory tract infection). As a result, up to 50% of VLBW infants who have CLD are rehospitalized for pulmonary decompensation in the first year after discharge from the NICU. The infant who has CLD requires increased calories and protein to grow compared with unaffected age- and birthweight-matched preterm infants. Good nutrition can help the at-risk growing preterm infant who has CLD in attaining good overall physical growth, pulmonary development, and neurodevelopment as well as optimizing immune function.

References:

American Academy of Pediatrics Committee on Fetus and Newborn. Postnatal corticosteroids to treat or prevent chronic lung disease in preterm infants. Pediatrics. 2002;109;330-338. Available at: http://pediatrics.aappublications.org/cgi/content/full/109/2/330

Bancalari EH. Bronchopulmonary dysplasia and neonatal chronic lung disease. In: Martin RJ, Fanaroff AA, Walsh MC, eds. Fanaroff and Martin's Neonatal-Perinatal Medicine. 8th ed. Philadelphia, Pa: Mosby Elsevier; 2006:1155-1167

Baraldi E, Filippone M. Chronic lung disease after premature birth. N Engl J Med. 2007;357:1946-1955. Extract available at: http://content.nejm.org/cgi/content/extract/357/19/1946

Bhandari A, Panitch HB. Pulmonary outcomes in bronchopulmonary dysplasia. Semin Perinatol. 2006;30:219-226. Abstract available at: http://www.ncbi.nlm.nih.gov/pubmed/16860162

Biniwale MA, Ehrenkranz RA. The role of nutrition in the prevention and management of bronchopulmonary dysplasia. Semin Perinatol. 2006;30:200-208. Abstract available at: http://www.ncbi.nlm.nih.gov/pubmed/16860160

Shaw NJ, Kotecha S. Management of infants with chronic lung disease of prematurity in the United Kingdom. Early Hum Dev. 2005;81:165-170. Abstract available at:

Page 618: AAP PREP 2009

2009 PREP SA on CD-ROM

page 618

http://www.ncbi.nlm.nih.gov/pubmed/15748971

Vaucher YE. Bronchopulmonary dysplasia: an enduring challenge. Pediatr Rev. 2002;23:349-358. Available at: http://pedsinreview.aappublications.org/cgi/content/full/23/10/349

Page 619: AAP PREP 2009

2009 PREP SA on CD-ROM

page 619

Question: 179

You are evaluating a 10-month-old boy brought to the emergency department because of fussiness for 1 day. His mother reports that she was carrying him while answering the phone yesterday and that he fell from her arms onto the linoleum floor. Physical examination reveals a thin boy who is crying. He resists weight-bearing on the left leg, but you cannot elicit specific tenderness. He has bruises on the left temporal region, upper arm, and thighs. You suspect nonaccidental trauma and order a skeletal survey.

Of the following, the skeletal survey finding that is MOST specific for nonaccidental trauma is

A. linear nondisplaced skull fracture

B. long bone nondisplaced shaft fracture

C. metaphyseal chip fracture

D. spiral tibial fracture

E. subperiosteal new bone formation

Page 620: AAP PREP 2009

2009 PREP SA on CD-ROM

page 620

Critique: 179 Preferred Response: C

Whenever child abuse is suspected, it is critical for the clinician to evaluate the child fully for any and all potential manifestations of such abuse. The evaluation begins with a thorough trauma history and social history, paying close attention to the child-parent interaction, and a complete physical examination. When bruises or other lesions are present, or the history is inconsistent with the mechanism of action for the lesions, child abuse should be strongly suspected, and further evaluation is required. Such evaluation includes a complete skeletal survey to look for fractures. A skeletal survey consists of radiographs of the skull, lateral cervical spine, ribs, pelvis, thoracolumbar spine, arms, hands, legs, and feet. Anteroposterior and lateral views should be obtained to examine all aspects of the bones fully. A skeletal survey can provide information about the age of a fracture, although some fractures, such as those of the ribs, may not be recognized early. A repeat survey 2 weeks later can be helpful to identify healing fractures that exhibit new bone formation. Certain types of fractures have a high specificity for nonaccidental trauma. Among the most specific are metaphyseal chip fractures (Item C179A) and metaphyseal bucket handle fractures (Item C179B), both of which are seen in children younger than 1 year of age. Other concerning findings include fractures of the posterior ribs (Item C179C), scapula, spinous process, and sternum as well as multiple fractures, fractures of different ages, femur fractures in children younger than 1 year of age (Item C179D), and complex skull fractures. Common skeletal survey findings that have a low specificity for child abuse include simple linear skull fractures, nondisplaced long bone fractures, and subperiosteal new bone formation. A nondisplaced spiral fracture of the tibia, also known as a "toddler's fracture" (Item C179E), is the result of a twisting motion of the tibia and can occur in young children who have just learned to walk. Even in these types of fractures, however, it is important to obtain a complete trauma history. If the history is inconsistent with the injury, a full evaluation for nonaccidental trauma is indicated. Further evaluation includes imaging of the head (either computed tomography scan or magnetic resonance imaging) to look for hematomas, ophthalmologic examination to look for retinal hemorrhages (Item C179F), and urinalysis and liver and pancreatic enzyme measurement to look for evidence of blunt abdominal trauma. Any suspicious findings warrant mandatory reporting to child protective services.

References:

Jenny C; Committee on Child Abuse and Neglect. Evaluating infants and young children with multiple fractures. Pediatrics. 2006;118:1299-1303. Available at: http://pediatrics.aappublications.org/cgi/content/full/118/3/1299

Kellogg ND and the Committee on Child Abuse and Neglect. Evaluation of suspected child physical abuse. Pediatrics. 2007;119:1232-1241. Available at: http://pediatrics.aappublications.org/cgi/content/full/119/6/1232

Sirotnak AP, Grigsby T, Krugman RD. Physical abuse of children. Pediatr Rev. 2004;25:264-277. Available at: http://pedsinreview.aappublications.org/cgi/content/full/25/8/264

Page 621: AAP PREP 2009

2009 PREP SA on CD-ROM

page 621

Critique: 179

Metaphyseal fracture (arrow) of the left proximal humerus in an infant who had been physically abused. (Courtesy of D. Krowchuk)

Page 622: AAP PREP 2009

2009 PREP SA on CD-ROM

page 622

Critique: 179

Bucket handle metaphyseal fracture of the radius (arrow) in a child who has been physically abused. (Courtesy of S. Sinal)

Page 623: AAP PREP 2009

2009 PREP SA on CD-ROM

page 623

Critique: 179

Healing posterior rib fractures (arrows) in an infant who has been physically abused. (Courtesy of D. Krowchuk)

Page 624: AAP PREP 2009

2009 PREP SA on CD-ROM

page 624

Critique: 179

Right femur fracture in a 2-month-old child who had been physically abused. (Courtesy of D. Krowchuk)

Page 625: AAP PREP 2009

2009 PREP SA on CD-ROM

page 625

Critique: 179

Oblique (left) and anteroposterior (right) views of the distal tibia show a nondisplaced spiral (toddler's) fracture. (Courtesy of D. Mulvihill)

Page 626: AAP PREP 2009

2009 PREP SA on CD-ROM

page 626

Critique: 179

Retinal hemorrhages and papilledema in an infant who had been shaken. (Courtesy of M. Rimsza)

Page 627: AAP PREP 2009

2009 PREP SA on CD-ROM

page 627

Question: 180

A medical student notes on rounds that a 2-year-old girl admitted for pneumonia has a complete blood count (CBC) that includes a hematocrit of 35% (0.35), hemoglobin of 11.5 g/dL (115.0 g/L), mean corpuscular volume of 68.0 fL, and platelet and white blood cell counts that are normal for age. During the bedside encounter with the child’s mother, you advise her to start the child on a multivitamin with iron and have her primary care physician obtain another CBC in a month or so. The medical student asks why you recommended iron supplementation when the child has a normal hematocrit.

Of the following, the BEST reason to prescribe supplemental iron therapy for this child at this time is to prevent

A. diminished cognitive abilities

B. fatigue

C. rapid progression to anemia

D. recurrent infections

E. short stature

Page 628: AAP PREP 2009

2009 PREP SA on CD-ROM

page 628

Critique: 180 Preferred Response: A

The child described in the vignette likely has iron deficiency, as evidenced by her low mean corpuscular volume. Providing iron supplementation may improve her cognition. It is not clear whether effects on cognition and behavior caused by iron deficiency are completely reversible with iron therapy. Of the two studies that treated children for 2 months or longer, one reported dramatic benefits for development and the other did not. Evidence suggests that children who had iron deficiency as toddlers may have slightly impaired cognition in elementary school, even if they were treated with iron and the anemia resolved. Finally, according to some evidence, iron deficiency may cause or be associated with symptoms of attention-deficit/hyperactivity disorder that may be improved with iron therapy. For many children, such as the girl described in the vignette, iron deficiency is revealed on a complete blood count obtained because of a febrile illness. Fever may cause transient anemia and microcytosis due to hemolysis, and repeat screening is recommended when the child recovers. Routine iron supplementation with a multivitamin with iron should be prescribed until it is determined whether iron therapy is required. Iron deficiency has not been proven to cause fatigue in the absence of anemia, and there is no definitive association between isolated iron deficiency or anemia and short stature. Iron deficiency does not cause recurrent infection. Iron deficiency does not lead rapidly to anemia; rather, anemia may develop over weeks to months.

References:

Konofal E, Lecendreux M, Arnulf I, Mouren MC. Iron deficiency in children with attention-deficit/hyperactivity disorder. Arch Pediatr Adolesc Med. 2004;158:1113-1115.

Lozoff B, De Andraca I, Castillo M, Smith JB, Walter T, Pino P. Behavioral and developmental effects of preventing iron-deficiency anemia in healthy full-term infants. Pediatrics. 2003;112:846-854. Available at: http://pediatrics.aappublications.org/cgi/content/full/112/4/846

Lozoff B, Jimenez E, Smith JB. Double burden of iron deficiency in infancy and low socioeconomic status: a longitudinal analysis of cognitive test scores to age 19 years. Arch Pediatr Adolesc Med. 2006;160:1108-1113.

Martins S, Logan S, Gilbert R. Iron therapy for improving psychomotor development and cognitive function in children under the age of three with iron deficiency anaemia. Cochrane Database Syst Rev. 2001;2;CD001444. Available at: http://www.mrw.interscience.wiley.com/cochrane/clsysrev/articles/CD001444/frame.html

Wu AC, Lesperance L, Bernstein H. Screening for iron deficiency. Pediatr Rev. 2002;23:171-178. Available at: http://pedsinreview.aappublications.org/cgi/content/full/23/5/171

Page 629: AAP PREP 2009

2009 PREP SA on CD-ROM

page 629

Question: 181

The youngest child in a family affected by neurofibromatosis 1, who is 5 years old, has just had the diagnosis confirmed. You begin the process of counseling the family.

Of the following, the MOST accurate statement about potential medical complications in affected children is that they

A. are at increased risk of developing cataracts

B. have an increased incidence of acoustic neuroma

C. have an increased incidence of childhood hypertension

D. should be restricted from contact sports

E. should be screened annually for optic gliomas using computed tomography scan

Page 630: AAP PREP 2009

2009 PREP SA on CD-ROM

page 630

Critique: 181 Preferred Response: C

Neurofibromatosis (NF) is an autosomal dominant neurocutaneous disorder of which there are at least two types. Type 1 has an incidence of approximately 1 in 3,500 people, and its characteristic features are the cutaneous café au lait macules and benign neurofibromas (Item C181). Type 2 has an incidence of approximately 1 in 40,000, and a characteristic feature is vestibular schwannoma. Individuals affected by NF-1 are at risk for the development of a number of complications, including optic pathway tumors, skeletal deformations, speech impediments, and learning disabilities. In addition, there is variable association with hypertension, short stature, constipation, and chronic headache. Patients who have NF-2 are not at increased risk for such complications. The child described in the vignette is at risk for the development of hypertension, particularly during ages 2 through 10 years. Although most patients have idiopathic hypertension, NF-1 can be associated with renal artery stenosis and pheochromocytoma. This child's health supervision visits should include a detailed history and accurate blood pressure measurements to ensure early identification of this complication. The child is not at an increased risk for the development of cataracts, and there is no reason to restrict him from contact sports if he has no other complications. Because approximately 15% of individuals who have NF-1 develop optic pathway tumors, periodic ophthalmologic evaluation is important. However, the potential high cumulative radiation dose of annual computed tomography scanning likely outweighs the benefits; the necessity and frequency of such imaging should be determined in consultation with the ophthalmologist. Acoustic neuroma, now referred to as vestibular schwannoma, is seen in NF-2, not NF-1.

References:

Hersh JH, Committee on Genetics. Health supervision for children with neurofibromatosis. Pediatrics. 2008;121:633-642. Available at: http://pediatrics.aappublications.org/cgi/content/full/121/3/633

Lama G, Graziano L, Calabrese E, et al. Blood pressure and cardiovascular involvement in children with neurofibromatosis type 1. Pediatr Nephrol. 2004;19:413-418. Abstract available at: http://www.ncbi.nlm.nih.gov/pubmed/14991390

Viskochil DH. Neurocutaneous disorders. In: Rudolph C, Rudolph A, eds. Rudolph's Pediatrics. 21st ed. New York, NY: McGraw Hill Medical Publishing Division; 2003:769-774

Page 631: AAP PREP 2009

2009 PREP SA on CD-ROM

page 631

Critique: 181

Café au lait macules and neurofibromas (arrow) are observed in neurofibromatosis type 1. (Courtesy of the Media Lab at Doernbecher)

Page 632: AAP PREP 2009

2009 PREP SA on CD-ROM

page 632

Question: 182

During the routine health supervision visit for a 4-month-old infant, you note low tone and poor visual interaction. His head shape is symmetric and his head circumference is 36 cm (<2nd percentile). He had been born at term with a head circumference of 32 cm (2nd percentile).

Of the following, the MOST helpful initial diagnostic test to explain the cause of the infant’s abnormal examination findings is

A. brain magnetic resonance imaging

B. electroencephalography

C. head ultrasonography

D. high-resolution karyotyping

E. three-dimensional head computed tomography scan

Page 633: AAP PREP 2009

2009 PREP SA on CD-ROM

page 633

Critique: 182 Preferred Response: A

The child described in the vignette had microcephaly at birth and has continued to develop below the standard head growth curve, increasing by just 4 cm, instead of 7 cm, during the first 4 months. Such slow growth most likely is due to a primary brain problem rather than any skull anomaly (Item C182). Microcephaly in the presence of abnormal neurodevelopment, as described for this infant, indicates a need for a thorough evaluation to determine the neurologic cause of symptoms. Magnetic resonance imaging (MRI) allows detailed assessment of all brain structures, including the brainstem and gray and white matter integrity, features that can guide additional evaluation. Head circumference should be measured and plotted at birth and at all subsequent health supervision visits, particularly for the first 3 postnatal years, because head growth normally corresponds with rapid brain growth in early childhood. Suboptimal head growth during the first year and microcephaly at 1 year of age are associated with subnormal developmental outcomes in children who had hypoxic-ischemic injuries at term birth. Head ultrasonography is a reasonable alternative to MRI because it does not require sedation and, therefore, is easier on the child (and family). However, it has lower resolution for peripheral gray matter and deep brainstem structures. This is a better choice for evaluation of a large head (macrocephaly) because it can diagnose hydrocephalus accurately. Electroencephalography findings are too nonspecific to guide diagnosis or treatment. High-resolution karyotyping may be reasonable, particularly if the infant displays other dysmorphic features or congenital brain structural anomalies, because chromosomal abnormalities can result in microcephaly. However, this evaluation can be deferred until after additional information about brain structure has been obtained. For example, if there is evidence of a stroke, hypoxic-ischemic injury, or a congenital infection, karyotyping may not be needed. Three-dimensional computed tomography scan is used to diagnose craniosynostosis. Craniosynostosis usually presents with an unusual head shape due to closure of one or more skull sutures. This infant's symmetric microcephaly plus an abnormal neurodevelopmental history make a bony abnormality in the skull unlikely.

References:

Kinsman SL, Johnston MV. Congenital anomalies of the central nervous system. In: Kliegman RM, Behrman RE, Jenson HB, Stanton BF, eds. Nelson Textbook of Pediatrics. 18th ed. Philadelphia, Pa: Saunders Elsevier; 2007:2443-2456

Shevell M, Ashwal S, Donley D, et al. Practice parameter: evaluation of the child with global developmental delay. Report of the Quality Standards Subcommittee of the American Academy of Neurology and The Practice Committee of the Child Neurology Society. Neurology. 2003;60:367-380. Available at: http://www.neurology.org/cgi/content/full/60/3/367

Shevell M, Majnemer A, Platt RW, Webster R, Birnbaum R. Developmental and functional outcomes in children with global developmental delay or developmental language impairment. Dev Med Child Neurol. 2005;47:678-683. Available at: http://www.blackwell-synergy.com/doi/abs/10.1111/j.1469-8749.2005.tb01053.x

Page 634: AAP PREP 2009

2009 PREP SA on CD-ROM

page 634

Critique: 182

Page 635: AAP PREP 2009

2009 PREP SA on CD-ROM

page 635

Question: 183

A 2-day-old infant is transferred from the regular nursery to the neonatal intensive care unit for evaluation and management of poor feeding and lethargy. A serum metabolic panel reveals a carbon dioxide concentration of 12.0 mEq/L (12.0 mmol/L) and a borderline low white blood cell count. After stopping all feedings, a septic evaluation is performed, and intravenous antibiotics are started. During this time, she becomes alert and vigorous, and her carbon dioxide value normalizes. Three days later, results of the septic evaluation are negative, and the infant resumes human milk feedings. Initially she does well, but after 2 days, she begins to vomit and becomes less active. Serum metabolic panel shows a glucose concentration of 35.0 mg/dL (1.9 mmol/L), a carbon dioxide concentration of 8.0 mEq/L (8.0 mmol/L), and an anion gap of 25; the serum ammonia value is twice the upper limit of normal. The baby is again made NPO and given intravenous fluids.

Of the following, the MOST critical diagnostic test for this baby is

A. lysosomal enzyme analysis

B. peroxisomal enzyme analysis

C. serum galactose-1-phosphate measurement

D. serum toxicology screen

E. urine organic acids measurement

Page 636: AAP PREP 2009

2009 PREP SA on CD-ROM

page 636

Critique: 183 Preferred Response: E

The finding of metabolic acidosis with an increased anion gap in an infant or child who is acutely ill, such as the infant described in the vignette, is an important clue to the possible diagnosis of an inborn error of metabolism. Even so, it is important to remember that any condition that produces a metabolic acidosis, such as infection and necrosis, can present similarly. Of the inborn metabolic errors, those characteristically associated with overwhelming metabolic acidosis in the young infant are the organic acid disorders, especially propionic, methylmalonic, and isovaleric acidemias. The infant in the vignette has propionic acidemia. Evaluation of the infant or child who has acidosis and an elevated anion gap should include measurement of plasma lactate, plasma pyruvate, complete blood and platelet counts, serum ammonia, urine organic and amino acids, plasma amino acids, and total and free carnitine as well as an acylcarnitine profile. Metabolic acidosis can interfere with normal urea cycle function, leading to elevated serum ammonia concentrations; it also can depress bone marrow production. Lysosomal storage disorders, such as Hurler and Hunter syndromes, typically do not present in the newborn period with metabolic imbalance; they become apparent in toddlers who exhibit decreased acquisition of skills and progressive deformation. Peroxisomal enzyme disorders that present in the newborn period, such as Zellweger syndrome, are associated with altered sensorium, hypotonia, and hepatomegaly and are not responsive to the withholding of feedings. Serum galactose-1-phosphate concentrations are elevated in the most common form of galactosemia. Galactosemia presents after days to weeks of human or cow milk feedings and is associated with jaundice, hepatomegaly, and Escherichia coli sepsis. A serum toxicology screen in this neonate would not be expected to reveal the source of her illness, and the infant's presentation is not characteristic of maternal substance abuse.

References:

Burton BK. Inborn errors of metabolism in infancy: a guide to diagnosis. Pediatrics. 1998;102:e69-e78. Available at: http://pediatrics.aappublications.org/cgi/content/full/102/6/e69

Enns GM. Inborn errors of metabolism masquerading as hypoxic-ischemic encephalopathy. NeoReviews. 2005;6:e549-e558. Available for subscription at: http://neoreviews.aappublications.org/cgi/content/full/6/12/e549

Hoffmann GF, Nyhan WL, Zschocke J, Kahler SG, Mayatepek E. Approach to the patient with metabolic disease. In: Inherited Metabolic Diseases. Philadelphia, Pa: Lippincott Williams & Wilkins; 2002:19-94

Nyhan WL, Barshop BA, Ozand PT. Organic acidemias. In: Atlas of Metabolic Diseases. 2nd ed. London, England: Hodder Arnold; 2005:1-108

Page 637: AAP PREP 2009

2009 PREP SA on CD-ROM

page 637

Question: 184

A 17-year-old young man comes to the clinic in the juvenile detention center with a penile discharge. He has no other symptoms. He was tested 1 week ago at a sexually transmitted infections clinic, and results of the rapid urine testing by nucleic acid amplification are positive for Neisseria gonorrhoeae and negative for Chlamydia trachomatis.

Of the following, the MOST appropriate treatment regimen is

A. benzathine penicillin G 2.4 million units intramuscularly in a single dose

B. ceftriaxone 125 mg intramuscularly in a single dose

C. ciprofloxacin 500 mg orally in a single dose

D. ofloxacin 400 mg orally in a single dose

E. spectinomycin 2 g intramuscularly in a single dose

Page 638: AAP PREP 2009

2009 PREP SA on CD-ROM

page 638

Critique: 184 Preferred Response: B

Gonorrhea is the second most commonly reported notifiable disease in the United States. Although the Centers for Disease Control and Prevention (CDC) Sexually Transmitted Diseases Treatment Guidelines, 2006, recommended treating uncomplicated gonococcal infections of the cervix, urethra, and rectum with either ceftriaxone, cefixime, ciprofloxacin, ofloxacin, or levofloxacin as single-dose therapy, in April 2007, the CDC issued an update to the guidelines, indicating that fluoroquinolones no longer are recommended for the treatment of gonococcal infections. Current recommendations for uncomplicated gonococcal infections of the cervix, urethra, and rectum include ceftriaxone 125 mg intramuscularly or cefixime 400 mg orally, both in a single dose, plus treatment for chlamydia if chlamydial infection has not been ruled out. These recommendations apply to all adult and adolescent patients, regardless of travel history or sexual behavior. Alternative regimens include spectinomycin 2 g in a single intramuscular dose or parenteral cephalosporin single-dose regimens, including ceftizoxime 500 mg intramuscularly, cefoxitin 2 g intramuscularly with probenecid 1 g orally, or cefotaxime 500 mg intramuscularly. Spectinomycin is recommended for persons who have penicillin or cephalosporin allergies. Benzathine penicillin is not recommended due to increased resistance to this drug.

References:

Centers for Disease Control and Prevention. Update to CDC's sexually transmitted diseases treatment guidelines, 2006: fluoroquinolones no longer recommended for treatment of gonococcal infections. MMWR Morbid Mortal Wkly Rep. 2007;56:332-336. Available at: http://www.cdc.gov/mmwr/preview/mmwrhtml/mm5614a3.htm

Workowski KA, Berman SM, Centers for Disease Control and Prevention. Sexually transmitted diseases treatment guidelines, 2006. MMWR Recomm Rep. 2006;55(RR11):1-94. Available at: http://www.cdc.gov/mmwr/preview/mmwrhtml/rr5511a1.htm

Page 639: AAP PREP 2009

2009 PREP SA on CD-ROM

page 639

Question: 185

A 3-year-old boy is admitted to the hospital for fever, cough, and increasing respiratory insufficiency of 2 days’ duration. Chest radiography demonstrates a right middle lobe and lower lobe pneumonia with a significant pleural effusion (Item Q185). You aspirate a sample of pleural fluid and send it to the laboratory for analysis.

Of the following, the MOST likely expected laboratory findings are

A. White Blood Cells: Moderate to high

Lactate Dehydrogenase (U/L): 2 to 3 times the serum concentrationProtein (g/dL): <3Gram Stain: Gram-positive cocciB. White Blood Cells: Moderate to high

Lactate Dehydrogenase (U/L): Equal to the serum concentrationProtein (g/dL): <3Gram Stain: Negative

C. White Blood Cells: Moderate to high

Lactate Dehydrogenase (U/L): 2 to 3 times the serum concentrationProtein (g/dL): >3Gram Stain: Gram-positive cocci

D. White Blood Cells: Rare

Lactate Dehydrogenase (U/L): Equal to the serum concentrationProtein (g/dL): >3Gram Stain: Negative

E. White Blood Cells: Rare

Lactate Dehydrogenase (U/L): Equal to the serum concentrationProtein (g/dL): <3Gram Stain: Gram-positive cocci

Page 640: AAP PREP 2009

2009 PREP SA on CD-ROM

page 640

Question: 185

(Courtesy of B. Poss)

Page 641: AAP PREP 2009

2009 PREP SA on CD-ROM

page 641

Critique: 185 Preferred Response: C

The child described in the vignette has a pleural effusion and lobar pneumonia. Pleural effusions in pediatric patients have a variety of causes, including pneumonia, trauma, malignancy, renal disease, liver failure, and congestive heart disease. Parapneumonic effusions (those associated with an underlying pneumonia) account for up to 70% of pediatric pleural effusions. Pleural effusions may be seen in up to 40% of bacterial pneumonias; progression to pus (empyema) occurs in more than 50% of these cases and is increasing in both incidence and associated complications. Classification of pleural fluid into transudates and exudates may assist the clinician in determining causes and guiding treatment (Item C185). Transudates are plasma ultrafiltrates and usually result from renal and liver disease or congestive heart failure. Exudates arise from inflammatory processes (such as pneumonia, malignancies, trauma, or systemic inflammatory diseases) or impaired lymphatic drainage, often following thoracic surgery (eg, chylothorax). Empyema, a type of exudate, is characterized by the presence of white blood cells, a positive Gram stain, or frank pus. The child described in the vignette most likely has an exudate based on the lactate dehydrogenase and protein values. A Gram stain may reveal bacteria (in this case, a gram-positive cocci), although bacterial culture results may be negative, especially if the patient has received prior antibiotic therapy. The pH value of the fluid should be low due to consumption of glucose by the bacteria, and there should be moderate-to-high white blood cell counts, consistent with an empyema. Lactate dehydrogenase values often are two to three times the serum concentrations, and protein values frequently are 3.0 g/dL (30.0 g/L) or greater in pleural effusions. Tube thoracostomy, potentially aided by video-assisted thoracic surgery, and antibiotics are the recommended treatment.

References:

Efrati O, Barak A. Pleural effusions in the pediatric population. Pediatr Rev. 2002:23:417-426. Available at: http://pedsinreview.aappublications.org/cgi/content/full/23/12/417

Schultz KD, Fan LL, Pinksy J, et al. The changing face of pleural empyemas in children: epidemiology and management. Pediatrics. 2004;113:1735-1740. Available at: http://pediatrics.aappublications.org/cgi/content/full/113/6/1735

Winnie GB. Pleurisy, pleural effusions, and empyema. In: Kliegman RM, Behrman RE, Jenson HB, Stanton BF, eds. Nelson Textbook of Pediatrics. 18th ed. Philadelphia, Pa: Saunders Elsevier; 2007:1832-1834

Page 642: AAP PREP 2009

2009 PREP SA on CD-ROM

page 642

Question: 186

You are seeing a short 9-year-old boy. He is growing steadily in height just below the third percentile on the growth curve. His parents ask if he will be very short when he finishes growing.

Of the following, the MOST important information needed to answer this question is

A. ethnicity of family

B. parent heights

C. sibling heights

D. upper-to-lower segment ratio

E. weight-for-age curve

Page 643: AAP PREP 2009

2009 PREP SA on CD-ROM

page 643

Critique: 186 Preferred Response: B

Short stature is a statistical diagnosis representing a stature less than the third percentile for height (3% of all children). Most children growing along lower percentiles, such as the boy described in the vignette, are not growing slowly by the time they reach middle childhood; they dropped to this percentile some time in the first 2 years after birth and now have a normal growth rate. Most either have short parents or will advance through puberty slowly (constitutional delay) and end up taller than anticipated by their childhood position on the growth curve. Accordingly, parent heights are the most pertinent facts needed to answer the question about this child's adult height. Family ethnicity is only generally useful in evaluating short stature; within-family heights are much more important for the individual child. Sibling height, unless the siblings have reached full adult height, are not as useful as adult parental height because the sibling adult heights still are unknown. Children who have particularly short legs or short spines may have an underlying chondrodystrophy or skeletal dysplasia. Measurement of upper-to-lower segment ratio can be a useful clinical examination when considering these conditions, although they are very rare. Children who are particularly underweight for height or very overweight for height may grow poorly. However, most overweight children grow exceedingly well because of their overnourished status. A very malnourished child may have a decreased growth rate. The steady growth in the child in the vignette suggests that nutritional status is not the most important determinant of his present height. Children who have normal growth rates are unlikely to have underlying endocrine or metabolic disorders or serious underlying illnesses to explain their stature. In contrast, children who have growth attenuation are much more likely to have underlying and perhaps specifically remediable causes for short stature.

References:

Ferry RJ Jr. Short stature. eMedicine Specialties, Pediatrics: General Medicine, Endocrinology. 2007. Available at: www.emedicine.com/ped/topic2087.htm

Plotnick LP, Miller RS. Growth, growth hormone, and pituitary disorders. In: McMillan JA, Feigin RD, DeAngelis C, Jones MD Jr. Oski's Pediatrics, Principles & Practice. Philadelphia, Pa: Lippincott, Williams & Wilkins; 2006:2084-2092

Rogol AD. Causes of short stature. UpToDate Online 15.3. 2008. Available for subscription at: http://www.uptodateonline.com/utd/content/topic.do?topicKey=pediendo/2279

Page 644: AAP PREP 2009

2009 PREP SA on CD-ROM

page 644

Question: 187

A 4-year-old boy cannot attend a local nursery school because he is not toilet trained. His development is otherwise normal. His parents explain that when they attempt to put him on the toilet, he refuses and runs out of the bathroom. They ask how they can train him to use the toilet.

Of the following, the MOST appropriate approach is to

A. develop a behavioral modification program to encourage him to use the toilet

B. have the parents gently scold him when he has accidents

C. insist he sit on the toilet every 2 hours during the day

D. recommend the family find a different nursery school that allows children who are not toilet trained

E. tell the parents to have him clean his own clothes after toilet accidents

Page 645: AAP PREP 2009

2009 PREP SA on CD-ROM

page 645

Critique: 187 Preferred Response: A

In the United States, 98% of children are continent during the day by the time they are 36 months old. Toilet training usually requires about 3 to 6 months for successful completion. For children such as the boy described in the vignette, who appears to be late in achieving toilet training, a behavioral modification program can be established to encourage use of the toilet. It is best to have the child take responsibility for being toilet trained, and a reward system using a star chart to earn a desired object may help him to meet this goal. Punishing him for not using the toilet or insisting that he clean his own clothes could lead to noncompliance and adversely affect his self-esteem. Insisting a child sit on the potty chair or forcing him or her to sit may increase the child's resistance. Often, a parent pushing a child to become toilet trained due to a preschool requirement leads to a power struggle, and the child does not achieve continence. Recommending that the parents find a different nursery school that does not require a child to be toilet trained may alleviate some of their stress, but it does not address how to toilet train a child. Toilet training begins with the parent encouraging the child to practice running to the potty chair. The parent should praise or reward the child for complying with the practice session. The child should be changed after any accidents, and the parents should avoid use of physical or verbal punishment. The use of underwear, time-in (provide the child with positive reinforcement), and incentives may help to increase a child's motivation.

References:

Parker S. Toilet training. In: Parker S, Zuckerman B, Augustyn M, eds. Developmental and Behavioral Pediatrics: A Handbook for Primary Care. 2nd ed. Philadelphia, Pa: Lippincott Williams & Wilkins; 2005:355-357

Schmitt B. Toilet training: getting it right the first time. Contemp Pediatr. 2004;21:105

Wolraich ML, Tippins S, ed. Guide to Toilet Training. Elk Grove Village, Ill: American Academy of Pediatrics; 2003

Page 646: AAP PREP 2009

2009 PREP SA on CD-ROM

page 646

Question: 188

The nurse caring for a 5-day-old infant you have hospitalized calls your office to report that the infant’s blood culture is growing gram-positive rods. You admitted the infant to the hospital because of a rectal temperature of 102.0°F (38.9°C) measured by his mother at home.

Of the following, the MOST likely pathogen is

A. Enterococcus sp

B. Escherichia coli

C. Listeria monocytogenes

D. Proteus mirabilis

E. Staphylococcus epidermidis

Page 647: AAP PREP 2009

2009 PREP SA on CD-ROM

page 647

Critique: 188 Preferred Response: C

Listeria monocytogenes is a small, motile, gram-positive rod. Three major serotypes of Listeria infect humans: 1a, 1b, and 4b. Infections in the newborn result from asymptomatic fecal or vaginal carriage in pregnant women, with the organism colonizing the baby during passage through the birth canal. Although bacterial infections due to group B Streptococcus and gram-negative organisms such as Escherichia coli or Klebsiella pneumoniae are more common, neonatal sepsis and meningitis due to Listeria represent a major risk to the neonate, especially in certain geographic regions of the United States (eg, the southwestern area). Illness with Listeria can present as early-onset disease (from birth to 7 days) or late-onset disease (>7 days after birth). The typical characteristics of early-onset infections are bacteremia or pneumonia and preterm birth; late-onset disease usually results in meningitis. For older infants and children, listerial infection is associated with ingestion of contaminated foods; unpasteurized milk, soft cheese, and prepared meats (eg, hot dogs, deli meats) are the sources of many infections. Most of these infections present with fever, malaise, headache, gastrointestinal tract symptoms, or back pain. Bacteremia and meningitis also can occur. Proteus mirabilis and E coli are gram-negative rods, and Enterococcus sp and Staphylococcus epidermidis are gram-positive cocci.

References:

American Academy of Pediatrics. Listeria monocytogenes infections (listeriosis). In: Pickering LK, Baker CJ, Long SS, McMillan JA, eds. Red Book: 2006 Report of the Committee on Infectious Diseases. 27th ed. Elk Grove Village, Ill: American Academy of Pediatrics; 2006:426-428

Posfay-Barbe KM, Wald ER. Listeriosis. Pediatr Rev. 2004;25:151-159. Available at: http://pedsinreview.aappublications.org/cgi/content/full/25/5/151

Page 648: AAP PREP 2009

2009 PREP SA on CD-ROM

page 648

Question: 189

A mother calls you to report that her 7-year-old son came home with a notice from school stating that a child in his class was diagnosed with mumps. The mother does not know the immunization status of the infected child but states that her son has received two measles-mumps-rubella (MMR) vaccines and is up to date on all his other immunizations. Her son has been asymptomatic, with no fever or other systemic complaints.

Of the following, the MOST appropriate action is to

A. administer a dose of mumps immune globulin to her son

B. confirm that her son has received two doses of MMR vaccine

C. keep her son home from school for 9 days to observe for the development of symptoms

D. treat her son with a course of ribavirin

E. vaccinate her son immediately with another dose of MMR to prevent infection from this exposure

Page 649: AAP PREP 2009

2009 PREP SA on CD-ROM

page 649

Critique: 189 Preferred Response: B

In the United States, mumps vaccine is administered as part of the measles-mumps-rubella (MMR) vaccine routinely to children at 12 to 15 months of age, with a second dose of MMR typically administered at 4 to 6 years of age. Protective efficacy of the vaccine is estimated to be more than 95%. In cases of exposure, such as described in the vignette, it is important to ensure that the exposed person has received the recommended number of doses of MMR vaccine because mumps outbreaks have occurred in people in highly immunized populations who previously have received only a single dose of mumps-containing vaccine. Therefore, the most appropriate action is to confirm with the mother that her son has received two doses of MMR vaccine. Mumps vaccine has not been demonstrated to be effective in preventing infection after exposure. However, the vaccine can be administered after exposure to provide protection against subsequent exposures in persons who are not fully vaccinated. Fully immunized persons do not need to be excluded from school after exposure. Students who are not fully immunized are excluded from school until they are immunized, after which they can be readmitted immediately to school. Students who refuse mumps vaccination because of medical, religious, or other reasons should be excluded from school for at least 26 days after the onset of parotitis in the last person who has mumps in the affected school. Persons who have mumps are excluded from school for 9 days from the onset of their parotid swelling. Immune globulin (IG) and mumps IG are not effective as postexposure prophylaxis measures. In fact, mumps IG no longer is available in the United States. Treatment of the disease is supportive; no effective antiviral agents are available.

References:

American Academy of Pediatrics. Mumps. In: Pickering LK, Baker CJ, Long SS, McMillan JA, eds. Red Book: 2006 Report of the Committee on Infectious Diseases. 27th ed. Elk Grove Village, Ill: American Academy of Pediatrics; 2006:464-468

Centers for Disease Control and Prevention (CDC). Notice to readers: updated recommendations of the Advisory Committee on Immunization Practices (ACIP) for the control and elimination of mumps. MMWR Morb Mortal Wkly Rep. 2006;55:629-630. Available at: http://www.cdc.gov/mmwr/preview/mmwrhtml/mm55e601a1.htm

Page 650: AAP PREP 2009

2009 PREP SA on CD-ROM

page 650

Question: 190

A 4-year-old boy presents with periorbital edema. He is receiving no medications, and his family history is negative for renal disease. On physical examination, he is afebrile; his heart rate is 88 beats/min, respiratory rate is 18 breaths/min, and blood pressure is 106/62 mm Hg; and he has periorbital (Item Q190A) and pitting pretibial edema (Item Q190B). Laboratory evaluation shows normal electrolyte values, blood urea nitrogen of 14.0 mg/dL (5.0 mmol/L), creatinine of 0.3 mg/dL (26.5 mcmol/L), and albumin of 1.6 g/dL (16.0 g/L). Urinalysis demonstrates a specific gravity of 1.020; pH of 6.5; 3+ protein; and negative blood, leukocyte esterase, and nitrite. Microscopy results are normal. Additionally, complement component (C3 and C4) values are normal, and results of serologic testing for antinuclear antibody, hepatitis B and C, and human immunodeficiency virus are negative.

Of the following, you are MOST likely to advise the parents that

A. a renal biopsy is warranted to determine the optimal treatment

B. disease relapse can be expected in fewer than 25% of those achieving remission

C. patients who relapse have a similar prognosis as those who do not respond to steroids

D. remission is expected in more than 75% of patients who receive corticosteroid treatment

E. tacrolimus is the preferred treatment for patients who do not respond to corticosteroids

Page 651: AAP PREP 2009

2009 PREP SA on CD-ROM

page 651

Question: 190

Periorbital edema, as exhibited by the boy in the vignette. (Courtesy of M. Rimsza)

Page 652: AAP PREP 2009

2009 PREP SA on CD-ROM

page 652

Question: 190

Pretibial pitting edema (arrow), as exhibited by the boy in the vignette. (Reprinted with permission from Cavanaurgh RM Jr. Orthostatic edema in adolescents: more than walking on water. Pediatr Rev. 2005;26:115-124.)

Page 653: AAP PREP 2009

2009 PREP SA on CD-ROM

page 653

Critique: 190 Preferred Response: D

The boy described in the vignette has signs and symptoms of nephrotic syndrome. Causes of nephrotic syndrome can be subdivided into primary glomerulopathies (eg, minimal change disease, focal segmental glomerulosclerosis, mesangial proliferative glomerulonephritis) and secondary glomerulopathies (eg, lupus nephritis, membranoproliferative glomerulonephritis due to hepatitis C, and human immunodeficiency virus nephropathy). The initial evaluation of new-onset nephrotic syndrome centers on establishing whether it is due to a primary glomerulopathy and warrants an empiric trial of corticosteroids. Clinical features that suggest secondary glomerulopathy include moderate-to-severe hypertension or gross hematuria. Laboratory markers suggestive of a secondary glomerulopathy include azotemia, hypocomplementemia, a positive antinuclear antibody finding, or a positive serologic test for human immunodeficiency virus or hepatitis B or C. The normal blood pressure and renal function as well as the absence of hematuria, hypocomplementemia, or a positive serologic test for lupus or viral infection described for the boy in the vignette are consistent with primary glomerulopathy. Accordingly, this child should receive a 4- to 6-week trial of daily corticosteroids because nearly 90% of childhood nephrotic syndrome cases are sensitive to steroids. Although 90% of pediatric patients achieve a state of remission with corticosteroids, 50% to 70% experience at least one relapse during their disease course. Unless clinical or serologic findings raise the possibility of an underlying glomerulonephritis, renal biopsy is not warranted. The steroid response, not renal biopsy findings, is the best predictor of the prognosis for childhood nephrotic syndrome. Steroid-sensitive patients, even those who have a relapsing course, have a better prognosis than steroid-resistant patients. The approximately 10% of patients who are steroid-resistant have a guarded prognosis, with a risk of developing chronic renal failure. Steroid-resistant patients require a renal biopsy to establish histologic information. In most circumstances, the first-line medication for steroid-resistant nephrotic syndrome is cyclosporine. More recently, another calcineurin inhibitor, tacrolimus, has been used in children who fail to respond to cyclosporine. At present, tacrolimus is considered a second- or third-line treatment for steroid-resistant nephrotic syndrome.

References:

Niaudet P. Steroid-sensitive idiopathic nephrotic syndrome in children. In: Avner ED, Harmon WE, Niaudet P, eds. Pediatric Nephrology. 5th ed. Philadelphia, Pa: Lippincott Williams & Wilkins; 2004:543-556

Niaudet P. Steroid-resistant idiopathic nephrotic syndrome in children. In: Avner ED, Harmon WE, Niaudet P, eds. Pediatric Nephrology. 5th ed. Philadelphia, Pa: Lippincott Williams & Wilkins; 2004:557-573

Valentini RP, Smoyer WE. Nephrotic syndrome. In: Kher KK, Schnaper HW, Makker SP, eds. Clinical Pediatric Nephrology. 2nd ed. London, England: Informa Healthcare; 2007:155-194

Page 654: AAP PREP 2009

2009 PREP SA on CD-ROM

page 654

Question: 191

You are evaluating a 14-year-old girl for seasonal allergic rhinitis. Despite a regimen of multiple allergy medications, she continues to have significant sneezing, rhinorrhea, and nasal congestion. You decide to evaluate for possible allergic triggers and discuss the advantages and disadvantages of allergy skin testing and blood testing.

Of the following, a TRUE statement regarding allergy skin and blood testing is that

A. infants younger than 1 year of age cannot undergo skin testing

B. patients may experience anaphylaxis during aeroallergen or food skin testing

C. patients need to fast prior to blood allergy testing

D. patients need to stop their antihistamines prior to blood allergy testing

E. the negative predictive value of aeroallergen skin testing is poor

Page 655: AAP PREP 2009

2009 PREP SA on CD-ROM

page 655

Critique: 191 Preferred Response: B

Two primary diagnostic tools are used to determine the role of indoor and outdoor aeroallergens as triggers for allergic rhinitis or allergic asthma: skin testing and blood testing. Aeroallergen skin testing involves the application of specific allergens (eg, oak, Bermuda grass, cat, ragweed) on the skin, typically using a prick or puncture method. Although sometimes uncomfortable for infants and toddlers, allergy skin testing is tolerated extremely well by most children and adolescents and can be performed at any age. The advantages of skin testing are that a broad array of allergens can be tested, testing materials are inexpensive, and results are immediately evident to the patient. One disadvantage is that patients must stop their antihistamine medication(s) 1 week prior to skin testing. Also, although most patients tolerate the local pruritus experienced at "positive" skin test sites, those who are very sensitive (eg, severe food anaphylaxis) may experience a systemic reaction with even a simple skin test. For patients who have a history of severe anaphylaxis to a specific allergen, allergists may choose to perform serum immunoglobulin (Ig) E testing instead of skin testing because blood testing does not have a risk for anaphylaxis. In the past, serum IgE testing employed primarily the radioallergosorbent test (RAST) method. Because of the significant variability in results between laboratories, RAST has been replaced in most institutions with the more sensitive and reproducible CAP-system fluorescein enzyme immunoassay. This system uses a cellulose matrix system. The advantage of serum IgE testing is that it is not affected by medications (ie, patients do not need to stop an antihistamine). Patients do not need to fast prior to either allergy skin or blood testing. While ongoing studies are comparing the sensitivity and specificity of skin testing compared with the CAP system fluorescein enzyme immunoassay, skin testing is regarded as more sensitive and specific. Finally, although skin testing is considered "inexpensive," most general pediatricians find the cost of an allergy consultation with skin testing to be more expensive than a routine battery of serum IgE tests for aeroallergens or food. The availability and clinical application of serum IgE testing continues to expand, but clinicians who do not seek allergy consultation should be comfortable with interpretation and application of test results for a specific clinical scenario (eg, a wheat IgE of 10 kU/L in a patient who has atopic dermatitis has little to no clinical significance).

References:

Cartwright RC, Dolen WK. Consultation with the specialist: who needs allergy testing and how to get it done. Pediatr Rev. 2006;27:140-146. Available at: http://pedsinreview.aappublications.org/cgi/content/full/27/4/140

Mahr TA, Sheth K. Update on allergic rhinitis. Pediatr Rev. 2005;26:284-289. Available at: http://pedsinreview.aappublications.org/cgi/content/full/26/8/284

Page 656: AAP PREP 2009

2009 PREP SA on CD-ROM

page 656

Question: 192

A 3-year-old boy is brought to the emergency department at 8 am after his parents found him unresponsive in bed. The last time they had seen him awake was at 2 am, when they found him playing in the living room as they were cleaning up after a cocktail party. On physical examination, the child has diaphoresis and moans to painful stimuli. His vital signs include a temperature of 96.4°F (35.8°C), heart rate of 145 beats/min, respiratory rate of 20 breaths/min, blood pressure of 84/34 mm Hg, and oxygen saturation of 97% in room air. His pupils are mid-sized and sluggishly reactive.

Of the following, the MOST important test to obtain at this time is

A. acetylcholinesterase determination

B. bedside glucose determination

C. blood alcohol determination

D. serum osmolality

E. urine toxicology screen

Page 657: AAP PREP 2009

2009 PREP SA on CD-ROM

page 657

Critique: 192 Preferred Response: B

The approach to any pediatric patient who presents with acute alteration in mental status should include assessment of airway patency, adequacy of oxygenation, ventilation, and perfusion as well as a bedside glucose determination. Hypoglycemia may result from a variety of causes, including inborn errors of metabolism, poisonings, sepsis, shock, liver failure, and starvation. Signs and symptoms of hypoglycemia are related to counterregulatory epinephrine release and cerebral glucopenia. These include diaphoresis, tachycardia, tremulousness, pallor, irritability, somnolence, and coma. Unrecognized hypoglycemia can cause mortality as well as significant morbidity from seizure activity and if chronic, impaired brain development. Because hypoglycemia is treated easily with intravenous dextrose, oral glucose, or in some settings, glucagon, rapid diagnosis is critical. A number of poisonings are well known causes of hypoglycemia in children and include oral hypoglycemic agents, insulin, beta-blockers, salicylates, and ethanol. Ethanol intoxication with hypoglycemia is a likely cause of the symptoms described for the patient in the vignette because of the parents' report of unsupervised activity in an environment where ethanol was accessible. Other clues to ethanol ingestion in children who have hypoglycemia include an anion gap acidosis, an osmolar gap, and an elevated lactate value. The hypoglycemia caused by ethanol is related to gluconeogenesis inhibition and is not responsive to glucagon. Although acetylcholinesterase and blood alcohol determinations, measurement of serum osmolality, and a urine toxicology screen may be indicated in the evaluation of a patient who has acute alteration of mental status, the bedside glucose determination is the most appropriate initial test because it permits rapid identification of hypoglycemia and prompt treatment.

References:

Ernst AA, Jones K, Nick TG, Sanchez J. Ethanol ingestion and related hypoglycemia in a pediatric and adolescent emergency department population. Acad Emerg Med. 1996;3:46-49. Abstract available at: http://www.ncbi.nlm.nih.gov/pubmed/8749967

Sperling M. Hypoglycemia. In: Kleigman RM, Behrman RE, Jenson HB, Stanton BF, eds. Nelson Textbook of Pediatrics. 18th ed. Philadelphia, Pa: Saunders Elsevier; 2007:655-670

Sunehag A, Haymond MW. Etiology of hypoglycemia in infants and children. UpToDate Online 15.3. 2008. Available for supscription at:http://www.utdol.com/utd/content/topic.do?topicKey=pediendo/11162&selectedTitle=4~29&source=search_result

Page 658: AAP PREP 2009

2009 PREP SA on CD-ROM

page 658

Question: 193

A 3-year-old child presents to your office with chronic recurrent diarrhea of 3 months’ duration. He attends child care during the week. He is one of four children in the family, the oldest of whom is 8 years old. Stool microscopic analysis identifies Giardia lamblia. You treat the boy with metronidazole for 10 days. On a follow-up visit 30 days after initiating treatment, the mother states that the symptoms initially improved, but have recurred.

Of the following, the BEST explanation for the persistent symptoms is

A. Clostridium difficile infection

B. metronidazole resistance

C. persistent giardiasis

D. superimposed milk protein allergy

E. undiagnosed celiac disease

Page 659: AAP PREP 2009

2009 PREP SA on CD-ROM

page 659

Critique: 193 Preferred Response: C

Based on the documented history of giardiasis and ongoing diarrhea for the patient described in the vignette, the most likely cause of his symptoms is persistent giardiasis, most likely from reinfection from one of his siblings or a family pet. Many Giardia infections are asymptomatic, and siblings, other children in child care, or family pets frequently can be reservoirs for the organism. Therefore, a patient treated with a course of metronidazole is susceptible to reinfection if he or she is exposed to asymptomatic carriers. Metronidazole resistance is extremely rare in Giardia parasites, and based on the history provided, there is no reason to suspect Clostridium difficile infection, celiac disease, or food allergy at this time. Giardia is a flagellated enteric-coated protozoan that is the most common cause of intestinal parasitic infections in the United States. The parasite exists in two forms, cysts and trophozoites. Cysts (Item C193A) are extremely hardy and can survive in cold water for weeks to months. The cysts also are resistant to chlorination. Once animals (eg, dogs, cats, beavers) or humans ingest contaminated water, the cysts mature into trophozoites (Item C193B) in the intestine, where they cause small intestinal inflammation, villous atrophy, and malabsorption. Asymptomatic carriage is more common than symptomatic disease. However, in patients who do develop symptoms, the most common features are diarrhea, nausea, distention, cramping, and anorexia. Because Giardia is a small bowel pathogen, it does not cause rectal bleeding. The most important aspect to diagnosing Giardia infection is having a strong clinical suspicion. A number of good diagnostic tests of the stool are available, including microscopic examination for ova and parasites (stool O and P), enzyme-linked immunofluorescence assays, and direct fluorescence antibody tests. The microscopic examination ("O and P" test) has poorer sensitivity (50%) from a single sample and ideally requires microscopic examination of three separate samples. In contrast, the Giardia antigen-based tests have a sensitivity of approximately 90% from a single stool sample. However, the O and P examination can identify other potential pathogens (Blastocystis, ameba) that are not identified with the antigen tests. In some cases, endoscopy with biopsy may be useful to exclude Giardia or other pathogens and to characterize the extent of villous atrophy. Suspected or established giardiasis can be treated with either metronidazole (5 mg/kg per dose administered three times a day for 7 days) or nitazoxanide for 3 days (100 mg bid for children younger than 3 years, 200 mg bid for children ages 4 through 11 years, 500 mg bid for patients older than 12 years of age). If asymptomatic carriers are suspected in the family, it may be prudent to evaluate and treat the entire family.

References:

American Academy of Pediatrics. Giardia intestinalis infections (giardiasis). In: Pickering LK, Baker CJ, Long SS, McMillan JA, eds. Red Book: 2006 Report of the Committee on Infectious Diseases. 27th ed. Elk Grove Village, Ill: American Academy of Pediatrics; 2006:296-301

Huang DB, White AC. An updated review on Cryptosporidium and Giardia. Gastroenterol Clin North Am. 2006;35:291-314. Abstract available at: http://www.ncbi.nlm.nih.gov/pubmed/16880067

Page 660: AAP PREP 2009

2009 PREP SA on CD-ROM

page 660

Critique: 193

Photomicrograph of a Giardia lamblia cyst using an iodine stain. (Courtesy of the Public Health Image Library, Centers for Disease Control and Prevention)

Page 661: AAP PREP 2009

2009 PREP SA on CD-ROM

page 661

Critique: 193

Scanning electron micrograph demonstrates a Giardia lamblia trophozoite. (Courtesy of the Public Health Image Library, Centers for Disease Control and Prevention)

Page 662: AAP PREP 2009

2009 PREP SA on CD-ROM

page 662

Question: 194

You admit a 39 weeks’ gestation male who has respiratory distress to the intensive care nursery. His mother had a negative group B Streptococcus screening culture and did not receive antibiotics in labor. She did not have chorioamnionitis or prolonged rupture of the fetal membranes. However, the amniotic fluid was meconium-stained at the time of delivery, and the infant required tracheal intubation, with resultant meconium suctioned from below the vocal cords. Apgar scores were 3 and 7 at 1 and 5 minutes, respectively. On physical examination, he has marked work of breathing with tachypnea and retractions and episodic cyanosis when agitated. Breath sounds are coarse and equal. There is no heart murmur. While receiving hood oxygen at an FiO2 of 0.50, his oxygen saturation by pulse oximetry is 85%. You obtain a chest radiograph.

Of the following, the radiographic findings MOST expected for this infant are

A. air bronchograms, diffusely hazy lung fields, and low lung volume

B. cardiomegaly, hazy lung fields, and pulmonary vascular engorgement

C. fluid density in the horizontal fissure, hazy lung fields with central vascular prominence, and normal lung volume

D. gas-filled loops of bowel in the left hemithorax and opacification of the right lung field

E. patchy areas of diffuse atelectasis, focal areas of air-trapping, and increased lung volumes

Page 663: AAP PREP 2009

2009 PREP SA on CD-ROM

page 663

Critique: 194 Preferred Response: E

The infant described in the vignette, who was born at term following exposure to meconium-stained amniotic fluid (MSAF), has respiratory distress and hypoxemia. Radiographic evidence of meconium aspiration syndrome (MAS), atelectasis and hyperinflation, is seen in Item C194A. Meconium aspiration may follow in utero fetal gasping and aspiration of MSAF in the fetus that has intrauterine growth restriction or asphyxia or after delivery of the newborn who has meconium in the oropharynx, nasopharynx, or trachea. If respiratory distress follows, the condition often is described as MAS. The prevalence of MAS among infants born following MSAF is less than 10%, but tends to rise beyond 40 weeks' gestation (Item C194B), especially in cases of nonreassuring fetal heart rate monitoring or perinatal asphyxia. Affected infants present with clinical manifestations of impaired pulmonary compliance and hypoxemia. The chest radiograph is essential in confirming the diagnosis. In the classic case of MAS, a diffuse pneumonitis ensues, the lung often is hyperinflated, and there are patchy infiltrates and opacification of the pulmonary parenchyma. Complication with air-leak syndromes such as pneumothorax or pneumomediastinum is not uncommon. In some cases, chest radiography demonstrates pulmonary hypoperfusion without any apparent infiltrates or air leaks, and these cases generally represent hypoxia-mediated pulmonary vascular reactivity and pulmonary hypertension that may have been initiated in utero. Air bronchograms in diffusely hazy low volume lungs are most consistent with surfactant deficiency-related respiratory distress syndrome (Item C194C) in preterm infants. The same findings in a term infant might be seen in congenital pneumonia. Cardiomegaly with hazy lung fields and pulmonary vascular engorgement is seen in left-sided obstructive cardiac disease states or infants who have pulmonary overcirculation (eg, truncus arteriosus, aortic stenosis, anomalous pulmonary venous return, transposition of the great arteries) (Item C194D). Fluid densities in the horizontal fissure with hazy lung fields, normal lung volumes, and central vascular prominence are seen in retained fetal lung fluid, otherwise known as transient tachypnea of the newborn (Item C194E). When gas-filled loops of bowel are seen in the chest, diaphragmatic hernia is the diagnosis (Item C194F).

References:

Aly H. Respiratory disorders in the newborn: identification and diagnosis. Pediatr Rev. 2004;25:201-208. Available at: http://pedsinreview.aappublications.org/cgi/content/full/25/6/201

Dargaville PA, Copnell B for the Australian and New Zealand Neonatal Network. The epidemiology of meconium aspiration syndrome: incidence, risk factors, therapies, and outcome. Pediatrics. 2006;117:1712-1721. Available at: http://pediatrics.aappublications.org/cgi/content/full/117/5/1712

Miller MJ, Fanaroff AA, Martin RJ. Respiratory disorders in preterm and term infants. In: Martin RJ, Fanaroff AA, Walsh MC, eds. Fanaroff and Martin's Neonatal-Perinatal Medicine. 8th ed. Philadelphia, Pa: Mosby Elsevier; 2006:1122-1145

Ross MG. Meconium aspiration syndrome-more than intrapartum meconium. N Engl J Med. 2005;353:946-948. Extract available at: http://content.nejm.org/cgi/content/extract/353/9/946

Page 664: AAP PREP 2009

2009 PREP SA on CD-ROM

page 664

Critique: 194

Meconium aspiration: There are areas of atelectasis (arrows) and hyperinflation (seen best at the bases). (Reprinted with permission from Aly H. Respiratory disorders in the newborn: identification and diagnosis. Pediatr Rev. 2004;25:201-208.)

Page 665: AAP PREP 2009

2009 PREP SA on CD-ROM

page 665

Critique: 194

Incidence of meconium aspiration syndrome related to gestational age. (Reprinted with permission from Dargaville PA, Copnell B for the Australian and New Zealand Neonatal Network. The epidemiology of meconium aspiration syndrome: incidence, risk factors, therapies, and outcome. Pediatrics. 2006;117:1712-1721.)

Page 666: AAP PREP 2009

2009 PREP SA on CD-ROM

page 666

Critique: 194

Underinflation, a "ground-glass" appearance, and air bronchograms (arrows) are characteristic of respiratory distress syndrome. (Courtesy of B. Carter)

Page 667: AAP PREP 2009

2009 PREP SA on CD-ROM

page 667

Critique: 194

Chest radiograph from an infant who has transposition of the great vessels revealing prominent pulmonary vessels (arrows), suggesting pulmonary overcirculation. (Reprinted with permission from:Aly H. Respiratory disorders in the newborn: identification and diagnosis. Pediatr Rev. 2004;25:201-208.)

Page 668: AAP PREP 2009

2009 PREP SA on CD-ROM

page 668

Critique: 194

Chest radiograph in transient tachypnea of the newborn shows increased pulmonary interstitial markings and fluid in the interlobar fissures (arrows). (Reprinted with permission from Aly H. Respiratory disorders in the newborn: identification and diagnosis. Pediatr Rev. 2004;25:201-208.)

Page 669: AAP PREP 2009

2009 PREP SA on CD-ROM

page 669

Critique: 194

Plain radiograph of the chest and abdomen in a patient who has congenital diaphragmatic hernia shows bowel in the left chest, with displacement of the heart to the right. (Courtesy of B. Carter)

Page 670: AAP PREP 2009

2009 PREP SA on CD-ROM

page 670

Question: 195

A mother brings her 5-year-old girl to your office because she noticed a "lump" in her daughter’s neck over the past several weeks. The girl appears well and has normal vital signs and no fever. A 1x1-cm slightly soft mass (Item Q195) is apparent in the middle of her neck, and when she swallows, the mass moves vertically. There is no drainage or overlying erythema.

Of the following, the MOST likely diagnosis is

A. branchial cleft cyst

B. cystic hygroma

C. reactive lymphadenopathy

D. thyroglossal duct cyst

E. thyroid nodule

Page 671: AAP PREP 2009

2009 PREP SA on CD-ROM

page 671

Question: 195

Mass, as described for the child in the vignette. (Courtesy of M. Rimsza)

Page 672: AAP PREP 2009

2009 PREP SA on CD-ROM

page 672

Critique: 195 Preferred Response: D

The differential diagnosis of neck masses in children is extensive and includes inflammatory masses, neoplasms, and congenital lesions. The most common masses are inflammatory and include reactive lymphadenopathy and lymphadenitis. Reactive lymphadenopathy occurs when an area drained by the lymph node becomes infected, as in anterior cervical adenopathy with streptococcal or viral pharyngitis. The affected lymph nodes are enlarged, mobile, and usually tender, but overt signs of infection, such as erythema and warmth, are absent. Lymphadenitis occurs when the node itself becomes infected, and erythema, tenderness, warmth, and occasionally fluctuance are present. Congenital lesions, although present from birth, may not become evident until later in childhood, often after a simple upper respiratory tract infection. These result from failure of branchial clefts to close properly during embryonic development. The most common is the branchial cleft cyst (Item C195A), which may be unilateral or bilateral and is nonmobile. A thyroglossal duct cyst is a soft midline mass (Item C195B) that moves vertically when the child swallows or protrudes the tongue, as described for the girl in the vignette. Drainage onto the skin may be seen with these lesions. Both types of lesions may become infected and appear indistinguishable from lymphadenitis (especially unilateral branchial cleft cysts), but their persistence after appropriate antibiotic treatment suggests the diagnosis. Neoplasms, such as leukemia and lymphoma, may present with enlarged lymph nodes in the cervical area. Characteristics include nontender, firm nodes that are matted to the adjacent tissues. A cystic hygroma (Item C195C), or lymphangioma, is a soft mass of dilated lymph vessels. These benign tumors of the lymphatic system can grow to a large size and infiltrate local structures and blood vessels. Thyroid nodules are infrequent in children. They may be firm or soft masses and are usually lateral rather than midline. Movement on swallowing is not typical.

References:

Camitta BM. The lymphatic system. In: Kliegman RM, Behrman RE, Jenson HB, Stanton BF, eds. Nelson Textbook of Pediatrics. 18th ed. Philadelphia, Pa: Elsevier Saunders; 2007:2092-2096

Tracy TF Jr, Muratore CS. Management of common head and neck masses. Semin Pediatr Surg. 2007;16:3-13. Abstract available at: http://www.ncbi.nlm.nih.gov/pubmed/17210478

Waldhausen JH. Branchial cleft and arch anomalies in children. Semin Pediatr Surg. 2006;15:64-69. Abstract available at: http://www.ncbi.nlm.nih.gov/pubmed/16616308

Page 673: AAP PREP 2009

2009 PREP SA on CD-ROM

page 673

Critique: 195

Branchial cleft cysts do not appear in the midline. Typically, they are located laterally along the anterior border of the sternocleidomastoid muscle. (Courtesy of D. Epstein)

Page 674: AAP PREP 2009

2009 PREP SA on CD-ROM

page 674

Critique: 195

A thyroglossal duct cyst is a soft midline mass that moves vertically when the child swallows or protrudes the tongue. (Courtesy of M. Rimsza)

Page 675: AAP PREP 2009

2009 PREP SA on CD-ROM

page 675

Critique: 195

Cystic hygromas represent macrocystic lymphatic malformations. They typically are soft and not localized only to the midline neck. (Courtesy of the Media Lab at Doernbecher)

Page 676: AAP PREP 2009

2009 PREP SA on CD-ROM

page 676

Question: 196

You have been treating a 2-year-old girl for 1 month with ferrous sulfate after her hemoglobin was 10.0 g/dL (100.0 g/L) and hematocrit was 29% (0.29). Today, her reticulocyte count is 4.2% (0.042), hemoglobin is 11.5 g/dL (115.0 g/L), and hematocrit is 33% (0.33). The nurse practitioner student with whom you are working asks if she can stop the iron supplement.

Of the following, the BEST reason for continuing iron therapy in this child is to

A. increase her appetite

B. prevent infection

C. prevent lead poisoning

D. reinforce vitamin supplementation

E. replenish iron stores

Page 677: AAP PREP 2009

2009 PREP SA on CD-ROM

page 677

Critique: 196 Preferred Response: E

Treatment of iron deficiency anemia involves iron replacement, usually with ferrous sulfate, for at least 2 months after the anemia has been corrected to replenish iron stores. Most children who have mild iron deficiency anemia are asymptomatic and are diagnosed following routine screening as part of health supervision visits, screening based on dietary risk factors, or a complete blood count obtained for evaluation of illness, particularly fever without a focus. Contrary to popular belief, there is no evidence that treatment of iron deficiency stimulates appetite. Treatment of iron deficiency also does not prevent lead poisoning, although lead poisoning and iron deficiency may coexist because iron deficiency may lead to pica. Neither iron supplementation nor therapy has been shown to prevent clinical infection. However, studies in children of low socioeconomic status have shown that iron supplementation may improve T-cell counts, delayed hypersensitivity skin reactions, and interleukin-2 production. Vitamin and mineral supplementation is not recommended for children who are eating normal diets and have no evidence for deficiency; such supplementation may increase the risk of ingesting toxic amounts of iron.

References:

Glader B. Iron deficiency anemia. In: Kliegman RM, Behrman RE, Jenson HB, Stanton BF, eds. Nelson Textbook of Pediatrics. 18th ed. Philadelphia, Pa: Elsevier Saunders; 2007:2014-2016

Kleinman RE. Nutrition and immunity. In: Pediatric Nutrition Handbook. 5th ed. Elk Grove Village, Ill: American Academy of Pediatrics; 2004:609-628

Richardson M. Microcytic anemia. Pediatr Rev. 2007;28:5-14. Available at: http://pedsinreview.aappublications.org/cgi/content/full/28/1/5

Page 678: AAP PREP 2009

2009 PREP SA on CD-ROM

page 678

Question: 197

You are performing screening sports participation examinations at the local high school. One of the students, a 16-year-old boy, reports that his father has hypertrophic cardiomyopathy but that none of his three older brothers has it. He also reports that he was seen by a cardiologist at age 10 years and was "fine." As you take his history, you find that he has never had shortness of breath, chest pain, exercise intolerance, dizziness, or fainting. He has always participated in sports and has excelled.

Of the following, the BEST plan of management for this boy is

A. chest radiography

B. electrocardiography

C. genetic testing for hypertrophic cardiomyopathy

D. referral to a cardiologist

E. unrestricted sports participation unless symptoms develop

Page 679: AAP PREP 2009

2009 PREP SA on CD-ROM

page 679

Critique: 197 Preferred Response: D

Hypertrophic cardiomyopathy (HCM) typically is inherited as an autosomal dominant trait due to a gene abnormality on chromosome 15. However, multiple genes have been found to be responsible for this disorder, and this fact contributes to the variability of severity. Interestingly, HCM may not be apparent on echocardiography until adolescence and even young adulthood in some patients, necessitating regular, repeat evaluation for asymptomatic children who have an affected first-degree relative. Alternatively, genetic testing may be an option for some, but genetic variability and the expense of testing may make this impractical in some cases. Accurate diagnosis of HCM in children and adolescents is essential because the condition can be the source of morbidity and mortality. Thoughtful counseling is an important component of care for the affected child; some sports and activities, such as competitive soccer, basketball, and football, are believed to be particularly high risk for the development of arrhythmia and even sudden death. In fact, HCM is the most common reason for a young person to experience sudden death during sports activities, occurring with nearly twice the frequency of coronary artery anomalies. The boy in the vignette has a family history of HCM (present in his father); each of his siblings has as great as a 50% chance of having inherited the HCM gene from the father. Current recommendations suggest evaluating such children regularly (perhaps as often as yearly) with detailed history, physical examination, and diagnostic testing that includes electrocardiography and echocardiography (Item C197). Therefore, the boy in the vignette and his siblings should be referred to a cardiologist for further evaluation. Chest radiography, a relatively nonspecific imaging study for cardiac structure, may yield normal results in patients who have HCM and would not be used either to confirm or rule out the diagnosis. Similarly, electrocardiography tracings can appear normal in a small percentage of patients who have HCM, making this study inappropriate for confirmation or ruling out of the diagnosis. Genetic testing for the HCM gene can be very effective if the specific gene mutation is well-identified in the index case of the family, but undertaking such testing for the boy in the vignette without testing his father first is not productive. Recognition of HCM and referral to a cardiologist for thorough preparticipation screening are critical.

References:

Berger S, Utech L, Hazinski MF. Sudden death in children and adolescents. Pediatr Clin North Am. 2004;51:1653-1677. Abstract available at: http://www.ncbi.nlm.nih.gov/pubmed/15561179

Corrado D, Basso C, Schiavon M, Thiene G. Screening for hypertrophic cardiomyopathy in young athletes. N Engl J Med. 1998;339:364-369. Abstract available at: http://www.ncbi.nlm.nih.gov/pubmed/9691102

Ho CY, Seidman CE. A contemporary approach to hypertrophic cardiomyopathy. Circulation. 2006;113:e858-e862. Available at: http://circ.ahajournals.org/cgi/content/full/113/24/e858

Maron BJ, Chaitman BR, Ackerman MJ, et al. Recommendations for physical activity and recreational sports participation for young patients with genetic cardiovascular diseases. Circulation. 2004;109:2807-2816. Available at: http://circ.ahajournals.org/cgi/content/full/109/22/2807

Maron BJ, Thompson PD, Ackerman MJ, et al. AHA scientific statements. Recommendations and considerations related to preparticipation screening for cardiovascular abnormalities in competitive athletes: 2007 update. A scientific statement from the American Heart Association Council on Nutrition,

Page 680: AAP PREP 2009

2009 PREP SA on CD-ROM

page 680

Physical Activity, and Metabolism: endorsed by the American College of Cardiology Foundation. Circulation. 2007;115:1643-1655. Available at: http://circ.ahajournals.org/cgi/content/full/115/12/1643

Page 681: AAP PREP 2009

2009 PREP SA on CD-ROM

page 681

Question: 198

A 4-year-old boy recently underwent hematopoietic stem cell transplantation for acute myelogenous leukemia. Fourteen days after his transplant, he experiences a seizure and confusion. He is receiving cyclosporine, prednisone, ganciclovir, fluconazole, cefotaxime, tobramycin, and omeprazole. Magnetic resonance imaging shows signal changes in bilateral occipital lobes.

Of the following, the MOST likely cause of the seizures is

A. cyclosporine

B. fluconazole

C. ganciclovir

D. prednisone

E. tobramycin

Page 682: AAP PREP 2009

2009 PREP SA on CD-ROM

page 682

Critique: 198 Preferred Response: A

Seizures in children are not commonly iatrogenic, but medications used in certain disciplines, most commonly oncology and psychiatry, may precipitate them. Chemotherapy for childhood cancers or organ transplantation often has adverse effects on the central nervous system, causing neuronal cell death, changes in white matter, or vascular lesions. Patients receiving cyclosporine or intrathecal methotrexate may develop diffuse or predominantly occipital white matter changes. The child described in the vignette has characteristic changes associated with cyclosporine. In addition to seizures, occipital blindness or strokelike weakness may occur. The changes seen on magnetic resonance imaging can be reversible. Fluconazole, ganciclovir, prednisone, and tobramycin are unlikely causes of seizures. Other medications that may cause or exacerbate seizures in children include isoniazid, theophylline, cocaine, psychostimulants, bupropion, insulin, and oral hypoglycemic agents.

References:

Abbott MB, Levin RH, Wu S. Medication potpourri. Pediatr Rev. 2006;27:283-288. Available at: http://pedsinreview.aappublications.org/cgi/content/full/27/8/283

Norman JK, Parke JT, Wilson DA, McNall-Knapp RY. Reversible posterior leukoencephalopathy syndrome in children undergoing induction therapy for acute lymphoblastic leukemia. Pediatr Blood Cancer. 2007;49:198-203. Abstract available at: http://www.ncbi.nlm.nih.gov/pubmed/16123992

Pound CM, Keene DL, Udjus K, Humphreys P, Johnston DL. Acute encephalopathy and cerebral vasospasm after multiagent chemotherapy including PEG-asparaginase and intrathecal cytarabine for the treatment of acute lymphoblastic leukemia. J Pediatr Hematol Oncol. 2007;29:183-186. Abstract available at: http://www.ncbi.nlm.nih.gov/pubmed/17356399

Page 683: AAP PREP 2009

2009 PREP SA on CD-ROM

page 683

Question: 199

During the health supervision visit of a 2-week-old infant, you note that his weight remains below his birthweight. The baby was delivered at term by a midwife in the parents’ home. There were no complications, and the parents have declined all perinatal testing. His mother says that he breastfeeds well, and her milk supply is good compared with that for her previous two children. Recently, though, the infant has been vomiting after feedings. On physical examination, he has total body jaundice, and his liver is enlarged to palpation. He is alert.

Of the following, the test that is MOST likely to aid in diagnosis is

A. abdominal ultrasonography

B. serum transaminases measurement

C. total and direct bilirubin measurement

D. urine organic acids measurement

E. urine reducing substances measurement

Page 684: AAP PREP 2009

2009 PREP SA on CD-ROM

page 684

Critique: 199 Preferred Response: E

The infant described in the vignette has a history and physical findings suggestive of galactosemia, an autosomal recessive inborn error of metabolism that results in the inability to oxidize galactose. Galactose dimerizes with glucose to form lactose, which is abundant in human and cow milk. Individuals who have classic galactosemia are deficient in galactose-1-phosphate uridyltransferase (GALT) activity, resulting in an elevated concentration of galactose, a reducing substance, in the urine. Although the presence of reducing substances in the urine is not specific for galactosemia, this finding, especially in the presence of poor weight gain, jaundice, and hepatomegaly, is strongly suggestive of galactosemia. Galactosemia can be detected in almost 100% of affected infants by newborn screening programs that test for it. The infant who has classic galactosemia typically presents within days of the initiation of milk feedings with vomiting and jaundice. If untreated, the child develops hepatomegaly and failure to thrive. Escherichia coli sepsis may occur, and approximately 10% of infants have cataracts at the time of diagnosis. Treatment is aimed at removing lactose from the diet, usually by feeding the baby a soy-based formula. The natural history of galactosemia is more complicated than once believed. Studies are showing that, despite early diagnosis and strict dietary management, affected older children and adults can have learning disabilities, neurologic disorders, verbal dyspraxia, and ovarian failure. For infants presenting with signs of galactosemia, abdominal ultrasonography may show hepatomegaly. Serum transaminases are elevated, as are total and direct bilirubin values. However, these abnormalities are not sufficiently informative to point to a diagnosis. Urine organic acids assessment may document elevated lactate, which is a nonspecific finding.

References:

Elsas LJ. Galactosemia. GeneReviews. 2007. Available at: http://www.geneclinics.org/servlet/access?db=geneclinics&site=gt&id=8888891&key=xAcWBcrjmZrVo&gry=&fcn=y&fw=4Vlc&filename=/profiles/galactosemia/index.html

Nyhan WL, Barshop BA, Ozand PT. Organic acidemias. In: Atlas of Metabolic Diseases. 2nd ed. London, England: Hodder Arnold; 2005:1-108

Page 685: AAP PREP 2009

2009 PREP SA on CD-ROM

page 685

Question: 200

A 16-year-old young man presents to the emergency department with a 12-hour history of pain in the scrotal area. He states that the pain started gradually and describes it as on the left side and moderate in intensity. He is sexually active and uses condoms. He has some burning pain with urination, but no penile discharge. He has felt warm but has not taken his temperature. He has had no vomiting or diarrhea. He has had no previous similar symptoms. On physical examination, the young man is afebrile and has normal findings on abdominal evaluation. He has moderate swelling of the left scrotum without erythema and marked tenderness that involves more of the posterolateral area. The testicular position is lower on the left than on the right. The left spermatic cord is very tender. Urinalysis shows more than 10 white blood cells per high-power field on a first-void specimen.

Of the following, the MOST likely diagnosis is

A. epididymitis

B. testicular torsion

C. testicular tumor

D. urinary tract infection

E. varicocele

Page 686: AAP PREP 2009

2009 PREP SA on CD-ROM

page 686

Critique: 200 Preferred Response: A

The differential diagnosis for a painful scrotal mass or swelling in an adolescent includes testicular torsion, torsion of the spermatic cord or appendix testis, epididymitis (Item C200), orchitis, trauma with hematoma, and incarcerated hernia. Other causes could be Henoch-Schönlein purpura or other trauma such as an infected piercing or insect sting. The features exhibited by the young man in the vignette that support the diagnosis of epididymitis include: sexual activity, gradual pain onset, localization of pain to the spermatic cord and posterolateral area of the scrotum, dysuria, and pyuria. With testicular torsion, a true surgical emergency, pain often is more abrupt in onset, and nausea and vomiting are frequent. The affected testis frequently is located more superiorly than the contralateral testis. Testicular tumors and varicoceles generally are not painful and are not accompanied by spermatic cord tenderness. A urinary tract infection is unlikely in an adolescent who has no congenital urinary tract abnormalities and would not produce a painful, swollen testis.

References:

Adelman WP, Joffe A. Scrotal disorders. In: Neinstein, LS, ed. Adolescent Health Care: A Practical Guide. 5th ed. Philadelphia, Pa: Lippincott Williams & Wilkins; 2008:401-410

Centers for Disease Control and Prevention. Update to CDC's sexually transmitted diseases treatment guidelines, 2006: fluoroquinolones no longer recommended for treatment of gonococcal infections. MMWR Morbid Mortal Wkly Rep. 2007;56:332-336. Available at: http://www.cdc.gov/mmwr/preview/mmwrhtml/mm5614a3.htm

Workowski AK, Berman SM, Centers for Disease Control and Prevention. Sexually transmitted diseases treatment guidelines, 2006. MMWR Recomm Rep. 2006;55(RR11):1-94. Available at: http://www.cdc.gov/mmwr/preview/mmwrhtml/rr5511a1.htm

Page 687: AAP PREP 2009

2009 PREP SA on CD-ROM

page 687

Critique: 200

Epididymitis is characterized by erythema and swelling of the scrotum. (Courtesy of J. Zenel)

Page 688: AAP PREP 2009

2009 PREP SA on CD-ROM

page 688

Question: 201

A 1-year-old boy who is intubated for severe asthma is demonstrating significant acute respiratory and cardiac deterioration, as evidenced by tachycardia, tachypnea, decreased blood pressure, and oxygen saturation of 75%. During your examination, you note a marked shift of the trachea to the left and markedly decreased aeration on the right side.

Of the following, the MOST likely cause of this boy’s sudden respiratory deterioration is

A. endotracheal tube obstruction

B. exacerbation of the underlying asthma

C. intubation of the right mainstem bronchus

D. tension pneumothorax

E. ventilator-associated pneumonia

Page 689: AAP PREP 2009

2009 PREP SA on CD-ROM

page 689

Critique: 201 Preferred Response: D

The tachypnea, tachycardia, decreased breath sounds on the involved side, and decreased oxygenation described for the boy in the vignette are strongly suggestive of pneumothorax, the accumulation of air within the extrapleural space. Pneumothoraces typically are classified as simple, communicating, or tension. Simple pneumothoraces are the most common and have no communication with the atmosphere. They usually are a result of a spontaneous pneumothorax with or without underlying lung pathology, blunt chest trauma, or mechanical ventilation. A communicating pneumothorax normally is caused by open chest trauma, although rapid accumulation of pleural air is possible in the spontaneously breathing patient due to entrainment of air with inspiration. Tension pneumothoraces represent a medical emergency and develop when continued accumulation of air produces a mediastinal shift to the contralateral side and subsequent compression and decreased function of the lung and vascular structures. The tracheal shift and hypotension reported for the boy in the vignette indicate that he has a tension pneumothorax. Rapid diagnosis and treatment of symptomatic pneumothoraces is critical. Chest radiography (Item C201) is the gold standard for diagnosis, but clinical conditions may necessitate rapid treatment with thoracentesis or thoracic tube placement prior to imaging. Development of a pneumothorax along with other potential ventilator complications always should be considered promptly when a mechanically ventilated patient deteriorates. Endotracheal tube obstruction normally presents with bilateral decreased aeration, difficulty in passing a suction catheter, and lack of tracheal shift. A right bronchus intubation is characterized by decreased aeration on the left side and oxygen desaturation, but cardiovascular compromise is less common than with a tension pneumothorax. Progression of underlying disease processes such as asthma or development of ventilator-associated pneumonia is more gradual than described in the vignette.

References:

Chase MA, Wheeler DS. Disorders of the pediatric chest. In: Wheeler DS, Wong HR, Shanley T, eds. Pediatric Critical Care Medicine: Basic Science and Clinical Evidence. New York, NY: Springer-Verlag; 2007:361-375

Winnie GB. Pnuemothorax. In: Kliegman RM, Behrman RE, Jenson HB, Stanton BF, eds. Nelson Textbook of Pediatrics. 18th ed. Philadelphia, Pa: Saunders Elsevier; 2007:1835-1836

Page 690: AAP PREP 2009

2009 PREP SA on CD-ROM

page 690

Critique: 201

Chest radiography demonstrating tension pneumothorax: There is collapse of the right lung, flattening of the right hemidiaphragm, and shift of the heart and mediastinum to the left. (Courtesy of D. Mulvihill)

Page 691: AAP PREP 2009

2009 PREP SA on CD-ROM

page 691

Question: 202

During the health supervision visit of a 9}-year-old girl, you note that her height is just above the 97th percentile for age and her weight is at the 85th percentile. Her mother is 5 feet 5 inches tall and father is 5 feet 10 inches. Her parents ask if she will be very tall when she has finished growing.

Of the following, the MOST important element of the physical examination to help answer this question is

A. arm span measurement

B. body mass index

C. eye examination

D. Sexual Maturity Rating

E. upper-to-lower segment ratio

Page 692: AAP PREP 2009

2009 PREP SA on CD-ROM

page 692

Critique: 202 Preferred Response: D

Most children of tall stature either have tall parents or have matured early and are tall in late childhood, although adult height after early puberty will be well within the mid-parental range. Therefore, if the girl described in the vignette is in puberty, an explanation for her tall stature at age 9 years will be apparent, and a bone age radiograph can be used to predict her ultimate adult height. Accordingly, it is most important to determine her Sexual Maturity Rating. Lack of puberty suggests that she will, indeed, be tall. Further studies might be indicated to look for other reasons for her tall stature, particularly if she has had growth acceleration. Rarely, children of tall stature have Marfan syndrome, which is characterized by long arm span and a decreased body mass index. Increased body mass index and obesity may be associated with tall stature and early puberty. If there is worry about growth hormone excess and a pituitary tumor, eye examination and careful assessment of visual fields should be undertaken, but growth hormone-producing pituitary tumors often are too small to produce visual symptoms. They are detected best by physical examination showing signs of growth hormone excess: widened spaces between the teeth, large hands and feet, and increased soft-tissue thickening. Laboratory measurement of insulin-like growth factor I and magnetic resonance imaging of the pituitary may be helpful. Measurement of the upper-to-lower segment ratio may be useful in the clinical diagnosis of Marfan syndrome because long legs compared with the trunk result in a lower ratio than expected for age. Other congenital causes of tall stature, such as Sotos syndrome, are associated with specific physical and behavioral findings, including large head circumference and decreased intellectual functioning. Diagnostic genetic testing is available for this disorder.

References:

Boom JA. Normal pediatric growth. UpToDate Online 15.3. 2008.Available for subscription at: http://www.uptodateonline.com/utd/content/topic.do?topicKey=gen_pedi/13648

Cohen P, Shim M. Hyperpituitarism, tall stature, and overgrowth syndromes. In: Kliegman RM, Behrman RE, Jenson HB, Stanton BF, eds. Nelson Textbook of Pediatrics. 18th ed. Philadelphia, Pa: Saunders Elsevier; 2007:2303-2307

Richmond EJ, Rogol AD. The child with abnormally rapid growth. UpToDate Online 15.3. 2008. Available for subscription at: http://www.uptodateonline.com/utd/content/topic.do?topicKey=pediendo/7226

Page 693: AAP PREP 2009

2009 PREP SA on CD-ROM

page 693

Question: 203

A mother brings in her 10-year-old daughter and 8-year-old son because they are fighting constantly. The son says he hates having a sister and complains that his parents favor her and give her everything she wants. The daughter says that her brother is spoiled and always touches her stuff. The mother is frustrated by their constant fighting and asks for assistance in handling the children.

Of the following, the BEST initial guidance for the mother is to

A. explain that this is typical of siblings and she should ignore the behavior

B. give her a list of books on parenting

C. refer her to a behavioral therapist to improve her parenting skills

D. suggest she use behavioral modification techniques to diminish fighting

E. tell her to return in 6 months if the siblings are still fighting

Page 694: AAP PREP 2009

2009 PREP SA on CD-ROM

page 694

Critique: 203 Preferred Response: D

Although sibling rivalry is common, the actions described for the children in the vignette indicate the need for the mother to employ behavioral techniques to decrease their fighting in the home. Children initially should be allowed to resolve their differences, but parents need to intervene if one sibling is being abused either physically or verbally. When the fighting is heading toward a dangerous situation, the parents need to describe the actions of the siblings, establish limits, and separate the siblings. The purpose of the behavioral intervention is to open lines of communication so the siblings can begin to work out their differences. If the behavior continues to be challenging and not responsive to such initial parental interventions, referral to a therapist may be considered. Telling the mother either to ignore the sibling rivalry or wait 6 months will not aid in changing the children's behaviors. A book on sibling rivalry may supplement an initial therapeutic plan, but the family should be provided direct guidance on implementing a behavior intervention strategy. A common time for the emergence of sibling rivalry is after a recent birth. It is common for a child to be upset and have some regression in behavioral skills following the birth of a new sibling. Parents need to intervene more with younger children to prevent any injuries.

References:

Faber A, Mazlish E. When the kids fight: how to step in so we can step out. In: Siblings Without Rivalry: How to Help Your Children Live Together So You Can Live Too. New York, NY: Quill; 2002:146-177

Needlman R. Sibling rivalry. In: Parker S, Zuckerman B, Augustyn M, eds. Developmental and Behavioral Pediatrics: A Handbook for Primary Care. 2nd ed. Philadelphia, Pa: Lippincott Williams & Wilkins; 2005:412-415

Page 695: AAP PREP 2009

2009 PREP SA on CD-ROM

page 695

Question: 204

You are evaluating an 8-month-old boy who is having multiple "coughing spells." During these spells, the boy sometimes turns blue and even vomits. You inform the mother that you are going to prescribe an antimicrobial agent. She wants to know why you are giving her infant an antimicrobial agent when he needs something for the cough.

Of the following, the BEST reason to prescribe an antimicrobial agent for this boy is that treatment will decrease the

A. chance of death

B. cough

C. hypoxic episodes

D. infectivity

E. posttussive vomiting

Page 696: AAP PREP 2009

2009 PREP SA on CD-ROM

page 696

Critique: 204 Preferred Response: D

The clinical findings for the child described in the vignette indicate the diagnosis of pertussis (whooping cough). The clinical course of pertussis can be ameliorated with antimicrobial agents only in patients whose disease is limited to mild upper respiratory tract symptoms (catarrhal stage). Once patients develop the paroxysmal cough (paroxysmal stage), the use of antimicrobial agents does not shorten the disease course or affect the cough, posttussive vomiting, hypoxic episodes, or chance of death. However, antimicrobial therapy is used for patients in the paroxysmal stage to decrease the spread of Bordetella pertussis to others. Erythromycin products have been the treatment of choice for pertussis until recently. Data now demonstrate that azithromycin and clarithromycin have microbiologic effectiveness similar to erythromycin in individuals 6 months of age or older. The advantages of the newer macrolides over erythromycin for the treatment of pertussis are: 1) higher tissue concentrations, 2) longer half-life, 3) fewer daily doses required (one or two per day), and 4) short treatment courses (5 to 7 days). Azithromycin and clarithromycin also are approved as first-line agents for prophylaxis of exposed individuals. In most situations of treatment or prophylaxis, patients tolerate and are more compliant with azithromycin or clarithromycin than erythromycin products. Erythromycin should not be used in infants younger than 1 month of age due to the agent's association with infantile pyloric stenosis. There are no safety data for the use of clarithromycin in neonates, making azithromycin the macrolide of choice for this age group. Trimethoprim-sulfamethoxazole is an alternative agent that can be used for children older than 2 months of age who cannot take macrolides for pertussis treatment or prophylaxis.

References:

American Academy of Pediatrics. Pertussis (whooping cough). In: Pickering LK, Baker CJ, Long SS, McMillan JA, eds. Red Book: 2006 Report of the Committee on Infectious Diseases. 27th ed. Elk Grove Village, Ill: American Academy of Pediatrics; 2006:498-520

Tiwari T, Murphy RV, Moran J. Recommended antimicrobial agents for the treatment and postexposure prophylaxis of pertussis. 2005 CDC guidelines. MMWR Recomm Rep. 2005;54(RR14):1-16. Available at: http://www.cdc.gov/mmwr/preview/mmwrhtml/rr5414a1.htm

Waseem M, Kin LL. Index of suspicion: case 6. Pediatr Rev. 2005;26:23-33. Available at: http://pedsinreview.aappublications.org/cgi/content/full/26/1/23

Page 697: AAP PREP 2009

2009 PREP SA on CD-ROM

page 697

Question: 205

You are evaluating a 17-month-old previously healthy girl who presents with an 8-month history of recurrent cellulitis and abscesses on her lower right abdomen. Eight months ago, she developed a "pimple" on her abdomen that rapidly enlarged to the size of a golf ball and became very red, hard, and tender over 2 days. The lesion spontaneously drained a purulent material and resolved. Over the last 8 months, the girl has had nine similar episodes. She was seen on several occasions in an urgent care center and each time was placed on a course of cephalexin, which resulted in no improvement until the lesion drained spontaneously. The patient has no fever with the episodes. Physical examination shows a 2x3-cm erythematous, indurated, very tender, fluctuant lesion on the patient’s right flank.

Of the following, the MOST likely organism causing this patient’s recurrent infections is

A. methicillin-resistant Staphylococcus aureus

B. methicillin-sensitive Staphylococcus aureus

C. Staphylococcus epidermidis

D. Streptococcus pneumoniae

E. Streptococcus pyogenes

Page 698: AAP PREP 2009

2009 PREP SA on CD-ROM

page 698

Critique: 205 Preferred Response: A

The first-generation cephalosporins (eg, cephalexin) have a spectrum of activity focused primarily on gram-positive bacteria. They have been used widely as alternatives to penicillin for the treatment of staphylococcal and nonenterococcal streptococcal infections. These agents are effective in the treatment of infections caused by susceptible strains of groups A, B, C, and G streptococci; S pneumoniae; viridans streptococci; methicillin-susceptible Staphylococcus aureus (MSSA); and some strains of S epidermidis. Historically, first-generation cephalosporins have been the mainstay of therapy for skin and soft-tissue infections. The gram-negative spectrum of the first-generation cephalosporins is much more limited. These drugs are effective against susceptible strains of Neisseria gonorrhoeae, Escherichia coli, Klebsiella sp, and Proteus mirabilis. However, they have poor activity against penicillin-resistant pneumococci, Haemophilus influenzae, and Moraxella catarrhalis and are not recommended in the treatment of sinusitis, otitis media, and lower respiratory tract infections. All isolates of methicillin-resistant S aureus (MRSA) are resistant to the cephalosporin class of antibiotics. Antibiotics used to treat MRSA include clindamycin, trimethoprim-sulfamethoxazole, and linezolid. The patient described in the vignette has had episodes of recurrent cellulitis and abscesses that improve only after spontaneous drainage, despite receiving a first-generation cephalosporin for treatment. The most common organisms associated with skin and soft-tissue infections are S aureus and S pyogenes (group A streptococci). The organism most likely to be the cause of this patient's infection is MRSA, which is associated with recurrent episodes of cellulitis and abscesses. MRSA is resistant to first-generation cephalosporins such as cephalexin. MSSA and S pyogenes both are susceptible to the first-generation cephalosporins, and infection caused by these agents should improve with treatment. S pneumoniae is a cause of preseptal cellulitis and rarely lymphadenitis, but it is not associated with cellulitis and abscesses. S epidermidis usually does not cause cellulitis or soft-tissue infections.

References:

Andes DR, Craig WA. Cephalosporins. In: Mandell GL, Bennett JE, Dolin R, eds. Mandell, Douglas, and Bennett's Principles and Practice of Infectious Diseases. 6th ed. Philadelphia, Pa: Elsevier Churchill Livingstone; 2005:294-310

Jantausch BA. Peripheral brain: cephalosporins. Pediatr Rev. 2003;24:128-136. Available at: http://pedsinreview.aappublications.org/cgi/content/full/24/4/128

Page 699: AAP PREP 2009

2009 PREP SA on CD-ROM

page 699

Question: 206

A mother brings in her 4-year-old daughter because of decreased energy following a 3-day history of diarrhea without vomiting. On physical examination, the girl’s temperature is 100.2°F (37.9°C), heart rate is 130 beats/min, respiratory rate is 18 breaths/min, and blood pressure is 122/84 mm Hg. She has pale conjunctivae, a hyperdynamic precordium, and mild pretibial edema. Laboratory evaluation reveals: · Sodium, 133.0 mEq/L (133.0 mmol/L) · Potassium, 5.2 mEq/L (5.2 mmol/L) · Chloride, 100.0 mEq/L (100.0 mmol/L) · Bicarbonate, 16.0 mEq/L (16.0 mmol/L)· Albumin, 2.5 g/dL (25.0 g/L) · Blood urea nitrogen, 40.0 mg/dL (14.3 mmol/L)· Creatinine, 1.4 mg/dL (123.8 mcmol/L)· Hemoglobin, 6.1 g/dL (610.0 g/L) · White blood cell count, 21.5x103/mcL (21.5x109/L)· Platelet count, 90.0x103/mcL (90.0x109/L)

Of the following, the MOST likely additional laboratory abnormality is

A. a low reticulocyte count

B. elevated lactate dehydrogenase concentration

C. elevated parathyroid hormone concentration

D. positive Coombs test

E. prolonged prothrombin time

Page 700: AAP PREP 2009

2009 PREP SA on CD-ROM

page 700

Critique: 206 Preferred Response: B

The diarrheal prodrome, tachycardia, hyperdynamic precordium, anemia, thrombocytopenia, and elevated creatinine value described for the girl in the vignette suggests the diagnosis of hemolytic-uremic syndrome (HUS). HUS is a microangiopathic process often linked to Shiga toxin-producing enterohemorrhagic Escherichia coli, which results in the triad of microangiopathic hemolytic anemia, thrombocytopenia, and acute renal failure (ARF). HUS is categorized as a thrombotic microangiopathy that has a predilection for arterioles and capillaries, especially those involving the renal circulation. When HUS is preceded by diarrhea (90% of cases), it is designated D(+). The diarrheal prodrome may be bloody or nonbloody and is associated most commonly with enterohemorrhagic E coli, with serotype O157:H7 being the most common in the United States. This Shiga toxin-producing E coli often is acquired by ingesting undercooked hamburger, unpasteurized apple cider, or other contaminated foods. Patients who have this organism in their gastrointestinal tracts can develop diarrhea (bloody or nonbloody) alone or HUS. Interestingly, only 5% to 8% of children who have hemorrhagic colitis from a Shiga toxin-producing enterohemorrhagic E coli actually develop HUS, which suggests that virulence and host factors likely play a role in who develops HUS. Those who exhibit HUS develop endothelial cell injury and small vessel thromboses involving the glomerular capillary, arteriolar, and interlobular arteries. The resultant ischemic injury affects the glomeruli and tubules, leading to ARF. The endothelial injury results in von Willebrand factor release, platelet adhesion, and microthrombi formation. This thrombotic microangiopathic process leads to a hemolytic anemia and thromobocytopenia. Children who have D(+) HUS may have symptoms mimicking those of ulcerative colitis or an acute abdomen. To detect HUS in its early stages, it is important to monitor hemoglobin, platelet count, and serum creatinine as well as urinalysis in a child who has severe colitis. Because oliguria due to dehydration is common in children who have severe diarrhea, the diagnosis of ARF due to HUS can be delayed. When HUS is suspected, a peripheral blood smear should be obtained. A smear that shows evidence of fragmented red blood cells (schistocytes) (Item C206), low platelet number, and reticulocytosis is diagnostic. Of note, the reticulocyte count may not be markedly elevated in severe renal dysfunction due to impaired erythropoietin production. Leukocytosis is common, with higher white blood cell counts associated with a worse renal prognosis. In addition, due to the severity of the hemolysis, other laboratory abnormalities include elevated lactate dehydrogenase and low haptoglobin values. The Coombs test is negative because this is not an antibody-mediated process. The prothrombin and partial thromboplastin times are normal because HUS is not a consumptive coagulopathy. The parathyroid hormone concentration would not be expected to be elevated in the setting of acute renal failure; this is a manifestation of renal osteodystrophy seen in severe forms of chronic kidney disease. Treatment of HUS is largely supportive, including intravenous volume expansion in those who do not have oliguria and dialysis in those who have oligoanuria. Of note, empiric antibiotics should be avoided in children who have bloody diarrhea because if the diarrhea is due to enterohemorrhagic E coli, antibiotic therapy is associated with an increased risk of developing HUS.

References:

Ake JA, Jelacic S, Ciol MA, et al. Relative nephroprotection during Escherichia coli O157:H7 infections: association with intravenous volume expansion. Pediatrics. 2005;115:e673-e680. Available at: http://pediatrics.aappublications.org/cgi/content/full/115/6/e673

Oakes RS, Siegler RL, McReynolds MA, Pysher T, Pavia AT. Predictors of fatality in postdiarrheal hemolytic uremic syndrome. Pediatrics. 2006;117:1656-1662. Available at:

Page 701: AAP PREP 2009

2009 PREP SA on CD-ROM

page 701

http://pediatrics.aappublications.org/cgi/content/full/117/5/1656

Mahan JD. Hemolytic uremic syndrome. In: Kher KK, Schnaper HW, Makker SP, eds. Clinical Pediatric Nephrology. 2nd ed. London, England: Informa Healthcare; 2007:235-244

Wong CS, Jelacic S, Habeeb RL, Watkins SL, Tarr PI. The risk of the hemolytic-uremic syndrome after antibiotic treatment of Escherichia coli O157:H7 infections. N Engl J Med. 2000;342:1930-1936. Available at: http://content.nejm.org/cgi/content/full/342/26/1930

Page 702: AAP PREP 2009

2009 PREP SA on CD-ROM

page 702

Critique: 206

Schistocytes (arrows) are observed on the peripheral blood smear of patients who have hemolytic-uremic syndrome. (Courtesy of S. Dabbagh)

Page 703: AAP PREP 2009

2009 PREP SA on CD-ROM

page 703

Question: 207

A 16-year-old girl who has moderate persistent asthma presents to the emergency department with coughing, wheezing, and increasing dyspnea. She states that she was feeling fine until she was exposed to cologne that one of her classmates was wearing. An ambulance was called after her symptoms did not improve following administration of two puffs of her beta2 agonist inhaler. On physical examination, the teenager has a respiratory rate of 30 breaths/min, heart rate of 90 beats/min, and pulse oximetry of 98% on room air. She has difficulty completing a sentence and points to her neck, saying it is "hard to get air in." Her lungs are clear to auscultation, and rhinolaryngoscopy demonstrates adduction of one of the vocal cords during inspiration. Pulmonary function testing shows a blunted inspiratory loop (Item Q207).

Of the following, the MOST likely cause for this patient’s symptoms is

A. allergic rhinitis

B. asthma exacerbation

C. habit cough

D. sinusitis

E. vocal cord dysfunction

Page 704: AAP PREP 2009

2009 PREP SA on CD-ROM

page 704

Question: 207

Blunted inspiratory flow curve in pink (arrow), as exhibited by the patient in the vignette. A normal curve is shown in blue. (Courtesy of K. Waibel)

Page 705: AAP PREP 2009

2009 PREP SA on CD-ROM

page 705

Critique: 207 Preferred Response: E

The teenager described in the vignette has signs and symptoms consistent with vocal cord dysfunction (VCD), a condition that can mimic or coexist with asthma. In contrast to an asthma exacerbation, the key features of VCD exhibited by this girl include a normal room air pulse oximetry reading, failure to improve with her beta2 agonist inhaler, clear lungs, and difficulty with inspiration instead of expiration. A blunted inspiratory loop on spirometry also is supportive of VCD, although affected patients usually have normal spirometry readings when not experiencing symptoms. Triggers for VCD can include viral upper respiratory tract infections, chemicals, fumes, pollution, emotional changes, laughing, exercise, gastroesophageal reflux (GER), and cold air. GER can cause cough and be a trigger for asthma. It may worsen during exercise, eating, or when supine. Although GER is a cause of chronic cough, the patient in the vignette does not admit to GER symptoms, making this diagnosis unlikely. Postnasal drip syndrome, now termed upper airway cough syndrome, can result in coughing due to allergic rhinitis, nonallergic rhinitis, or sinusitis. The lack of nasal congestion, rhinorrhea, or postnasal drip for this girl makes this an unlikely cause of her acute symptoms. Psychogenic cough, also called habit cough syndrome, is a well-described chronic cough that may begin after a viral upper respiratory tract infection. The cough usually is nonproductive and does not occur during sleep. Teenagers who have asthma may use coughing as a method to avoid school (factitious or malingering), but the girl in the vignette is not having symptoms during a specific class or time of day.

References:

Liu AH, Covar RA, Spahn JD, Leung DYM. Childhood asthma. In: Kleigman RM, Behrman RE, Jenson HB, Stanton BF, eds. Nelson Textbook of Pediatrics. 18th ed. Philadelphia, Pa: Saunders Elsevier; 2007:953-969

Weinberger M, Abu-Hasan M. Pseudo-asthma: when cough, wheezing, and dyspnea are not asthma. Pediatrics. 2007;120:855-864. Available at: http://pediatrics.aappublications.org/cgi/content/full/120/4/855

Page 706: AAP PREP 2009

2009 PREP SA on CD-ROM

page 706

Question: 208

A 7-year-old boy is brought to the emergency department because of altered mental status. His parents report that he was well when he came home from school today, but when he came in the house for dinner after playing outside with his friends, he complained of abdominal pain and had an episode of nonbilious and nonbloody emesis. Over the next 30 minutes, he became increasingly lethargic until his parents could not arouse him. They called emergency medical services, and he was transported to the emergency department by ambulance. On physical examination, he is unresponsive and drooling, his temperature is 98.8°F (37.1°C), heart rate is 50 beats/min, respiratory rate is 36 breaths/min, blood pressure is 100/60 mm Hg, and oxygen saturation is 82% on room air. His pupils are mid-size and sluggishly reactive, and his breath sounds are coarse bilaterally, with increased work of breathing. You suspect a toxin exposure.

Of the following, the MOST appropriate treatment of this patient is

A. atropine

B. N-acetylcysteine

C. naloxone

D. octreotide

E. physostigmine

Page 707: AAP PREP 2009

2009 PREP SA on CD-ROM

page 707

Critique: 208 Preferred Response: A

The patient described in the vignette is exhibiting the classic symptoms of cholinergic poisoning. These symptoms can be remembered using the mnemonic "SLUDGE" (salivation, lacrimation, urination, diarrhea, gastric emesis) and are due to irreversible inhibition of acetylcholinesterase and excess acetylcholine at the neuromuscular junction. The resulting overstimulation of cholinergic receptors causes the muscarinic symptoms noted previously. Nicotinic symptoms, which include muscle twitching, weakness, and paralysis, also may result. Organophosphate exposure can occur though ingestion as well as dermal absorption and inhalation. These compounds are found commonly in the home environment as components of lawn and garden care products, scabicides, and interior insecticides. The treatment of cholinergic poisoning involves patient stabilization, decontamination, and administration of antidotes. Significantly symptomatic patients may require intubation for airway protection and pulmonary toilet as well as assisted ventilation with 100% oxygen for respiratory muscle weakness and hypoxia. Systemic decontamination with activated charcoal as well as dermal decontamination should be performed. Dermal decontamination may be facilitated by cleansing the skin with a dilute bleach solution. Contaminated clothes should be discarded because laundering may not remove the toxin. The mainstay of stabilization and treatment for cholinergic poisonings is the administration of atropine. Atropine competes with acetylcholine at the cholinergic receptors and decreases the muscarinic cholinergic effects. The doses of atropine used in this setting are higher than those used for symptomatic bradycardia from other causes. An initial atropine dose of 0.05 mg/kg should be administered and doubled every 3 to 5 minutes until the pulmonary muscarinic symptoms (bronchorrhea, bronchospasm) are controlled. Nicotinic neuromuscular symptoms are treated by adding pralidoxime, a cholinesterase reactivating agent, to atropine therapy. N-acetylcysteine is used to treat acetaminophen poisoning, which causes few, if any, acute symptoms. Naloxone is an opiate antagonist used to treat symptomatic opiate overdoses, which are characterized by miosis, bradycardia, and respiratory depression without bronchorrhea. Octreotide is a somatostatin analog that inhibits insulin release and is indicated in the treatment of sulfonylurea overdoses, which cause profound hypoglycemia unresponsive to dextrose administration. Physostigmine is a cholinergic agent that may be used in significantly symptomatic anticholinergic poisonings. Because the patient's symptoms are not consistent with overdoses of any of these drugs, treatment with these agents is not indicated.

References:

Bird S. Organophosphate and carbamate toxicity. UpToDate Online 15.3. 2008. Available for subscription at:http://www.utdol.com/utd/content/topic.do?topicKey=ad_tox/9425&selectedTitle=1~150&source=search_result

Karr CJ, Solomon GM, Brock-Utne AC. Health effects of common home, lawn, and garden pesticides. Pediatr Clin North Am. 2007;54:63-80. Abstract available at: http://www.ncbi.nlm.nih.gov/pubmed/17306684

Peter JV, Moran JL, Graham PL. Advances in the management of organophosphate poisoning. Expert Opin Pharmacother. 2007;8:1451-1464. Abstract available at: http://www.ncbi.nlm.nih.gov/pubmed/17661728

Page 708: AAP PREP 2009

2009 PREP SA on CD-ROM

page 708

Question: 209

You are evaluating a 2-day-old term infant because of abdominal distention. He fed normally the first day after birth, but has had progressively increasing vomiting, which now is bilious. Physical examination demonstrates upslanted palpebral fissures, a prominent tongue, and mild hypotonia. Upon passage of a nasogastric tube, you aspirate 80 mL of green-yellow material from his stomach. Abdominal radiographs, including a left lateral decubitus film, reveal dilated loops of bowel and air-fluid levels but no evidence of pneumatosis (Item Q209).

Of the following, the condition that BEST explains this baby’s clinical findings is

A. duodenal atresia

B. Hirschsprung disease

C. meconium ileus

D. necrotizing enterocolitis

E. neonatal intussusception

Page 709: AAP PREP 2009

2009 PREP SA on CD-ROM

page 709

Question: 209

(Courtesy of D. Mulvihill)

Page 710: AAP PREP 2009

2009 PREP SA on CD-ROM

page 710

Critique: 209 Preferred Response: B

The infant described in the vignette has clinical features of Down syndrome (Item C209A). Infants who have Down syndrome are at risk for a number of gastrointestinal malformations, most notably duodenal atresia and Hirschsprung disease. The air in the distal small bowel apparent in the radiograph obtained for the infant in the vignette excludes duodenal atresia and annular pancreas. The lack of pneumatosis on the radiograph for this term infant makes necrotizing enterocolitis unlikely. Neonatal intussusception is extremely rare, and meconium ileus is associated with cystic fibrosis, not Down syndrome. Hirschsprung disease is characterized by congenital absence of a portion of the enteric nervous system (aganglionosis). The aganglionic segment of bowel typically begins at the anal verge and extends proximally. Disease limited to the rectosigmoid colon ("short segment" Hirschsprung) accounts for 80% to 90% of cases. The remainder of cases can involve a larger portion of the distal colon ("long segment Hirschsprung"), the entire colon ("total colonic Hirschsprung"), or the colon and small bowel. The aganglionic segment of bowel is unable to contract, leading to either severe constipation or a functional bowel obstruction. It can be difficult to distinguish between functional constipation and Hirschsprung disease in childhood. Some distinguishing features are shown in Item C209B. In general, patients who have Hirschsprung disease have a history of delayed passage of meconium; 95% of affected infants fail to pass meconium in the first day after birth. Stools of infants and toddlers who have Hirschsprung disease often are very thin. Because of the hypertensive anal sphincter and aperistaltic distal colon, affected children almost never have encopresis. On rectal examination, an infant or child who has short segment Hirschsprung may have an "explosion of stool" after the examiner's finger is removed. If Hirschsprung disease is suspected strongly, the patient should undergo further diagnostic testing. Anorectal manometry demonstrates a hypertensive anal sphincter that fails to relax. Barium enema may demonstrate a narrowed rectosigmoid (aganglionic segment) (Item C209C) and a dilated proximal colon, with a "transition zone" (area of caliber change between the normal and aganglionic segment). The definitive test for Hirschsprung disease is rectal biopsy, which demonstrates absent ganglion cells in the submucosa and muscularis propria of the rectum. If Hirschsprung disease is identified, surgical resection of the aganglionic colon and anastomosis of the normal colon to the anorectal canal is the treatment of choice.

References:

de Lorijn F, Kremer LC, Reitsma JB, Benninga MA. Diagnostic tests in Hirschsprung disease: a systematic review. J Pediatr Gastroenterol Nutr. 2006;42:496-505. Abstract available at: http://www.ncbi.nlm.nih.gov/pubmed/16707970

Imseis E, Gariepy CE. Hirschsprung disease. In: Walker WA, Goulet O, Kleinman RE, Sherman PM, Shneider BL, Sanderson IR, eds. Pediatric Gastrointestinal Disease. 4th ed. Hamilton, Ontario, Canada: BC Decker; 2004:1031-1043

Page 711: AAP PREP 2009

2009 PREP SA on CD-ROM

page 711

Critique: 209

Facial features of Down syndrome in infants include upslanted palpebral fissures, a flat face, and a broad nasal root. (Courtesy of M. Rimsza)

Page 712: AAP PREP 2009

2009 PREP SA on CD-ROM

page 712

Critique: 209

Page 713: AAP PREP 2009

2009 PREP SA on CD-ROM

page 713

Critique: 209

Spot radiograph from a barium enema series in a patient who has Hirschsprung disease showing a contracted rectum (representing the aganglionic segment) with a short transition zone to a normal dilated proximal rectum. (Courtesy of D. Mulvihill)

Page 714: AAP PREP 2009

2009 PREP SA on CD-ROM

page 714

Question: 210

You are called to the neonatal intensive care unit to examine a newborn who has abdominal distention and respiratory distress. She was born at 38 weeks’ gestation and weighs 4 kg. Apgar scores were 3 and 6 at 1 and 5 minutes, respectively. She required tracheal intubation and assisted ventilation. On physical examination, she has a large, distended, and tense abdomen without bowel sounds. The abdominal wall is not erythematous, and there is no clearly palpable mass. She does not display other evidence of body wall or scalp edema. The breath sounds are coarse and equal bilaterally. There is no heart murmur. Radiograph of the chest appears normal, but abdominal radiography shows background granular density, paucity of intraluminal bowel gas, and a calcified mass in the left lower quadrant (Item Q210).

Of the following, the BEST explanation for this infant’s abdominal findings is

A. congenital lymphangioma

B. erythroblastosis fetalis

C. meconium peritonitis

D. ovarian cyst

E. urinary ascites

Page 715: AAP PREP 2009

2009 PREP SA on CD-ROM

page 715

Question: 210

(Courtesy of B. Carter)

Page 716: AAP PREP 2009

2009 PREP SA on CD-ROM

page 716

Critique: 210 Preferred Response: C

The respiratory distress described for the infant in the vignette may be related to diaphragmatic impingement from abdominal distention. Evaluation of the cause of abdominal distention includes a plain radiograph that, in this case, has findings consistent with ascites and a calcified mass (Item C210A). Subsequent abdominal ultrasonography may prove to be valuable diagnostically. The mass is a meconium pseudocyst, indicative of a bowel rupture with contained (walled-off) meconium that has become calcified. This condition, seen in meconium peritonitis, usually is associated with meconium ileus and is a result of cystic fibrosis (CF). An autosomal recessive disease, CF is believed to affect 1 in 2,000 to 4,000 live births of white children. It occurs much less frequently in African Americans (estimated at 1 in 17,000 live births) and is rare in infants of Asian descent (estimated at 1 in 90,000 live births). The disease is related to altered epithelial cell ion transport and is associated with exocrine pancreatic insufficiency and pulmonary decompensation in later life. More than 1,000 mutations (mapped to the 7th chromosome) have been identified, but the most common defect is in a chloride channel transmembrane transport protein regulated by the delta-F508 mutation, which accounts for more than 70% of all CF cases. Problems that may present in the neonatal period include: 1. Meconium ileus, a thick meconium obstruction of the distal ileum characteristically presenting clinically as a small bowel obstruction. This condition occurs in approximately 17% of infants who have CF. A history may indicate polyhydramnios, a prenatal ultrasonographically observed bowel dilation, or delayed postnatal passage of stool. Physical examination may reveal abdominal distention and bile-stained emesis. A plain abdominal radiograph may show stacked loops of variably dilated bowel, soap-suds bubbly-like appearance of meconium stool (most often in the right lower quadrant), and a ground-glass character of the distal bowel with a relative paucity of distal bowel gas. A contrast enema may reveal a microcolon and failure to pass contrast beyond the ileocecal valve. Surgical exploration and removal of inspissated meconium is generally necessary, as may be a temporizing ileostomy and later reanastamosis. 2. Meconium peritonitis is associated with bowel obstruction leading to perforation and spillage of meconium into the peritoneal cavity, occasionally involving a walled-off calcified meconium "pseudocyst" apparent on plain abdominal radiograph or abdominal ultrasonography. The perforation may have occurred in utero and may be associated with fetal ascites or hydrops, pulmonary hypoplasia and respiratory failure, and impaired bowel or liver function. The newborn may have ascites or anasarca on physical examination, with a tense, distended abdomen. Plain abdominal radiography may reveal a diffuse ground-glass density and little intraluminal bowel gas; there also may be diffuse or focal calcification (Item C210B). Medical management is directed at stabilizing pulmonary function, fluid and electrolyte balance, and hepatic function before addressing the surgical condition. 3. Prolonged jaundice with a predominance of conjugated hyperbilirubinemia also is seen in infants who have CF and may result from inspissated bile or prolonged use of parenteral nutrition. Congenital lymphangiomas may present with ascites and hydrothoraces requiring drainage; analysis of the fluid confirms the diagnosis. Erythroblastosis fetalis is an isoimmune hemolytic condition that involves fetal hydrops (neonatal anasarca) and profound anemia, but it is not related to bowel obstruction and does not result in meconium peritonitis. Ovarian cysts may be massive, occasionally resulting in bowel compression and extrinsic obstruction that can require surgical excision. They are discernible by abdominal and pelvic ultrasonography as being distinct from the bowel lumen. Urinary ascites develops following urinary tract obstruction, with perforation and leakage of urine into the peritoneal space. It occurs most commonly in boys who have posterior urethral valves. Analysis of the ascites fluid reveals an elevated creatinine concentration.

References:

Page 717: AAP PREP 2009

2009 PREP SA on CD-ROM

page 717

Albanese CT, Sylvester KG. Pediatric surgery. In: Doherty GM, Way LW. Current Surgical Diagnosis and Treatment. 12th ed. New York, NY: The McGraw-Hill Companies, Inc; 2006:chap 45

Chaudry G, Navarro OM, Levine DS, Oudjhane K. Abdominal manifestations of cystic fibrosis in children. Pediatr Radiol. 2006;36:233-240. Abstract available at: http://www.ncbi.nlm.nih.gov./pubmed/16391928

Davis PB. Cystic fibrosis. Pediatr Rev. 2001;22:257-264. Available at: http://pedsinreview.aappublications.org/cgi/content/full/22/8/257

Kaye CI and the Committee on Genetics. Newborn screening fact sheets. Pediatrics. 2006;118:e934-e963. Available at: http://pediatrics.aappublications.org/cgi/content/full/118/3/e934

Page 718: AAP PREP 2009

2009 PREP SA on CD-ROM

page 718

Critique: 210

Abdominal radiograph in meconium peritonitis reveals a hazy appearance to the abdomen that suggests ascites and a calcified mass (arrow) representing a meconium pseudocyst. (Courtesy of B. Carter)

Page 719: AAP PREP 2009

2009 PREP SA on CD-ROM

page 719

Critique: 210

Anteroposterior radiograph of the chest and abdomen demonstrates a paucity of gas in the abdomen, with scattered calcifications typical of meconium peritonitis. (Courtesy of D. Mulvihill)

Page 720: AAP PREP 2009

2009 PREP SA on CD-ROM

page 720

Question: 211

A 13-year-old girl comes to your office with a 1-day history of right eye pain and tearing. She denies trauma, but says she rubbed her eyes a lot the day before because it was windy outside. Her right bulbar and palpebral conjunctivae are very injected, and copious clear discharge is present. There is no hyphema, and the pupils are normal. She complains of pain with the eye examination. After applying fluorescein to the eye, you see a single linear abrasion on the cornea. When you evert the eyelid, you find no foreign body.

Of the following, the MOST appropriate management for this condition is

A. oral analgesic

B. oral antistaphylococcal antibiotic

C. tight patching of the eye

D. topical anesthetic drops

E. topical steroid drops

Page 721: AAP PREP 2009

2009 PREP SA on CD-ROM

page 721

Critique: 211 Preferred Response: A

Corneal abrasions occur commonly in children as a result of vigorous rubbing of the eye or a foreign body. Symptoms are tearing, photophobia, and pain. The physical examination may reveal injection of the conjunctivae, copious clear discharge, or a dull corneal light reflex. It is important to perform fluorescein dye staining in children suspected of having a corneal abrasion. This dye stains the damaged cornea but is not taken up by intact corneal epithelium. The staining is seen best with a Wood lamp (Item C211), but large abrasions may be visible with regular white light or an ophthalmoscope. The presence of a linear abrasion, as described for the girl in the vignette, suggests the possibility of a retained foreign body underneath the eyelid, and care should be taken to evert the eyelid to evaluate for this. Topical anesthetic drops, such as tetracaine, may be used to facilitate the eye examination and may provide significant relief to the patient, but providing these for home use is not recommended because they may slow healing and mask persistence of symptoms. Oral analgesics usually are all that is needed to control pain while the abrasion heals. Topical nonsteroidal anti-inflammatory drops also may provide relief, but topical steroids are not indicated in the management of uncomplicated corneal abrasions (due to concerns about adverse effects on wound healing and exacerbation of coexisting viral infection). Antibiotic ointment may provide lubrication until the abrasion has healed, but the risk of secondary infection is low, so routine use of antibiotics is not needed. Patching does not speed the healing process and has been shown to interfere with activities of daily living; it no longer is recommended for routine care of simple corneal abrasions. Most corneal abrasions heal within 2 to 3 days, so if symptoms continue beyond this period, evaluation by an ophthalmologist is warranted to rule out secondary infection or retained foreign body.

References:

Calder LA, Balasubramanian S, Fergusson D. Topical nonsteroidal anti-inflammatory drugs for corneal abrasions: meta-analysis of randomized trials. Acad Emerg Med. 2005;12: 467-473. Abstract available at: http://www.ncbi.nlm.nih.gov/pubmed/15860701

Michael JG, Hug D, Dowd MD. Management of corneal abrasion in children: a randomized clinical trial. Ann Emerg Med. 2002;40:67-72. Abstract available at: http://www.ncbi.nlm.nih.gov/pubmed/12085075

Stout AU. Technical tip: corneal abrasions. Pediatr Rev. 2006;27:433-434. Available at: http://pedsinreview.aappublications.org/cgi/content/full/27/11/433

Turner A, Rabiu M. Patching for corneal abrasion. Cochrane Database Syst Rev. 2006;2:CD004764. Available at: http://www.mrw.interscience.wiley.com/cochrane/clsysrev/articles/CD004764/frame.html

Page 722: AAP PREP 2009

2009 PREP SA on CD-ROM

page 722

Critique: 211

Corneal abrasion (arrow) demonstrated after flourescein staining and examination with a Wood lamp. (Courtesy of Wake Forest University Eye Center)

Page 723: AAP PREP 2009

2009 PREP SA on CD-ROM

page 723

Question: 212

A previously healthy 15-year-old girl returns from summer camp in the mountains complaining of dysuria, frequency, and urgency. You diagnose cystitis and prescribe trimethoprim-sulfamethoxazole. Her mother phones 3 days later to report that the girl is very tired and appears pale. You advise her mother to bring her to your office. On examination, she appears pale and your order laboratory tests. The girl’s hemoglobin is 8.5 g/dL (85.0 g/L), a decrease from the value of 11.5 g/dL (115.0 g/L) that was measured during her pre-camp physical examination. Her reticulocyte count is 5.0% (0.050), and the red cell indices are normal except for mild microcytosis with a mean corpuscular volume of 76 fL. You review a smear (Item Q212).

Of the following, the MOST likely cause of this girl’s rapid onset of anemia is

A. glucose-6-phosphate dehydrogenase deficiency

B. hemoglobin SC disease

C. hereditary elliptocytosis

D. inadequate dietary iron

E. pyelonephritis

Page 724: AAP PREP 2009

2009 PREP SA on CD-ROM

page 724

Question: 212

(Courtesy of Gulati G, Caro J. Blood Cells - An Atlas of Morphology with Clinical Relevance. Chicago, Ill: ASCP Press; 2007. ©2007, American Society for Clinical Pathology.)

Page 725: AAP PREP 2009

2009 PREP SA on CD-ROM

page 725

Critique: 212 Preferred Response: A

The girl described in the vignette has a peripheral blood smear and clinical signs and symptoms consistent with hemolytic anemia following the administration of trimethoprim-sulfamethoxazole. In people who have glucose-6-phosphatase dehydrogenase (G6PD) deficiency, hemolysis can occur after the administration of sulfonamides and other antibiotics. G6PD deficiency is the most common disease-producing enzymopathy in humans. Inherited as an X-linked disorder, primarily as single-base mutations in the G6PD locus at Xq28, G6PD deficiency affects 400 million people worldwide. The highest prevalence rates (with gene frequencies from 5% to 25%) are found in tropical Africa, the Middle East, tropical and subtropical Asia, some areas of the Mediterranean, and Papua New Guinea. The defect occurs in approximately 13% of African American males. Homozygous women are found in populations in which the frequency of G6PD deficiency is high. Heterozygous (carrier) women may develop hemolytic attacks. The gene confers some protection against malaria, which probably accounts for its high gene frequency in modern populations. The G6PD enzyme catalyzes the oxidation of glucose-6-phosphate to 6-phosphogluconate while reducing the oxidized form of nicotinamide adenine dinucleotide phosphate (NADP+) to nicotinamide adenine dinucleotide phosphate (NADPH). NADPH, a required cofactor in many biosynthetic reactions, maintains glutathione in its reduced form. Reduced glutathione is a scavenger for dangerous oxidative metabolites in the cell. Red blood cells depend on G6PD activity as the only protection against oxidative stresses. People deficient in G6PD, therefore, are at risk for hemolysis and its sequelae when exposed to oxidative stress. The degree of hemolysis varies with dose of the inciting agent. The enzymatic defect in Americans of African descent is statistically less severe than that of people of Mediterranean descent. The enzyme also is found in lower quantities in older red blood cells due to senescence of the enzyme. Affected individuals typically do not display symptoms or signs in the absence of oxidative stress. However, those who have very low enzyme concentrations may present with neonatal jaundice and acute hemolytic anemia. Neonatal jaundice usually appears by age 1 to 4 days, at the same time as or slightly earlier than so-called physiologic jaundice; kernicterus is rare. Acute hemolytic anemia results from stress factors such as oxidative drugs or chemicals, infection, or ingestion of fava beans (common to a Mediterranean diet). Gallstones may be a prominent feature in affected individuals. As with hemolysis of different causes, jaundice and splenomegaly may be present during a crisis. Immediately after an episode, younger red blood cells are released that contain a higher concentration of enzyme. Accordingly, testing should be delayed in the event of undiagnosed hemolysis in children for a few weeks after the hemolysis to allow G6PD to decrease to normal concentrations. The clinician should have a high suspicion for G6PD deficiency in immigrants of the ethnic groups noted previously and consider testing potentially affected males who exhibit hemolysis as part of a significant illness (such as diabetic acidosis, hepatitis) or trauma, especially prior to administering medications that may precipitate hemolysis. Precipitants include antibacterials (especially sulfonamides), antimalarials (chloroquine, primaquine), and other medications (aspirin, vitamin K analogs). Chemicals that may induce hemolysis include naphthalene, which is found in mothballs. Neonatal screening and health education have been effective in some countries at detecting individuals at risk and preventing hemolysis. Widespread screening of asymptomatic individuals is rare in the United States because most affected individuals have only a mild form of the disease. Hemoglobin SC disease and hereditary elliptocytosis are unlikely diagnoses in a previously healthy 15-year-old girl because these conditions are associated with clinical disease in early childhood. In addition, hemoglobin SC disease often is diagnosed in the neonatal period because of the availability of neonatal screening. Hereditary elliptocytosis (Item C212) also is detected in infancy and early childhood

Page 726: AAP PREP 2009

2009 PREP SA on CD-ROM

page 726

based on hemolysis without stressors. Iron deficiency does not cause hemolysis or jaundice. Pyelonephritis may result from ascending infection from cystitis but normally is associated with fever and other constitutional symptoms in addition to fatigue. Hemolysis can result from infection in persons unaffected by G6PD deficiency but usually not to the degree noted in the child described in the vignette.

References:

Carter SM, Gross SJ. Glucose-6-phosphate dehydrogenase deficiency. eMedicine Specialties, Medicine, Ob/Gyn, Psychiatry, and Surgery, Hematology. 2005. Available at:http://www.emedicine.com/med/topic900.htm

Frank JE. Diagnosis and management of G6PD deficiency. Am Fam Physician. 2005;72:1277-1282. Available at: http://www.aafp.org/afp/20051001/1277.html

Segal GB. Enzymatic defects. In: Kliegman RM, Behrman RE, Jenson HB, Stanton BF, eds. Nelson Textbook of Pediatrics. Philadelphia, Pa: Saunders Elsevier; 2007:2039-2041

Segel GB, Hirsh MG, Feig SA. Managing anemia in a pediatric office practice: part 2. Pediatr Rev. 2002;23:111-122. Available at: http://pedsinreview.aappublications.org/cgi/content/full/23/4/111

Page 727: AAP PREP 2009

2009 PREP SA on CD-ROM

page 727

Critique: 212

Elliptocytes (oval-shaped red blood cells), as seen in hereditary elliptocytosis. (Courtesy of Gulati G, Caro J. Blood Cells - An Atlas of Morphology with Clinical Relevance. Chicago, Ill: ASCP Press; 2007. ©2007, American Society for Clinical Pathology.)

Page 728: AAP PREP 2009

2009 PREP SA on CD-ROM

page 728

Question: 213

A 13-year-old boy who has a bicuspid aortic valve and aortic stenosis with a 20-mm Hg (mild) gradient by echocardiography (Item Q213) is interested in participating in sports. He asks for your advice.

Of the following, the BEST response is that

A. aerobic activities should be well tolerated

B. football is contraindicated

C. he should not participate in weight training

D. he should refrain from highly competitive sports

E. wrestling is contraindicated

Page 729: AAP PREP 2009

2009 PREP SA on CD-ROM

page 729

Critique: 213 Preferred Response: A

Congenital heart disease is the most commonly encountered malformation, with an incidence of approximately 8 in 1,000 live births or slightly less than 1%. Today, congenital heart disease is diagnosed early, often in utero, and medical, catheter-based, and surgical management strategies have progressed to a point that most affected children lead active and healthy lives. Exercise and sports participation are healthy and normal components of many children's lives, including those affected by congenital heart disease. Various professional organizations have recommended levels of sports participation based on the underlying cardiac malformation and its degree of severity. The most common congenital heart abnormality is a bicommissural (bicuspid) aortic valve (Item C213), as described for the boy in the vignette. Although many of the children who have bicommissural valves do not have associated aortic stenosis during childhood, it is the most common cause of aortic stenosis in children. The pressure gradient across the aortic valve during systole determines the severity of the stenosis. Pressure gradients that are less than 25 mm Hg generally are considered mild, those between 25 and 50 mm Hg are considered moderate, and those in excess of 50 mm Hg are considered severe. Patients who have aortic stenosis should be monitored regularly, with electrocardiography and echocardiography supplementing a thorough history and physical examination because aortic stenosis can progress with patient growth and age. Patients who have mild aortic stenosis, such as the boy described in the vignette, generally have no restrictions on physical activity and exercise. Children who have moderate aortic stenosis may participate in self-limited aerobic activities and should avoid isometric activities such as weightlifting and wrestling, which increase the systemic afterload and workload of the left ventricle. The boy in the vignette should be able to play football or any other competitive sport if the remainder of his physical examination and cardiac evaluation is benign. He also may participate in weight training, particularly the lighter weights that build endurance and muscle speed rather than the heavier weights that build large muscle bulk. Wrestling is not contraindicated for children who have asymptomatic mild aortic stenosis, although it may be in patients who have moderate aortic stenosis.

References:

Dickhuth H-H, Kececioglu D, Schumacher YO. FIMS Position Statement: Congenital Heart Disease and Sports. International Federation of Sports Medicine; January 2006. Available at: http://www.fims.org/default.asp?PageID=120975716

Maron BJ, Chaitman BR, Ackerman MJ, et al. Recommendations for physical activity and recreational sports participation for young patients with genetic cardiovascular diseases. Circulation. 2004;109:2807-2816. Available at: http://circ.ahajournals.org/cgi/content/full/109/22/2807

Singh A, Silberbach M. Consultation with the specialist: cardiovascular preparticipation sports screening. Pediatr Rev. 2006;27:418-424. Available at: http://pedsinreview.aappublications.org/cgi/content/full/27/11/418

Stefani L, Galanti G, Tonicelli L, et al. Bicuspid aortic valve in competitive athletes. Br J Sports Med. 2008;42:31-35. Abstract available at: http://www.ncbi.nlm.nih.gov/pubmed/17548371

Page 730: AAP PREP 2009

2009 PREP SA on CD-ROM

page 730

Critique: 213

In a bicuspid aortic valve, one of the commisures fails to form (arrow), resulting in two rather than three leaflets. (Courtesy of P Lynch)

Page 731: AAP PREP 2009

2009 PREP SA on CD-ROM

page 731

Question: 214

The mother of a 7-year-old girl who has epilepsy phones your office because her child has developed a rash. The mother is worried that the rash may be due to her new antiseizure medication.

Of the following, the MOST appropriate next step is to

A. defer evaluation until the next health supervision visit

B. examine the child in your office promptly

C. refer the child to a dermatologist

D. refer the child to her neurologist

E. send the child to the laboratory to obtain a complete blood count

Page 732: AAP PREP 2009

2009 PREP SA on CD-ROM

page 732

Critique: 214 Preferred Response: B

Antiseizure medications can cause a wide variety of adverse effects, including central nervous system effects such as sedation, dizziness, personality changes, and occasionally, worsening of seizures. The neurologist, psychiatrist, or other physician who prescribes such medications should be familiar with adverse effect profiles of these medications. Adverse effects of antiseizure medications may occur outside the central nervous system as well. Bone marrow suppression and liver toxicity are among the more serious possible systemic effects. Rashes are common in children and can occur as an adverse effect of most antiseizure medications. Such drug rashes tend to occur during the first month of use, as described for the girl in the vignette. Rapid assessment of the child is important because in some cases, the rash may be a harbinger of a more global, serious allergic reaction or of Stevens-Johnson syndrome (erythema multiforme major) (Item C214). A general pediatrician often is better trained than a neurologist in the identification of the many rashes that can occur in childhood. Therefore, prompt assessment should be undertaken by the primary care physician in the office. A dermatologist referral may be needed, and laboratory testing may be useful, but these decisions should be made after the clinical assessment by the child's primary care physician.

References:

French JA, Kanner AM, Bautista J, et al. Efficacy and tolerability of the new antiepileptic drugs I: treatment of new onset epilepsy. Report of the Therapeutics and Technology Assessment Subcommittee and Quality Standards Subcommittee of the American Academy of Neurology and the American Epilepsy Society. Neurology. 2004;62:1252-1260. Available at: http://www.neurology.org/cgi/content/full/62/8/1252

Johnston MV. Seizures in childhood. In: Kliegman RM, Behrman RE, Jenson HB, Stanton BF, eds. Nelson Textbook of Pediatrics. 18th ed. Philadelphia, Pa: Saunders Elsevier; 2007:2457-2475

Page 733: AAP PREP 2009

2009 PREP SA on CD-ROM

page 733

Critique: 214

Stevens-Johnson syndrome due to lamotrigine: There is involvement of conjunctivae and oral mucosa as well as erythematous eroded patches. (Courtesy of M Rimsza)

Page 734: AAP PREP 2009

2009 PREP SA on CD-ROM

page 734

Question: 215

A newborn male experiences prolonged oozing following circumcision. Hematologic evaluation reveals that he has less than 1% of factor VIII clotting activity and a prolonged partial thromboplastin time, consistent with severe hemophilia A. His family history is negative for any individuals affected by clotting disorders.

Of the following, the MOST accurate statement for counseling this child’s parents is that

A. another family member likely is affected, but the condition is so mild that the person has not been diagnosed

B. in families such as this, 50% of affected boys have a spontaneous gene mutation

C. molecular genetic testing can detect mutations in 50% of affected individuals

D. severe hemophilia is the rarest of all types

E. there is an 80% chance that the mother is a hemophilia carrier

Page 735: AAP PREP 2009

2009 PREP SA on CD-ROM

page 735

Critique: 215 Preferred Response: E

One third to one half of all males who have hemophilia A have no family history of the disorder, as described for the infant in the vignette. In these cases, it is important to recognize that multiple genetic scenarios can result in such an outcome. In about 20% of the cases involving no previous family history for hemophilia, the mother is not a carrier, in which case her son has a de novo disease-causing mutation. In some of these situations, the son is a somatic mosaic for the mutation, ie, the mutation is not present in all of the cells of his body. In 80% of such cases, the mother is a carrier, and she might have a de novo gene mutation. This mutation could have occurred in the egg or sperm cell from which she was conceived (in which case it is present in all of her cells). Alternatively, it may be due to a somatic mutation that occurred in early embryogenesis or to germline mosaicism, in which case it is present in only some of her germ cells. The mother could have inherited the disease-causing mutation from her mother or unaffected father (if he is a somatic or germline mosaic). Finally, the mother could have inherited the mutation from a previous generation, which may have been passed on only through daughters, for example, so that no one was affected with hemophilia A. Cases in which the mutation for hemophilia A is present in a mosaic state are unusual, and the gene mutation runs true in families. Therefore, every boy in a family who has the same mutation will have the same severity of disease. Severe hemophilia (<1% normal factor VIII activity) is the most common of all types of hemophilia A. Molecular genetic testing can detect mutations in 98% of those who have severe disease.

References:

Hamosh A. Clinical case studies illustrating genetic principles. In: Nussbaum RL, McInnes RR, Willard HR, eds. Thompson & Thompson Genetics in Medicine. 7th ed. Philadelphia, Pa: Elsevier Saunders; 2007:268-269

Johnson MJ, Thompson AR. Hemophilia A. GeneReviews. 2005. Available at: http://www.geneclinics.org/servlet/access?db=geneclinics&site=gt&id=8888891&key=xAcWBcrjmZrVo&gry=&fcn=y&fw=8cy5&filename=/profiles/hemo-a/index.html

Page 736: AAP PREP 2009

2009 PREP SA on CD-ROM

page 736

Question: 216

An 18-year-old young woman reports that she has "bumps" in her vaginal area. She recently became sexually active with a single partner. She says that the lesions are not tender, and she has no vaginal discharge or itching. Genital examination reveals several clusters of flesh-colored, pedunculated lesions, primarily in the posterior fourchette, compatible with genital warts (Item Q216). You counsel her about treatment options.

Of the following, the MOST accurate statement regarding management and treatment of genital warts is that

A. human papillomavirus infectivity is eradicated by treatment of external warts

B. no definitive evidence supports the superiority of any of the available genital wart treatments

C. single treatment with clinician- or patient-applied methods eradicates all lesions in most patients

D. waiting for spontaneous resolution of warts is not acceptable

E. with her lesions, the patient is not currently a candidate for the human papillomavirus vaccine

Page 737: AAP PREP 2009

2009 PREP SA on CD-ROM

page 737

Question: 216

Papules (arrow), as described for the patient in the vignette. (Courtesy of M. Rimsza)

Page 738: AAP PREP 2009

2009 PREP SA on CD-ROM

page 738

Critique: 216 Preferred Response: B

Human papillomavirus (HPV) is the cause of genital warts, which usually appear as flat, papular, or pedunculated lesions of the genital mucosa (Item C216). Research data indicate that currently available therapies for the treatment of genital warts might reduce, but probably do not eradicate, HPV infectivity. When no treatment is given, genital warts may resolve spontaneously, remain unchanged, or increase in number or size. An acceptable alternative for some patients is to forego treatment and wait for spontaneous resolution. No definitive evidence suggests that any of the available treatments, including topical patient- or clinician-applied medications (podofilox, imiquimod, podophyllin resin, or bichloro- or trichloroacetic acid), cryotherapy, surgical removal, intralesional interferon, or laser therapy, is superior to any other. Most patients require a course of therapy rather than a single treatment to eradicate warts. Vaccination against HPV still is recommended for a patient who has external warts because the vaccine can protect against other strains of HPV to which the patient may not have been exposed. References:

Markowitz LE, Dunne EF, Saraiya M, Lawson HW, Chesson H, Unger ER. Quadrivalent human papillomavirus vaccine: recommendations of the Advisory Committee on Immunization Practices (ACIP). MMWR Recomm Rep. 2007;56(RR02):1-24. Available at: http://www.cdc.gov/mmwr/preview/mmwrhtml/rr5602a1.htm

Workowski KA, Berman SM, Centers for Disease Control and Prevention. sexually transmitted diseases treatment guidelines, 2006. MMWR Recomm Rep. 2006;55(RR11):1-94. Available at: http://www.cdc.gov/mmwr/preview/mmwrhtml/rr5511a1.htm

Page 739: AAP PREP 2009

2009 PREP SA on CD-ROM

page 739

Critique: 216

Filiform papules characteristic of human papillomavirus infection on mucosal surfaces. (Courtesy of M. Rimsza)

Page 740: AAP PREP 2009

2009 PREP SA on CD-ROM

page 740

Question: 217

A 2-year-old girl is brought to the emergency department after being found unconscious at her grandparent’s home. Her mother reports that she was in her usual good health when she was dropped off at her grandparents 2 hours ago and that there is no history of trauma. Of note, the grandmother found a spilled, opened bottle of her "blood pressure" medicine in the bathroom. On physical examination, the girl is somnolent but arouses with stimulation. There is no sign of trauma on physical examination. Her temperature is 98.0°F (37.0°C), heart rate is 60 beats/min, respiratory rate is 25 breaths/min, and oxygen saturation is 93% on room air. Her pupils are 2 mm and reactive bilaterally. Her mouth and mucous membranes are dry, and she has no rashes. You order serum electrolyte measurement and a urine toxicology screen.

Of the following, the MOST appropriate additional tests to obtain are

A. chest radiography and electrocardiography

B. computed tomography (CT) scan of the abdomen and electrocardiography

C. CT scan of the head and electrocardiography

D. CT scan of the head and lumbar puncture

E. electrocardiography and skull radiographs

Page 741: AAP PREP 2009

2009 PREP SA on CD-ROM

page 741

Critique: 217 Preferred Response: C

A child who has an altered level of consciousness can present a diagnostic dilemma. Patient age, a clear history of the present illness, and a comprehensive past medical history can help guide the diagnostic evaluation and narrow the differential diagnosis. A useful guide for remembering potential causes of an altered level of consciousness is the mnemonic AEIOU TIPS (tips on the vowels) (Item C217). Toxic ingestions are a common cause of an acute altered level of consciousness in a toddler. Serum and urine toxicology screens should be performed if the ingestion is not witnessed because multiple drugs may have been ingested. The previously healthy child described in the vignette has signs and symptoms consistent with a clonidine overdose, as evidenced by her being awakened with stimulation and having dry mucous membranes and pupillary constriction. The onset of action of this drug is less than 1 hour, and toxic effects include neurologic and respiratory depression, hypotonia, bradycardia, hypotension, and cardiac arrhythmias. Treatment is supportive, with both intensive care unit monitoring and baseline electrocardiography indicated. In this case, CT scan of the head also is indicated because the child might have had an unwitnessed fall. Primary pulmonary processes rarely present as an altered level of consciousness unless there is an associated oxygen deficiency. Therefore, chest radiography is not necessary in this case. Abdominal CT scan is indicated if abdominal trauma or an intra-abdominal infectious process is suspected. Lumbar puncture should be undertaken if meningitis is suspected, but the child in the vignette is afebrile and was reported in good health just 2 hours ago. Skull radiography has limited utility in the evaluation of an altered level of consciousness.

References:

Avner JR. Altered states of consciousness. Pediatr Rev. 2006:27:331-338. Available at: http://pedsinreview.aappublications.org/cgi/content/full/27/9/331

Frankel LR. Neurological emergencies and stabilization. In: Kliegman RM, Behrman RE, Jenson HB, Stanton BF, eds. Nelson Textbook of Pediatrics. 18th ed. Philadelphia, Pa: Saunders Elsevier: 2007:405-412

Page 742: AAP PREP 2009

2009 PREP SA on CD-ROM

page 742

Critique: 217

Page 743: AAP PREP 2009

2009 PREP SA on CD-ROM

page 743

Question: 218

The parents of a 6-year-old boy are concerned because he has been developing pubic hair over the past 6 months. On physical examination, you note a recent growth spurt, Sexual Maturity Rating 3 pubic hair, a penis that is 8 cm in length and androgenized, and testes that are 5 mL in volume. Other findings are normal. His bone age is 7 years. You order measurements of serum testosterone, 17-hydroxyprogesterone, dehydroepiandrosterone, luteinizing hormone, and follicle-stimulating hormone.

Of the following, the MOST important additional test is measurement of serum

A. adrenocorticotropic hormone

B. estradiol

C. free testosterone

D. human chorionic gonadotropin

E. prolactin

Page 744: AAP PREP 2009

2009 PREP SA on CD-ROM

page 744

Critique: 218 Preferred Response: D

The child described in the vignette has sexual precocity, with testes that are increased in volume and definite evidence of increased phallus size and pubic hair. Therefore, it is likely that the increased androgen is being produced by the child's testes. Such production could be related to autonomous testicular functioning, as in "testitoxicosis," or gonadotropin-independent sexual precocity, but it is more likely related to testicular stimulation by gonadotropins. With true central sexual precocity, luteinizing hormone (LH), follicle-stimulating hormone (FSH), and circulating concentrations of testosterone are elevated. However, testosterone also could be produced if the testes are stimulated by human chorionic gonadotropin (HCG), which can mimic LH and stimulate growth of the Leydig cells that produce testosterone. HCG may be produced by germ cell or other tumors located in the central nervous system, the mediastinum, the liver (hepatoblastomas), and other locations. This hormone should be measured specifically in the blood as beta-HCG. Elevation of HCG concentrations causes pubertal change in boys but not in girls because girls require LH and FSH to stimulate ovarian estrogen production. Boys who have sexual precocity as a result of HCG secretion have a smaller testicular volume than expected for pubertal stage because FSH-stimulated Sertoli cell numbers do not increase. Measurement of adrenocorticotropic hormone is not useful because concentrations of this hormone fluctuate with stress and are elevated persistently only in the presence of Cushing disease or adrenal insufficiency, not precocious puberty. Serum estradiol values are likely to be slightly elevated because testosterone is converted to estrogen peripherally in fat and in the liver, so if testosterone or other androgen concentrations are elevated, so are estrogens. Estrogen values in this child are likely to be low because no breast enlargement is reported. Measurement of free testosterone is unlikely to be useful in a child whose physical examination reveals so much androgen effect. Free testosterone measurement may be useful to examine androgen effect when normal testosterone concentrations are associated with mild clinical hyperandrogenism in women. Serum prolactin does not stimulate production of androgen in boys. Sometimes, prolactinomas are associated with mild hyperandrogenism and irregular menses or amenorrhea in women.

References:

Ferry RJ Jr. Precocious pseudopuberty. eMedicine Specialties, Pediatrics: General Medicine, Endocrinology. 2007. Available at: http://www.emedicine.com/ped/topic1881.htm

Kaplowitz P. Precocious puberty. eMedicine Specialties, Pediatrics: General Medicine, Endocrinology. 2007. Available at: http://www.emedicine.com/ped/topic1882.htm

Muir A. Precocious puberty. Pediatr Rev. 2006;27:373-381. Available at: http://pedsinreview.aappublications.org/cgi/content/full/27/10/373

Rivarola MA, Belgorsky A, Mendilaharzu H, Vidal G. Precocious puberty in children with tumours of the suprasellar and pineal areas: organic central precocious puberty. Acta Paediatr. 2001;90:751-756. Abstract available at: http://www.blackwell-synergy.com/doi/abs/10.1111/j.1651-2227.2001.tb02800.x?journalCode=apa

Saenger P. Overview of precocious puberty. UpToDate Online 15.3. 2008. Available for subscription at: http://www.uptodateonline.com/utd/content/topic.do?topicKey=pediendo/14867

Page 745: AAP PREP 2009

2009 PREP SA on CD-ROM

page 745

Question: 219

During the health supervision visit for an infant, her mother mentions that the child has been tolerating solid foods with no problem. When placed on her back to be examined, she brings her feet to her mouth. Her mother holds a small mirror to the child’s face to distract her during your examination, and the baby reaches for the mirror and pats her image.

Of the following, these developmental milestones are MOST typical for an infant whose age is

A. 2 months

B. 4 months

C. 6 months

D. 9 months

E. 12 months

Page 746: AAP PREP 2009

2009 PREP SA on CD-ROM

page 746

Critique: 219 Preferred Response: C

Tolerating solid foods well, placing her feet to her mouth while supine, and reaching for a mirror and patting the image are socioemotional milestones most typical of a 6-month-old child. At 2 months of age, infants bring their hands to their mouths, swipe at dangling objects with their hands, and grasp and shake hand toys. A 4-month-old infant has a spontaneous social smile and, when shown a mirror, smiles and vocalizes. An infant of this age also brings items to his or her mouth, can maintain an upright position if placed upright with the assistance of pillows, and knows the difference between strangers and family. A 9-month-old infant can imitate nursery games in response to a mother's demonstrations and may give a toy in response to a request. A 12-month-old infant points for requests, may throw a toy in play or refusal, helps with dressing, may hug a stuffed animal, and may offer a toy to an image in a mirror. References:

Knobloch H, Stevens FM, Malone AF. The revised developmental stages. In: Manual of Developmental Diagnosis: The Administration and Interpretation of the Revised Gesell and Amatruda Developmental and Neurologic Examination. Albany, NY: Developmental Evaluation Materials, Inc; 1987:17-120

Whitaker T, Palmer F. The developmental history. In: Accardo PJ. Capute & Accardo's Neurodevelopmental Disabilities in Infancy and Childhood. Volume I: Neurodevelopmental Diagnosis and Treatment. 3rd ed. Baltimore, Md: Paul H. Brookes Publishing Co; 2008:297-310

Page 747: AAP PREP 2009

2009 PREP SA on CD-ROM

page 747

Question: 220

You admitted a patient to the hospital yesterday who had acute onset of fever (temperature of 103.0oF [39.4oC]), a petechial rash, meningismus, and shock. She required blood pressure support and mechanical ventilation during the night. As per the protocol for your hospital, you placed this child into respiratory isolation upon admission. Today you are told that her blood culture is growing Neisseria meningitidis. The nurse taking care of her asks you how long the child needs to remain in respiratory isolation.

Of the following, the BEST answer is until the child

A. completes 1 day of antimicrobial therapy

B. defervesces

C. is clinically stable

D. is extubated

E. is proven not to have meningitis

Page 748: AAP PREP 2009

2009 PREP SA on CD-ROM

page 748

Critique: 220 Preferred Response: A

Standard isolation precautions should be observed for patients who have meningococcal disease, and they should remain in respiratory isolation until 24 hours after the initiation of effective antimicrobial therapy. The presence of continued fever, prolonged intubation, clinical instability, or meningitis is irrelevant to the issue of isolation. Patients treated for invasive disease with any antimicrobial agent other than cefotaxime or ceftriaxone also must receive 2 days of rifampin, one dose of ceftriaxone, or a single dose of ciprofloxacin prior to hospital discharge to eradicate the nasopharyngeal carriage of Neisseria meningitidis. Family members and individuals who have had close contact with the patient also should be given chemoprophylaxis. Exposed individuals who have received either the polysaccharide or conjugate meningococcal vaccine still should receive chemoprophylaxis because the vaccine may not contain the serotype that is specific for the patient's illness (eg, serotype B). Invasive disease caused by N meningitidis usually presents in one of three forms. Meningococcemia is characterized by symptoms suggestive of an upper respiratory tract infection and complaints of headache, lethargy, myalgias, or joint pain. Within hours, a petechial or purpuric rash (Item C220) may develop, as might circulatory collapse. Meningococcal meningitis often begins with similar symptoms to meningococcemia, but affected children become irritable and have signs of meningeal irritation on physical examination. Occult bacteremia with N meningitidis also has been described. Affected children present with a febrile illness, and a blood culture grows N meningitidis, but the children never progress to the clinical picture of fulminant meningococcemia.

References:

American Academy of Pediatrics. Meningococcal infections. In: Pickering LK, Baker CJ, Long SS, McMillan JA, eds. Red Book: 2006 Report of the Committee on Infectious Diseases. 27th ed. Elk Grove Village, Ill: American Academy of Pediatrics; 2006:452-460

Bilukha OO, Rosenstein N. Prevention and control of meningococcal disease: recommendations of the Advisory Committee on Immunization Practices (ACIP). MMWR Recomm Rep. 2005;54(RR07):1-21. Available at: http://www.cdc.gov/mmwr/preview/mmwrhtml/rr5407a1.htm

Hugo B. Index of suspicion: case 4. Pediatr Rev. 2005;26:295-301. Available at: http://pedsinreview.aappublications.org/cgi/content/full/26/8/295

Notice to readers: revised recommendations of the Advisory Committee on Immunization Practices to vaccinate all persons aged 11-18 years with meningococcal conjugate vaccine. MMWR Morbid Mortal Wkly Rep. 2007;56:794-795. Available at: http://www.cdc.gov/mmwr/preview/mmwrhtml/mm5631a3.htm

Page 749: AAP PREP 2009

2009 PREP SA on CD-ROM

page 749

Critique: 220

Meningococcemia is characterized by a petechial and purpuric eruption. (Courtesy of G. Schutze)

Page 750: AAP PREP 2009

2009 PREP SA on CD-ROM

page 750

Question: 221

You are speaking to the mother of a previously healthy boy who has just broken out with chickenpox lesions. His mother states that one of her son’s classmates also has the disease. No one else in the household is ill. He did not receive varicella vaccine, but all of his other immunizations are up to date. His mother asks whether her son is at risk for developing a severe case of the disease.

Of the following, the factor that is MOST likely to increase his risk for moderate-to-severe varicella disease is

A. age younger than 12 years

B. being a secondary case in the household

C. concurrent ear infection

D. receipt of a recent course of antibiotics

E. receipt of short-term salicylate therapy

Page 751: AAP PREP 2009

2009 PREP SA on CD-ROM

page 751

Critique: 221 Preferred Response: B

Chickenpox or varicella infection is caused by the varicella-zoster virus. Infection tends to be more severe in adolescents and adults and in those who have immunocompromising conditions. Complications may include bacterial superinfection of the skin lesions (Item C221A), pneumonia (Item C221B), central nervous system involvement (cerebellar ataxia, encephalitis), thrombocytopenia, glomerulonephritis, arthritis, and hepatitis. The decision of whether a patient should receive antiviral therapy, the route by which it is administered, and the duration of therapy is determined by specific host factors, the extent of infection, the timing of exposure, and initial response to therapy. Oral acyclovir is not recommended for routine use in healthy children who have varicella. However, oral acyclovir should be considered in immunocompetent hosts who are at increased risk for moderate-to-severe varicella. Persons who fall into this category include: those who are older than 12 years of age; those who have chronic cutaneous or pulmonary disorders; persons receiving long-term salicylate therapy; people receiving short, intermittent, or aerosolized courses of corticosteroids; and secondary household cases (disease usually is more severe than in the primary case). The factor that would increase the risk for developing moderate-to-severe varicella disease for the patient described in the vignette is being a secondary case in the household. His age, ear infection, or receipt of antibiotics or short-term salicylate therapy is not associated with increased risk for moderate-to-severe disease. Intravenous antiviral therapy is recommended for immunocompromised patients who have varicella, including patients being treated with chronic corticosteroids. To maximize efficacy, therapy should be initiated early in the course of the illness, preferably within 24 hours of rash onset. Oral acyclovir generally should not be used to treat immunocompromised children who have varicella because of poor oral bioavailability. However, high-dose oral acyclovir therapy may be used in selected immunocompromised patients who are believed to be at lower risk of developing severe varicella. Such patients include those infected with human immunodeficiency virus who have relatively normal concentrations of CD4+ lymphocytes and children who have leukemia in whom careful follow-up is guaranteed. Varicella-zoster immune globulin, if available, or intravenous immunoglobulin administered shortly after exposure can prevent or modify the course of varicella disease, but immune globulin preparations are not effective once disease is established.

References:

American Academy of Pediatrics. Varicella-zoster infections. In: Pickering LK, Baker CJ, Long SS, McMillan JA, eds. Red Book: 2006 Report of the Committee on Infectious Diseases. 27th ed. Elk Grove Village, Ill: American Academy of Pediatrics; 2006:711-725

Arvin AM. Antiviral therapy for varicella and herpes zoster. Semin Pediatr Infect Dis. 2002;13:12-21. Abstract available at: http://www.ncbi.nlm.nih.gov/pubmed/12118839

Balfour HH Jr, Rotbart HA, Feldman S, et al. Acyclovir treatment of varicella in otherwise healthy adolescents. The Collaborative Acyclovir Varicella Study Group. J Pediatr. 1992;120:627-633. Abstract available at: http://www.ncbi.nlm.nih.gov/pubmed/1313098

Whitley RJ. Approaches to the treatment of varicella-zoster virus infections. Contrib Microbiol. 1999;3:158-172

Whitley RJ. Therapeutic approaches to varicella-zoster virus infections. J Infect Dis. 1992;166(suppl 1):S51-S57. Abstract available at: http://www.ncbi.nlm.nih.gov/pubmed/1378081

Page 752: AAP PREP 2009

2009 PREP SA on CD-ROM

page 752

Critique: 221

Lesions of varicella may become infected secondarily with Streptococcus pyogenes or Staphylococcus aureus. (Reprinted with permission from English R. Varicella. Pediatr Rev. 2003;24:372-379. Photo courtesy of J. Zenel.)

Page 753: AAP PREP 2009

2009 PREP SA on CD-ROM

page 753

Critique: 221

The chest radiograph in varicella pneumonia is characterized by bilateral infiltrates that, in the early stages, may have a nodular component (arrow). (Courtesy of the Red Book® Online.)

Page 754: AAP PREP 2009

2009 PREP SA on CD-ROM

page 754

Question: 222

A father brings in his 8-year-old son because the boy has been "feeling tired" for the past few weeks. The remainder of the history is unremarkable. His weight is at the 5th percentile, height is less than the 5th percentile, temperature is 98.6°F (37°C), heart rate is 88 beats/min, respiratory rate is 16 breaths/min, and blood pressure is 124/84 mm Hg. Urinalysis findings include a specific gravity of 1.005, pH of 6.5, no blood, and 2+ protein. Other laboratory results are: · Sodium, 134.0 mEq/L (134.0 mmol/L) · Potassium, 5.4 mEq/L (5.4 mmol/L) · Chloride, 96.0 mEq/L (96.0 mmol/L) · Bicarbonate, 14.0 mEq/L (14.0 mmol/L) · Blood urea nitrogen, 96.0 mg/dL (34.3 mmol/L) · Creatinine, 8.4 mg/dL (742.6 mcmol/L)

Of the following, the MOST likely additional finding expected for this child is

A. enlarged kidneys on ultrasonography

B. hypomagnesemia

C. increased parathyroid hormone concentration

D. low insulin-like growth factor I concentration

E. reticulocytosis

Page 755: AAP PREP 2009

2009 PREP SA on CD-ROM

page 755

Critique: 222 Preferred Response: C

The boy described in the vignette has fatigue, growth failure, and elevated blood urea nitrogen (BUN) and creatinine values, which are consistent with a diagnosis of chronic renal failure (CRF). CRF in children is characterized by the marked loss of excretory function. The normal glomerular filtration rate (GFR) is 100 to 120 mL/min per 1.73 m2, and children who have severe renal impairment should be considered for renal replacement therapy (dialysis or transplantation) when the GFR approaches 20 mL/min per 1.73 m2. Prior to reaching this degree of renal dysfunction, other abnormalities occur due to impaired renal excretory function, reduced functional nephron mass, and impaired endocrine function. Because of the CRF, affected children often have poor appetites and impaired growth, leading to failure to thrive and short stature in those who have long-standing disease. More specifically, affected children actually have elevated growth hormone and insulin-like growth factor (IGF) I concentrations, but because of high concentrations of IGF-binding proteins, the bioavailability is reduced, resulting in abnormal growth. Decreased urine output with impaired excretion of sodium and water can lead to hypertension. Other minerals, including phosphorus, potassium, and magnesium, tend to accumulate in renal failure. With decreased nephron mass, 25-hydroxyvitamin D3 exiting the liver cannot be activated in the kidney to 1,25-dihydoxyvitamin D3 due to low concentrations of 1-alpha hydroxylase activity. Low bioactive forms of vitamin D3 accompanied by phosphorus retention and secondary hyperparathyroidism lead to renal osteodystrophy (renal rickets) (Item C222). The low erythropoietin values that are seen in advanced renal disease (usually when the GFR is less than 35 mL/min per 1.73 m2) lead to anemia associated with a low reticulocyte count. Although children who have acute renal failure often have inflamed kidneys that are enlarged on ultrasonography, children who have CRF usually have small, shrunken kidneys on ultrasonography. Thus, kidney size sometimes can help distinguish between acute and chronic renal failure.

References:

Fine RN, Whyte DA, Boydstun II. Conservative management of chronic renal insufficiency. In: Avner ED, Harmon WE, Niaudet P, eds. Pediatric Nephrology. 5th ed. Philadelphia, Pa: Lippincott Williams & Wilkins; 2004:1291-1311

Wong CS, Mak RH. Chronic kidney disease. In: Kher KK, Schnaper HW, Makker SP, eds. Clinical Pediatric Nephrology. 2nd ed. London, England: Informa Healthcare; 2007:339-352

Page 756: AAP PREP 2009

2009 PREP SA on CD-ROM

page 756

Critique: 222

Anteroposterior radiograph of the knees shows diffuse demineralization of both cortical and trabecular bone and fraying of the metaphyses. (Courtesy of D. Mulvihill)

Page 757: AAP PREP 2009

2009 PREP SA on CD-ROM

page 757

Question: 223

A 4-year-old girl presents with a 2-week history of bilateral discolored rhinorrhea, nasal congestion, and decreased oral intake. Her mother states that at the onset of this illness, she developed clear rhinorrhea 2 days after attending child care. Despite using over-the-counter antihistamines and decongestants, the child’s symptoms have persisted. A quick review of her chart shows that her immunizations are up to date, including her pneumococcal conjugate vaccine series and her annual influenza vaccination. On physical examination, the child has appropriate vital signs for her age, infraorbital edema bilaterally, and yellowish mucus in her nares. You suspect acute bacterial rhinosinusitis (ABRS) and discuss evaluation and treatment options with the mother.

Of the following, a TRUE statement regarding this child’s condition is that

A. a sinus radiograph should be performed prior to initiating antibiotic therapy for ABRS

B. ABRS can be distinguished easily from a viral upper respiratory tract infection

C. allergic rhinitis is the most common risk factor for developing ABRS

D. the gold standard test for organism identification in ABRS is a nasal swab culture

E. the most likely bacterial pathogen is Haemophilus influenzae

Page 758: AAP PREP 2009

2009 PREP SA on CD-ROM

page 758

Critique: 223 Preferred Response: E

Acute bacterial rhinosinusitis (ABRS) is defined as sinusitis symptoms lasting for fewer than 4 weeks. Common symptoms include purulent nasal discharge and sinus pressure. Because the symptoms of ABRS are difficult to distinguish from a viral upper respiratory tract infection (URI), symptoms of a viral URI that persist past 7 to 10 days, as described for the girl in the vignette, generally are regarded as indicative of bacterial sinusitis. Viral URI is the most common preceding condition for ABRS, but allergic rhinitis is a precipitant in approximately 20% of cases. In 2001, the American Academy of Pediatrics published management guidelines for the evaluation and treatment of ABRS. Because the initial symptoms of ABRS usually are indistinguishable from a viral URI, physical examination, laboratory tests, and radiographic studies such as a sinus radiograph or sinus computed tomography scan are not helpful in distinguishing a prolonged viral URI from bacterial sinusitis. A Waters view sinus radiograph may help confirm unilateral sinusitis by showing maxillary opacification, but it does not need to be performed prior to initiating antibiotics. An antral puncture is considered the gold standard for isolation of the causative organism, but it is invasive and rarely performed today. A nasal swab culture of the nares may isolate the causative organism, but it also will include many other normal flora. With the initiation of the pneumococcal conjugate vaccine, nontypeable Haemophilus influenzae has replaced Streptococcus pneumoniae as the predominant bacterial pathogen in ABRS. When determining appropriate antibiotic choices for the treatment of ABRS, the clinician should consider the most likely pathogen, recent antibiotic use, and local antibiotic resistance rates. Depending on the organism, 15% to 80% of ABRS cases resolve spontaneously without treatment. Most clinicians, however, are reluctant to use a "watch-and-wait" approach after 10 to 14 days of unremitting symptoms. Initial recommended oral therapies include high-dose amoxicillin (90 mg/kg per day), amoxicillin/clavulanate, cefdinir, cefpodoxime proxetil, or cefuroxime axetil. Patients who have had recent antibiotic therapy or who attend child care may require amoxicillin/clavulanate or intramuscular ceftriaxone as initial therapy. Many adjunctive therapies for ABRS are available, including nasal corticosteroids, oral decongestants, cough expectorants, pine oil, nasal saline rinses, and nasal decongestants. However, none has shown clinical benefit.

References:

American Academy of Pediatrics Subcommittee on Management of Sinusitis and Committee on Quality Improvement. Clinical practice guideline: management of sinusitis. Pediatrics. 2001;108:798-808. Available at: http://pediatrics.aappublications.org/cgi/content/full/108/3/798

Brook I, Foote PA, Hausfeld JN. Frequency of recovery of pathogens causing acute maxillary sinusitis in adults before and after introduction of vaccination of children with the 7-valent pneumococcal vaccine. J Med Microbiol. 2006;55:943-946. Available at: http://jmm.sgmjournals.org/cgi/content/full/55/7/943

Taylor A, Adam HM. In brief: sinusitis. Pediatr Rev. 2006;27:395-397. Available at: http://pedsinreview.aappublications.org/cgi/content/full/27/10/395

Page 759: AAP PREP 2009

2009 PREP SA on CD-ROM

page 759

Question: 224

A 5-year-old boy is brought to the emergency department following 2 days of headache, nausea, and vomiting. His mother reports that he has had no fever or diarrhea and that everyone at home, "including the dog," has the same symptoms. Physical examination demonstrates a heart rate of 120 beats/min, respiratory rate of 24 breaths/min, blood pressure of 100/60 mm Hg, and oxygen saturation of 100% on room air. The boy is mildly irritable, and his mucous membranes appear bright red. His lungs are clear, and abdominal examination findings are unremarkable. As you are completing your evaluation, the mother tells you that the furnace in their house has been malfunctioning.

Of the following, the MOST appropriate next step is to

A. administer 2 L of oxygen by nasal cannula

B. arrange for emergent hyperbaric oxygen therapy

C. monitor the patient’s oxygen saturation continuously with pulse oximetry

D. obtain a venous carboxyhemoglobin measurement

E. obtain arterial blood gases

Page 760: AAP PREP 2009

2009 PREP SA on CD-ROM

page 760

Critique: 224 Preferred Response: D

Carbon monoxide (CO) is an odorless, colorless gas that, when inhaled in a closed space, leads to nonspecific clinical symptoms that can be mistaken easily for those seen in a variety of unrelated medical conditions. Because unrecognized CO poisoning can lead to significant morbidity, including delayed cognitive and focal neurologic deficits, as well as mortality, physicians should have a high index of suspicion for this diagnosis when examining patients who have vague, "flulike" symptoms. This is especially important if there is no accompanying fever or others in the same environment have similar symptoms, such as described in the vignette. Although a "cherry red" appearance of the skin and mucous membranes traditionally was believed to be a clue to the diagnosis, this now is recognized as a highly insensitive sign. CO poisoning results from CO binding to hemoglobin 250 times more readily than oxygen. The resultant carboxyhemoglobin (COHgb) is unable to transport oxygen to tissues and interferes with oxygen dissociation from the remaining oxyhemoglobin. Tissue hypoxia results, with effects manifested in every organ system. Central nervous system (headache, confusion, lethargy, dizziness, coma), pulmonary (dyspnea on exertion, tachypnea), cardiovascular (tachycardia, palpitations), and gastrointestinal (nausea and vomiting) symptoms predominate. Evaluation and treatment of CO poisoning include removal of the patient from the contaminated environment, patient stabilization, high-flow oxygen administration, and measurement of a COHgb concentration. A COHgb value of greater than 3% to 5% in a nonsmoker is indicative of exposure. Although symptomatic patients typically have values greater than 10%, such values do not correlate well with type or magnitude of symptoms. Pulse oximetry cannot distinguish between oxyhemoglobin and COHgb, so oxygen saturation measurements typically are normal. Arterial blood gases should be measured in those who have severe symptoms to quantify the degree of acidosis, an indicator of anaerobic metabolism in the face of poor tissue oxygenation. The goal of oxygen therapy in the treatment of CO poisoning is to convert COHgb rapidly to oxyhemoglobin. The administration of 100% oxygen using a nonrebreather mask can decrease the half-life of COHgb from 300 minutes to 90 minutes and should be continued until the COHgb concentration is less than 5%. Although hyperbaric oxygen can decrease the COHgb half-life further to 30 minutes, its benefit in improving patient outcome has not been demonstrated clearly. At present, expert consensus guidelines recommend its use only for severe intoxications.

References:

Clardy PF, Manakar S. Carbon monoxide poisoning. UpToDate Online 15.3. 2008. Available for subscription at: http://www.utdol.com/utd/content/topic.do?topicKey=ad_tox/2932&selectedTitle=1~26&source=search_result

Juurlink DN, Buckley NA, Stanbrook MB, Isbister GK, Bennett M, McGuigan MA. Hyperbaric oxygen for carbon monoxide poisoning. Cochrane Database Syst Rev. 2005;1:CD002041. Available at: http://www.mrw.interscience.wiley.com/cochrane/clsysrev/articles/CD002041/frame.html

Kind T, Etzel RA. In brief: carbon monoxide. Pediatr Rev. 2005;26:150-151. Available at: http://pedsinreview.aappublications.org/cgi/content/full/26/4/150

Page 761: AAP PREP 2009

2009 PREP SA on CD-ROM

page 761

Question: 225

A 15-year-old girl presents with a history of abdominal pain. She is a competitive runner and has required frequent ibuprofen for treatment of knee pain. On physical examination, you note epigastric tenderness. Fecal occult blood test results are positive.

Of the following, the test that is MOST likely to provide a definitive diagnosis is

A. abdominal computed tomography scan

B. Helicobacter pylori serology

C. serum amylase measurement

D. upper endoscopy with biopsies

E. upper gastrointestinal radiographic series

Page 762: AAP PREP 2009

2009 PREP SA on CD-ROM

page 762

Critique: 225 Preferred Response: D

The epigastric pain, use of nonsteroidal anti-inflammatory agents, and guaiac-positive stool described for the patient in the vignette suggest the possibility of peptic ulcer disease. Peptic ulcers are erosions of the gastric or duodenal mucosa that disrupt the epithelium and predispose to abdominal pain or gastrointestinal bleeding. The most common predisposing factors placing patients at risk for the development of gastric ulcers are Helicobacter pylori infection, medications (particularly chemotherapeutic agents or nonsteroidal anti-inflammatory drugs), Crohn disease, vasculitis, or severe illness requiring intensive care unit hospitalization. Ulcers also may be idiopathic and without an identifiable inflammatory or infectious cause. If peptic ulcer is suspected and definitive diagnosis is required, the test of choice is upper gastrointestinal endoscopy or esophagogastroduodenoscopy (EGD). EGD is superior to other imaging modalities such as ultrasonography or abdominal computed tomography scan for identifying mucosal lesions and allows for biopsy of the stomach to evaluate for Crohn disease or H pylori. If a patient has a bleeding ulcer (Item C225) that has a visible vessel, therapeutic endoscopy (in which the bleeding area is injected with epinephrine or cauterized) can be performed at the same time as the diagnostic procedure. Because endoscopy is an invasive test that requires sedation or anesthesia, the benefits of the procedure need to be weighed against the risks. In particular, the endoscopy should be undertaken with caution in a patient who has a bleeding disorder or a critically ill patient who has gastrointestinal bleeding. In such patients, empiric therapy with proton pump antagonists and hemodynamic stabilization should be attempted before endoscopy is performed. H pylori serology and serum amylase measurement cannot diagnose peptic ulcer disease. Upper gastrointestinal radiographic series and abdominal computed tomography scan have poor sensitivity for ulcers.

References:

De Giacomo C. Helicobacter pylori gastritis and peptic ulcer disease. In: Guandalini S, ed. Textbook of Pediatric Gastroenterology and Nutrition. London, England: Taylor & Francis; 2004:73-94

Fox VL. Pediatric endoscopy. Gastrointest Endosc Clin North Am. 2000;10: 175-194. Abstract available at: http://www.ncbi.nlm.nih.gov/pubmed/10618461

Page 763: AAP PREP 2009

2009 PREP SA on CD-ROM

page 763

Critique: 225

Bleeding duodenal ulcer (arrow). (Courtesy of A. Bousvaros)

Page 764: AAP PREP 2009

2009 PREP SA on CD-ROM

page 764

Question: 226

You are seeing a 30-year-old multigravid woman for prenatal counseling. She has had immune thrombocytopenic purpura for the past 5 years, and her spleen was removed 2 years ago. She asks you about the effects that her disease might have on her unborn child.

Of the following, you are MOST likely to tell her that

A. if her newborn has thrombocytopenia, he or she will be treated with intravenous immunoglobulin

B. maternal platelet counts predict fetal risks of intracranial hemorrhage

C. maternal platelet transfusion during pregnancy will minimize the risk for neonatal thrombocytopenia

D. operative delivery of the newborn will reduce the risk of intracranial hemorrhage

E. the newborn will require a platelet transfusion soon after birth

Page 765: AAP PREP 2009

2009 PREP SA on CD-ROM

page 765

Critique: 226 Preferred Response: A

The fact that the mother described in the vignette has immune thrombocytopenia is noteworthy. Because this condition is associated with immunoglobulin G (IgG) directed against maternal platelet antigens, transplacental acquisition of these antibodies by the fetus may occur after 30 weeks' gestation. As such, maternal-fetal surveillance is recommended, but no direct correlation exists between maternal platelet counts and fetal platelet counts or the risk of intracranial hemorrhage. Maternal platelet transfusion during pregnancy may help avoid concerns for the mother, but not the fetus or newborn. Although the risk for intracranial hemorrhage is only about 5% overall, it is greatest when fetal platelet counts are less than 20.0x103/mcL (20.0x109/L) because spontaneous intracranial hemorrhage may occur. An operative delivery of the newborn does not reduce the risk of intracranial hemorrhage significantly. Treatment of the newborn generally is supportive, with attention to the risk of thrombocytopenia significant enough to warrant platelet transfusion. Ninety percent of affected newborns require no treatment. However, platelet transfusion may be in order for any infant of a mother who has immune thrombocytopenic purpura when the neonate exhibits bleeding or has a platelet count of less than 20.0x103/mcL (20.0x109/L). The best treatment for affected infants is intravenous immunoglobulin (IVIg), which blocks circulating maternal IgG directed against platelet antigens and allows the newborn's platelet counts to rise. Platelet transfusion for the newborn is less effective than IVIg and may risk exposure to platelet-associated antigens that could induce a postnatal immune thrombocytopenia.

References:

Buyon JP, Nugent D, Mellins E, Sandborg C. Maternal immunologic diseases and neonatal disorders. NeoReviews. 2002;3:e3-e10. Available for subscription at: http://neoreviews.aappublications.org/cgi/content/full/3/1/e3

Murphy MF, Bussel JB. Advances in the management of alloimmune thrombocytopenia. Br J Haematol. 2007;136:366-378. Abstract available at: http://www.ncbi.nlm.nih.gov/pubmed/17233844

Wong W, Glader B. Approach to the newborn who has thrombocytopenia. NeoReviews. 2004;5:e444-e450. Available for subscription at: http://neoreviews.aappublications.org/cgi/content/full/5/10/e444

Page 766: AAP PREP 2009

2009 PREP SA on CD-ROM

page 766

Question: 227

You are addressing a group of expectant mothers who are due to deliver their infants in the next few weeks. You discuss the benefits of breastfeeding and explain that it is the best nutrition for most babies. One woman asks you if it is acceptable to breastfeed if she has had hepatitis in the past. You explain that there are only a few infections that would prevent a mother from being able to breastfeed her baby.

Of the following, breastfeeding is MOST likely to be contraindicated if a mother

A. has active untreated pulmonary tuberculosis

B. has genital herpes without breast lesions

C. is a cytomegalovirus carrier

D. is hepatitis B surface antigen-positive

E. is hepatitis C antibody-positive

Page 767: AAP PREP 2009

2009 PREP SA on CD-ROM

page 767

Critique: 227 Preferred Response: A

Human milk is the optimal nutrition for infants. Benefits include transference of protective maternal antibodies, improved bonding between mother and child, and probable improvement in cognitive and developmental function of the infant. Most mothers can breastfeed successfully, although there are contraindicated conditions for both infants and mothers. Infants who have galactosemia should not receive human milk, and infants who have other forms of metabolic disease, such as urea cycle defects or phenylketonuria, may receive only a limited amount. Mothers infected with human immunodeficiency virus (in the United States) or human T-cell lymphotrophic virus-1 or -2 and those who have active untreated tuberculosis or active herpes lesions on the breast should not breastfeed their infants. However, mothers who are hepatitis B surface antigen-positive or hepatitis C antibody-positive, are cytomegalovirus carriers, or have genital herpes without breast lesions can breastfeed safely. Maternal medications that preclude breastfeeding include antineoplastic agents, immunosuppressants, lithium, and radiopharmaceutical agents. Several disorders of the breast may make breastfeeding difficult, but they are not contraindications to breastfeeding. Previous breast surgery may cause ineffective lactation, but this varies among mothers. Women who have inverted or flat nipples may experience difficulties with latch-on, but this can be improved with early feedings, use of nipple shields, and lactation consultation. Use of a breast pump also may help. Women who have breast cancer may be able to breastfeed if they are not taking antineoplastic medications. Mastitis, inflammation of the breast usually caused by obstruction of ducts, may make breastfeeding painful, but more frequent nursing is the best recommendation to help resolve this condition. Sore or cracked nipples may develop, especially if the infant has oral-motor dysfunction, and adjusting the infant's latch-on may improve these symptoms. A mother who has Candida infection of the breast may continue to breastfeed, but both she and her infant should be treated for the infection to avoid a cycle of reinfection.

References:

American Academy of Pediatrics Section on Breastfeeding. Breastfeeding and the use of human milk. Pediatrics. 2005;115:496-506. Available at: http://pediatrics.aappublications.org/cgi/content/full/115/2/496

Chandran L, Gelfer P. Breastfeeding: the essential principles. Pediatr Rev. 2006;27:409-417. Available at: http://pedsinreview.aappublications.org/cgi/content/full/27/11/409

Powers NG, Slusser W. Breastfeeding update 2: clinical lactation management. Pediatr Rev. 1997;18:147-161. Available at: http://pedsinreview.aappublications.org/cgi/content/full/18/5/147

Page 768: AAP PREP 2009

2009 PREP SA on CD-ROM

page 768

Question: 228

A 6-month-old girl, who was born in Nigeria, presents for an urgent visit as soon as the family arrives in the United States because of fever and irritability. Physical examination reveals a fussy infant who has anorexia, a temperature of 100°F (37.8°C), and swelling of all of the fingers of the right hand (Item Q228). The remainder of the examination findings are negative.

Of the following, the MOST likely cause of this pattern of swelling in this child is

A. cellulitis

B. juvenile idiopathic arthritis

C. malaria

D. sickle cell disease

E. trauma

Page 769: AAP PREP 2009

2009 PREP SA on CD-ROM

page 769

Question: 228

Swelling of the digits, as exhibited by the infant in the vignette. (Courtesy of M. Rimsza)

Page 770: AAP PREP 2009

2009 PREP SA on CD-ROM

page 770

Critique: 228 Preferred Response: D

Sickle cell disease (SCD) usually is diagnosed in the United States in early infancy because of mandated newborn screening. If screening is not performed, the diagnosis may be delayed until the development of symptoms from a vaso-occlusive crisis. In infants and toddlers, the first vaso-occlusive crisis may be heralded by painful and swollen hands or feet, also known as dactylitis or "hand and foot syndrome." The examination findings for the child described in the vignette suggest the diagnosis of dactylitis that is most likely due to SCD. Dactylitis due to SCD usually presents with swelling and tenderness of the hands (Item C228) or feet that is self-limited and resolves in 5 to 31 days. Fever and leukocytosis also may be noted. Radiographs of the hands and feet taken 7 to 14 days after the swelling begins may reveal periosteal new bone formation or intramedullary densities. Such lesions resolve in 2 to 3 months. Diagnostic confusion with cellulitis or osteomyelitis (which rarely may occur in conjunction with dactylitis) is frequent, but dactylitis often involves most, if not all, of the digits equally and is not accompanied by overlying cellulitis of the skin. Trauma generally does not involve all digits equally without contusion or laceration of the overlying skin. Arthritis, including juvenile idiopathic arthritis, usually affects only a few joints and is uncommon in this age group. Malaria causes fever and anemia but does not commonly cause dactylitis or other extremity findings. Dactylitis may be seen in other conditions, including psoriasis, insect bites with angioedema, and other conditions that are more common in older children and adults.

References:

American Academy of Pediatrics Section on Hematology/Oncology and Committee on Genetics. Health supervision for children with sickle cell disease. Pediatrics. 2002;109:526-535. Available at: http://pediatrics.aappublications.org/cgi/content/full/109/3/526

Gill FM, Sleeper LA, Weiner SJ, et al for the Cooperative Study of Sickle Cell Disease. Clinical events in the first decade in a cohort of infants with sickle cell disease. Blood. 1995;86:776-783. Available at: http://bloodjournal.hematologylibrary.org/cgi/reprint/86/2/776

Meremikwu MM. Sickle cell disease (updated). BMJ Clinical Evidence. 2007. Available for subscription at: http://clinicalevidence.bmj.com/ceweb/conditions/bly/2402/2402.jsp

Miller ST, Sleeper LA, Pegelow CH, et al. Prediction of adverse outcomes in children with sickle cell disease. N Engl J Med. 2000:342:2:83-89. Abstract available at: http://www.ncbi.nlm.nih.gov/pubmed/10631276

Sickle-cell dactylitis. In: Wheeless' Textbook of Orthopaedics. Available at: http://www.wheelessonline.com/ortho/sickle_cell_dactylitis

Page 771: AAP PREP 2009

2009 PREP SA on CD-ROM

page 771

Critique: 228

Vaso-occlusive crises in infants who have sickle cell disease often are characterized by swelling and tenderness of the fingers or toes. (Courtesy of M. Rimsza)

Page 772: AAP PREP 2009

2009 PREP SA on CD-ROM

page 772

Question: 229

You work as a voluntary attending pediatrician in the resident continuity clinic at your local hospital. You are precepting a resident, who tells you that she has just evaluated a 16-year-old varsity volleyball player. The girl’s height is 71 inches, weight is 125 lb, and blood pressure is 115/74 mm Hg. The resident is concerned about scoliosis and a 3/6 holosystolic murmur heard at the cardiac apex with radiation to the left axilla (Item Q229).

Of the following, the MOST likely diagnosis for this patient is

A. Ehlers—Danlos syndrome

B. infective endocarditis

C. Marfan syndrome

D. rheumatic heart disease

E. Williams syndrome

Page 773: AAP PREP 2009

2009 PREP SA on CD-ROM

page 773

Critique: 229 Preferred Response: C

The patient in the vignette has several physical examination findings that suggest the diagnosis of Marfan syndrome, including scoliosis, tall stature, and a holosystolic murmur that could be due to mitral valve disease. The murmur is heard at the cardiac apex and is of the quality and location typical for mitral regurgitation (Item C229A), a common result of the prolapsing and redundant valve. The patient should be referred to a specialist, usually a geneticist, who will use the Ghent criteria to determine if the child has Marfan syndrome. Marfan syndrome is an autosomal dominant connective tissue disorder that is caused by mutations in the fibrillin gene on chromosome 15. The abnormal fibrillin affects the organization of extracellular myofibrils that support the formation of elastin networks in connective tissue. The abnormal fibrillin is associated with variable cardiovascular, skeletal, and ocular features. Not all people who have Marfan syndrome have cardiovascular involvement, but the blood vessels, cardiac valves, and myocardium all contain substantial amounts of fibrillin. Cardiovascular involvement in children most commonly results in dilation of the aortic root at the sinuses of Valsalva and abnormality of the atrial ventricular valves. The mitral valve frequently is affected and demonstrates dysplasia, prolapse, and dysfunction, as may the tricuspid valve. The skeletal findings of Marfan syndrome may include scoliosis, a tall and thin habitus, pectus deformities (Item C229B), arachnodactyly (Item C229C), and hypermobility of the joints. The ocular findings may include ectopia lentis (lens dislocation) (Item C229D). Often, the arm span-to-height ratio exceeds 1.05, and the upper-to-lower segment ratio is less than 0.88. Patients in whom Marfan syndrome is diagnosed or highly suspected should be counseled to avoid participation in competitive contact sports. Avoidance of contact sports is important because there is an increased risk of cardiac, skeletal, and ophthalmologic problems resulting from injury. A specific potential problem with the cardiovascular system is tearing of the weakened, abnormal aortic wall, which can lead to a catastrophic outcome. It is important to remember that sports practice often can be as competitive and intense as the games; both should be avoided. Consultation with an experienced pediatric cardiologist is important, and echocardiographic imaging of the heart is essential to evaluate for the possibility of cardiovascular involvement. The patient described in the vignette does not have the findings of either Ehlers-Danlos or Williams syndromes. She has no findings to suggest infective carditis or rheumatic fever/heart disease.

References:

Maron BJ, Chaitman BR, Ackerman MJ, et al. Recommendations for physical activity and recreational sports participation for young patients with genetic cardiovascular diseases. Circulation. 2004;109:2807-2816. Available at: http://circ.ahajournals.org/cgi/content/full/109/22/2807

Maron BJ, Thompson PD, Ackerman MJ, et al. AHA scientific statements. Recommendations and considerations related to preparticipation screening for cardiovascular abnormalities in competitive athletes: 2007 update. A scientific statement from the American Heart Association Council on Nutrition, Physical Activity, and Metabolism: endorsed by the American College of Cardiology Foundation. Circulation. 2007;115:1643-1655. Available at: http://circ.ahajournals.org/cgi/content/full/115/12/1643

Moodie DS. AAP: health supervision for children with Marfan syndrome. Clin Pediatr (Phila). 1997;36:489

Peirpont MEM. Connective tissue diseases. In: Moller JH, Hoffman JIE, eds. Pediatric Cardiovascular Medicine. Philadelphia, Pa: Churchill Livingstone; 2000:901-912

Page 774: AAP PREP 2009

2009 PREP SA on CD-ROM

page 774

von Kodolitsch Y, Robinson PN. Marfan syndrome: an update of genetics, medical and surgical management. Heart. 2007;93:755-760. Extract available at: http://heart.bmj.com/cgi/content/extract/93/6/755

Page 775: AAP PREP 2009

2009 PREP SA on CD-ROM

page 775

Critique: 229

Pectus carinatum in a patient who has Marfan syndrome. (Courtesy of T. Jewett)

Page 776: AAP PREP 2009

2009 PREP SA on CD-ROM

page 776

Critique: 229

Arachnodactyly is a feature of Marfan syndrome. (Courtesy of M. Rimsza)

Page 777: AAP PREP 2009

2009 PREP SA on CD-ROM

page 777

Critique: 229

Subluxed lens: The lens is displaced inferiorly (arrows show the border of the lens). In Marfan syndrome, the lens typically is displaced superiorly and temporally. (Courtesy of the Wake Forest University Eye Center)

Page 778: AAP PREP 2009

2009 PREP SA on CD-ROM

page 778

Question: 230

The parents of a 6-month-old previously well infant bring her to your office. She had been developing normally, but she stopped interacting with her parents over the last 24 hours. For several days prior to this development, she had had unusual spells during which her head and chin dropped to her chest. Now she is having clusters of these spells involving head drop and body flexion. On physical examination, there is no bruising. The infant is afebrile and alert, her tone is low, and she does not make persistent eye contact or track visually. You refer her to the emergency department, where results of a complete blood count, electrolyte panel, urinalysis, and a noncontrast head computed tomography scan are normal.

Of the following, the test that is MOST likely to reveal the correct diagnosis is

A. electroencephalography

B. electroretinography

C. lumbar puncture

D. muscle biopsy

E. serum lactate measurement

Page 779: AAP PREP 2009

2009 PREP SA on CD-ROM

page 779

Critique: 230 Preferred Response: A

The infant described in the vignette has an encephalopathy, a "confusional" state. It is critical to consider focal, ischemic, ictal, infectious, and toxic/metabolic causes for any patient presenting with mental status changes. The noncontrast head computed tomography scan and laboratory tests were appropriate initial decisions in the emergency department to rule out emergent, treatable problems. For this infant, the key to the diagnosis is recognition that the cluster of body spasms combined with the loss of developmental milestones is characteristic of infantile spasms, which should be assessed by electroencephalography (EEG). EEG is used to identify the neurophysiologic pattern that confirms the diagnosis of infantile spasms. The characteristic diffuse, slow, disorganized, and high-amplitude pattern is known as "hypsarrhythmia." Infantile spasms describes a syndrome of epilepsy and encephalopathy in infancy, with a peak onset at 6 months. The spasms (Item C230) can range from subtle, quick movements involving just the head to dramatic "clasp-knife" full body spasms. Infantile spasms can have a number of causes or can be idiopathic. The prognosis is often poor, particularly if the child's neurologic development was abnormal prior to the onset of spasms. Potential causes of the infantile spasms may be structural, genetic, or metabolic. Electroretinography (ERG) is used to assess the neurophysiology of the retina. For example, ERG responses are abnormal in Leber congenital amaurosis and various rod/cone dystrophies. This child's new visual impairment associated with hypotonia and spasms is not likely related to retinal pathology, but more likely is due to a central nervous system problem causing abnormal visual processing. A lumbar puncture should be considered in an infant who has fever and an acute encephalopathy to rule out infectious causes, but establishing the diagnosis of infantile spasms is more important in this case. A lumbar puncture, if it is obtained, might be more valuable as part of a metabolic evaluation, as would measurement of serum lactate and a muscle biopsy, if needed to assess for the possibility of a mitochondrial disorder.

References:

Kossoff EH. Infantile spasms. In: Singer HS, Kossoff EH, Hartman AL, Crawford TO, eds. Treatment of Pediatric Neurologic Disorders. Boca Raton, Fla: Taylor & Francis; 2005:111-116

Mackay MT, Weiss SK, Adams-Webber T, et al. Practice parameter: medical treatment of infantile spasms. Report of the American Academy of Neurology and the Child Neurology Society. Neurology. 2004;62:1668-1681. Available at: http://www.neurology.org/cgi/content/full/62/10/1668

Page 780: AAP PREP 2009

2009 PREP SA on CD-ROM

page 780

Question: 231

While examining an infant in the newborn nursery, you note that the pupil of one eye seems abnormally large, and little of the iris is visible. The baby appears otherwise normal. A subsequent ophthalmologic evaluation confirms the diagnosis of partial aniridia.

Of the following, the MOST accurate statement regarding the diagnosis is that

A. aniridia is associated with hepatoblastoma

B. few individuals who have aniridia have an affected parent

C. isolated aniridia has an autosomal recessive inheritance pattern

D. molecular testing is available to determine risk for Wilms tumor

E. routine abdominal ultrasonography should be performed every 3 months until age 5 years in affected individuals

Page 781: AAP PREP 2009

2009 PREP SA on CD-ROM

page 781

Critique: 231 Preferred Response: D

Aniridia is defined as complete or partial iris hypoplasia (Item C231), often associated with foveal hypoplasia and reduced visual acuity. Sometimes it can be detected in the newborn period with the use of direct ophthalmoscopy; the red reflex may appear unusual in shape or is too large due to the lack of intervening iris tissue. Infants who have such a finding should be referred to ophthalmology for confirmation and further delineation of the defect. The ophthalmologist plays a major role in the management of affected children. Isolated aniridia is an autosomal dominant trait, and most affected individuals have an affected parent. It is important to note that aniridia is associated with Wilms tumor in some individuals. It is now possible to evaluate a blood specimen from an individual who has aniridia to determine if he or she has a PAX6 gene mutation, which is detectable in 90% of isolated cases. If a PAX6 mutation is detected, Wilms tumor is not a concern. However, if no mutation of PAX6 is detected, the aniridia could be part of a condition, such as aniridia-Wilms tumor or the Wilms tumor-aniridia-genital anomalies-retardation (WAGR) syndrome. These conditions are caused by genes adjacent to PAX6, which, when deleted, result in aniridia plus other anomalies. In such cases, it is important to follow a regular surveillance protocol for Wilms tumor that includes regular urinalysis and routine renal ultrasonography. Wilms tumor also occurs with increased frequency in conditions such as Beckwith-Wiedemann syndrome (BWS) and hemihypertrophy. In these conditions, published surveillance protocols for the development of associated neoplasms should be followed. Aniridia is not associated with an increased risk for hepatoblastoma, although BWS and hemihypertrophy are.

References:

Clericuzio C. Recognition and management of childhood cancer syndromes: a systemic approach. Am J Med Genet. 1999;25:89:81-90. Abstract available at: http://www.ncbi.nlm.nih.gov/pubmed/10559762

Hingorani M, Moore A. Aniridia. GeneReviews. 2005. Available at: http://www.geneclinics.org/servlet/access?db=geneclinics&site=gt&id=8888891&key=xAcWBcrjmZrVo&gry=&fcn=y&fw=581E&filename=/profiles/aniridia/index.html

Lapunzina P. Risk of tumorigenesis in overgrowth syndromes: a comprehensive review. Am J Med Genet C Semin Med Genet. 2005;137:53-71. Abstract available at: http://www.ncbi.nlm.nih.gov/pubmed/16010678

Page 782: AAP PREP 2009

2009 PREP SA on CD-ROM

page 782

Critique: 231

Aniridia appears as a large pupil with little iris. (Courtesy of the Media Lab at Doernbecher)

Page 783: AAP PREP 2009

2009 PREP SA on CD-ROM

page 783

Question: 232

A 15-year-old girl presents with vaginal pain and burning accompanied by feelings of warmth and generalized muscle aches for the past 24 hours. She has had no previous similar symptoms. She is sexually active and does not use barrier methods for contraception. On physical examination, you find multiple shallow ulcers of the labia minora that are surrounded by erythema and are exquisitely tender to touch (Item Q232). There is no vaginal discharge.

Of the following, the most appropriate treatment is

A. acyclovir 5 to 10 mg/kg intravenously every 8 hours for 2 to 7 days

B. acyclovir 400 mg orally once a day for 1 year

C. acyclovir 400 mg orally 3 times a day for 7 to 10 days

D. acyclovir 800 mg orally twice a day for 2 days

E. no antiviral chemotherapy

Page 784: AAP PREP 2009

2009 PREP SA on CD-ROM

page 784

Question: 232

(Courtesy of M. Rimsza)

Page 785: AAP PREP 2009

2009 PREP SA on CD-ROM

page 785

Critique: 232 Preferred Response: C

Most genital herpes simplex virus (HSV) infections are caused by HSV type 2 (HSV-2), although the prevalence of HSV type 1 (HSV-1) appears to be increasing. Primary infection with either of the two types may go undetected, but the labial lesions (Item C232), systemic symptoms of generalized muscle aches, and no previous symptoms described for the girl in the vignette suggest that this is a primary HSV infection. The clinical diagnosis should be confirmed by type-specific laboratory testing. Recurrences and subclinical shedding are much less frequent for genital HSV-1 infection than HSV-2 infection, and the prognosis and counseling can be influenced by the type detected. Antiviral therapy for a first clinical episode of genital herpes is recommended, even if manifestations are mild, because some patients develop severe or prolonged symptoms at a later time. A 7- to 10-day course of oral acyclovir (400 mg three times a day)is recommended. Famciclovir and valacyclovir are other treatment options.

References:

American Academy of Pediatrics. Herpes simplex. In: Pickering LK, Baker CJ, Long SS, McMillan JA, eds. Red Book: 2006 Report of the Committee on Infectious Diseases. 27th ed. Elk Grove Village, Ill: American Academy of Pediatrics; 2006:361-371

Workowski KA, Berman SM, Centers for Disease Control and Prevention. Sexually transmitted diseases treatment guidelines, 2006. MMWR Recomm Rep. 2006;55(RR11):1-94. Available at: http://www.cdc.gov/mmwr/preview/mmwrhtml/rr5511a1.htm

Page 786: AAP PREP 2009

2009 PREP SA on CD-ROM

page 786

Critique: 232

Infection of the genitalia with herpes simplex virus produces painful ulcers. (Courtesy of M. Rimsza)

Page 787: AAP PREP 2009

2009 PREP SA on CD-ROM

page 787

Question: 233

A 16-year-old boy presents to the emergency department with an acute change in his mental status. According to his parents, he was previously healthy and has suffered no recent trauma. On physical examination, he is somnolent, has pinpoint pupils and mild hypotension, and demonstrates shallow breathing.

Of the following, the test that is MOST likely to help determine the cause of his altered level of consciousness is

A. brainstem auditory evoked response

B. chest radiography

C. electroencephalography

D. serum amino acid measurement

E. urine toxicology screen

Page 788: AAP PREP 2009

2009 PREP SA on CD-ROM

page 788

Critique: 233 Preferred Response: E

Evaluation of the patient who has an altered level of consciousness consists of a thorough medical history, a detailed physical examination, and appropriate diagnostic studies. A sudden onset over minutes in a previously healthy individual suggests trauma or a cerebrovascular accident; effects of toxic ingestions may present over several hours; and a more gradual onset suggests infection, metabolic disease, or a space-occupying lesion. Vital signs, level of consciousness, pupil size and reaction, motor responses, and skin findings can be crucial in narrowing potential causes and determining appropriate diagnostic studies. Diagnostic studies might include blood and urine testing (for potential metabolic, infectious, and toxic causes), lumbar puncture, computed tomography scan and magnetic resonance imaging (with or without angiography), brainstem evoked potentials, electroencephalography, and electrocardiography. The teenager described in the vignette has had an acute mental status change, and his somnolence, hypotension, shallow and slow respirations, and pinpoint pupils are consistent with an opiate ingestion. Therefore, toxicology screens for confirmation as well as to evaluate for coingestions are indicated. Evoked potentials may be useful in testing the integrity of the brainstem and in providing prognostic information for comatose patients, but results are nonspecific and rarely helpful for initial diagnosis. Chest radiography generally is of little value in the assessment of a comatose patient. Although seizures may produce an altered level of consciousness, no suggestion of seizure activity is present in this patient that would necessitate electroencephalography. The patient's age makes an inborn error of metabolism unlikely, and serum amino acid measurement can be deferred.

References:

Avner JR. Altered states of consciousness. Pediatr Rev. 2006:27:331-338. Available at: http://pedsinreview.aappublications.org/cgi/content/full/27/9/331

Frankel LR. Neurological emergencies and stabilization. In: Kliegman RM, Behrman RE, Jenson HB, Stanton BF, eds. Nelson Textbook of Pediatrics. 18th ed. Philadelphia, Pa: Saunders Elsevier; 2007:405-412

Page 789: AAP PREP 2009

2009 PREP SA on CD-ROM

page 789

Question: 234

The parents of a 3-year-old boy in whom you recently diagnosed type 1 diabetes mellitus are anxious about providing the best diabetes control for their son, but wish to avoid frequent fingersticks to measure blood glucose. They have read that a hemoglobin A1c gives a measure of blood glucose control and correlates with long-term complications of diabetes. They request that this blood test be obtained at weekly intervals to give them assurance of good control.

Of the following, the MOST important information to provide them about hemoglobin A1c measurement is that it

A. can replace self blood glucose monitoring

B. is not useful in children younger than 5 years of age

C. should be obtained every month

D. should be obtained every 2 to 3 months

E. should be obtained every 6 months

Page 790: AAP PREP 2009

2009 PREP SA on CD-ROM

page 790

Critique: 234 Preferred Response: D

The hemoglobin A1c (HbA1c) is a measure of nonenzymatic glycosylation of hemoglobin A and correlates with concentrations of circulating blood glucose over the life of the red blood cell. It is not a useful measure of daily changes in blood glucose or of amplitude of excursion of blood glucose over the course of a day. Because the red cell has a life of approximately 120 days, measurement of HbA1c more frequently than every 2 months is not likely to be helpful in clinical management. In general, this measurement is made every 2 to 3 months. Results permit broad adjustments in insulin therapy and monitoring of large trends in management. If the HbA1c is measured less frequently, glycemic control may deteriorate without recognition. As children have been reported to say, it is "the test that does not lie." It is useful in a child of any age, although the presence of fetal hemoglobin in very young children may make interpretation of the results more difficult. In addition, measurement of HbA1c in children who have hemoglobinopathies may not be entirely accurate. Instead, measurement of total glycosylated hemoglobin can be substituted. For children who have rapid red cell destruction, the published relationship between serum glucose and HbA1c may be inaccurate. Self blood glucose monitoring gives a relatively dynamic picture of blood glucose changes over the course of a day and allows decisions to be made about adjustment of basal insulin requirements as well as adjustments for meals, exercise, and other life activities. A combination of self blood glucose monitoring and periodic measurement of HbA1c permits reasoned decisions and improved glycemic control in this most difficult disorder.

References:

McCulloch DK. Estimation of blood glucose control in diabetes mellitus. UpToDate Online 15.3. 2008. Available for subscription at: http://www.uptodateonline.com/utd/content/topic.do?topicKey=diabetes/7913

McCulloch DK. Glycemic control and vascular complications in type 1 diabetes. UpToDate Online 15.3. 2008. Available for subscription at: http://www.uptodateonline.com/utd/content/topic.do?topicKey=diabetes/10573

Silverstein J, Klingensmith G, Copeland K, et al, Care of children and adolescents with type 1 diabetes: a statement of the American Diabetes Association. Diabetes Care. 2005;28:186-212. Available at: http://care.diabetesjournals.org/cgi/content/full/28/1/186

Page 791: AAP PREP 2009

2009 PREP SA on CD-ROM

page 791

Question: 235

A mother brings in her child for a health supervision visit. He is able to pull to stand, take a few independent steps, and use two fingers to grasp pieces of cereal.

Of the following, these developmental milestones are MOST typical for a child whose age is

A. 6 months

B. 9 months

C. 12 months

D. 15 months

E. 18 months

Page 792: AAP PREP 2009

2009 PREP SA on CD-ROM

page 792

Critique: 235 Preferred Response: C

The motor milestones described in the vignette are appropriate for a 12-month-old child. Such children can pull to a stand and cruise, take a few independent steps (Item C235A), and use a neat pincer grasp to handle a raisin or pellet. Motor milestones for a 6-month-old child include sitting with support, rolling back to front (Item C235B), and transferring an object between hands. A 9-month-old child can sit indefinitely, may creep on hands and knees, and can use a radial digital grasp to pick up a cube. A 15-month-old child can run stiff-legged. An 18-month-old child can walk up and down stairs with one hand held, climb on a chair to reach items, scribble spontaneously, turn two or three pages at once in a picture book, drink from a regular cup, and get a spoon to his or her mouth. Parents typically first report delays in motor milestones between the ages of 6 and 12 months. A child's motor developmental milestones are determined from both parental history and direct observation during the neurodevelopmental evaluation. A motor quotient may be determined by dividing the developmental age level by the chronologic age and multiplying it by 100. Gross motor quotients below 50 usually are indicative of cerebral palsy. Motor quotients in the 50 to 75 range may be more indicative of low muscle tone and developmental coordination disorder. Children who have milder motor delays may have comorbid neurodevelopmental disorders such as learning disorders or attention-deficit/hyperactivity disorder.

References:

Accardo PJ, Accardo J, Capute A. A neurodevelomental perspective on the continuum of developmental disabilities. In: Accardo PJ. Capute & Accardo's Neurodevelopmental Disabilities in Infancy and Childhood. Volume I: Neurodevelopmental Diagnosis and Treatment. 3rd ed. Baltimore, Md: Paul H. Brookes Publishing Co; 20083-10

Blasco PA. Motor delays. In: Parker S, Zuckerman B, Augustyn M, eds. Developmental and Behavioral Pediatrics: A Handbook for Primary Care. 2nd ed. Philadelphia, Pa: Lippincott Williams & Wilkins; 2005:242-247

Knobloch H, Stevens FM, Malone AF. The revised developmental stages. In: Manual of Developmental Diagnosis: The Administration and Interpretation of the Revised Gesell and Amatruda Developmental and Neurologic Examination. Albany, NY: Developmental Evaluation Materials, Inc; 1987:17-120

Page 793: AAP PREP 2009

2009 PREP SA on CD-ROM

page 793

Question: 236

A 5-year-old girl presents approximately 96 hours after being bitten by a dog on her leg. Her mother states that she developed fever and swelling of the leg around the bite site over the past 12 hours. Physical examination reveals a nontoxic-appearing girl who has a temperature of 101.8°F (38.8°C) and an open wound with visible purulence and surrounding erythema.

Of the following, the MOST likely pathogen responsible for these symptoms is

A. Eikenella corrodens

B. Kingella kingae

C. Pasteurella multocida

D. Staphylococcus aureus

E. Streptococcus pyogenes

Page 794: AAP PREP 2009

2009 PREP SA on CD-ROM

page 794

Critique: 236 Preferred Response: D

Approximately 10% to 15% of patients who sustain a bite wound from a dog develop infection in the wound. The patient described in the vignette developed an infection 4 days after sustaining her injury and has visible purulence at the bite site. Late infections that produce purulence are usually due to Staphylococcus aureus. Infections with Pasteurella multocida can occur following an animal bite, but they usually develop rapidly (within 24 hours) and exhibit erythema, tenderness, and edema. Eikenella corrodens is associated more commonly with human than animal bites. Although dogs have been found to be colonized with group A Streptococcus (GAS), wound infections with this organism after a bite are uncommon. In addition, GAS is less likely to produce overt purulence than S aureus. Kingella kingae are found in the human oropharynx and are an important pathogen in bone and joint disease among children younger than 5 years of age. References:

American Academy of Pediatrics. Bite wounds. In: Pickering LK, Baker CJ, Long SS, McMillan JA, eds. Red Book: 2006 Report of the Committee on Infectious Diseases. 27th ed. Elk Grove Village, Ill: American Academy of Pediatrics; 2006:191-195

Sagerman PJ. Wounds. Pediatr Rev. 2005;26:43-49. Available at: http://pedsinreview.aappublications.org/cgi/content/full/26/2/43

Page 795: AAP PREP 2009

2009 PREP SA on CD-ROM

page 795

Question: 237

A 10-year-old boy was bitten by a dog 2 days ago while visiting relatives in rural Mexico. He was playing outside with his cousin when a stray dog suddenly ran up and bit him on the arm. After the incident, the dog ran off and could not be found. His mother washed the wound with soap and water, but no other medical attention was sought at that time. Physical examination today reveals a moderately deep bite wound on the boy’s right forearm that is erythematous, mildly indurated, and tender, with seropurulent drainage. You prescribe appropriate antibiotic therapy.

Of the following, the MOST appropriate postexposure prophylaxis regimen for this patient is

A. rabies immune globulin alone

B. rabies immune globulin and rabies vaccine

C. rabies immune globulin and tetanus vaccine

D. rabies vaccine and tetanus vaccine

E. rabies vaccine alone

Page 796: AAP PREP 2009

2009 PREP SA on CD-ROM

page 796

Critique: 237 Preferred Response: B

Infection with the rabies virus produces an acute illness that has rapidly progressive central nervous system findings, including anxiety, dysphagia, seizures, and encephalitis that, in most cases, progresses to death. Postexposure prophylaxis for rabies is recommended for all persons: 1) bitten by wild mammalian carnivores, bats, or domestic animals that may be infected and 2) who report an open wound, scratch, or mucous membrane that has been contaminated with saliva or other potentially infectious material from a rabid animal or human. Prophylaxis should be initiated as soon as possible after bites by known or suspected rabid animals. The goal of postexposure prophylaxis is to prevent virus from entering neural tissue. Prompt local treatment of the wound is critical. All wounds should be flushed thoroughly and cleaned with soap and water. If possible, wounds should not be sutured. The need for tetanus prophylaxis and antibiotic therapy also should be considered. After wound care is completed, concurrent use of passive (human rabies immune globulin) (HRIG) and active (rabies vaccine) immunoprophylaxis is required for optimal therapy. Prophylaxis should begin as soon as possible after exposure, ideally within 24 hours, but prophylaxis still should be initiated if indicated, regardless of the interval between exposure and initiation of therapy. Therefore, the boy described in the vignette should receive both HRIG and rabies vaccine. HRIG should be used concomitantly with the first dose of vaccine for postexposure prophylaxis. If vaccine is not available immediately, HRIG should be given alone and immunization started as soon as possible. Similarly, if HRIG is not available immediately, vaccine should be administered and HRIG given if it can be obtained within 7 days after initiating immunization. If administration of both vaccine and HRIG is delayed, both should be used regardless of the interval between exposure and treatment. The recommended dose of HRIG is 20 IU/kg. As much of the dose as possible should be used to infiltrate the wound, if present. The remainder is administered intramuscularly. A 1.0-mL dose of rabies vaccine is given intramuscularly in the deltoid region or the anterolateral aspect of the thigh on the first day of postexposure prophylaxis, and repeated doses are provided on days 3, 7, 14, and 28 after the first dose for a total of five doses. HRIG or rabies vaccine alone or in combination with tetanus vaccine is inadequate postexposure prophylaxis.

References:

American Academy of Pediatrics. Rabies. In: Pickering LK, Baker CJ, Long SS, McMillan JA, eds. Red Book: 2006 Report of the Committee on Infectious Diseases. 27th ed. Elk Grove Village, Ill: American Academy of Pediatrics; 2006:552-559

Centers for Disease Control and Prevention. Human rabies prevention - United States 1999: recommendations of the Advisory Committee on Immunization Practices (ACIP). MMWR Recomm Rep. 1999;48(RR-1):1-21. Available at: http://www.cdc.gov/mmwr/preview/mmwrhtml/00056176.htm

Rupprecht CE, Gibbons RV. Prophylaxis against rabies. N Engl J Med. 2004;351:2626-2635. Extract available at: http://content.nejm.org/cgi/content/extract/351/25/2626

Page 797: AAP PREP 2009

2009 PREP SA on CD-ROM

page 797

Question: 238

An 8-year-old boy presents with gross hematuria associated with intermittent right-sided flank pain. There is no history of dysuria, urgency, frequency, or trauma. Physical examination reveals a temperature of 98.6°F (37°C), heart rate of 76 beats/min, respiratory rate of 20 breaths/min, blood pressure of 106/66 mm Hg, and no abdominal or costovertebral angle tenderness. Urinalysis shows a specific gravity of 1.025, pH of 6, 3+ blood, and trace protein. Microscopy documents 20 to 50 red blood cells/high-power field. Renal ultrasonography reveals a normal bladder with mild hydronephrosis on the right and an echogenic focus (Item Q238) with shadowing in the right kidney.

Of the following, the MOST likely additional expected laboratory feature contributing to this patient’s condition is

A. a positive urine culture

B. elevated urinary calcium excretion

C. elevated urinary citrate excretion

D. hypercalcemia

E. metabolic alkalosis

Page 798: AAP PREP 2009

2009 PREP SA on CD-ROM

page 798

Question: 238

Longitudinal ultrasonography scan of the abdomen reveals an echogenic focus with distal shadowing in a mildly dilated renal collecting system. (Courtesy of D. Mulvihill)

Page 799: AAP PREP 2009

2009 PREP SA on CD-ROM

page 799

Critique: 238 Preferred Response: B

The renal colic (intermittent flank pain) and gross hematuria combined with red blood cells in the urine and hydronephrosis with a shadowing echogenic focus in the kidney described for the patient in the vignette represent urolithiasis. Children who have urolithiasis require treatment for the stone via either surgical extraction or extracorporeal shock wave lithotripsy under the guidance of a pediatric urologist. The pediatrician needs to recognize the underlying risk factors for stone development to aid in the prevention of a recurrence. Urolithiasis in the pediatric patient usually involves the ureters and upper urinary tract, with a few patients exhibiting stones within the bladder. The most common types of renal stones in decreasing order of occurrence are calcium oxalate, calcium phosphate, mixed (calcium oxalate and calcium phosphate), struvite (magnesium ammonium phosphate), cystine, and uric acid. The evaluation for urolithiasis often begins with a 24-hour urine collection to look at urine volume and measure creatinine as reference points to ensure adequate fluid intake and adequate 24-hour urine collection. In addition, the 24-hour urine collection is aimed at measuring promoters of stone formation (calcium, oxalate, uric acid, and cystine) and inhibitors of stone formation (magnesium and citrate). The most common metabolic abnormality in a child who has stones is hypercalciuria. If hypercalciuria is present, the child should have serum calcium, phosphorus, ionized calcium, intact parathyroid hormone, and vitamin D (25-hydoxyvitamin D3 and 1,25-dihydoxyvitamin D3) measured. A child who has a renal stone also might have a urinary tract infection (UTI). Some urinary pathogens that produce urease (especially Proteus sp) can lead to an environment favorable to struvite stone formation. Also, even calcium-containing stones can obstruct urine flow and result in the development of a UTI. However, a UTI is not likely in a patient who has no dysuria, urgency, or frequency, such as the boy in the vignette. Risk factors for the development of renal stones include hypercalciuria, hyperoxaluria, decreased urinary magnesium, and decreased urinary citrate. Although hypercalciuria is commonly associated with renal stones, hypercalcemia is not a typical finding. One condition associated with hypercalciuria and renal stones is renal tubular acidosis. Metabolic alkalosis is seen when calcium phosphate stones are formed, but it is far less likely than hypercalciuria to be a risk factor for stone formation for the patient in the vignette.

References:

Alon US, Srivastava T. Urolithiasis. In: Kher KK, Schnaper HW, Makker SP, eds. Clinical Pediatric Nephrology. 2nd ed. London, England: Informa Healthcare; 2007:539-551

Milliner DS. Urolithiasis. In: Avner ED, Harmon WE, Niaudet P, eds. Pediatric Nephrology. 5th ed. Philadelphia, Pa: Lippincott Williams & Wilkins; 2004:1091-1111

Page 800: AAP PREP 2009

2009 PREP SA on CD-ROM

page 800

Question: 239

A 12-year-old boy presents with a 3-year history of hay fever in the spring. He experiences daily nasal congestion, sneezing, and rhinorrhea from March to May that worsens when he is outside. He is asymptomatic for the remainder of the year, but his parents are concerned because his symptoms interfere with outdoor sports activities. Use of over-the-counter first-generation antihistamines resulted in undesirable sedation.

Of the following, the BEST initial medication to treat this patient is a(an)

A. intranasal corticosteroid

B. intranasal decongestant

C. nasal saline spray

D. oral decongestant

E. oral leukotriene antagonist

Page 801: AAP PREP 2009

2009 PREP SA on CD-ROM

page 801

Critique: 239 Preferred Response: A

The boy described in the vignette presents with classic symptoms of allergic rhinitis. The approach to medical management of allergic rhinitis should take into account the patient's primary symptoms, past experiences with medications, and any coexisting medical problems. Nasal corticosteroids have become a first-line therapy for allergic rhinitis, with studies demonstrating improvement in nasal pruritus, sneezing, nasal obstruction, and rhinorrhea. Further, nasal steroids typically are well tolerated and have minimal adverse effects. Nasal decongestants may be purchased over the counter and markedly reduce nasal congestion, but recurrent or prolonged use can result in rhinitis medicamentosa. Nasal saline is an adjunct for nasal toilet, but it is inferior to nasal corticosteroids for the treatment of allergic rhinitis. Oral decongestants may be taken separately or in combination with an oral antihistamine. They can improve nasal congestion, but they may increase blood pressure or cause insomnia. Oral leukotriene antagonists have been approved for allergic rhinitis and mild persistent asthma. Although this class of medication generally has an excellent safety profile and is not a steroid, it is inferior to nasal corticosteroids for treatment of nasal allergic rhinitis symptoms.

References:

Atkins D, Leung DYM. Principles of treatment of allergic disease. In: Kliegman RM, Behrman RE, Jenson HB, Stanton BF, eds. Nelson Textbook of Pediatrics. 18th ed. Philadelphia, Pa: Saunders Elsevier; 2007:942-948

Mahr TA, Sheth K. Update on allergic rhinitis. Pediatr Rev. 2005;26:284-289. Available at: http://pedsinreview.aappublications.org/cgi/content/full/26/8/284

Page 802: AAP PREP 2009

2009 PREP SA on CD-ROM

page 802

Question: 240

You are examining a 7-year-old boy who has a 2-day history of abdominal pain. The pain began 2 nights ago after he ate pizza with the rest of his family and initially was crampy and diffuse. No one else became ill. He continued to complain of pain through the day yesterday, and this morning he vomited once, prompting his mother to bring him to the office. The emesis was nonbilious and nonbloody, and he has had no diarrhea, fever, or urinary symptoms. On physical examination, the boy is afebrile, has normal vital signs, and has diminished bowel sounds with involuntary guarding in the right lower quadrant. There are no peritoneal signs, and Rovsing, obturator, and psoas signs are negative.

Of the following, the study MOST likely to confirm the diagnosis is

A. abdominal radiograph

B. complete blood count

C. computed tomography scan of the abdomen and pelvis

D. C-reactive protein determination

E. procalcitonin determination

Page 803: AAP PREP 2009

2009 PREP SA on CD-ROM

page 803

Critique: 240 Preferred Response: C

Appendicitis is the most common abdominal surgical condition in children. Obstruction of the appendix with fecal matter or other debris leads to edema, bacterial overgrowth, mucosal breakdown, and appendiceal inflammation. Early diagnosis and surgical treatment are the keys to favorable outcomes. Diagnosing appendicitis in children can be challenging. When present, the highly sensitive signs and symptoms of abdominal pain that migrates to the right lower quadrant, fever, and abdominal rebound or guarding often are enough to prompt surgical exploration. However, the presentation in children frequently is not classic. The difficulties encountered in evaluating preverbal patients, the overlap of appendicitis with many other causes of childhood abdominal pain, and the often atypical presentations contribute to the 28% to 57% appendicitis misdiagnosis rate in children. Laboratory and radiologic studies often are used as diagnostic adjuncts when appendicitis is considered in a patient who has an atypical presentation, such as the child described in the vignette. Abdominal computed tomography scan (Item C240) is the most useful for confirming the diagnosis because its sensitivities and specificities approach 100%. There is debate in the literature regarding the optimal use of oral or rectal contrast for such studies. Abdominal radiographs are not helpful. No laboratory tests are independently predictive of the presence or absence of appendicitis. Although procalcitonin has not been studied in this setting, neither the total white blood count, absolute neutrophil count, nor C-reactive protein measurement has been found to be adequately sensitive or specific. However, with growing concern about the long-term effects of ionizing radiation exposure in children, laboratory tests combined with clinical findings may prove useful in identifying low-risk individuals for whom imaging can be avoided.

References:

Bundy DG, Byerly JS, Liles EA, Perrin EM, Katznelson J, Rice HE. Does this child have appendicitis? JAMA. 2007;298:438-451. Abstract available at: http://www.ncbi.nlm.nih.gov/pubmed/17652298

Kwok MY, Kim MK, Gorelick MH. Evidence-based approach to the diagnosis of appendicitis in children. Pediatr Emerg Care. 2004;20:690-698

Wesson DE. Evaluation and diagnosis of appendicitis in childhood. UpToDate Online 15.3. 2008. Available at:http://www.utdol.com/utd/content/topic.do?topicKey=ped_surg/4980&selectedTitle=4~150&source=search_result

Page 804: AAP PREP 2009

2009 PREP SA on CD-ROM

page 804

Critique: 240

Computed tomography scan of the abdomen in a reformatted coronal projection shows a fluid-filled tubular structure with a thick wall consistent with an obstructed appendix. (Coutesy of D. Mulvihill)

Page 805: AAP PREP 2009

2009 PREP SA on CD-ROM

page 805

Question: 241

A 12-year-old boy who has a history of recurrent abdominal pain presents to your office for an annual health supervision visit. The boy complains of periumbilical pain, unrelated to meals, occurring twice a month and lasting 15 minutes. Physical examination findings are normal. Fecal occult blood test results are negative. His father, who is a physician, asks if the boy should undergo testing for Helicobacter pylori.

Of the following, a TRUE statement about H pylori infection is that

A. all children who have positive H pylori serologies should undergo endoscopy

B. antibiotic therapy for H pylori is most effective when combined with a proton pump inhibitor

C. H pylori is difficult to detect on gastric histology without special immunofluorescence staining

D. H pylori infection is less prevalent in children from the developing world

E. H pylori organisms rarely develop antibiotic resistance

Page 806: AAP PREP 2009

2009 PREP SA on CD-ROM

page 806

Critique: 241 Preferred Response: B

Helicobacter pylori is a curved, gram-negative bacillus that has a causative role in peptic ulcer disease. Approximately 50% of individuals worldwide are colonized with H pylori; the organism is more prevalent in the developing world. Approximately 20% of people infected with H pylori develop complications from the infection, but there is a long latency period between the time of infection (usually in childhood) and the time that symptoms develop. The complications of long-term infection may be serious, however, and include gastric and duodenal ulcers, gastric lymphoma (mucosal associated lymphoid tissue lymphoma [MALToma]), and atrophic gastritis. If symptoms or clinical findings suggest the presence of H pylori, a number of diagnostic tests can aid in establishing a diagnosis. The gold standard for H pylori diagnosis is upper gastrointestinal endoscopy with biopsy of the gastric antrum. Endoscopy often shows nodular inflammation of the antrum (Item C241A), and biopsy can demonstrate the characteristic organisms in the gastric epithelium. Most often, organisms can be seen easily on conventional hematoxylin and eosin stain; in cases involving few organisms, special staining is helpful (Item C241B). Given the invasiveness of endoscopy, this procedure should not be performed on every child who has abdominal pain or dyspeptic symptoms. If a clinician wants to rule out H pylori as a cause of gastrointestinal symptoms, available tests include a serum H pylori immunoglobulin G serology, a fecal sample for H pylori antigen, or a C13 urea breath test for H pylori. The limitation of these tests is the occurrence of both false-positive and false-negative results. In particular, the false-positive rate of serology is about 10%. Therefore, this test has a poor positive predictive value and probably should not be ordered if the clinical suspicion of H pylori infection is low (eg, in a child who has periumbilical or lower abdominal pain). Given that chronic abdominal pain and dyspeptic symptoms are so common in children and adults, the role of H pylori in chronic pain remains highly controversial. Even if a diagnostic test for H pylori has a positive result, it may be hard to prove a definitive association between the patient's symptoms and the H pylori. Therefore, the clinician and patient need to consider whether to perform definitive testing (ie, endoscopy) or treat empirically. The "test versus treat" question remains a clinical dilemma because empiric therapy is more cost-effective and less invasive, but endoscopy provides a definitive diagnosis and excludes other conditions (eg, reflux esophagitis). Treatment of H pylori infection usually involves two antibiotics (eg, amoxicillin and metronidazole or amoxicillin and clarithromycin) in conjunction with a proton pump inhibitor (PPI) (eg, omeprazole, lansoprazole, pantoprazole). The PPI enhances antimicrobial eradication because the organisms prefer the normal acid medium of the stomach. Unfortunately, antimicrobial resistance is common in H pylori, especially to metronidazole. Therefore, if H pylori is causing serious disease and has been treated, documentation of eradication is suggested either by negative fecal antigen studies or by endoscopy.

References:

Ford A, McNulty C, Delaney B, Moayyedi A. Helicobacter pylori infection. BMJ Clinical Evidence. 2007. Available for subscription at: http://clinicalevidence.bmj.com/ceweb/conditions/dsd/0406/0406.jsp

Gold BD, Colletti RB, Abbott M, et al; North American Society for Pediatric Gastroenterology and Nutrition. Helicobacter pylori infection in children: recommendations for diagnosis and treatment. J Pediatr Gastroenterol Nutr. 2000;31:490-497

Vilaichone RK, Mahachai V, Graham DY. Helicobacter pylori diagnosis and management. Gastroenterol Clin North Am. 2006;35:229-247. Abstract available at: http://www.ncbi.nlm.nih.gov/pubmed/16880064

Page 807: AAP PREP 2009

2009 PREP SA on CD-ROM

page 807

Critique: 241

H pylori infection produces a nodular inflammation of the gastric antrum. (Courtesy of A. Bousvaros)

Page 808: AAP PREP 2009

2009 PREP SA on CD-ROM

page 808

Critique: 241

Silver stain of a gastric biopsy demonstrating curved H pylori bacteria in a gastric crypt. (Courtesy of J. Glickman)

Page 809: AAP PREP 2009

2009 PREP SA on CD-ROM

page 809

Question: 242

A term newborn is delivered to a mother who has had a 5-day history of a nonspecific gastroenteritis, some loose stools, generalized malaise, and low-grade fever. The infant had a seizure at 6 hours of age and is ill, with an inspired oxygen requirement of 0.40, some petechiae, and oozing from the umbilicus and phlebotomy sites. He is irritable on neurologic examination. Laboratory findings include: · White blood cell count, 7.5x103/mcL (7.5x109/L) · Platelet count, 90.0x103/mcL (90.0x109/L) · Hematocrit, 45% (0.45) · Aspartate aminotransferase, 240.0 U/L · Alanine aminotransferase, 300.0 U/L · Fibrinogen, 90.0 mg/dL (2.6 mcmol/L) · Prothrombin time, 20 seconds · Partial thromboplastin time, 60 seconds · Internationalized Normalized Ratio (INR), 1.80 · Serum glucose, 90.0 mg/dL (5.0 mmol/L)

A lumbar puncture reveals 35 white blood cells, with 50% polymorphonuclear cells and 50% mononuclear cells; 1 red blood cell; glucose of 60.0 mg/dL (3.3 mmol/L); and protein of 100 mg/dL (1,000 g/L). No organisms are seen on cerebrospinal fluid (CSF) Gram stain.

Of the following, a TRUE statement about this patient’s meningitis is that

A. gram-negative organisms are unlikely to be causative

B. group B streptococcal meningitis is likely to be the cause

C. infection likely is related to maternal enteroviral infection

D. the abnormal CSF glucose and protein values indicate bacterial meningitis

E. the abnormal liver function test results and CSF cell counts indicate herpes simplex virus infection

Page 810: AAP PREP 2009

2009 PREP SA on CD-ROM

page 810

Critique: 242 Preferred Response: C

The infant described in the vignette has symptoms of respiratory distress, neurologic irritability, and coagulopathy. The likely viral gastroenteritis suffered by his mother was of short enough duration not to have conferred any protective antibody transfer to the fetus. His laboratory findings are most concerning for thrombocytopenia, coagulopathy, and elevated hepatic transaminases. His lumbar puncture results indicate a pleocytosis consistent with meningitis. Neonatal meningitis of either viral or bacterial cause typically occurs in the face of a sepsis syndrome that may be associated with known risk factors prenatally (maternal fever, chorioamnionitis, prolonged rupture of membranes) or postnatally (bacteremia, urinary tract infection, or respiratory distress), with attendant signs of apnea, lethargy, acidosis, and hypoglycemia. The infection most often is bacterial. Physical examination findings can include a bulging or tense anterior fontanelle, irritability, fever, emesis, coma, and seizures; neuromotor tone may be increased or decreased. CSF values in bacterial meningitis generally include elevated protein values to more than 150 mg/dL (1,500 g/L), white blood cell count of more than 100, and a glucose value less than 50% of that measured in the blood. These values may differ greatly from those seen in viral (aseptic) meningitis of the newborn, such as that affecting the infant described in the vignette. Neonatal enteroviral infection may follow maternal infection, characteristically in the spring and summer seasons. Clinically, affected newborns manifest illness with a sepsis syndrome, coagulopathy, and meningitis. Hepatopathy and respiratory distress also may be significant. The mortality rate of neonatal bacterial meningitis generally is cited at 10%, but both mortality and late morbidity vary with the cause of meningitis in the newborn. The most common pathogens for neonatal meningitis include group B Streptococcus, Escherichia coli, other gram-negative rods, and Listeria monocytogenes. Although neonatal group B streptococcal meningitis is common, it generally is associated with later onset (after 7 days), a shocklike state, and CSF findings consistent with bacterial meningitis, as noted previously. Gram-negative meningitis varies with perinatal exposure, prior maternal or neonatal antibiotic use, comorbidities, and neonatal intensive care unit (NICU) practices. S pneumoniae and Haemophilus influenzae are uncommon causes of meningitis in the newborn. Staphylococcal infections may be increasing causes of concern in the NICU. Methicillin-resistant S aureus meningitis may require aggressive therapy and isolation. Herpes simplex viral meningoencephalitis is associated with a hemorrhagic pleocytosis, apnea, seizures, coagulopathy, and hepatic transaminase values generally greater than 1,000 U/L.

References:

Heath PT, Nik Yusoff NK, Baker CJ. Neonatal meningitis. Arch Dis Child Fetal Neonatal Ed. 2003;88:F173-F178. Abstract available at: http://www.ncbi.nlm.nih.gov/pubmed/12719388

Klinger G, Chin C-N, Beyene J, Perlman M. Predicting the outcome of neonatal bacterial meningitis. Pediatrics. 2000;106:477-482. Available at: http://pediatrics.aappublications.org/cgi/content/full/106/3/477

Miyairi I, Berlingieri D, Protic J, Belko J. Neonatal invasive group A streptococcal disease: case report and review of the literature. Pediatr Infect Dis J. 2004;23:161-165. Abstract available at: http://www.ncbi.nlm.nih.gov/pubmed/14872185

Moylett EH. Neonatal Candida meningitis. Semin Pediatr Infect Dis. 2003;14:115-122. Abstract available at: http://www.ncbi.nlm.nih.gov/pubmed/12881799

Page 811: AAP PREP 2009

2009 PREP SA on CD-ROM

page 811

Philip AGS. Neonatal meningitis in the new millennium. NeoReviews. 2003;4:e73-e80. Available for subscription at: http://neoreviews.aappublications.org/cgi/content/full/4/3/e73

Polin RA, Harris MC. Neonatal bacterial meningitis. Semin Neonatol. 2001;6:157-172. Abstract available at: http://www.ncbi.nlm.nih.gov/pubmed/11483021

Page 812: AAP PREP 2009

2009 PREP SA on CD-ROM

page 812

Question: 243

A 5-month-old boy is brought to the emergency department by his mother because of decreased activity and vomiting for 1 day. She reports occasional foul-smelling stools but no recent changes in stool pattern. There has been no fever. As a neonate, the boy had difficulty gaining weight and prolonged jaundice, but he has not required hospitalization. Physical examination reveals an ill-appearing child who has mild dehydration, a heart rate of 120 beats/min, and otherwise normal vital signs. He appears somewhat cachectic, and his weight is at the 3rd percentile. Laboratory values include a normal complete blood count and urinalysis, sodium of 134.0 mEq/L (134.0 mmol/L), chloride of 86.0 mEq/L (86.0 mmol/L), potassium of 3.8 mEq/L (3.8 mmol/L), and carbon dioxide of 31.0 mEq/L (31.0 mmol/L). Blood urea nitrogen and creatinine values are within normal limits.

Of the following, the MOST likely diagnosis is

A. Bartter syndrome

B. congenital adrenal hyperplasia

C. cystic fibrosis

D. Fanconi syndrome

E. hypertrophic pyloric stenosis

Page 813: AAP PREP 2009

2009 PREP SA on CD-ROM

page 813

Critique: 243 Preferred Response: C

Cystic fibrosis is an autosomal recessive disorder affecting many children and adolescents. It is caused by a defect in a chloride channel, the cystic fibrosis transmembrane conductance regulator (CFTR), on the apical membranes of the linings of the airways, intestinal tract, vas deferens, biliary tree, pancreatic ducts, and sweat ducts. The result is ineffective secretion of fluids from affected areas and an increased sodium and chloride sweat concentration, the latter being the basis for diagnostic testing. Affected children have varying degrees of mucoid airway obstruction, with secondary bacterial infections, failure to thrive, intestinal obstruction, pancreatic and biliary dysfunction, and infertility. Hypochloremic metabolic alkalosis with dehydration, as described for the boy in the vignette, is a common feature because of the high salt loss from the sweat glands. Foul-smelling stools, prolonged jaundice, and poor weight gain are additional features of the disease. Therefore, the boy in the vignette should undergo testing for cystic fibrosis (eg, sweat chloride testing and genetics testing). Bartter syndrome results from a defect in chloride reabsorption in the loop of Henle. Clinical features include failure to thrive, polyuria, and vomiting. Hypochloremia and metabolic alkalosis can occur, but hypokalemia, which can be severe, is usual and due to urinary potassium wasting. Infants who have the salt-losing form of congenital adrenal hyperplasia also may present with vomiting and failure to thrive, but typical electrolyte abnormalities are hyponatremia, hypochloremia, and hyperkalemia. Fanconi syndrome is characterized by abnormal proximal renal tubule function. Excessive bicarbonaturia causes metabolic acidosis with hyperchloremia, and hypokalemia may be seen. Clinical features in infancy can be similar to those of the previously described diseases, with failure to thrive, vomiting, and polyuria being common. Infants who have hypertrophic pyloric stenosis often have hypochloremic metabolic alkalosis, but this condition is seen in younger infants and is the result of substantial vomiting of a long duration.

References:

Boat TF, Acton JD. Cystic fibrosis. In: Kliegman RM, Behrman RE, Jenson HB, Stanton BF, eds. Nelson Textbook of Pediatrics. 18th ed. Philadelphia, Pa: Saunders Elsevier; 2007:1803-1816

Davis PB. Cystic fibrosis. Pediatr Rev. 2001;22:257-264. Available at: http://pedsinreview.aappublications.org/cgi/content/full/22/8/257

Page 814: AAP PREP 2009

2009 PREP SA on CD-ROM

page 814

Question: 244

A 15-month-old girl presents to the emergency department with a temperature of 103°F (39.5°C) during respiratory virus season. Physical examination reveals rhinorrhea and mild cough but no other focus of infection. However, she has diffuse bruises in various stages of healing on her abdomen, subscapular area, and both extensor and flexor surfaces of her extremities. Laboratory studies reveal a white blood cell count of 9.2x103/mcL (9.2x109/L) with a normal differential count, platelet count of 376.0x103/mcL (376.0x109/L), hemoglobin of 13.0 g/dL (130.0 g/L), and hematocrit of 39% (0.39).

Of the following, the BEST next step in the evaluation of this child is

A. computed tomography scan of the brain

B. measurement of factor VIII

C. measurement of von Willebrand factor

D. prothrombin time and partial thromboplastin time

E. radiographic skeletal survey

Page 815: AAP PREP 2009

2009 PREP SA on CD-ROM

page 815

Critique: 244 Preferred Response: E

Bruises on the back (Item C244), abdomen, and flexor surfaces in various stages of healing, as reported for the child in the vignette, are suspicious for nonaccidental trauma. Thus, the child should undergo a bone survey to look for occult fractures. Bruising in older infants and toddlers often results from the combination of increasing mobility and developing coordination and stability. However, bruising that relates to falls associated with creeping, crawling, cruising, and early walking is seen primarily on the front of the body, especially on the forehead, forearms, and pretibial areas of the legs, where the child "meets" obstacles in the environment. The appearance of a bruise depends on its age, location, and depth as well as the child's skin pigmentation. A review of the literature about "dating" bruises in children reveals that a bruise may appear red, blue, or purple at any time from onset to resolution but often contains yellow discoloration after 18 hours from onset. However, even multiple bruises resulting from the same injury at the same time in the same person may "age" differently, so it may be difficult to age bruises accurately. Coagulation disorders such as hemophilia and von Willebrand disease may present with bruising but often are associated with other physical findings and a history of mucosal bleeding. Studies that can assess for a coagulation disorder, such as measuring prothrombin time, partial thromboplastin time, von Willebrand factor, and factor VIII, are unlikely to yield positive results for the child described in the vignette. In the absence of signs of head contusion, irritability, focal neurologic signs, or developmental delay, computed tomography scan of the brain is unlikely to be helpful and would result in unnecessary exposure to ionizing radiation. Although not specifically addressed in this vignette, some recommend evaluation for possible soft-tissue injury, including renal, hepatic, and splenic contusions or laceration, with abdominal ultrasonography. Minor elevations in liver enzymes and microscopic hematuria may be found with less severe injury. Ophthalmologic examination may detect retinal hemorrhages, which indicates a potential need for brain imaging, preferably with magnetic resonance imaging in a neurologically stable child. Evaluation of the family and caretaker systems by social services and possibly a report for evaluation by child protective services is clearly indicated when possible child abuse is suspected. Development of a safety plan may improve the outcome for women who are in abusive relationships with domestic partners, which is a risk factor for child abuse, and decrease episodes of violence, at least in the short term.

References:

Barlow J, Johnston I, Kendrick D, Polnay L, Stewart-Brown S. Individual and group-based parenting programmes for the treatment of physical child abuse and neglect. Cochrane Database Syst Rev. 2006;3:CD005463. Available at: http://www.mrw.interscience.wiley.com/cochrane/clsysrev/articles/CD005463/frame.html

Coulter K. In brief: bruising and skin trauma. Pediatr Rev. 2000;21:34-35. Available at: http://pedsinreview.aappublications.org/cgi/content/full/21/1/34

Kellogg ND and the Committee on Child Abuse and Neglect. Evaluation of suspected child physical abuse. Pediatrics. 2007;119:1232-1241. Available at: http://pediatrics.aappublications.org/cgi/content/full/119/6/1232

Klevens J, Sadowski L. Intimate partner violence towards women (update). BMJ Clinical Evidence.

Page 816: AAP PREP 2009

2009 PREP SA on CD-ROM

page 816

2007. Available for subscription at: http://clinicalevidence.bmj.com/ceweb/conditions/woh/1013/1013_I5.jsp

Labbé J, Caouette G. Recent skin injuries in normal children. Pediatrics. 2001;108:271-276. Available at: http://pediatrics.aappublications.org/cgi/content/full/108/2/271

Maguire S, Mann MK, Sibert J, Kemp A. Are there patterns of bruising in childhood which are diagnostic or suggestive of abuse? A systematic review. Arch Dis Child. 2005;90:182-186. Abstract available at: http://www.ncbi.nlm.nih.gov/pubmed/15665178

Sugar NF, Taylor JA, Feldman KW and the Puget Sound Pediatric Research Network. Bruises in infants and toddlers: those who don't cruise rarely bruise. Arch Pediatr Adolesc Med. 1999;153:399-403. Available at: http://archpedi.ama-assn.org/cgi/content/full/153/4/399

Page 817: AAP PREP 2009

2009 PREP SA on CD-ROM

page 817

Critique: 244

Bruising on the back may be the result of nonaccidental trauma. (Courtesy of D. Krowchuk)

Page 818: AAP PREP 2009

2009 PREP SA on CD-ROM

page 818

Question: 245

You receive a telephone call from the physician mother of a 1-week-old patient who was born at 24 weeks' gestation. He is being treated in the neonatal intensive care unit and has been stable on the ventilator. She is concerned because when she visited him this morning, his blood pressure was 44/26 mm Hg. His mean arterial pressure was 30 mm Hg. She is worried that his blood pressure is low and that this may be harmful.

Of the following, the MOST accurate statement regarding blood pressure in the preterm infant is that

A. blood pressure values for preterm infants should be compared with those for term infants

B. blood pressure values vary indirectly with gestational age

C. mean arterial pressure should be no less than the corrected gestational age in weeks

D. patent ductus arteriosus narrows the pulse pressure by raising the diastolic pressure

E. systemic hypertension typically occurs coincidentally with pulmonary hypertension

Page 819: AAP PREP 2009

2009 PREP SA on CD-ROM

page 819

Critique: 245 Preferred Response: C

Preterm birth can be associated with a number of morbidities, including hyper- or hypotension. There are well-described normative values for blood pressure in a healthy term newborn, but normal blood pressure values for the preterm infant are much less clear. In fact, there may be no true correct or expected blood pressure for the preterm infant, particularly the child who is born extremely preterm. Blood pressure increases during the first few days and weeks after birth for each gestational age. Neonatologists generally agree that the mean arterial blood pressure for a preterm infant should not be less than the corrected gestational age in weeks. Thus, for example, a 26-week-gestation infant should have a mean arterial blood pressure in excess of 26 mm Hg. Beyond this simple rule, a given blood pressure value can be considered adequate if there is no evidence of metabolic acidosis, elevated lactate concentration, or inadequate end-organ perfusion. Just as infant and child blood pressure norms are not compared with adult norms, the normative blood pressure for a term infant is not used to assess the blood pressure of the preterm infant. As noted previously, blood pressure varies directly, not indirectly, with advancing gestational age. A patent ductus arteriosus widens the blood pressure by allowing blood to be diverted away from the higher-resistance systemic circulation and toward the lower-resistance pulmonary circulation. As this "steal" becomes greater, the diastolic pressure decreases and the pulse pressure (difference between the systolic and diastolic pressure) increases. Finally, systemic hypertension usually does not occur with pulmonary hypertension. In fact, it is not uncommon to note systemic hypotension with pronounced pulmonary hypertension as the failing right ventricle leads to a decrease in cardiac output.

References:

Padbury JF. Neonatal hypotension and hypovolemia. In: Rudolph C, Rudolph A, eds. Rudolph's Pediatrics. 21st ed. New York, NY: McGraw Hill Medical Publishing Division; 2003:137-140

Nwankwo MU, Lorenz JM, Gardiner JC. A standard protocol for blood pressure measurement in the newborn. Pediatrics. 1997;99:E10. Available at: http://pediatrics.aappublications.org/cgi/content/full/99/6/e10

Weindling AM, Subhedar NV. The definition of hypotension in very low-birthweight infants during the immediate neonatal period. NeoReviews. 2007;8:e32. Available for subscription at: http://neoreviews.aappublications.org/cgi/content/full/8/1/e32

Zubrow AB, Hulman S, Kushner H, Falkner B. Determinants of blood pressure in infants admitted to neonatal intensive care units: a prospective multicenter study. Philadelphia Neonatal Blood Pressure Study Group. J Perinatol. 1995;15:470-479. Abstract available at: http://www.ncbi.nlm.nih.gov/pubmed/8648456

Page 820: AAP PREP 2009

2009 PREP SA on CD-ROM

page 820

Question: 246

A 17-year-old boy who receives carbamazepine for epilepsy presents to the emergency department after a 40-minute generalized tonic-clonic seizure. He has been well, and there is no history of trauma. On physical examination, he answers a few questions, but he is sleepy and confused. He is afebrile, and his vital signs are normal. Although he is uncooperative, he moves all limbs spontaneously with good strength.

Of the following, the diagnostic test that is MOST likely to explain this seizure is

A. magnetic resonance imaging

B. noncontrast head computed tomography scan

C. prolonged electroencephalography

D. serum anticonvulsant measurement

E. serum electrolyte measurement

Page 821: AAP PREP 2009

2009 PREP SA on CD-ROM

page 821

Critique: 246 Preferred Response: D

The adolescent described in the vignette has a prior diagnosis of epilepsy. He now is in a postictal state after status epilepticus, but during the assessment in the emergency department appears to be returning to normal consciousness. The most likely causes for a prolonged seizure in this setting are his epilepsy and failure to take his medication. Accordingly, measuring his serum drug concentrations should be most helpful in explaining the seizure. Brief or prolonged seizures can be symptomatic of intracranial lesions such as ischemic strokes, trauma, hemorrhages, neoplasms, or focal or generalized brain infections. They also can occur due to metabolic derangements involving glucose or electrolytes. It is important to consider this broad range of causes in any child presenting to the emergency department with seizure, particularly one in status epilepticus, defined as any seizure lasting 30 minutes or longer. If the seizure has a focal onset, there is a residual focal deficit, or the seizure is accompanied by fever and prolonged confusion, imaging studies should be ordered emergently. For infants or for those children whose histories suggest electrolyte disturbance, glucose and electrolyte testing can be helpful. Electroencephalography (EEG) may be helpful if confusion is prolonged because occasionally the prolonged confusion may be due to nonconvulsive status epilepticus. In this case, however, the child's postictal state is resolving. Therefore, EEG will not add anything useful to initial medical decisions. For the child or adolescent who has a prior diagnosis of epilepsy, the most likely cause of the prolonged seizure is noncompliance with medications. Illicit drug use also may be involved and is worth considering in the differential diagnosis. In addition, excessively high serum drug concentrations of carbamazepine, sometimes due to drug-drug interactions, occasionally can precipitate seizures. Newer antiseizure medications are used more frequently in children. Unfortunately, many hospital laboratories are unable to measure these rapidly. A history obtained from the family ("He just ran out of medicine, and we couldn't get to the pharmacy") or a phone call to the patient's neurologist ("This has happened before - he doesn't take his medication and misses appointments") may help clarify the diagnosis in these cases.

References:

Riviello JJ Jr, Ashwal S, Hirtz D, et al. Practice parameter: diagnostic assessment of the child with status epilepticus (an evidence-based review). Report of the Quality Standards Subcommittee of the American Academy of Neurology and the Practice Committee of the Child Neurology Society. Neurology. 2006;67:1542-1550. Available at: http://www.neurology.org/cgi/content/full/67/9/1542

Page 822: AAP PREP 2009

2009 PREP SA on CD-ROM

page 822

Question: 247

A mother brings in her 4-month-old baby because she is concerned about the infant’s head shape. The baby is growing and developing normally. Physical examination findings are normal except for a flat occiput and a wide biparietal diameter with a flat forehead. The head circumference is normal, and the anterior fontanelle is small but patent.

Of the following, this baby’s findings MOST likely are caused by

A. bilateral coronal suture synostosis

B. bilateral lambdoid suture synostosis

C. metopic suture synostosis

D. positional plagiocephaly

E. sagittal suture synostosis

Page 823: AAP PREP 2009

2009 PREP SA on CD-ROM

page 823

Critique: 247 Preferred Response: A

Craniosynostosis, the premature fusion of one or more cranial sutures, may have a pre- or postnatal onset. The causes are varied and include mechanical forces, suboptimal brain growth, gene mutations, and metabolic derangements. Craniosynostosis may be primary, in the case of a genetic cause, or secondary, as can be seen with vitamin D deficiency or hyperthyroidism. Sutural formation and skull growth are exceedingly complex. Histologically, craniosynostosis begins at a single point and spreads along the suture. Skull growth typically occurs in a direction perpendicular to each of the major sutures (Item C247A). The infant described in the vignette demonstrates a short anteroposterior (AP) dimension to the skull and a wide biparietal dimension with flattening of the forehead. These findings are consistent with bilateral coronal synostosis, in which growth of the skull perpendicular to the coronal sutures is limited (Item C247B). Coronal suture synostosis may be isolated but frequently is associated with large thumbs or great toes and often is due to dominant mutations in fibroblast growth factor receptor (FGFR) genes. For infants who have this presentation, it is imperative to take a family history to determine if there are similarly affected relatives. Metopic suture synostosis results in trigonocephaly, or a "keel-shaped" skull (Item C247C). Sagittal suture synostosis causes the head shape to be scaphocephalic (long in the AP dimension and narrow in the biparietal dimension) (Item C247D). Posterior plagiocephaly (flattening of one or both sides of the occiput) that results from premature closure of the lambdoid suture(s) is rare and often is confused with positional plagiocephaly (Item C247E). Since the advent of the recommendation that infants be placed on their backs to sleep, there has been a dramatic increase in positional plagiocephaly, which is characterized by unilateral flattening of the occiput and compensatory prominence of the forehead on the same side. Positional plagiocephaly can be ameliorated by the use of a helmet molded to fit the infant's skull; ideally, this therapy should be initiated prior to 9 months of age.

References:

Cohen MM Jr. Fibroblast growth factor receptor mutations. In: Cohen MM Jr, MacLean RE, eds. Craniosynostosis: Diagnosis, Evaluation, and Management. 2nd ed. New York, NY: Oxford University Press; 2000:77-94

Cohen MM Jr. History, terminology, and classification of craniosynostosis. In: Cohen MM Jr, MacLean RE, eds. Craniosynostosis: Diagnosis, Evaluation, and Management. 2nd ed. New York, NY: Oxford University Press; 2000:103-111

Robin NH, Falk MJ, Hlademan-Englert CR. FGFR-related craniosynostosis syndromes. GeneReviews. 2007. Available at: http://www.geneclinics.org/servlet/access?db=geneclinics&site=gt&id=8888891&key=xAcWBcrjmZrVo&gry=&fcn=y&fw=MR4U&filename=/profiles/craniosynostosis/index.html

Page 824: AAP PREP 2009

2009 PREP SA on CD-ROM

page 824

Critique: 247

Skull growth normally occurs in a direction perpendicular to each of the sutures. (Courtesy of A. Johnson)

Page 825: AAP PREP 2009

2009 PREP SA on CD-ROM

page 825

Critique: 247

Premature fusion of the coronal sutures (blue) results in brachycephaly, a prominent and flattened frontal bone, flattening of the occiput, and anterior displacement of the vertex. (Courtesy of A. Johnson)

Page 826: AAP PREP 2009

2009 PREP SA on CD-ROM

page 826

Critique: 247

Metopic synostosis (blue) results in a triangular-shaped forehead and bossing of the parieto-occipital regions. (Courtesy of A. Johnson)

Page 827: AAP PREP 2009

2009 PREP SA on CD-ROM

page 827

Critique: 247

Sagittal suture synostosis (blue) causes the skull to be elongated (scaphocephaly). (Courtesy of A. Johnson)

Page 828: AAP PREP 2009

2009 PREP SA on CD-ROM

page 828

Critique: 247

In positional plagiocephaly (left), when viewed from above, the head has a parallelogram shape, with unilateral occipitoparietal flattening, displacement of the ipsilateral ear anteriorly, and bossing of the ipsilateral frontal skull. In contrast, in unilateral lambdoidal synostosis (right, [blue]), the head has a trapezoidal shape, with unilateral occipital flattening, posterior displacement of the ipsilateral ear, and bossing of the contralateral frontal skull. (Courtesy of A. Johnson)

Page 829: AAP PREP 2009

2009 PREP SA on CD-ROM

page 829

Question: 248

The parents of a 14-year-old girl are concerned about her weight loss. Her weight today is 20 lb less than a documented weight obtained 1 year ago at her camp physical examination. She complains of frequent nausea, decreased appetite, and early satiety, even after eating very small portions. She has no vomiting or diarrhea, but frequent constipation. She complains of increased fatigue but is still able to participate in diving 5 days a week. She is doing well in school academically. She attained menarche at 12 years of age and had monthly periods for about 18 months, but she has had no menses for the past 7 months. She has been a vegetarian for the past 18 months and feels she is at a good weight currently. On physical examination, her body mass index is 17.0. Her urine pregnancy test result is negative.

Of the following, the MOST likely diagnosis is

A. anorexia nervosa

B. depression

C. hypothalamic tumor

D. hypothyroidism

E. inflammatory bowel disease

Page 830: AAP PREP 2009

2009 PREP SA on CD-ROM

page 830

Critique: 248 Preferred Response: A

The diagnosis of an eating disorder is not always straightforward, but the decreased appetite and early satiety, large weight loss, amenorrhea, and disturbed body image with a low body mass index reported for the girl in the vignette strongly suggest anorexia nervosa. Participating in a body-conscious sport such as diving, becoming a vegetarian as a way to restrict the diet, and the presence of constipation also may be associated with this condition. The lack of diarrhea or hematochezia makes inflammatory bowel disease less likely. Weight loss, doing well in school, and maintaining a strong exercise program are uncommon in hypothyroidism. Depression can accompany an eating disorder, but the early satiety, constipation, amenorrhea, and good school performance are unlikely to occur if depression is the sole diagnosis. A hypothalamic tumor typically presents with neurologic symptoms or signs of increased intracranial pressure. A limited laboratory evaluation that includes a complete blood count; metabolic panel; urinalysis; measurement of free thyroxine, thyroid-stimulating hormone, luteinizing hormone, follicle-stimulating hormone, and prolactin; and electrocardiography may be useful in ruling out other possible organic causes of this patient's weight loss and symptoms.

References:

Fisher M. Treatment of eating disorders in children, adolescents, and young adults. Pediatr Rev. 2006;27:5-16. Available at: http://pedsinreview.aappublications.org/cgi/content/full/27/1/5

Rome ES, Ammerman S, Rosen DS, et al. Children and adolescents with eating disorders: the state of the art. Pediatrics. 2003;111:e98-e108. Available at: http://pediatrics.aappublications.org/cgi/content/full/111/1/e98

Rosen DS. Eating disorders in children and young adolescents: etiology, classification, clinical features, and treatment. Adolesc Med. 2003;14:49-59. Abstract available at: http://www.ncbi.nlm.nih.gov/pubmed/12529190

Page 831: AAP PREP 2009

2009 PREP SA on CD-ROM

page 831

Question: 249

You are evaluating an 8-month-old infant in preparation for administering chloral hydrate to perform a sedated brainstem auditory evoked potentials test.

Of the following, a CONTRAINDICATION to proceeding with the planned sedation is

A. absence of intravenous access

B. history of delivery at 36 weeks estimated gestational age

C. ingestion of infant formula 3 hours ago

D. ingestion of apple juice 3 hours ago

E. no previous anesthetic administration

Page 832: AAP PREP 2009

2009 PREP SA on CD-ROM

page 832

Critique: 249 Preferred Response: C

The goal of procedural sedation is to provide anxiolysis and analgesia and minimize patient movement. Sedation should be viewed as a continuum; the American Academy of Pediatrics has defined four levels of sedation. Minimal sedation (formerly anxiolysis) is defined as a drug-induced state in which the patient retains the ability to respond normally to verbal commands, and cardiovascular and respiratory functions are unaffected. Moderate sedation (formerly conscious sedation) is a drug-induced depression of consciousness in which patients still should respond to verbal and physical stimulation. Cardiovascular status is maintained, but the clinician must be able to recognize and respond to potential airway compromise. Deep sedation is defined as further depression of the level of consciousness, with partial or complete loss of protective airway reflexes and the need for assistance with airway maintenance. General anesthesia is the deepest level of sedation that is characterized by loss of consciousness and airway protective reflexes. Impairment of respiratory and cardiovascular function is common. Provision of safe and effective sedation requires appropriate patient selection, evaluation, and preparation. Appropriate monitoring and resuscitation equipment, thorough documentation, and trained personnel are essential. A contraindication to proceeding with the planned sedation for the patient in the vignette is the ingestion of formula 3 hours ago, which increases the risk of aspiration due to potential loss of airway reflexes. Children should not ingest formula or food within 6 hours or human milk within 4 hours of sedation. Clear liquids such as juice are allowed up to 2 hours prior to a planned elective sedation and may help reduce gastric contents. Neither the history of birth at 36 weeks estimated gestational age nor the lack of previous anesthetic administration are contraindications, but the preanesthetic history should explore any complications at birth as well as any family history of difficulty with sedation or anesthesia. Because preterm infants are at increased risk of apnea and bradycardia following sedation until they have achieved a postconceptual age of 52 weeks, they deserve additional monitoring. Pediatric sedations frequently are achieved with oral medications, but personnel and equipment to obtain urgent intravenous access should be readily available.

References:

American Academy of Pediatrics, American Academy of Pediatric Dentistry, Coté CJ, Wilson S, AAP Work Group on Sedation. Guidelines for monitoring and management of pediatric patients during and after sedation for diagnostic and therapeutic procedures: an update. Pediatrics. 2006;118:2587-2602. Available at: http://pediatrics.aappublications.org/cgi/content/full/118/6/2587

Koh JL, Palermo T. Conscious sedation: reality or myth? Pediatr Rev. 2007:28:243-248. Available at: http://pedsinreview.aappublications.org/cgi/content/full/28/7/243

Wetzel R. Anesthesia and perioperative care. In: Kliegman RM, Behrman RE, Jenson HB, Stanton BF, eds. Nelson Textbook of Pediatrics. 18th ed. Philadelphia, Pa: Saunders Elsevier; 2007:460-474

Page 833: AAP PREP 2009

2009 PREP SA on CD-ROM

page 833

Question: 250

You observe a child entering the waiting room, accompanied by her mother. She looks at the receptionist and says "Hi." While holding her doll, the child turns to her mother and says "juice." The mother gives her a cup of juice, and the child says "doll" and tries to give the doll a drink. The mother shakes her head, and the child says "no." The child then points to her own mouth, smiles, and says "mouth." The mother takes a tissue to clean the doll’s face. The child says "me" and begins to imitate her mother’s action with another wipe. The child looks at her mother, says "ma ma," and gives her mother a hug.

Of the following, these developmental milestones are MOST typical for a child whose age is

A. 12 months

B. 15 months

C. 18 months

D. 24 months

E. 30 months

Page 834: AAP PREP 2009

2009 PREP SA on CD-ROM

page 834

Critique: 250 Preferred Response: C

The developmental milestones described in the vignette are appropriate for a child of 18 months of age. A healthy 18-month-old child may speak 7 to 10 words spontaneously and correctly, use words for wants or needs, identify one or more body parts, and imitate household tasks. Typically developing infants of 12 months of age can understand that a particular set of sounds represents a certain object or action and may be just beginning to say their first words. A 15-month-old child can say four to six specific words, use jargon, and indicate some desire or need by pointing. An 18-month-old child may have a vocabulary of 20 to 29 words and will point to four body parts on request. A 24-month-old child typically has a vocabulary of more than 50 words, is beginning to use two- to three-word sentences, and will point to seven body parts and name at least one part. A 30-month-old child can speak in eight- or nine-word sentences, identify his or her sex, name seven body parts, and relate events that occurred 2 to 3 days ago. When evaluating a child's language, cognitive, and behavioral development, the clinician needs to determine if the language development is appropriate to the cognitive development and if there are any atypical social behaviors.

References:

Dedrick C. Developmental milestones. Developmental Behavioral Pediatrics Online. 2005. Available at: http://dbpeds.org/articles/detail.cfm?TextID=%20701

Knobloch H, Stevens FM, Malone AF. The revised developmental stages. In: Manual of Developmental Diagnosis. Albany, NY: Developmental Evaluation Materials, Inc; 1987:17-120

Page Glascoe F. Developmental screening. In: Parker S, Zukerman B, Augustyn M, eds. Developmental and Behavioral Pediatrics: A Handbook for Primary Care. 2nd ed. Philadelphia, Pa: Lippincott Williams & Wilkins; 2005:41-50

Whitaker T, Palmer F. The developmental history. In: Accardo PJ. Capute & Accardo's Neurodevelopmental Disabilities in Infancy and Childhood. Volume I: Neurodevelopmental Diagnosis and Treatment. 3rd ed. Baltimore, Md: Paul H. Brookes Publishing Co; 2008:297-310

Page 835: AAP PREP 2009

2009 PREP SA on CD-ROM

page 835

Question: 251

The hospital laboratory calls your office to tell you that the rapid plasma reagin (RPR) test on the cord blood of a newborn you saw yesterday in the hospital is positive at 1:4. You recall that the physical examination findings for the infant were normal.

Of the following, the MOST appropriate approach to this infant is to

A. begin penicillin therapy

B. obtain further maternal history

C. place the infant in contact isolation

D. repeat the RPR in 3 months

E. report the case to the Department of Health

Page 836: AAP PREP 2009

2009 PREP SA on CD-ROM

page 836

Critique: 251 Preferred Response: B

Two serologic tests are used routinely to screen a patient for syphilis. The rapid plasma regain (RPR) and the Venereal Disease Research Laboratory (VDRL) tests both are based on the serologic detection of immunoglobulin G (IgG) antibody titers against lipoidal antigen from Treponema pallidum. These tests are inexpensive, provide rapid results, and can be quantitated. The quantitation tends to correlate with disease activity. Results of both of these nontreponemal screening tests should be confirmed with a treponemal test such as the fluorescent treponemal antibody absorption (FTA-ABS) or the T pallidum particle agglutination (TP-PA). The treponemal tests are specific for the diagnosis of syphilis, are nonquantitative, and usually remain positive for life. Maternal passage of IgG antibody begins after 28 weeks of pregnancy. Therefore, results of cord blood testing of a newborn, such as in the infant described in the vignette, reflect the mother's status. Because the infant has a positive cord blood titer at 1:4 without signs or symptoms of congenital syphilis, more maternal history is required. The RPR or VDRL can be falsely positive due to concomitant viral infections (eg, Epstein-Barr virus), connective tissue disease, malignancy, intravenous drug abuse, endocarditis, laboratory error, contamination of the specimen by Wharton jelly when cord blood is used, or even pregnancy, thus necessitating the need for more history. Until more history is obtained, there is no indication for penicillin therapy. Unless the mother has a history of untreated syphilis, there is no indication to make a report to the health department. Newborns who have open skin lesions or snuffles (chronic rhinorrhea) (Item C251) due to syphilis can be contagious, but because the patient in the vignette is asymptomatic, there is no need for isolation. Waiting 3 months and repeating the testing is not the best approach because if the mother does have syphilis without a history of treatment, the infant requires therapy before leaving the hospital. For patients who acquire syphilis through sexual contact, a positive RPR or VDRL with a positive confirmatory FTA-ABS or TP-PA confirms the diagnosis.

References:

American Academy of Pediatrics. Syphilis. In: Pickering LK, Baker CJ, Long SS, McMillan JA, eds. Red Book: 2006 Report of the Committee on Infectious Diseases. 27th ed. Elk Grove Village, Ill: American Academy of Pediatrics; 2006:631-644

Hyman EL, Adam HM. In brief: syphilis. Pediatr Rev. 2006;27:37-39. Available at: http://pedsinreview.aappublications.org/cgi/content/full/27/1/37

Page 837: AAP PREP 2009

2009 PREP SA on CD-ROM

page 837

Critique: 251

Rhinorrhea (snuffles) in an infant who has congenital syphilis. (Courtesy of M. Rimsza)

Page 838: AAP PREP 2009

2009 PREP SA on CD-ROM

page 838

Question: 252

You are speaking to a group of neonatal nurses about the laboratory methods that can be used to make the diagnosis of human immunodeficiency virus infection/acquired immune deficiency syndrome in high-risk infants.

Of the following, the test that is MOST likely to confirm the diagnosis is

A. cord blood culture

B. DNA polymerase chain reaction

C. neonatal specific immunoglobulin

D. p24 antigen

E. RNA polymerase chain reaction

Page 839: AAP PREP 2009

2009 PREP SA on CD-ROM

page 839

Critique: 252 Preferred Response: B

Diagnosing human immunodeficiency virus (HIV) infection in neonates and infants, especially in the perinatal period, is complicated by the prolonged persistence of maternal antibody following delivery. Standard serologic tests for the detection of HIV are not reliable until infants are approximately 18 months of age. In the neonatal period, infected infants produce only small amounts of HIV-specific immunoglobulin G antibodies to a restricted number of antigens, and these antibodies are obscured by maternal antibody, making them very difficult to measure. The preferred test for the diagnosis of HIV infection in infants is HIV nucleic acid detection by polymerase chain reaction (PCR) assay of DNA extracted from peripheral blood mononuclear cells. About 30% of infants who are infected with HIV have a positive DNA PCR assay result in samples obtained before 48 hours of age. A positive result identifies infants who were infected in utero. Approximately 93% of infected infants have detectable HIV DNA by 2 weeks of age, and 100% of infected infants have positive HIV DNA PCR assay results by 1 month of age. A single HIV DNA PCR assay has a sensitivity of 95% and a specificity of 97% on samples collected from infants 1 to 36 months of age. The HIV DNA PCR assay is more sensitive on a single assay than viral culture. Several alternative diagnostic methods have been suggested. Viral isolation by culture is expensive, has limited availability, and requires up to 28 days for positive results. This test no longer is recommended and has been replaced by the DNA PCR assay. The p24 antigen detection test is substantially less sensitive than HIV DNA PCR assay or culture. Results may vary, and the test is not recommended. HIV RNA PCR is not recommended for routine testing of infants and children younger than 18 months of age because a negative result does not exclude HIV infection definitely, especially if infection is acquired at the time of delivery, when HIV RNA concentrations in the plasma may be quite low.

References:

American Academy of Pediatrics. Human immunodeficiency virus infection. In: Pickering LK, Baker CJ, Long SS, McMillan JA, eds. Red Book: 2006 Report of the Committee on Infectious Diseases. 27th ed. Elk Grove Village, Ill: American Academy of Pediatrics; 2006:378-401

Maldarelli F. Diagnosis of human immunodeficiency virus infection. In: Mandell GL, Bennett JE, Dolin R, eds. Mandell, Douglas, and Bennett's Principles and Practice of Infectious Diseases. 6th ed. Philadelphia, Pa: Elsevier Churchill Livingstone; 2005:1506-1526

Pollack H, Zhan MX, Ilmet-Moore T, Ajuang-Simbiri K, Krasinski K, Borkowsky W. Ontogeny of anti-human immunodeficiency virus (HIV) antibody production in HIV-1-infected infants. Proc Natl Acad Sci USA. 1993;90:2340-2344. Available at: http://www.pnas.org/cgi/reprint/90/6/2340

Read JS and the Committee on Pediatric AIDS. Diagnosis of HIV-1 infections in children younger than 18 months in the United States. Pediatrics. 2007;120:e1547-e1562. Available at: http://pediatrics.aappublications.org/cgi/content/full/120/6/e1547

Page 840: AAP PREP 2009

2009 PREP SA on CD-ROM

page 840

Question: 253

The parents of a 3-year-old boy who has polyuria and polydipsia ask if anything can be done for their child’s symptoms and what the prognosis is for toilet training. After confirming a normal serum glucose value and a negative urine culture, you arrange for a water deprivation test at the hospital. The test begins at 9 am, and assessments are made hourly. At 0900, the patient weighs 14.1 kg, the serum osmolality is 290 mOsm/kg H2O, and the urine osmolality is 120 mOsm/kg H2O. The measurements made over the course of the test are summarized in Item Q253. Per protocol, the patient is given no food or fluids intravenously or orally. Aqueous vasopressin is administered subcutaneously at 1101, immediately after the 1100 laboratory samples are taken. After 4 hours, the test is stopped, and the patient is allowed to drink to prevent hypovolemia.

Of the following, the MOST likely diagnosis is

A. central diabetes insipidus

B. nephrogenic diabetes insipidus

C. no abnormalities in urinary concentrating ability

D. primary (psychogenic) polydipsia

E. inconclusive results necessitating repeat testing

Page 841: AAP PREP 2009

2009 PREP SA on CD-ROM

page 841

Question: 253

Page 842: AAP PREP 2009

2009 PREP SA on CD-ROM

page 842

Critique: 253 Preferred Response: B

When a child presents with polyuria and polydipsia, it is imperative for the clinician to exclude diabetes mellitus. Once excluded by a normal glucose value, as occurred in the vignette, the clinician should test for normal urinary concentrating ability, which usually is measured on a first morning urine specimen, making it difficult to obtain in the incontinent child. In this situation, a water deprivation test may be necessary to assess urinary concentrating ability appropriately. The test is conducted in the hospital due to the risks of dehydration in the patient who has diabetes insipidus (DI) and cannot concentrate urine adequately. Younger children who are not yet toilet trained require a urinary catheter to monitor urine output and allow for urine osmolar testing. In a healthy person, the water deprivation test results in weight loss, increased serum osmolality, and progressive increase in urine osmolality. The effective urinary concentrating ability demonstrates normal antidiuretic hormone (ADH) production and normal renal response to hypovolemia. In contrast, when the 4-hour test was administered to the boy in the vignette, he lost weight, developed a progressive increase in serum osmolality, but maintained a fixed urine osmolality. These findings are consistent with DI. A trial of aqueous vasopressin (VP) was given in an effort to differentiate between central DI (VP-responsive) and nephrogenic DI (VP-unresponsive). The patient's lack of response to VP indicates that he has nephrogenic DI (NDI). NDI is an X-linked disorder in 90% of cases and is caused by mutations in the V2R gene (located on the X chromosome), which encodes for the arginine vasopressin type 2 receptor (V2R) that is expressed on the basolateral membrane of the collecting tubules. When circulating antidiuretic hormone is presented to the V2 receptor, water channels (aquaporin-2 [AQP2]) translocate to the luminal surface of the collecting tubule, resulting in water reabsorption. Ten percent of NDI cases are autosomal recessive and caused by mutations in the AQP2 gene. Alternative clinical possibilities in the setting of polyuria and polydipsia include primary (psychogenic) polydipsia, a diagnosis that is excluded readily by a water deprivation test. Affected patients have normal ADH production and normal tubular response. Accordingly, when the plasma osmolality reaches 295 mOsm/kg H2O, the urine osmolality exceeds 800 mOsm/kg H2O. Results of the water deprivation test for the boy in the vignette are conclusive, demonstrating an abnormality in urinary concentrating ability that failed to respond to volume depletion (excluding primary polydipsia) and aqueous vasopressin (excluding central DI).

References:

Goodyer P. Disorders of tubular transport. In: Kher KK, Schnaper HW, Makker SP, eds. Clinical Pediatric Nephrology. 2nd ed. London, England: Informa Healthcare; 2007:317-336

Knoers NVAM, Monnens LAH. Nephrogenic diabetes insipidus. In: Avner ED, Harmon WE, Niaudet P, eds. Pediatric Nephrology. 5th ed. Philadelphia, Pa: Lippincott Williams & Wilkins; 2004:777-787

Rose BD, Post TW. Hyperosmolal states-hypernatremia. In: Clinical Physiology of Acid-base and Electrolyte Disorders. 5th ed. New York, NY: McGraw-Hill Medical Publishing Division; 2001:746-793

Page 843: AAP PREP 2009

2009 PREP SA on CD-ROM

page 843

Question: 254

An 18-year-old boy who has mild persistent asthma presents to the emergency department with a 2-week history of coughing and wheezing that has not improved with twice-daily use of his beta2 agonist metered dose inhaler (MDI). On physical examination, the teenager is breathing comfortably but often coughs and has audible expiratory wheezing. His vital signs are appropriate for age, but a room air pulse oximetry reading is 95%. Chest radiography shows some peribronchial streaking but no infiltrate, no consolidation, and a normal cardiac silhouette. His only other medication is a medium-dose inhaled corticosteroid.

Of the following, the MOST appropriate management step is to

A. begin a course of outpatient oral steroids

B. change his steroid inhaler to one that combines a steroid and long-acting beta2 agonist

C. double his steroid MDI dose

D. increase his rescue beta2 agonist to every 4 hours

E. observe the patient

Page 844: AAP PREP 2009

2009 PREP SA on CD-ROM

page 844

Critique: 254 Preferred Response: A

The primary goal of managing asthma exacerbations is early treatment, with special attention paid to infants and those who are at high risk for fatal asthma. Early treatment strategies include use of a written asthma plan, recognition of symptoms, appropriate intensification of therapy, and removal of possible allergens or irritants. The teenager described in the vignette has a history of persistent asthma and presents with a typical asthma exacerbation of moderate severity. The 2007 Expert Panel Report 3: Guidelines for the Diagnosis and Management of Asthma (available at www.nhlbi.nih.gov/guidelines/asthma/asthgdln.pdf) provides an excellent resource to determine the appropriate course of action for such patients. Two primary therapies advocated for asthma exacerbations are short-acting beta2 agonists (SABAs) and oral corticosteroids (evidence level A). For patients who have mild asthma exacerbations, a 24- to 48-hour course of a SABA is recommended. Patients who respond completely to this intervention need no further therapy. Patients who have moderate, severe, or life-threatening exacerbations should seek medical attention, and corticosteroids should be initiated promptly. Neither increasing the patient's SABA to every 4 hours nor observing him is an appropriate option because he has had symptoms for 2 weeks without improvement. Initiation of a combination steroid/long-acting beta2 agonist inhaler (eg, fluticasone/salmeterol or budesonide/formoterol) would not benefit him because it would only add a longer-duration beta2 agonist to his current medication regimen. Some clinicians advocate doubling the inhaled steroid dose at the onset of asthma symptoms. However, this approach has not been shown to be effective in reducing the severity or preventing progression of exacerbations. An emergency department study has supported quadrupling the dose of an inhaled corticosteroid for 7 days at the onset of worsening symptoms. This may obviate the need for oral steroids, but does not have the level of evidence as is present for oral corticosteroids.

References:

Harrison TW, Oborne J, Newton S, Tattersfield AE. Doubling the dose of inhaled corticosteroid to prevent asthma exacerbations: randomised controlled trial. Lancet. 2004;363:271-275. Abstract available at: http://www.ncbi.nlm.nih.gov/pubmed/14751699

Keeley D, McKean M. Asthma and other wheezing disorders in children. BMJ Clinical Evidence. 2006. Available for subscription at: http://clinicalevidence.bmj.com/ceweb/conditions/chd/0302/0302.jsp#Q1

National Heart, Lung, and Blood Institute. National Asthma Education and Prevention Program. Expert Panel Report 3: Guidelines for the Diagnosis and Management of Asthma. 2007. Available at: www.nhlbi.nih.gov/guidelines/asthma/asthgdln.pdf

Page 845: AAP PREP 2009

2009 PREP SA on CD-ROM

page 845

Question: 255

A 15-year-old boy presents to the emergency department after falling off of his skateboard. He was skating downhill at high speed when he hit a bump and fell off his board. He fell onto his left shoulder and struck his abdomen on the curb. He now complains of left shoulder pain. On physical examination, his heart rate is 110 beats/min, respiratory rate is 24 breaths/min and shallow due to pain, and blood pressure is 130/75 mm Hg. He refuses to move his left shoulder. His lung sounds are clear, and his abdomen is diffusely tender. Radiographs of his left shoulder are reported as normal. You order an abdominal computed tomography (CT) scan.

Of the following, the CT scan is MOST likely to demonstrate

A. abdominal aortic dissection

B. duodenal hematoma

C. pancreatic transection

D. retroperitoneal hemorrhage

E. splenic laceration

Page 846: AAP PREP 2009

2009 PREP SA on CD-ROM

page 846

Critique: 255 Preferred Response: E

Blunt abdominal trauma in children is second only to head injury as a cause of injury-related mortality. The most commonly injured intra-abdominal organs are the liver and spleen, comprising 60% of childhood blunt abdominal injuries, with renal, pancreatic, and bowel injuries occurring much less frequently. Most pediatric solid organ injuries are diagnosed by computed tomography (CT) scan, although ultrasonography is being used increasingly as an initial screening test. Most solid organ injuries are managed nonoperatively. The patient described in the vignette demonstrates classic signs and symptoms of a splenic injury, with left upper quadrant tenderness and left shoulder pain due to diaphragmatic irritation. Blunt force from the fall onto the curb is the likely mechanism. The CT scan is the diagnostic modality of choice because it can demonstrate the injury and extent of splenic damage (Item C255). With the exception of patients who have massive splenic disruption and hemodynamic instability, spleen preservation is preferred to splenectomy, with 90% to 98% of patients recovering fully without surgery. Although thoracic aortic dissection may be seen following blunt chest trauma (steering wheel versus chest) or a high-velocity deceleration mechanism, abdominal aortic dissection is an uncommon injury, especially in children, following blunt trauma. Duodenal hematomas typically are seen after a focal blow to the upper abdomen (the so-called "handlebar injury") or after a deceleration mechanism in a restrained passenger following a high-speed motor vehicle crash. Accounting for only 2% of blunt abdominal injuries in one study, these injuries are notoriously difficult to diagnose. They often are missed on initial, noncontrast abdominal CTs, and affected patients may present days after the initial trauma with vomiting and diffuse abdominal pain. A high index of suspicion related to the mechanism of injury is the key to timely diagnosis. Pancreatic injuries are uncommon and occur from similar mechanisms as intestinal injuries. These injuries also may be missed on initial CT scans. Elevated pancreatic enzyme values and persistent upper abdominal pain should prompt evaluation for a pancreatic injury. Conservative treatment in patients without pancreatic duct injury usually leads to full recovery. Retroperitoneal hemorrhage may result from significant renal trauma. Physical examination findings include flank ecchymoses and back pain. A CT scan should demonstrate both the presence of retroperitoneal blood as well as its source in the trauma victim.

References:

Holmes JF, Sokolove PE, Brant WE, et al. Identification of children with intra-abdominal injuries after blunt trauma. Ann Emerg Med. 2002;39:500-509. Abstract available at: http://www.ncbi.nlm.nih.gov/pubmed/11973557

Wegner S, Colletti JE, Van Wie D. Pediatric blunt abdominal trauma. Pediatr Clin North Am. 2006;53:243-256. Abstract available at: http://www.ncbi.nlm.nih.gov/pubmed/16574524

Page 847: AAP PREP 2009

2009 PREP SA on CD-ROM

page 847

Critique: 255

Axial computed tomography scan of the abdomen shows disruption of the spleen with a large hematoma. (Courtesy of D. Mulvihill)